Sunteți pe pagina 1din 134

SALES Case Digest (Atty.

Sarona)
Compiled by: Wigmore #wigmoreforever

PART I: CONCEPT OF SALE ownership by virtue of sale, as opposed to the ruling of both RTC
and CA. The execution of the heirs of Pido the Declaration of
Heirship and Waiver of Rights was held to be not tantamount to
1. BASIC CONCEPTS sale. Such declaration is only one whereby heirs adjudicate and
divide the estate left by the decedent among themselves as they
see fit. The Court further noted that waiver of hereditary rights is
1. Contract of Sale (Article 1458) different from sale of hereditary rights. Sale of hereditary rights
presupposes an existence of a contract of sale whereas waiver
TEODORO ACAP vs. CA of hereditary rights is an abdication or intentional relinquishment
G.R. No. 118114. December 7, 1995. of a known right with a knowledge of its existence and intention
to relinquish it in favor of other persons who are co-heirs in the
Padilla, J. succession. As delos Reyes is a stranger to the succession of
Cosme Pido, he cannot claim ownership over the lot on the sole
Doctrine: Ownership and real rights are acquired only pursuant basis of the document executed. Hence, private respondent
to a legal mode or process. While title is the juridical justification, delos Reyes had not acquired ownership over Lot 1130 and
mode is the actual process of acquisition or transfer of ownership consequently had no right to exact lease rentals from petitioner
over a thing in question. Acap.

FACTS: Teodoro Acap has been a tenant of a portion of land of


Lot No. 1130 Hinigaran Cadastre since 1960. Said lot was
formerly owned by Spouses Vasquez and Lorenza Oruma, which Toyota Shaw Inc. vs. Court of Appeals, and Sosa
upon their death was inherited by Felixberto. In 1975, Felixberto 244 SCRA 320
sold the lot to Cosme Pido. Acap remained to be a registered May 1995
tenant of the said land and religiously paid his leasehold rentals
to Pido and thereafter, upon his death, to his widow Laurenciana. FACTS:
On 1981, Pido’s wife and children executed a notarized
document denominated “Declaration of Heirship and Waiver of Luna L. Sosa and his son, Gilbert, went to purchase a yellow
Rights” of the land in favor Edy delos Reyes. Delos Reyes Toyota Lite Ace from the Toyota office at Shaw Boulevard, Pasig
alleged that he and Acap entered into an oral lease agreement (petitioner Toyota) on June 14, 1989 where they met Popong
whereby Acap undertook to pay him 10 cavans of rice per year Bernardo who was a sales representative of said branch. Sosa
as lease rental. From 1983 onwards Acap refused to pay further emphasized that he needed the car not later than June 17, 1989
lease rentals. In defense, Acap denied having entered in an oral because he, his family, and a balikbayan guest would be using it
lease agreement with delos Reyes and that he did not recognize on June 18 to go home to Marinduque where he will celebrate
his ownership over the land. As a matter of fact he alleged that his birthday on June 19. Bernardo assured Sosa that a unit
he continued to pay Laurenciana, Pido’s wife. Delos Reyes filed would be ready for pick up on June 17 at 10:00 in the morning,
a suit of recovery of possession against Acap and for the and signed the "Agreements Between Mr. Sosa & Popong
payment of rentals accruing to him as owner of the said lot. Trial Bernardo of Toyota Shaw, Inc.,” a document which did not
court rendered decision in favor of delos Reyes ruling that there mention anything about the full purchase price and the manner
was a perfected sale between heirs of Pido and delos Reyes the installments were to be paid. Sosa and Gilbert delivered the
over the said lot and ordered Acap to deliver possession of the down payment of P100,000.00 on June 15, 1989 and Bernardo
same to delos Reyes. Upon appeal, CA affirmed the lower court’s accomplished a printed Vehicle Sales Proposal (VSP) No. 928
decision. Hence, this petition. which showed Sosa’s full name and home address, that payment
is by "installment," to be financed by "B.A.," and that the
ISSUE: Whether delos Reyes acquired ownership over the lot in "BALANCE TO BE FINANCED" is "P274,137.00", but the spaces
question. provided for "Delivery Terms" were not filled-up.

HELD: NO. The Court noted that an asserted right or claim to When June 17 came, however, petitioner Toyota did not deliver
ownership or a real right over a thing arising from a juridical act, the Lite Ace. Hence, Sosa asked that his down payment be
however justified, is not per se sufficient to give rise to ownership refunded and petitioner Toyota issued also on June 17 a Far
over the res. That right or title must be completed by fulfilling East Bank check for the full amount of P100,000.00, the receipt
certain conditions imposed by law. Hence, ownership and real of which was shown by a check voucher of Toyota, which Sosa
rights are acquired only pursuant to a legal mode or process. signed with the reservation, "without prejudice to our future
While title is the juridical justification, mode is the actual process claims for damages." Petitioner Toyota contended that the B.A.
of acquisition or transfer of ownership over a thing in question. Finance disapproved Sosa’s the credit financing application and
Under Article 712 of the Civil Code, modes of acquisition may further alleged that a particular unit had already been reserved
either be original or derivative. Original modes of acquisition and earmarked for Sosa but could not be released due to the
include occupation, acquisitive prescription, law or intellectual uncertainty of payment of the balance of the purchase price.
creation. Derivative modes of acquisition on the other hand Toyota then gave Sosa the option to purchase the unit by paying
include succession mortis causa and tradition as a result of the full purchase price in cash but Sosa refused.
certain contracts such as sale, barter, donation, assignment or
mutuum. In the instant case, the Court determined whether delos The trial court found that there was a valid perfected contract of
Reyes acquired ownership over the lot in question through any of sale between Sosa and Toyota which bound the latter to deliver
the modes mentioned. It was ruled that he had not acquired the vehicle and that Toyota acted in bad faith in selling to another
the unit already reserved for Sosa, and the Court of Appeals
COMPILED BY: WIGMORE #WIGMOREFOREVER 1
SALES Case Digest (Atty. Sarona)
Compiled by: Wigmore #wigmoreforever

affirmed the said decision. NDC to sell the leased property in its favor.

ISSUE: Issue:
1. Whether or not there is a valid sale between NDC and PUP.
Was there a perfected contract of sale between respondent Sosa
and petitioner Toyota? Ruling
A contract of sale, as defined in the Civil Code, is a contract
COURT RULING: where one of the parties obligates himself to transfer the
ownership of and to deliver a determinate thing to the other or
The Supreme Court granted Toyota’s petition and dismissed others who shall pay therefore a sum certain in money or its
Sosa’s complaint for damages because the document entitled equivalent. It is therefore a general requisite for the existence of
“Agreements Between Mr. Sosa & Popong Bernardo of Toyota a valid and enforceable contract of sale that it be mutually
Shaw, Inc.,” was not a perfected contract of sale, but merely an obligatory, i.e., there should be a concurrence of the promise of
agreement between Mr. Sosa and Bernardo as private the vendor to sell a determinate thing and the promise of the
individuals and not between Mr. Sosa and Toyota as parties to a vendee to receive and pay for the property so delivered and
contract. transferred. The Civil Code provision is, in effect, a "catch-all"
provision which effectively brings within its grasp a whole gamut
There was no indication in the said document of any obligation of transfers whereby ownership of a thing is ceded for a
on the part of Toyota to transfer ownership of a determinate thing consideration.
to Sosa and neither was there a correlative obligation on the part All three (3) essential elements of a valid sale, without which
of the latter to pay therefor a price certain. The provision on the there can be no sale, were attendant in the "disposition" and
downpayment of P100,000.00 made no specific reference to a "transfer" of the property from NDC to PUP - consent of the
sale of a vehicle. If it was intended for a contract of sale, it could parties, determinate subject matter, and consideration therefor.
only refer to a sale on installment basis, as VSP No.928 Consent to the sale is obvious from the prefatory clauses of
executed on June 15, 1989 confirmed. The VSP also created no Memorandum Order No. 214 which explicitly states the
demandable right in favor of Sosa for the delivery of the vehicle acquiescence of the parties to the sale of the property.
to him, and its non-delivery did not cause any legally Furthermore, the cancellation of NDC's liabilities in favor of the
indemnifiable injury. National Government constituted the "consideration" for the sale.

Polytechnic University of the Philippines vs Court of Manila Metal Container Corporation vs Philippine
Appeals and Firestone Ceramics National Bank
National Development Corporation vs Firestone Ceramics [GR No. 166862, December 20, 2006]
Inc.
[GR No. 143513 and 143590. November 14, 2001] Callejo, Sr., J.:

Bellosilo, J.: Facts:


Facts: Petitioner was the owner of 8,015 square meters of parcel
Petitioner National Development Corp., a government owned of land located in Mandaluyong City, Metro Manila. To secure
and controlled corporation, had in its disposal a 10 hectares a P900,000.00 loan it had obtained from respondent Philippine
property. Sometime in May 1965, private respondent Firestone National Bank, petitioner executed a real estate mortgage over
Corporation manifested its desire to lease a portion of it for the lot. Respondent PNB later granted petitioner a new credit
ceramic manufacturing business. On August 24, 1965, both accommodation. On August 5, 1982, respondent PNB filed a
parties entered into a contract of lease for a term of 10 years petition for extrajudicial foreclosure of the real estate mortgage
renewable for another 10 years. Prior to the expiration of the and sought to have the property sold at public auction. After due
aforementioned contract, Firestone wrote NDC requesting for an notice and publication, the property was sold at public action
extension of their lease agreement. It was renewed with an where respondent PNB was declared the winning bidder.
express grant to Firestone of the first option to purchase the Petitioner sent a letter to PNB, requesting it to be granted an
leased premise in the event that it was decided "to dispose and extension of time to redeem/repurchase the property. Some PNB
sell the properties including the lot..." personnel informed that as a matter of policy, the bank does not
Cognizant of the impending expiration of the leased agreement, accept “partial redemption”. Since petitioner failed to redeem the
Firestone informed NDC through letters and calls that it was property, the Register of Deeds cancelled TCT No. 32098 and
renewing its lease. No answer was given. Firestone's issued a new title in favor of PNB.
predicament worsened when it learned of NDC's supposed plans Meanwhile, the Special Asset Management Department
to dispose the subject property in favor of petitioner Polytechnic (SAMD) had prepared a statement of account of petitioner’s
University of the Philippines. PUP referred to Memorandum obligation. It also recommended the management of PNB to
Order No. 214 issued by then President Aquino ordering the allow petitioner to repurchase the property for P1,574,560.oo.
transfer of the whole NDC compound to the National PNB rejected the offer and recommendation of SAMD. It instead
Government. The order of conveyance would automatically result suggested to petitioner to purchase the property
in the cancellation of NDC's total obligation in favor of the for P2,660,000.00, in its minimum market value. Petitioner
National Government. declared that it had already agreed to SAMD’s offer to purchase
Firestone instituted an action for specific performance to compel for P1,574,560.47 and deposited a P725,000.00.
COMPILED BY: WIGMORE #WIGMOREFOREVER 2
SALES Case Digest (Atty. Sarona)
Compiled by: Wigmore #wigmoreforever

Issue:
Whether or not petitioner and respondent PNB had entered HELD: No. The absence of a specific manner of payment in the
into a perfected contract for petitioner to repurchase the property terms and conditions of the contract makes it a contract to sell.
for respondent. Ownership was never transferred to the Cruzes. This is because
the manner of payment of the purchase price is an essential
Ruling: element before a valid and binding contract of sale can exist.
The SC affirmed the ruling of the appellate court that there Although the Civil Code does not expressly state that the minds
was no perfected contact of sale between the parties. of the parties must also meet on the terms or manner of payment
A contract is meeting of minds between two persons of the price, the same is needed, otherwise there is no sale.
whereby one binds himself, with respect to the other, to give Also, the Cruzes never transferred their house from the front
something or to render some service. Under 1818 of the Civil portion to the rear portion of the lot. It was evident in the contract
Code, there is no contract unless the following requisites concur: that they will transfer the house to the rear portion once they
1. Consent of the contracting parties; were able to buy it.
2. Objection certain which is the subject matter of the contract; The SC also ruled that the Fernandos were not buyers in bad
3. Cause of the obligation which is established. faith. There was no consummated sale between the Cruzes and
Contract is perfected by mere consent which is manifested the Gloriosos. In a contract to sell, there being no previous sale
by the meeting of the offer and the acceptance upon the thing of the property, a third person buying such property despite the
and causes which are to constitute the contract. Once perfected, fulfillment of the suspensive condition such as the full payment of
the bind between other contracting parties and the obligations the purchase price, for instance, cannot be deemed a buyer in
arising therefrom have the form of law between the parties and bad faith and the prospective buyer cannot seek the relief of
should be complied in good faith. The absence of any essential reconveyance of the property. There is no double sale in such
element will negate the existence of a perfected contract of sale. case. Title to the property will transfer to the buyer
after registration because there is no defect in the owner-seller’s
The court ruled in Boston Bank of the Philippines vs title per se, but the latter, of course, may be sued for damages by
Manalo: the intending buyer.
“A definite agreement as to the price is an essential
element of a binding agreement to sell personal or real property
because it seriously affects the rights and obligations of the
parties. Price is an essential element in the formation of a
binding and enforceable contract of sale. The fixing of the price Velarde vs CA
can never be left to the decision of one of the contracting parties.
But a price fixed by one of the contracting parties, if accepted by Facts: David Raymundo (private respondent) is the absolute and
the other, gives rise to a perfected sale.” registered owner of a parcel of land, located at 1918 Kamias St.,
In the case at bar, the parties to the contract is between Dasmariñas Village Makati, together with the house and other
Manila Metal Container Corporation and Philippine National Bank improvements, which was under lease. It was negotiated by
and not to Special Asset Management Department. Since the David’s father with plaintiffs Avelina and Mariano Velarde
price offered by PNB was not accepted, there is no contract. (petitioners). A Deed of Sale with Assumption of Mortgage was
Hence it cannot serve as a binding juridical relation between the executed in favor of the plaintiffs. Part of the consideration of the
parties. sale was the vendee’s assumption to pay the mortgage
obligations of the property sold in the amount of P 1,800,000.00
in favor of the Bank of the Philippine Islands. And while
their application for the assumption of the mortgage obligations is
Spouses Cruz vs Fernando not yet approved by the mortgagee bank, they have agreed to
pay the mortgage obligations on the property with the bank in the
477 SCRA 173 – Civil Law – Law on Sale – Manner of Payment name of Mr. David Raymundo. It was further stated that “in the
Essential in a Contract of Sale event Velardes violate any of the terms and conditions of the said
In 1983, Cruz executed a Kasunduan with the Gloriosos for the Deed of Real Estate Mortgage, they agree that the down
consideration of the rear portion of a 223 sq m lot. payment P800,000.00, plus all the payments made with the BPI
The Kasunduan provides that the lot will be sold at a P40 per sq on the mortgage loan, shall be forfeited in Favor of Mr.
m. That the portion of the lot to be sold is the rear portion of it. Raymundo, as and by way of liquidated damages, w/out
That upon selling, the Cruz will transfer their house from the front necessity of notice or any judicial declaration to that effect, and
portion to the rear portion of the land once it is bought. That they Mr. Raymundo shall resume total and complete ownership and
will have a right of way from the front portion going to the back possession of the property, and the same shall be deemed
end of the lot. The Cruz never gave anything to the Gloriosos for automatically cancelled”, signed by the Velardes.
there was an alleged failure to have the land surveyed. Due to
non payment, the Gloriosos instead sold the whole lot (back and Pursuant to said agreements, plaintiffs paid BPI the monthly
rear portion) to the Fernandos. interest loan for three months but stopped in paying the
In 1994, after repeated demands, the Fernandos filed a case in mortgage when informed that their application for the assumption
court for accion publiciana demanding the Cruz to vacate the lot of mortgage was not approved. The defendants through a
and to pay a rental of P500.00. The RTC ruled in favor of the counsel, wrote plaintiffs informing the latter that their non-
Fernandos. The CA affirmed the RTC ruling. payment to the mortgagee bank constituted non-performance of
their obligation and the cancellation and rescission of the
ISSUE: Whether or not what transpired between the Cruzes and intended sale. And after two days, the plaintiffs responded and
the Gloriosos was a contract of sale. advised the vendor that he is willing to pay provided that Mr.
COMPILED BY: WIGMORE #WIGMOREFOREVER 3
SALES Case Digest (Atty. Sarona)
Compiled by: Wigmore #wigmoreforever

Raymundo: (1) delivers actual possession of the property to square meters at the corner of Meralco Avenue and General
them not later than January 15, 1987 for their occupancy (2) Capinpin Street, Barrio Oranbo, Pasig City. On February 21,
causes the release of title and mortgage from the BPI and make 1994, the properties were offered for sale for P52,140,000.00 in
the title available and free from any liens and encumbrances (3) cash. The offer was made to Atty. Helena M. Dauz who was
executes an absolute deed of sale in their favor free from any acting for respondent spouses as undisclosed principals. Atty.
liens and encumbrances not later than Jan. 21, 1987. Dauz signified her clients’ interest in purchasing the properties
for the amount for which they were offered by petitioner, under
The RTC of Makati dismissed the complaint of the petitioners the following terms: the sum of P500,000.00 would be given as
against Mr. Raymundo for specific performance, nullity of earnest money and the balance would be paid in eight equal
cancellation, writ of possession and damages. However, their monthly installments from May to December, 1994.
Motion for Reconsideration was granted and the Court instructed
petitioners to pay the balance of P 1.8 million to private However, petitioner refused the counter-offer. On March 29,
respondent who, in turn were ordered to execute a deed of 1994, Atty. Dauz wrote another letter proposing the following
absolute sale and to surrender possession of the disputed terms for the purchase of the properties, viz: Enclosing herewith
property to petitioners. the sum of P1,000,000.00 representing earnest-deposit money,
subject to the following conditions.
Upon the appeal of the private respondent to the CA, the court 1. We will be given the exclusive option to purchase the property
upheld the earlier decision of the RTC regarding the validity of within the 30 days from date of your acceptance of this offer.
the rescission made by private respondents. 2. During said period, we will negotiate on the terms and
conditions of the purchase; SMPPI will secure the necessary
Issue: Whether the rescission of contract made by the private Management and Board approvals; and we initiate the
respondent is valid. documentation if there is mutual agreementbetween us.
3. In the event that we do not come to an agreement on this
Held: There is a breach of contract because the petitioners did transaction, the said amount of P1,000,000.00 shall be
not merely stopped paying the mortgage obligations but they refundable to us in full upon demand.
also failed to pay the balance purchase price. Their conditional
offer to Mr. Raymundo cannot take the place of actual Isidro A. Sobrecarey, San Miguel’s vice-president and operations
payment as would discharge the obligation of the buyer under manager for corporate real estate, indicated his conformity to the
contract of sale. offer by affixing his signature to the letter and accepted the
"earnest-deposit" of P1 million. Upon request of respondent
Mr. Raymundo’s source of right to rescind the contract is Art. spouses, Sobrecarey ordered the removal of the "FOR SALE"
1191 of the Civil Code predicated on a breach of faith by the sign from the properties. Atty. Dauz and Sobrecarey then
other party who violates the reciprocity between them. Moreover, commenced negotiations.
the new obligations as preconditions to the performance of the
petitioners’ own obligation were repudiation of an existing Sobrecarey informed Atty. Dauz that San Miguel was willing to
obligation, which was legally due and demandable under the sell the subject properties on a 90-day term. Atty. Dauz
contract of sale. countered with an offer of 6 months within which to pay.The
parties again met during which Sobrecarey informed Atty.
The breach committed by the petitioners was the non- Dauz that San Miguel had not yet acted on her counter-offer.
performance of a reciprocal obligation. The mutual restitution is Atty. Dauz proposed a 4-month period of amortization.
required to bring back the parties to their original situation prior to
the inception of the contract. The initial payment and the On April 25, 1994, Atty. Dauz asked for an extension of 45 days
mortgage payments advanced by petitioners should be returned within which to exercise her option to purchase the property,
by private respondents, lest the latter unjustly enriched at the adding that within that period to finalize the agreement." Her
expense of the other. Rescission creates the obligation request was granted. On July 7, 1994, petitioner, through its
to return the obligation of contract. To rescind, is to declare a president and chief executive officer, Federico Gonzales, wrote
contract void at its inception and to put an end to it as though it Atty. Dauz informing her that because the parties failed to agree
never was. on the terms and conditions of the sale despite the extension
granted by petitioner, the latter was returning the amount of P1
The decision of the CA is affirmed with modification that private million given as "earnest-deposit."
respondents are ordered to return to petitioners, the amount they
have received in advanced payment. Sps Huang demanded the execution within 5 days of a deed of
sale covering the properties. Sps Huang attempted to return the
"earnest-deposit" but San Miguel refused on the ground that
2. Stages in the life of Contract of Sale respondents’ option to purchase had already expired. On August
16, 1994, spouses Huang filed a complaint for specific
performance against San Miguel. SAN MIGUEL
SAN MIGUEL PROPERTIES v SPS HUANG
[G.R. No. 137290. July 31, 2000] (1) the alleged "exclusive option" of respondent spouses lacked
a consideration separate and distinct from the purchase price
FACTS: San Miguel Properties Philippines, Inc. is a domestic and was thus unenforceable and
corporation engaged in the purchase and sale of real properties.
Part of its inventory are two parcels of land totalling 1, 738 (2) the complaint did not allege a cause of action because there
was no "meeting of the minds" between the parties and,
COMPILED BY: WIGMORE #WIGMOREFOREVER 4
SALES Case Digest (Atty. Sarona)
Compiled by: Wigmore #wigmoreforever

therefore, no perfected contract of sale. existence of a perfected sale. In the absence of a perfected
contract of sale, it is immaterial whether Isidro A. Sobrecarey had
TC Dismissed. CA REVERSED: held that all the requisites of a the authority to enter into a contract of sale in behalf of petitioner.
perfected contract of sale had been complied with as the offer
made on March 29, 1994, in connection with which the earnest
money in the amount of P1 million was tendered by respondents,
had already been accepted by petitioner. The fact the parties had 2. ESSENTIAL CHARACTERISTICS OF A CONTRACT OF
not agreed on the mode of payment did not affect the contract as SALE
such is not an essential element for its validity. CA also ruled that
Sobrecarey had no authority to sell the subject real properties 1. Nominate and Principal
ISSUE
WON there was a perfected contract of sale between the parties 2. Consensual

RULING: No. On alleged payment and acceptance Earnest


money, Sps Huang did not give the P1 million as "earnest QUIJADA vs. COURT OF APPEALS
money" as provided by Art. 1482 of the Civil Code. They G.R. No. 126444 December 4, 1998
presented the amount merely as a deposit of what would
eventually become the earnest money or downpayment should a FACTS: Petitioners, as heirs of the late Trinidad Quijada, filed a
contract of sale be made by them. The amount was thus given complaint against private respondents for quieting of title,
not as a part of the purchase price and as proof of the perfection recovery of possession and ownership of parcels of land with
of the contract of sale but only as a guarantee that respondents claim for attorney's fees and damages. Petitioners are the
would not back out of the sale. children of the late Trinidad Corvera Vda, de Quijada. Trinidad
was one of the heirs of the late Pedro Corvera and inherited from
In the present case, the P1 million "earnest-deposit" could not Pedro the 2-hectare parcel of land subject of the case, situated in
have been given as earnest money as contemplated in Art. 1482 the barrio of San Agustin, Talacogon, Agusan del Sur. On April 5,
because, at the time when petitioner accepted the terms of 1956, Trinidad together with her sisters Leonila Corvera and Paz
respondents’ offer of March 29, 1994, their contract had not yet and brother Epapiadito executed a conditional deed of donation
been perfected. of subject land in favor of the Municipality of Talacogon, the
condition being that the parcel of land shall be used solely and
The first condition: option period of 30 days sufficiently shows exclusively as part of the campus of the proposed provincial high
that a sale was never perfected. Acceptance of this condition did school in Talacogon. Apparently, Trinidad remained in possession
not give rise to a perfected sale but merely to an option or an of the parcel of land despite the donation.
accepted unilateral promise. Art. 1479(2) states that an accepted
unilateral promise to buy or sell a determinate thing for a price On July 29, 1962, Trinidad sold (1) hectare of the subject parcel
certain is binding upon the promisor only if the promise is of land to defendant-appellant Regalado Mondejar and verbally
supported by a distinct consideration. There is no showing here sold the remaining (1) hectare also to defendant-appellant
of any consideration for the option. Lacking any proof of such without the benefit of a written deed of sale and evidenced solely
consideration, the option isunenforceable. by receipts of payment. In 1980, the heirs of Trinidad (deceased)
filed a complaint for forcible entry against Mondejar, which
Second condition: that, during the option period, the parties complaint was, however, dismissed for failure to prosecute.
would negotiate the terms and conditions of the purchase. In the
present case, the parties never got past the negotiation stage. In 1987, the proposed provincial high school having failed to
Nothing more than offers and counter-offers which did not materialize, the Sangguniang Bayan of the municipality of
amount to any final arrangement containing the essential Talacogon enacted a resolution reverting the (2) hectares of land
elements of a contract of sale. While the parties already agreed donated back to the donors. In the meantime, Regalado
on the real properties which were the objects of the sale and on Mondejar sold portions of the land to (respondents) Fernando,
the purchase price, the fact remains that they failed to arrive at Rodolfo Goloran , Efren Guden and Ernesto Goloran. On July 5,
mutually acceptable terms of payment, despite the 45-day 1988, Heirs of Trinidad filed this action alleging that their
extension given by petitioner. deceased mother never sold, conveyed, transferred or disposed
of the property in question to any person or entity much less to
Also, the manner of payment of the purchase price is an Regalado Mondejar save the donation made to the Municipality
essential element before a valid and binding contract of sale can of Talacogon in 1956 and that at the time of the alleged sale to
exist. An agreement on the manner of payment goes into the Regalado Mondejar by Trinidad Quijada, the land still belongs to
price such that a disagreement on the manner of payment is the Municipality of Talacogon, hence, the supposed sale is null
tantamount to a failure to agree on the price. The fact, therefore, and void.
that the petitioners delivered to the respondent the sum as part
of the DP that they had to pay cannot be considered as sufficient Respondents alleged that the land in dispute was sold to
proof of the perfection of any purchase and sale agreement Regalado Mondejar, the one (1) hectare on July 29, 1962, and
between the parties herein under Art. 1482 of the new Civil the remaining one (1) hectare on installment basis until fully paid.
Code, as some essential matter - the terms of the payment - still
had to be mutually covenanted Thus, it is not the giving of ISSUE: WON the sale to Mondejar was valid
earnest money, but the proof of the concurrence of all the
essential elements of the contract of sale which establishes the RULING: YES. The donation made by Trinidad Quijada and her
COMPILED BY: WIGMORE #WIGMOREFOREVER 5
SALES Case Digest (Atty. Sarona)
Compiled by: Wigmore #wigmoreforever

brother and sisters was subject to the condition that the donated
property shall be "used solely and exclusively as a part of the SUBJECT: A house and lot located at No. 7757 Sherwood Street,
campus of the proposed Provincial High School in Talacogon. It Marcelo Green Village, Parañaque, Metro Manila worth P630
further provides that should "the proposed PHS be discontinued 000.00.
or if the same shall be opened but for some reason or another,
the same may in the future be closed" the donated property shall FACTS: In the exercise of the authority of Special Power Of
automatically revert to the donor. Attorney, on January 20, 1989, the heirs of the late Francisco Q.
Laforteza represented by Roberto Z. Laforteza and Gonzalo Z.
When the Municipality's acceptance of the donation was made Laforteza, Jr. entered into a Memorandum of Agreement
known to the donor, the former became the new owner of the (Contract to Sell) with the plaintiff over the subject property for
donated property — donation being a mode of acquiring and the sum of SIX HUNDRED THIRTY THOUSAND PESOS
transmitting ownership — notwithstanding the condition imposed (P630,000.00) payable as follows:
by the donee. Accordingly, ownership is immediately transferred
to the donee and that ownership will only revert to the donor if (a) P30,000.00 as earnest money, to be forfeited in favor of the
the resolutory condition is not fulfilled (construction of the defendants if the sale is not effected due to the fault of the
school). plaintiff;

Thus, at the time of the sales made in 1962 towards 1968, the (b) P600,000.00 upon issuance of the new certificate of title in
alleged seller (Trinidad) could not have sold the lots since she the name of the late Francisco Q. Laforteza and upon execution
had earlier transferred ownership thereof by virtue of the deed of of an extra-judicial settlement of the decedent's estate with sale
donation. So long as the resolutory condition subsists and is in favor of the plaintiff (Par. 2, Exh. "E", record, pp. 335-336).
capable of fulfillment, the donation remains effective and the
donee continues to be the owner subject only to the rights of the Significantly, the fourth paragraph of the Memorandum of
donor or his successors-in-interest under the deed of donation. Agreement (Contract to Sell) dated January 20, 1989 (Exh. "E",
supra.) contained a provision as follows:
Since no period was imposed by the donor on when must the
donee comply with the condition, the latter remains the owner so . . . . Upon issuance by the proper Court of the new title, the
long as he has tried to comply with the condition within a BUYER-LESSEE shall be notified in writing and said BUYER-
reasonable period. Such period, however, became irrelevant LESSEE shall have thirty (30) days to produce the balance of
herein when the donee-Municipality manifested through a P600,000.00 which shall be paid to the SELLER-LESSORS upon
resolution that it cannot comply with the condition of building a the execution of the Extrajudicial Settlement with sale.
school and the same was made known to the donor.
On January 20, 1989, plaintiff paid the earnest money of THIRTY
What the donor sold was the land itself which she no longer THOUSAND PESOS (P30,000.00), plus rentals for the subject
owns. It would have been different if the donor-seller sold her property .
interests over the property under the deed of donation which is
subject to the possibility of reversion of ownership arising from On September 18, 1998 3, defendant heirs, through their counsel
the non-fulfillment of the resolutory condition. There is one thing wrote a letter to the plaintiff furnishing the latter a copy of the
which militates against the claim of Quijadas. Sale, being a reconstituted title to the subject property, advising him that he
consensual contract, is perfected by mere consent, which is had thirty (3) days to produce the balance of P600,000.00 under
manifested the moment there is a meeting of the minds as to the the Memorandum of Agreement which plaintiff received on the
offer and acceptance thereof on three (3) elements: subject same date.
matter, price and terms of payment of the price. On October 18, 1989, plaintiff sent the defendant heirs a letter
Ownership by the seller on the thing sold at the time of the requesting for an extension of the THIRTY (30) DAYS deadline
perfection of the contract of sale is not an element for its up to November 15, 1989 within which to produce the balance of
perfection. What the law requires is that the seller has the right to P600,000.00. Defendant Roberto Z. Laforteza, assisted by his
transfer ownership at the time the thing sold is delivered. counsel Atty. Romeo L. Gutierrez, signed his conformity to the
plaintiff's letter request. The extension, however, does not appear
A perfected contract of sale cannot be challenged on the ground to have been approved by Gonzalo Z. Laforteza, the second
of non-ownership on the part of the seller at the time of its attorney-in-fact as his conformity does not appear to have been
perfection; hence, the sale is still valid. Trinidad Quijada's heirs secured.
and successors-in-interest became the owners of the subject
property upon the reversion of the ownership of the land to them. On November 15, 1989, plaintiff informed the defendant heirs,
Consequently, ownership is transferred to respondent Mondejar through defendant Roberto Z. Laforteza, that he already had the
and those who claim their right from him. balance of P600,000.00 covered by United Coconut Planters
Bank Manager's Check dated November 15, 1989 . However, the
defendants, refused to accept the balance .Defendant Roberto Z.
G.R. No. 137552 June 16, 2000 Laforteza had told him that the subject property was no longer for
ROBERTO Z. LAFORTEZA, GONZALO Z. LAFORTEZA, sale .
MICHAEL Z. LAFORTEZA, DENNIS Z. LAFORTEZA, and LEA
Z. LAFORTEZA vs. ALONZO MACHUCA On November 20, defendants informed plaintiff that they were
canceling the Memorandum of Agreement (Contract to Sell) in
PARTIES: HEIRS OF FRANCISCO LAFORTEZA – SELLER view of the plaintiff's failure to comply with his contractual
ALONZO MACHUCA – BUYER obligations .
COMPILED BY: WIGMORE #WIGMOREFOREVER 6
SALES Case Digest (Atty. Sarona)
Compiled by: Wigmore #wigmoreforever

determinate thing for a price certain is binding upon the


Thereafter, plaintiff reiterated his request to tender payment of promissor if the promise is supported by a consideration distinct
the balance of P600,000.00. Defendants, however, insisted on from the price.
the rescission of the Memorandum of Agreement. Thereafter,
plaintiff filed the instant action for specific performance. In the present case, the six-month period merely delayed the
demandability of the contract of sale and did not determine its
LOWER COURT: The lower court rendered judgment in favor of perfection for after the expiration of the six-month period, there
the Alonzo Machuca and against the defendant heirs of the late was an absolute obligation on the part of the petitioners and the
Francisco Q. Laforteza,. respondent to comply with the terms of the sale.

Petitioners appealed to the Court of Appeals, CA: This affirmed


with the decision of the lower court.
VDA. DE APE VS CA
Hence this petition wherein the petitioners raise the issues:
FACTS: Cleopas Ape died in 1950 and left a parcel of land (Lot
ISSUES: 2319) to his 11 children. The children never formally divided the
(1) Whether or not the MOA is an OPTION CONTRACT, property amongst themselves except through hantal-hantal
CONTRACT TO SELL or a CONTRACT OF SALE. whereby each just occupied a certain portion and developed
2) WON the six-month period during which the respondent each.
would be in possession of the property as lessee was a
period within which to exercise an option. On the other hand, the spouses Lumayno were interested in the
land so they started buying the portion of land that each of the
HELD: In the case at bench, there was a perfected agreement heirs occupied. On 11 Apr 1973, one of the children, Fortunato,
between the petitioners and the respondent whereby the entered into a contract of sale with Lumayno. In exchange of his
petitioners obligated themselves to transfer the ownership of and lot, Lumayno agreed to pay P5,000.00. She paid in advance
deliver the house and lot located at 7757 Sherwood St., Marcelo P30.00. Fortunato was given a receipt prepared by Lumayno’s
Green Village, Parañaque and the respondent to pay the price son in law (Andres Flores). Flores also acted as witness.
amounting to six hundred thousand pesos (P600,000.00). All the Lumayno also executed sales transactions with Fortunato’s
elements of a contract of sale were thus present.The siblings separately.
elements of a valid contract of sale under Article 1458 of the Civil
Code are (1) consent or meeting of the minds; (2) determinate In 1973, Lumayno compelled Fortunato to make the the delivery
subject matter and (3) price certain money or its equivalent. to her of the registrable deed of sale over Fortunato’s portion of
the Lot No. 2319. Fortunato assailed the validity of the contract
Even assuming for the sake of argument that the petitioners of sale. He also invoked his right to redeem (as a co-owner) the
were ready to comply with their obligation (and Machuca cannot), portions of land sold by his siblings to Lumayno. Fortunato died
we find that rescission of the contract will still not prosper. The during the pendency of the case.
rescission of a sale of an immovable property is specifically
governed by Article 1592 of the New Civil Code, which reads: ISSUE: Whether or not there was a valid contract of sale?

In the sale of immovable property, even though it may have been HELD: No. Fortunato was a “no read no write” person. It was
stipulated that upon failure to pay the price at the time agreed incumbent for the the other party to prove that details of the
upon the rescission of the contract shall of right take place, the contract was fully explained to Fortunato before Fortunato signed
vendee may pay, even after the expiration of the period, as long the receipt.
as no demand for rescission of the contract has been made upon A contract of sale is a consensual contract, thus, it is perfected
him either judicially or by a notarial act. After the demand, the by mere consent of the parties. It is born from the moment there
court may not grant him a new term. is a meeting of minds upon the thing which is the object of the
sale and upon the price. Upon its perfection, the parties may
It is not disputed that the petitioners did not make a judicial or reciprocally demand performance, that is, the vendee may
notarial demand for rescission. compel the transfer of the ownership and to deliver the object of
the sale while the vendor may demand the vendee to pay the
2) WON the six-month period during which the respondent thing sold. For there to be a perfected contract of sale, however,
would be in possession of the property as lessee was a the following elements must be present: consent, object, and
period within which to exercise an option. price in money or its equivalent.
The six-month period, during which the respondent would be in
possession of the property as lessee, was clearly not a period For consent to be valid, it must meet the following requisites:
within which to exercise an option. An option is a contract (a) it should be intelligent, or with an exact notion of the matter to
granting a privilege to buy or sell within an agreed time and at a which it refers;
determined price. An option contract is a separate and distinct (b) it should be free and
contract from that which the parties may enter into upon the (c) it should be spontaneous. Intelligence in consent is vitiated
consummation of the option. An option must be supported by by error; freedom by violence, intimidation or undue influence;
consideration. An option contract is governed by the second spontaneity by fraud.
paragraph of Article 1479 of the Civil Code, which reads:
Lumayno claimed that she explained fully the receipt to
Art. 1479. An accepted unilateral promise to buy or to sell a Fortunato, but Flores’ testimony belies it. Flores said there was
COMPILED BY: WIGMORE #WIGMOREFOREVER 7
SALES Case Digest (Atty. Sarona)
Compiled by: Wigmore #wigmoreforever

another witness but the other was a maid who also lacked
education. Further, Flores himself was not aware that the receipt Thus Villanueva filed a Complaint for specific performance
was “to transfer the ownership of Fortunato’s land to her mom-in- which the RTC granted anchoring its judgment on the finding
law”. It merely occurred to him to explain the details of the receipt that there existed a perfected contract of sae between PNB and
but he never did. Villanueva.

PNB appealed to the CA which reversed and set aside the


decision, stating that in the case at bench, consent, in respect
VILLANUEVA VS. PNB to the price and manner of its payment, is lacking. The record
G.R. NO. 154493, DECEMBER 6, 2006 shows that appellant, thru Guevara's July 6, 1990 letter, made a
qualified acceptance of appellee's letter-offer dated June 28,
FACTS: 1990 by imposing an asking price of P2,883,300.00 in cash for
The Special Assets Management Department (SAMD) of the Lot 19. The letter dated July 6, 1990 constituted a counter-offer
Philippine National Bank (PNB) issued an advertisement for the (Art. 1319, Civil Code), to which appellee made a new proposal,
sale of certain PNB properties in Calumpang, General Santos i.e., to pay the amount of P2,883,300.00 in staggered amounts,
City, including Lots 17 and 19 with advertised floor prices of that is, P600,000.00 as downpayment and the balance within
P1,409,000.00 and P2,268,000.00 respectively. two years in quarterly amortizations.

Villanueva offered to purchase the lots for P3,677,000.00. He CA held that a qualified acceptance, or one that involves a new
also manifested that he was depositing P400,000.00 to show proposal, constitutes a counter-offer and a rejection of the
his good faith but with the understanding that said amount may original offer (Art. 1319). Consequently, when something is
be treated as part of the payment of the purchase price only desired which is not exactly what is proposed in the offer, such
when his offer is accepted by PNB. acceptance is not sufficient to generate consent because any
modification or variation from the terms of the offer annuls the
At the bottom of said letter there appears an unsigned marginal offer.
note stating that P400,000.00 was deposited into Villanueva's
account with PNB-General Santos Branch. ISSUE: W/N there was a perfected Contract of Sale between
respondents PNB and herein petitioner Villanueva. NO
PNB forwarded the letter of Villanueva to Ramon Guevara, Vice
President, SAMD. Guevara informed Villanueva that only Lot HELD:
No. 19 is available and that the asking price therefor is Contracts of sale are perfected by mutual consent whereby the
P2,883,300.00. Guevara further wrote: seller obligates himself, for a price certain, to deliver and
transfer ownership of a specified thing or right to the buyer over
If our quoted price is acceptable to you, which the latter agrees. Mutual consent being a state of mind,
please submit a revised offer to purchase. Sale shall its existence may only be inferred from the confluence of two
be subject to our Board of Director's approval and to acts of the parties: an offer certain as to the object of the
other terms and conditions imposed by the Bank on contract and its consideration, and an acceptance of the offer
sale of acquired assets. which is absolute in that it refers to the exact object and
consideration embodied in said offer. While it is impossible to
Instead of submitting a revised offer, Villanueva merely inserted expect the acceptance to echo every nuance of the offer, it is
at the bottom of Guevara's letter a marginal note, which reads: imperative that it assents to those points in the offer which,
under the operative facts of each contract, are not only material
CONFORME: but motivating as well. Anything short of that level of mutuality
PRICE OF P2,883,300.00 (downpayment of produces not a contract but a mere counter-offer awaiting
P600,000.00 and the balance payable in two (2) years acceptance. More particularly on the matter of the consideration
at quarterly amortizations.) of the contract, the offer and its acceptance must be unanimous
Villanueva paid P200,000.00 to PNB which the latter issued a both on the rate of the payment and on its term. An acceptance
receipt to acknowledge receipt of the "partial payment deposit of an offer which agrees to the rate but varies the term is
on offer to purchase." On the dorsal portion of Official Receipt ineffective.
No. 16997, Villanueva signed a typewritten note, stating:
Tracing the transactions and letters between Villanueva and
This is a deposit made to show the sincerity of my PNB, it can be said that there was no perfected contract of sale
purchase offer with the understanding that it shall be between the parties. The first letter of PNB stating that only Lot
returned without interest if my offer is not favorably 19 was available was certainly not an acceptance but a mere
considered or be forfeited if my offer is approved but I counter-offer. Further, such counter-offer imposed two more
fail/refuse to push through the purchase. conditions that Villanueva submit a revised offer to purchase
based on the new price and that such sae of property be
Thereafter, however, Guevara wrote Villanueva that SAMD is approved by the Board of Directors. However, Villanueva’s reply
deferring negotiations with him over said property and returning to said counter-offer was not an acceptance but a further
his deposit of P580,000.00. counter-offer since he qualified his acceptance proposing a two-
year payment
Undaunted, Villanueva attempted to deliver postdated checks
covering the balance of the purchase price but PNB refused the Moreover, Villanueva’s contention that the repudiation was
same. belated since PNB already agreed to his counter-offer when it
COMPILED BY: WIGMORE #WIGMOREFOREVER 8
SALES Case Digest (Atty. Sarona)
Compiled by: Wigmore #wigmoreforever

accepted his downpayment, the Court ruled that acceptance of They are to be performed simultaneously, so that the
Villanueva’s payments did not amount to an implied acceptance performance of one is conditioned upon the simultaneous
of his last counter-offer. PNB-GenSan Branch had no authority fulfillment of the other.
to bind PNB to a contract of Sale with Villanueva. Neither did
SAMD have authority to bind PNB. Both clearly stated that In the present case, the Deed of Sale contained a stipulation
whatever is offered will be subject to approval of PNB’s higher that the Corporation shall pay in full the downpayment upon
authorities. execution of the contract. However, based on Cortes’
admission, he agreed that the Corporation’s full payment of the
In sum, the amounts paid by petitioner were not in the nature of downpayment would depend upon the delivery of the TCTs of
downpayment or earnest money but were mere deposits or the three subject lots. As such, the corresponding reciprocal
proof of his interest in the purchase of Lot No. 19. Acceptance obligation of the Corporation’s payment was the transfer of titles
of said amounts by respondent does not presuppose perfection by Cortes. His obligation is not only to affix the signature in the
of any contract. Deed, but to set into motion the process that would facilitate
transfer of title of the lots.

As correctly found by the CA, Cortes never surrendered said


3. Bilateral and Reciprocal documents to the Corporation. Cortes avers that he delivered
the TCT’s through the broker’s son. He further avers that the
CORTES VS CA broker’s son delivered it to the broker, who in turn delivered
G.R. NO. 126083, JULY 12, 2006 them to the Corporation. However, Marcosa Sanchez’s
unrebutted testimony is that, she did not receive the TCTs. She
FACTS: also denied knowledge of delivery thereof to her son, Manny.
For the purchase price of 3.7M, Villa Esperanza Development
Corporation and Antonio Cortes entered into a contract of sale What further strengthened the findings of the Court of Appeals
over the lots located at Baclaran, Parañaque, Metro Manila. The that Cortes did not surrender the subject documents was the
Corporation advanced to Cortes the total sum of P1,213,000.00. offer of Cortes’ counsel at the pre-trial to deliver the TCTs and
Later, in September 1983, the parties executed a deed of the Deed of Absolute Sale if the Corporation will pay the
absolute sale on the following terms: balance of the down payment. Indeed, if the said documents
were already in the hands of the Corporation, there was no
The Corporation shall advance 2.2 M as need for Cortes’ counsel to make such offer.
downpayment, and Cortes shall likewise deliver the
TCT for the 3 lots. Considering that their obligation was reciprocal, performance
The balance of 1.5M shall be payable within a year thereof must be simultaneous. The mutual inaction of Cortes
from the date of the execution. and the Corporation therefore gave rise to a compensation
morae or default on the part of both parties because neither has
The Corporation filed the instant case for specific performance completed their part in their reciprocal obligation. Cortes is yet
seeking to compel Cortes to deliver the TCTs and the original to deliver the original copy of the notarized Deed and the TCTs,
copy of the Deed of Absolute Sale. According to the while the Corporation is yet to pay in full the agreed down
Corporation, despite its readiness and ability to pay the payment of P2,200,000.00. This mutual delay of the parties
purchase price, Cortes refused delivery of the sought cancels out the effects of default, such that it is as if no one is
documents. Cortes claimed that the owner’s duplicate copy of guilty of delay.
the three TCTs were surrendered to the Corporation and it is the
latter which refused to pay in full the agreed down payment. Additionally, under Article 1169 of the Civil Code, from the
moment one of the parties fulfills his obligation, delay by the
RTC rendered a decision rescinding the sale and directed other begins. Since Cortes did not perform his part, the
Cortes to return to the Corporation the amount of provision of the contract requiring the Corporation to pay in full
P1,213,000.00, plus interest. CA reversed the decision and the down payment never acquired obligatory force.
directed Cortes to execute a Deed of Absolute Sale conveying
the properties and to deliver the same to the Corporation
together with the TCTs, simultaneous with the Corporation’s ALMOCERA VS. ONG
payment of the balance of the purchase price of P2,487,000.00. 546 SCRA 164
G.R. NO. 170479
ISSUE: W/N there is delay in the performance of the parties’ FEBRUARY 18, 2008
obligations that would justify the rescission of the contract
of sale. FACTS:
Johnny Ong tried to acquire from Andre T. Almocera and First
HELD: Builder Multi-Purpose Cooperative (FBMC) a "townhome" in
Cebu City. As reflected in a Contract to Sell, the selling price of
There is no doubt that the contract of sale in question gave rise the unit was P3,400,000.00 pesos.
to a reciprocal obligation of the parties. Reciprocal obligations
are those which arise from the same cause, and which each Out of the purchase price, he was able to pay the amount of
party is a debtor and a creditor of the other, such that the P1,060,000.00.
obligation of one is dependent upon the obligation of the other.

COMPILED BY: WIGMORE #WIGMOREFOREVER 9


SALES Case Digest (Atty. Sarona)
Compiled by: Wigmore #wigmoreforever

Prior to the full payment of this amount, Ong claims that The contract was denominated as such and it contained the
defendants Andre Almocera and First Builders fraudulently provision that the unit shall be conveyed by way of an
concealed the fact that before and at the time of the perfection of Absolute Deed of Sale, together with the attendant
the aforesaid contract to sell, the property was already documents of Ownership – the Transfer Certificate of Title
mortgaged to and encumbered with the Land Bank of the and Certificate of Occupancy – and that the balance of the
Philippines (LBP). In addition, the construction of the house has contract price shall be paid upon the completion and delivery
long been delayed and remains unfinished. On March 13, 1999, of the unit, as well as the acceptance thereof by respondent.
Lot 4-a covering the unit was advertised in a local tabloid for All these clearly indicate that ownership of the townhouse
public auction for foreclosure of mortgage. It is the assertion of has not passed to respondent.
Ong that had it not for the fraudulent concealment of the
mortgage and encumbrance by defendants, he would have not The unit shall be completed and conveyed by way of an
entered into the contract to sell. Absolute Deed of Sale together with the attendant
documents of Ownership in the name of the BUYER – the
On the other hand, defendants assert that on March 20, 1995, Transfer Certificate of Title and Certificate of Occupancy
First Builders Multi-purpose Coop. Inc., borrowed money in the within a period of six (6) months from the signing of Contract
amount of P500,000.00 from Tommy Ong, plaintiff’s brother. This to Sell.
amount was used to finance the documentation requirements of
the LBP for the funding of the Atrium Town Homes. This loan will II. The respondent is justified in refusing to pay the
be applied in payment of one (1) town house unit which Tommy balance of the contract price.
Ong may eventually purchase from the project. When the project
was under way, Tommy Ong wanted to buy another townhouse From the terms of the contract, it is clear that petitioner and
for his brother, Johnny Ong, plaintiff herein, which then, the FBMC had the obligation to complete the townhouse unit
amount of P150,000.00 was given as additional partial payment. within six months from the signing of the contract. Upon
compliance therewith, the obligation of respondent to pay
However, the particular unit was not yet identified. It was only on the balance of P2,400,000.00 arises. Upon payment thereof,
January 10, 1997 that Tommy Ong identified Unit No. 4 plaintiff’s the townhouse shall be delivered and conveyed to
chosen unit and again tendered P350,000.00 as his third partial respondent upon the execution of the Absolute Deed of Sale
payment. When the contract to sell for Unit 4 was being drafted, and other relevant documents.
Tommy Ong requested that another contract to sell covering Unit
5 be made so as to give Johnny Ong another option to choose The evidence adduced shows that petitioner and FBMC
whichever unit he might decide to have. When the construction failed to fulfill their obligation -- to complete and deliver the
was already in full blast, defendants were informed by Tommy townhouse within the six-month period. With petitioner and
Ong that their final choice was Unit 5. It was only upon knowing FBMC’s non-fulfillment of their obligation, respondent
that the defendants will be selling Unit 4 to some other persons refused to pay the balance of the contract price. Respondent
for P4million that plaintiff changed his choice from Unit 5 to Unit does not ask that ownership of the townhouse be transferred
4. to him, but merely asks that the amount or down payment he
had made be returned to him.
In trying to recover the amount he paid as down payment for the
townhouse unit, Johnny Ong filed a complaint for Damages The contract subject of this case contains reciprocal
against Andre T. Almocera and FBMC alleging that they were obligations which were to be fulfilled by the parties, i.e., to
guilty of fraudulent concealment and breach of contract when complete and deliver the townhouse within six months from
they sold to him a townhouse unit without divulging that the the execution of the contract to sell on the part of petitioner
same, at the time of the perfection of their contract, was already and FBMC, and to pay the balance of the contract price
mortgaged with the Land Bank of the Philippines (LBP), with the upon completion and delivery of the townhouse on the part
latter causing the foreclosure of the mortgage and the eventual of the respondent.
sale of the townhouse unit to a third person.
In the case at bar, the obligation of petitioner and FBMC
In their Answer, Almocera and FBMC denied liability claiming that which is to complete and deliver the townhouse unit within
the foreclosure of the mortgage on the townhouse unit was the prescribed period, is determinative of the respondent’s
caused by the failure of Johnny Ong to pay the balance of the obligation to pay the balance of the contract price. With their
price of said townhouse unit. failure to fulfill their obligation as stipulated in the contract,
they incurred delay and are liable for damages. They cannot
ISSUES: insist that respondent comply with his obligation. Where one
I. WON it was a contract to sell or a contract of sale. of the parties to a contract did not perform the undertaking to
II. WON the respondent’s refusal to pay the balance of the which he was bound by the terms of the agreement to
purchase price is justified. -YES perform, he is not entitled to insist upon the performance of
the other party.
HELD:
I. It cannot be disputed that the contract entered into Petitioner insists there was no delay when the townhouse
by the parties was a contract to sell. In a contract unit was not completed within six months from the signing of
to sell, ownership is retained by the seller and is the contract inasmuch as the mere lapse of the stipulated six
not to pass to the buyer until full payment of the (6) month period is not by itself enough to constitute delay
price. on his part and that of FBMC, since the law requires that
there must either be judicial or extrajudicial demand to fulfill
COMPILED BY: WIGMORE #WIGMOREFOREVER 10
SALES Case Digest (Atty. Sarona)
Compiled by: Wigmore #wigmoreforever

an obligation so that the obligor may be declared in default. sale did not occur within a period of one year did not extinguish
He argues there was no evidence introduced showing that a the obligation of Fonacier to pay Gaite the balance of P65,000
prior demand was made by respondent before the original because it does not seem to be the intention of the parties to the
action was instituted in the trial court. contract.

We do not agree. The Court looked into several circumstances which lead them to
conclude that the sale of the iron ore is but a suspensive term.
Demand is not necessary in the instant case. Demand by First, the words of the contract express no contingency in the
the respondent would be useless because the impossibility buyer's obligation to pay.
of complying with their (petitioner and FBMC) obligation was
due to their fault. If only they paid their loans with the LBP, Second, in the usual course of business, an onerous contract is
the mortgage on the subject townhouse would not have most likely preferred by the parties in a sale.
been foreclosed and thereafter sold to a third person.
Nothing is found in the record to evidence that Gaite desired or
assumed to run the risk of losing his right over the ore without
getting paid for it, or that Fonacier understood that Gaite
4. Onerous assumed any such risk. This is proved by the fact that Gaite
insisted on a bond a to guarantee payment of the P65,000.00, an
GAITE VS. FONACIER not only upon a bond by Fonacier, the Larap Mines & Smelting
2 SCRA 831 Co., and the company's stockholders, but also on one by a
G.R. NO. L-11827 surety company; and the fact that appellants did put up such
JULY 31, 1961 bonds indicates that they admitted the definite existence of their
obligation to pay the balance of P65,000.00.
FACTS:
Isabelo Fonacier executed a ‘Deed of Assignment’ in favor of Assuming that there could be doubt whether by the wording of
Fernando Gaite as his true and lawful attorney-in-fact so that the the contract the parties indented a suspensive condition or a
latter may enter into a contract for the exploration and suspensive period (dies ad quem) for the payment of the
development of the mining claims owned by Fonacier. Gaite P65,000.00, the rules of interpretation would incline the scales in
executed a general assignment conveying the development and favor of "the greater reciprocity of interests", since sale is
exploitation of said mining claims to Larap Iron Mines owned by essentially onerous. The Civil Code of the Philippines, Article
him. 1378, paragraph 1, in fine, provides:

Fonacier decided to revoke the ‘Deed of Assignment’ to which “If the contract is onerous, the doubt shall be settled in
Gaite assented on the condition that Fonacier is to pay him favor of the greatest reciprocity of interests.”
P75,000 for the 24,000 metric tons of iron lodes already
extracted and to retain the company name Larap Iron MInes. There can be no question that greater reciprocity obtains if the
Fonacier already paid P10,000 leaving a balance of P65,000 buyer' obligation is deemed to be actually existing, with only its
which, as agreed by them, is to be derived from the local sale of maturity (due date) postponed or deferred, that if such obligation
Iron ore made by Larap Iron Mines. On December 8, 1954, were viewed as nonexistent or not binding until the ore was sold.
Fonacier issued a security bond to secure payment of balance
with Far Eastern Surety and Insurance Co. but the surety The only rational view that can be taken is that the sale of the ore
provided that liability to the company will only attach when there to Fonacier was a sale on credit, and not an aleatory contract
had been actual sale of iron ore by Larap Iron Mines for an where the transferor, Gaite, would assume the risk of not being
amount of not less than P65,000 and that the bond will paid at all; and that the previous sale or shipment of the ore was
automatically expire on December 8, 1955. not a suspensive condition for the payment of the balance of the
agreed price, but was intended merely to fix the future date of the
No sale of the iron ore was made thereafter. Gaite failed to pay payment.
Fonacier the balance and the surety company refused to pay
contending that the bond expired automatically.
5. Commutative
Gaite instituted the present case. Fonacier argued that the
payment of the P65,000 balance was subject to the condition BUENAVENTURA VS CA (Nov. 20, 2003)
that it would be paid out of the first sale of the iron ore by Larap
Mines which did not happen. FACTS:
Defendant spouses Leonardo Joaquin and Feliciana Landrito are
ISSUE: the parents of plaintiffs Consolacion, Nora, Emma and Natividad
WON the obligation of Fonacier to pay Gaite the balance of as well as of defendants Fidel, Tomas, Artemio, Clarita, Felicitas,
P65,000 was extinguished because the iron ore was not sold Fe, and Gavino, all surnamed JOAQUIN. The married Joaquin
within a year. children are joined in this action by their respective spouses.
Sought to be declared null and void ab initio are certain deeds of
HELD: sale of real property executed by defendant parents Leonardo
The shipment or local sale of the iron ore is not a condition Joaquin and Feliciana Landrito in favor of their co-defendant
precedent (or suspensive) to the payment of the balance of children.
P65,000.00, but was only a suspensive period or term. That the
COMPILED BY: WIGMORE #WIGMOREFOREVER 11
SALES Case Digest (Atty. Sarona)
Compiled by: Wigmore #wigmoreforever

The petitioners argue that the deeds of sale are simulated as payment, or even the breach of that manner of payment. If the
they are null and void ab initio because: real price is not stated in the contract, then the contract of sale is
valid but subject to reform.
1. There was no actual valid consideration for the deeds of
sale over the properties in litis; Petitioners’ failure to prove absolute simulation of price is
2. Assuming that there was consideration in the sums magnified by their lack of knowledge of their respondent siblings’
reflected in the questioned deeds, the properties are financial capacity to buy the questioned lots. On the other hand,
more than three-fold times more valuable than the the Deeds of Sale which petitioners presented as evidence
measly sums appearing therein; plainly showed the cost of each lot sold. Not only did
3. The deeds of sale do not reflect and express the true respondents’ minds meet as to the purchase price, but the real
intent of the parties (vendors and vendees); and price was also stated in the Deeds of Sale. As of the filing of the
4. The purported sale of the properties in litis was the complaint, respondent siblings have also fully paid the price to
result of a deliberate conspiracy designed to unjustly their respondent father.
deprive the rest of the compulsory heirs of their legitime.
III. Article 1355: Except in cases specified by law, lesion or
The trial court ruled in favor of the defendants. The Court of inadequacy of cause shall not invalidate a contract, unless
Appeals affirmed the trial court’s decision. there has been fraud, mistake or undue influence.

ISSUE: Art. 1470. Gross inadequacy of price does not affect a


(I) WON petitioners have a legal interest over the properties contract of sale, except as may indicate a defect in the
subject of the Deeds of Sale consent, or that the parties really intended a donation or
(II) WON the Deeds of Sale are void for lack of consideration some other act or contract.
(III) WON the Deeds of Sale are void for gross inadequacy of
price Petitioners failed to prove any of the instances mentioned in
Articles 1355 and 1470 of the Civil Code which would invalidate,
RULING: or even affect, the Deeds of Sale. Indeed, there is no
requirement that the price be equal to the exact value of the
I. No, petitioners do not have a legal interest over the properties subject matter of sale. All the respondents believed that they
subject of the Deeds of Sale. Petitioners failed to show any legal received the commutative value of what they gave.
right to the properties. In actions for the annulment of contracts,
such as this action, the real parties are those who are parties to Courts cannot follow one every step of his life and extricate him
the agreement or are bound either principally or subsidiarily from bad bargains, protect him from unwise investments, relieve
or are prejudiced in their rights with respect to one of the him from one-sided contracts, or annul the effects of foolish acts.
contracting parties and can show the detriment which would There must be, in addition, a violation of the law, the commission
positively result to them from the contract even though they did of what the law knows as an actionable wrong, before the courts
not intervene in it. are authorized to lay hold of the situation and remedy it.

Petitioners do not have any legal interest over the properties


subject of the Deeds of Sale. As the appellate court stated, 6. Sale is Title and Not Mode
petitioners’ right to their parents properties is merely
inchoate and vests only upon their parents death. While still SAN LORENZO DEVELOPMENT CORP. VS CA (Jan 21, 2005)
living, the parents of petitioners are free to dispose of their
properties. FACTS:
Respondents Miguel Lu and Pacita Zavalla, (hereinafter, the
In their overzealousness to safeguard their future legitime, Spouses Lu) owned two (2) parcels of land situated in Sta. Rosa,
petitioners forget that theoretically, the sale of the lots to their Laguna covered by TCT No. T-39022 and TCT No. T-39023 both
siblings does not affect the value of their parents’ estate. While measuring 15,808 square meters or a total of 3.1616 hectares.
the sale of the lots reduced the estate, cash of equivalent value
replaced the lots taken from the estate. According to Pablo Babasanta, Spouses Lu sold the two parcels
of land to him, for the price of P15.00 per square meter.
II. It is not the act of payment of price that determines the validity Babasanta made a downpayment of P50,000.00 as evidenced
of a contract of sale. Payment of the price has nothing to do with by a memorandum receipt issued by Pacita Lu of the same date.
the perfection of the contract. Payment of the price goes into the Several other payments totaling P200,000.00 were made by
performance of the contract. Failure to pay the consideration is Babasanta. Babasanta wrote a letter to Pacita Lu to demand the
different from lack of consideration. The former results in a right execution of a final deed of sale in his favor so that he could
to demand the fulfillment or cancellation of the obligation under effect full payment of the purchase price. Babasanta notified the
an existing valid contract while the latter prevents the existence spouses about having received information that the spouses sold
of a valid contract. the same property to another without his knowledge and
consent.
A contract of sale is not a real contract, but a consensual
contract. As a consensual contract, a contract of sale becomes a The Spouses Lu alleged that Pacita Lu obtained loans from
binding and valid contract upon the meeting of the minds as to Babasanta and when the total advances of Pacita reached
price. If there is a meeting of the minds of the parties as to the P50,000.00, the latter and Babasanta, without the knowledge
price, the contract of sale is valid, despite the manner of and consent of Miguel Lu, had verbally agreed to transform the
COMPILED BY: WIGMORE #WIGMOREFOREVER 12
SALES Case Digest (Atty. Sarona)
Compiled by: Wigmore #wigmoreforever

transaction into a contract to sell the two parcels of land to perfected contract to sell.
Babasanta with the P50,000.00 to be considered as the
downpayment for the property and the balance to be paid on or The distinction between a contract to sell and a contract of sale
before 31 December 1987. Respondents Lu added that as of is quite germane. In a contract of sale, title passes to the vendee
November 1987, total payments made by Babasanta amounted upon the delivery of the thing sold; whereas in a contract to sell,
to only P 200,000.00 and the latter allegedly failed to pay the by agreement the ownership is reserved in the vendor and is not
balance of P260,000.00 despite repeated demands. Babasanta to pass until the full payment of the price. In a contract of sale,
had purportedly asked Pacita for a reduction of the price from the vendor has lost and cannot recover ownership until and
P15.00 to P12.00 per square meter and when the Spouses Lu unless the contract is resolved or rescinded; whereas in a
refused to grant Babasantas request, the latter rescinded the contract to sell, title is retained by the vendor until the full
contract to sell and declared that the original loan transaction just payment of the price, such payment being a positive suspensive
be carried out in that the spouses would be indebted to him in condition and failure of which is not a breach but an event that
the amount of P200,000.00 prevents the obligation of the vendor to convey title from
becoming effective.
SLDC (San Lorenzo Development Corporation) filed a Motion to
Intervene and alleged that it had legal interest in the subject
matter under litigation because the two parcels of land involved The perfection of a contract of sale should not be confused with
had been sold to it in a Deed of Absolute Sale with Mortgage. It its consummation. In relation to the acquisition and transfer of
alleged that it was a buyer in good faith and for value and ownership, it should be noted that sale is not a mode, but merely
therefore it had a better right over the property in litigation. a title. A mode is the legal means by which dominion or
ownership is created, transferred or destroyed, but title is only
RTC rendered a decision upholding the sale made to SLDC and the legal basis by which to affect dominion or ownership. Under
ordered Spouses Lu to pay Babasanta the sum of P200,000. CA Article 712 of the Civil Code, ownership and other real rights over
reversed the decision declaring the sale between Babasanta and property are acquired and transmitted by law, by donation, by
Spouses Lu valid and the sale to SLDC null and void on the testate and intestate succession, and in consequence of certain
ground that it was a purchaser in bad faith. contracts, by tradition. Contracts only constitute titles or rights to
the transfer or acquisition of ownership, while delivery or tradition
ISSUE: is the mode of accomplishing the same. Therefore, sale by itself
1. WON the agreement between Babasanta and Spouses does not transfer or affect ownership; the most that sale does is
Lu was a contract to sell and not a contract of sale to create the obligation to transfer ownership. It is tradition or
2. WON the registration of the sale after the annotation of delivery, as a consequence of sale, that actually transfers
the notice of lis pendens obliterate the effects of ownership.
delivery and possession in good faith which admittedly
had occurred prior to SLDCs knowledge of the Explicitly, the law provides that the ownership of the thing sold is
transaction in favor of Babasanta acquired by the vendee from the moment it is delivered to him in
any of the ways specified in Article 1497 to 1501. The word
RULING: delivered should not be taken restrictively to mean transfer of
1. The agreement between Babasanta and Spouses Lu was a actual physical possession of the property. The law recognizes
contract to sell and not a contract of sale. two principal modes of delivery, to wit: (1) actual delivery; and (2)
legal or constructive delivery.
A document was presented showing that Pacita Lu
acknowledged receipt of P50,000 and that she agreed to sell the Actual delivery consists in placing the thing sold in the control
3.6 hectares at P15 per square meter. The receipt signed by and possession of the vendee. Legal or constructive delivery, on
Pacita Lu merely states that she accepted the sum of the other hand, may be had through any of the following ways:
P50,000.00 from Babasanta as partial payment of 3.6 hectares the execution of a public instrument evidencing the sale;
of farm lot situated in Sta. Rosa, Laguna. While there is no symbolical tradition such as the delivery of the keys of the place
stipulation that the seller reserves the ownership of the property where the movable sold is being kept; traditio longa manu or by
until full payment of the price which is a distinguishing feature of mere consent or agreement if the movable sold cannot yet be
a contract to sell, the subsequent acts of the parties convince us transferred to the possession of the buyer at the time of the sale;
that the Spouses Lu never intended to transfer ownership to traditio brevi manu if the buyer already had possession of the
Babasanta except upon full payment of the purchase price. object even before the sale; and traditio constitutum
possessorium, where the seller remains in possession of the
Babasantas letter dated 22 May 1989 was quite telling. He stated property in a different capacity.
therein that despite his repeated requests for the execution of the
final deed of sale in his favor so that he could effect full payment Respondent Babasanta did not acquire ownership by the mere
of the price, Pacita Lu allegedly refused to do so. In effect, execution of the receipt by Pacita Lu acknowledging receipt of
Babasanta himself recognized that ownership of the property partial payment for the property. For one, the agreement
would not be transferred to him until such time as he shall have between Babasanta and the Spouses Lu, though valid, was not
effected full payment of the price. Moreover, had the sellers embodied in a public instrument. Hence, no constructive delivery
intended to transfer title, they could have easily executed the of the lands could have been effected. For another, Babasanta
document of sale in its required form simultaneously with their had not taken possession of the property at any time after the
acceptance of the partial payment, but they did not. Doubtlessly, perfection of the sale in his favor or exercised acts of dominion
the receipt signed by Pacita Lu should legally be considered as a over it despite his assertions that he was the rightful owner of the
COMPILED BY: WIGMORE #WIGMOREFOREVER 13
SALES Case Digest (Atty. Sarona)
Compiled by: Wigmore #wigmoreforever

lands. Simply stated, there was no delivery to Babasanta, by means of a Letter of Guaranty from the DBP, which Norkis
whether actual or constructive, which is essential to transfer agreed to accept. Credit was extended to Nepales for the price of
ownership of the property. Thus, even on the assumption that the the motorcycle payable by DBP upon release of his
perfected contract between the parties was a sale, ownership motorcycle loan. As security for the loan, Nepales would execute
could not have passed to Babasanta in the absence of delivery, a chattel mortgage on the motorcycle in favor of DBP.
since in a contract of sale ownership is transferred to the vendee Petitioner issued a sales invoice which Nepales signed in
only upon the delivery of the thing sold. conformity with the terms of the sale. In the meantime, however,
the motorcycle remained in Norkis’ possession. On January 22,
2. No, it did not obliterate the delivery and possession in good 1980, the motorcycle was delivered ¬to a certain Julian Nepales,
faith. allegedly the agent of Alberto Nepales. The motorcycle met an
accident on February 3, 1980 at Binalbagan, Negros Occidental.
An investigation conducted by the DBP revealed that the unit
It must be stressed that as early as 11 February 1989, the was being driven by a certain Zacarias Payba at the time of the
Spouses Lu executed the Option to Buy in favor of SLDC upon accident. The unit was a total wreck was returned.
receiving P316,160.00 as option money from SLDC. After SLDC
had paid more than one half of the agreed purchase price of On March 20, 1980, DBP released the proceeds of private
P1,264,640.00, the Spouses Lu subsequently executed on 3 respondent’s motorcycle loan to Norkis in the total sum
May 1989 a Deed of Absolute Sale in favor or SLDC. At the time of P7,500. As the price of the motorcycle later increased to
both deeds were executed, SLDC had no knowledge of the prior P7,828 in March, 1980, Nepales paid the difference of P328
transaction of the Spouses Lu with Babasanta. Simply stated, and demanded the delivery of the motorcycle. When Norkis
from the time of execution of the first deed up to the moment of could not deliver, he filed an action for specific performance
transfer and delivery of possession of the lands to SLDC, it had with damages against Norkis in the RTC of Negros Occidental.
acted in good faith and the subsequent annotation of lis pendens He alleged that Norkis failed to deliver the motorcycle which
has no effect at all on the consummated sale between SLDC and he purchased, thereby causing him damages. Norkis answered
the Spouses Lu. that the motorcycle had already been delivered to
private respondent before the accident, hence, the risk of loss or
Section 52 of the Property Registration Decree (P.D. No. 1529) damage had to be borne by him as owner of the unit.
which reads, thus: Sec. 52. Constructive notice upon registration.
Every conveyance, mortgage, lease, lien, attachment, order, ISSUE:
judgment, instrument or entry affecting registered land shall, if Whether or not there has been a transfer of ownership of the
registered, filed, or entered in the office of the Register of Deeds motorcycle to Alberto Nepales.
for the province or city where the land to which it relates lies, be
constructive notice to all persons from the time of such HELD:
registering, filing, or entering. No.The issuance of a sales invoice does not prove transfer of
ownership of the thing sold to the buyer. An invoice is nothing
However, the constructive notice operates as such by the more than a detailed statement of the nature, quantity and cost
express wording of Section 52from the time of the registration of of the thing sold and has been considered not a bill of sale. In all
the notice of lis pendens which in this case was effected only on forms of delivery, it is necessary that the act of delivery whether
2 June 1989, at which time the sale in favor of SLDC had long constructive or actual, be coupled with the intention of delivering
been consummated insofar as the obligation of the Spouses Lu the thing. The act, without the intention, is insufficient. When the
to transfer ownership over the property to SLDC is concerned. motorcycle was registered by Norkis in the name of private
More fundamentally, a notice of lis pendens only serves as a respondent, Norkis did not intend yet to transfer the title or
warning to a prospective purchaser or incumbrancer that the ownership to Nepales, but only to facilitate the execution of a
particular property is in litigation; and that he should keep his chattel mortgage in favor of the DBP for the release of the
hands off the same, unless he intends to gamble on the results buyer’s motorcycle loan. The Letter of Guarantee (Exh. 5) issued
of the litigation. Precisely, in this case SLDC has intervened in by the DBP, reveals that the execution in its favor of a
the pending litigation to protect its rights. Obviously, SLDCs faith chattel mortgage over the purchased vehicle is a pre-requisite for
in the merit of its cause has been vindicated with the Courts the approval of the buyer's loan. If Norkis would not accede to
present decision which is the ultimate denouement on the that arrangement, DBP would not approve private respondent's
controversy. loan application and, consequently, there would be no sale

Article 1496 of the Civil Code which provides that “in the absence
of an express assumption of risk by the buyer, the things sold
remain at seller’s risk until the ownership thereof is transferred to
NORKIS DISTRIBUTOR VS. CA the buyer,” is applicable to this case, for there was neither an
G.R. NO. 91029, FEBRUARY 7,1991; 193 SCRA 694 actual nor constructive delivery of the thing sold, hence, the risk
of loss should be borne by the seller, Norkis, which was still the
FACTS: owner and possessor of the motorcycle when it was wrecked.
Petitioner Norkis Distributors, Inc. is the distributor of Yamaha This is in accordance with the well- known doctrine of res perit
motorcycles in Negros Occidental. On September 20, domino.
1979, private respondent Alberto Nepales bought from the Norkis
Bacolod branch a brand new Yamaha Wonderbike
motorcycle Model YL2DX. The price of P7,500.00 was payable

COMPILED BY: WIGMORE #WIGMOREFOREVER 14


SALES Case Digest (Atty. Sarona)
Compiled by: Wigmore #wigmoreforever

AZNAR V.YAPDIANGCO thereof by stealing the same while it was in the custody of the
13 SCRA 486 latter's son.

FACTS: There is no adequate evidence on record as to whether Irineo


Theodoro Santos advertised in the newspapers the sale of his Santos voluntarily delivered the key to the car to the unidentified
Ford Fairlane 500. After the advertisement, a certain de Dios, person who went with him and L. De Dios to the place on
claiming to be the nephew of Marella, went to the residence of Azcarraga where a sister of Marella allegedly lived. But even
Santos and expressing his uncle’s intent to purchase the car. if Irineo Santos did, it was not the delivery contemplated by
Since Santos wasn't around, it was Irineo who talked with de Article 712 of the Civil Code. For then, it would be indisputable
Dios. On being informed, Santos advised his son to see Marella, that he turned it over to the unidentified companion only so that
which the son did. Marella expressed his intention to purchase he may drive Irineo Santos and De Dios to the said place on
the car. A deed of sale was prepared and Irineo was instructed Azcarraga and not to vest the title to the said vehicle to him as
by his father not to part with the deed and the car without agent of Vicente Marella. Article 712 above contemplates that the
receiving the purchase price from Marella. When irineo and de act be coupled with the intent of delivering the thing.
Dios arrived at the residence of Marella, the latter averred that
his money was short and had to borrow from his sister. He then The lower court was correct in applying Article 559 of the Civil
instructed de Dios and Irineo to go the supposed house of the Code to the case at bar, for under it, the rule is to the effect that if
sister to obtain the money with an unidentified person. He also the owner has lost a thing, or if he has been unlawfully deprived
asked Irineo to leave the deed to have his lawyer see it. Relying of it, he has a right to recover it, not only from the finder, thief
on the good faith of Marella, Irineo did as requested. Upon or robber, but also from third persons who may have acquired it
arriving at thehouse of Marella’s supposed to be sister, de Dios in good faith from such finder, thief or robber.
and the unidentified person then disappeared together with the
car. This prompted Santos to report the incident to the The said article establishes 2 exceptions to the general rule of
authorities. irrevindicabilty—to wit, the owner has lost the thing or has been
unlawfully deprived thereof. In these cases, the possessor
Thereafter, Marella was able to sell the car to Aznar. And while in cannot retain the thing as against the owner who may recover it
possession of the car, police authorities confiscated the same. without paying any indemnity, except when the possessor
This prompted Aznar to file an action for replevin. acquired it in a public sale. Furthermore, the common law
principle that where one of two innocent persons must suffer a
ISSUE: W Aznar has the better title to the car. No fraud perpetrated by another, the law imposes the loss upon the
party who, by his misplaced confidence, has enable the fraud to
HELD: Vicente Marella did not have any title to the property be committed, cannot be applied in this case, which is covered
under litigation because the same was never delivered to him. by an express provision of law.
He sought ownership or acquisition of it by virtue of the
contract. Vicente Marella could have acquired ownership or title
to the subject matter thereof only by the delivery or tradition of
the car to him.
EQUATORIAL REALTY DEVELOPMENT, INC., vs. MAYFAIR
THEATER, INC., G.R. No. 133879 November 21, 2001
Under Article 712 of the Civil Code, "ownership and other real
rights over property are acquired and transmitted by law, by
donation, by testate and intestate succession, and
in consequence of certain contracts, by tradition." As interpreted FACTS: This case stemmed from a Civil Case entitled "Mayfair"
by this Court in a host of cases, by this provision, ownership is Theater, Inc. v. Carmelo and Bauermann, Inc., et al.," where
not transferred by contract merely but by tradition or delivery. Carmelo & Bauermann, Inc entered into a Contract of Lease with
Contracts only constitute titles or rights to the transfer Mayfair Theater Inc. ("Mayfair") for a period of 20 years. The
or acquisition of ownership, while delivery or tradition is the mode lease covered a portion of the second floor and mezzanine of a
of accomplishing the same. two-storey building it owned which respondent used as a movie
house known as Maxim Theater.
For the legal acquisition and transfer of ownership and other Two years later,Mayfair entered into a second Contract of Lease
property rights, the thing transferred must be delivered, with Carmelo for the lease of another portion of the latter's
inasmuch as, according to settled jurisprudence, the tradition of property .In that space, Mayfair put up another movie house
the thing is a necessary and indispensable requisite in the known as Miramar Theater. The Contract of Lease was likewise
acquisition of said ownership by virtue of contract. So long as for a period of 20 years.
property is not delivered, the ownership over it is not
transferred by contract merely but by delivery. Contracts only Both leases contained a provision granting Mayfair a right of first
constitute titles or rights to the transfer or acquisition of refusal to purchase the subject properties. However, on July 30,
ownership, while delivery or tradition is the method of 1978 — within the 20-year-lease term — the subject properties
accomplishing the same, the title and the method of acquiring were sold by Carmelo to Equatorial Realty Development, Inc.
it being different in our law. In the case on hand, the car in ("Equatorial"), without their first being offered to Mayfair.
question was never delivered to the vendee by the vendor as to
As a result of the sale of the subject properties to Equatorial,
complete or consummate the transfer of ownership by virtue of
Mayfair filed a Complaint for (a) the annulment of the Deed of
the contract. It should be recalled that while there was indeed a
Absolute Sale between Carmelo and Equatorial, (b) specific
contract of sale between Vicente Marella and Teodoro Santos,
performance, and (c) damages. After trial on the merits, the
the former, as vendee, took possession of the subject matter
COMPILED BY: WIGMORE #WIGMOREFOREVER 15
SALES Case Digest (Atty. Sarona)
Compiled by: Wigmore #wigmoreforever

lower court rendered a Decision in favor of Carmelo and the part of the vendor, and the assumption of the same by the
Equatorial. vendee."
The Supreme Court made the following pronouncement for this It is clear that petitioner never took actual
case: The Deed of Absolute Sale between petitioners Equatorial control and possession of the property sold, in view of
Realty Development, Inc. and Carmelo & Bauermann, Inc.is respondent's timely objection to the sale and the continued
deemed rescinded; Carmelo & Bauermann is ordered to return to actual possession of the property. The objection took the form of
petitioner Equatorial Realty Development the purchase price. a court action impugning the sale which, as we know, was
The latter is directed to execute the deeds and documents rescinded by a judgment rendered by this Court in the mother
necessary to return ownership to Carmelo & Bauermann of the case. It has been held that the execution of a contract of sale as
disputed lots. Carmelo & Bauermann is ordered to allow Mayfair a form of constructive delivery is a legal fiction. It holds true only
Theater, Inc. to buy the aforesaid lots. when there is no impediment that may prevent the passing of the
property from the hands of the vendor into those of the
The foregoing Decision of this Court became final and executory. vendee. When there is such impediment, "fiction yields to reality
Subsequently, Mayfair filed a Motion for Execution, which the — the delivery has not been effected."
trial court granted.
Hence, respondent's opposition to the transfer of the property by
However, Carmelo could no longer be located. Thus, following way of sale to Equatorial was a legally sufficient impediment that
the order of execution of the trial court, Mayfair deposited with effectively prevented the passing of the property into the latter's
the clerk of court. The lower court issued a Deed of hands.
Reconveyance in favor of Carmelo and a Deed of Sale in favor of
Mayfair. On the basis of these documents, the Registry of Deeds The execution of a public instrument gives rise, therefore, only to
of Manila canceled Equatorial's titles and issued new Certificates a prima facie presumption of delivery. Such presumption is
of Title in the name of Mayfair. destroyed when the instrument itself expresses or implies that
delivery was not intended; or when by other means it is shown
Meanwhile, barely five months after Mayfair had submitted its that such delivery was not effected, because a third person was
Motion for Execution before the RTC of Manila, Equatorial filed actually in possession of the thing. In the latter case, the sale
action for the collection of a sum of money against Mayfair, cannot be considered consummated.
claiming payment of rentals or reasonable compensation for the
defendant's use of the subject premises after its lease contracts Ownership is acquired, not by mere agreement, but by tradition
had expired. or delivery. Under the factual environment of this controversy as
found by this Court in the mother case, Equatorial was never put
ISSUE: Whether or not Equatorial is entitled to back rentals in actual and effective control or possession of the property
RULING: No. Equatorial is not entitled to back rentals. because of Mayfair's timely objection.

No right of ownership was transferred from Carmelo to Equatorial In short, the sale to Equatorial may have been valid from
in view of a patent failure to deliver the property to the buyer. inception, but it was judicially rescinded before it could be
consummated. Petitioner never acquired ownership, not because
Rent is a civil fruit that belongs to the owner of the property the sale was void, as erroneously claimed by the trial court, but
producing it by right of accession. Consequently and ordinarily, because the sale was not consummated by a legally
the rentals that fell due from the time of the perfection of the sale effective delivery of the property sold.
to petitioner until its rescission by final judgment should belong to
the owner of the property during that period.
By a contract of sale, "one of the contracting parties obligates III. DISTINGUISHED FROM OTHER
himself to transfer ownership of and to deliver a determinate TRANSACTIONS/CONTRACT
thing and the other to pay therefor a price certain in money or its
equivalent."
1. Distinguished from Barter
Ownership of the thing sold is a real right, which the buyer
acquires only upon delivery of the thing to him "in any of the 2. Distinguished from Donation
ways specified in articles 1497 to 1501, or in any other manner
signifying an agreement that the possession is transferred from 3. Distinguished from Contract for Piece of Work
the vendor to the vendee." This right is transferred, not merely by
contract, but also by tradition or delivery. And there is said to be
INCHAUSTI AND CO. vs. ELLIS CROMWELL, Collector of
delivery if and when the thing sold "is placed in the control and
Internal Revenue
possession of the vendee." Thus, it has been held that while the
execution of a public instrument of sale is recognized by law as G.R. No. L-6584 October 16, 1911
equivalent to the delivery of the thing sold, such constructive or
symbolic delivery, being merely presumptive, is deemed negated FACTS: Inchausti is engaged in the business of buying and
by the failure of the vendee to take actual possession of the land selling at wholesale hemp. It is customary to sell hemp in bales.
sold. The operation of bailing hemp is designated among merchants
by the word "prensaje."
In the Law on Sales, delivery may be either actual or
constructive, but both forms of delivery contemplate "the In all sales of hemp by the plaintiff firm, the price is quoted to the
absolute giving up of the control and custody of the property on buyer at so much per picul, no mention being made of bailing;
but with the tacit understanding, unless otherwise expressly

COMPILED BY: WIGMORE #WIGMOREFOREVER 16


SALES Case Digest (Atty. Sarona)
Compiled by: Wigmore #wigmoreforever

agreed, that the hemp will be delivered in bales and that, course of his business manufactures or procures for the general
according to the custom prevailing among hemp merchants and market, whether the same is on hand at the time or not, is a
dealers in the Philippine Islands, a charge, is to be made against contract for the sale of goods to which the statute of frauds
the buyer under the denomination of "prensaje." This charge is applies. But if the goods are to be manufactured especially for
made in the same manner in all cases, even when the operation the purchaser and upon his special order, and not for the general
of bailing was performed by the plaintiff or by its principal long market, the case is not within the statute.
before the contract of sale was made.
It is clear to our minds that in the case at bar the baling was
Plaintiff Inchausti has always paid to the defendant Collector of performed for the general market and was not something done
Internal Revenue or to his predecessor in the office of the by plaintiff which was a result of any peculiar wording of the
Collector of Internal Revenue the tax collectible upon the selling particular contract between him and his vendee. It is undoubted
price expressly agreed upon for all hemp sold by the plaintiff firm, that the plaintiff prepared his hemp for the general market.
but has not, until compelled to do so paid the said tax upon sums
received from the purchaser of such hemp under the
denomination of "prensaje."
CELESTINO CO VS CIR (G.R. NO. L-8506)
Subsequently, the defendant, acting in his official capacity as
Collector of Internal Revenue of the Philippine Islands, made FACTS: Celestino Co & Company is a duly registered general
demand in writing upon the plaintiff firm for the payment as tax co-partnership doing business under the trade name of “Oriental
on the sums of money collected from purchasers of hemp under Sash Factory”. From 1946 to 1951 it paid percentage taxes of
the denomination of "prensaje." 7% on the gross receipts of its sash, door and window factory, in
accordance with sec. 186 of the National Internal Revenue Code
The plaintiff firm paid to the defendant under protest that the tax which is a tax on the original sales of articles by manufacturer,
for the collected money under the denomination of "prensaje" is producer or importer.
illegal upon the ground that the said charge does not constitute a
part of the selling price of the hemp, but is a charge made for the However, in 1952 it began to claim only 3% tax under Sec. 191,
service of baling the hemp. which is a tax on sales of services. Petitioner claims that it does
It is the contention of the defendant that the said charge made not manufacture ready-made doors, sash and windows for the
under the denomination of "prensaje" is in truth and in fact a part public, but only upon special orders from the customers, hence, it
of the gross value of the hemp sold and of its actual selling price. is not engaged in manufacturing under sec 186, but only in sales
of services covered by sec 191.
ISSUE: Whether or not the amount collected under the
denomination “pensaje” is part of the selling price of the hemp. Having failed to convince BIR, petitioner went to the Court of Tax
RULING: Yes. It is considered part of the selling price and the Appeal where it also failed. CTA, in its decision, holds that the
tax was properly imposed. “petitioner has chosen for its tradename and has offered itself to
the public as a “Factory”, which means it is out to do business, in
The distinction between a contract of sale and one for work, its chosen lines on a big scale.
labor, and materials is tested by the inquiry whether the thing
transferred is one no in existence and which never would have As a general rule, sash factories receive orders for doors and
existed but for the order of the party desiring to acquire it, or a windows of special design only in particular cases but the bulk of
thing which would have existed and been the subject of sale to their sales is derived from a ready-made doors and windows of
some other person, even if the order had not been given. It is standard sizes for the average home.. Even if we were to believe
clear that in the case at bar the hemp was in existence in baled petitioner’s claim that it does not manufacture ready-made sash,
form before the agreements of sale were made, or, at least, doors and windows for the public and that it makes these articles
would have been in existence even if none of the individual sales only special order of its customers that does not make it a
here in question had been consummated. It would have been contractor within the purview of section 191 of the national
baled, nevertheless, for sale to someone else, since, according Internal Revenue Code. There are no less than fifty occupations
to the agreed statement of facts, it is customary to sell hemp in enumerated in the aforesaid section and after reading carefully
bales. When a person stipulates for the future sale of articles each and every one of them, we cannot find under which the
which he is habitually making, and which at the time are not business of manufacturing sash, doors and windows upon
made or finished, it is essentially a contract of sale and not a special order of customers fall under the category” mentioned
contract for labor. It is otherwise when the article is made under Sec 191.
pursuant to agreement. Where labor is employed on the
materials of the seller he cannot maintain an action for work and ISSUE: Whether the petitioner company provides special
labor. If the article ordered by the purchaser is exactly such as services or is engaged in manufacturing.
the plaintiff makes and keeps on hand for sale to anyone, and no
change or modification of it is made at the defendant's request, it HELD: The important thing to remember is that Celestino Co &
is a contract of sale, even though it may be entirely made after, Company habitually makes sash, windows and doors, as it has
and in consequence of, the defendant's order for it. represented in its stationery and advertisements to the public.
That it “manufactures” the same is practically admitted by
A contract to make is a contract of sale if the article ordered is
appellant itself. The fact that windows and doors are made by it
already substantially in existence at the time of the order and
only when customers place their orders, does not alter the nature
merely requires some alteration, modification, or adoption to the
of the establishment, for it is obvious that it only accepted such
buyer's wishes or purposes. It is also held in that state that a
orders as called for the employment of such material-moulding,
contract for the sale of an article which the vendor in the ordinary
COMPILED BY: WIGMORE #WIGMOREFOREVER 17
SALES Case Digest (Atty. Sarona)
Compiled by: Wigmore #wigmoreforever

frames, panels-as it ordinarily manufactured or was in a position


habitually to manufacture. The Oriental Sash Factory does HELD: No. The Supreme Court holds that the private respondent
nothing more than sell the goods that it mass-produces or is a “manufacturer” as defined in the Tax Code and not a
habitually makes; sash, panels, mouldings, frames, cutting them “contractor” under Section 205(e) of the Tax Code.
to such sizes and combining them in such forms as its customers
may desire. Petitioner CIR wants to impress upon this Court that under Article
1467, the true test of whether or not the contract is a piece of
Appellant invokes Article 1467 of the New Civil Code to bolster work (and thus classifying private respondent as a contractor) or
its contention that in filing orders for windows and doors a contract of sale (which would classify private respondent as a
according to specifications, it did not sell, but merely contracted manufacturer) is the mere existence of the product at the time of
for particular pieces of work or “merely sold its services”. the perfection of the contract such that if the thing already exists,
Said article reads as follows: the contract is of sale, if not, it is work. This is not the test
followed in this jurisdiction. Based on Art. 1467, what determines
Article 1467. A contract for the delivery at a certain price of an whether the contract is one of work or of sale is whether the thing
article which the vendor in the ordinary course of his business has been manufactured specially for the customer and “upon his
manufactures or procures for the general market, whether the special order.” Thus, if the thing is specially done at the order of
same is on hand at the time or not, is a contract of sale, but if the another, this is a contract for a piece of work. If, on the other
goods are to be manufactured specially for the customer and hand, the thing is manufactured or procured for the general
upon his special order, and not for the general market, it is market in the ordinary course of one’s business, it is a contract of
contract for a piece of work. sale.

In our opinion when this Factory accepts a job that requires the The distinction between a contract of sale and one for work,
use of extraordinary or additional equipment, or involves services labor and materials is tested by the inquiry whether the thing
not generally performed by it-it thereby contracts for a piece of transferred is one not in existence and which never would have
work — filing special orders within the meaning of Article 1467. existed but for the order of the party desiring to acquire it, or a
The orders herein exhibited were not shown to be special. They thing which would have existed and has been the subject of sale
were merely orders for work — nothing is shown to call them to some other persons even if the order had not been given. The
special requiring extraordinary service of the factory. The thought one who has ready for the sale to the general public finished
occurs to us that if, as alleged-all the work of appellant is only to furniture is a manufacturer, and the mere fact that he did not
fill orders previously made, such orders should not be called have on hand a particular piece or pieces of furniture ordered
special work, but regular work. The Supreme Court affirms the does not make him a contractor only.
assailed decision by the CTA.
A contract for the delivery at a certain price of an article which
the vendor in the ordinary course of his business manufactures
CIR VS ARNOLDUS CARPENTRY SHOP or procures for the – general market, whether the same is on
GR NO. 71122 hand at the time or not, is a contract of sale, but if the goods are
to be manufactured specially for the customer and upon his
FACTS: Arnoldus Carpentry Shop, Inc. is a domestic corporation special order, and not for the general market, it is a contract for a
which has been in existence since 1960 which has for its piece of work. The facts show that the company had a ready
purpose the “preparing, processing, buying, selling, exporting, stock of its shop products for sale to its foreign and local buyers.
importing, manufacturing, trading and dealing in cabinet shop As a matter of fact, the purchase orders from its foreign buyers
products, wood and metal home and office furniture, cabinets, showed that they ordered by referring to the models designated
doors, windows, etc., including their component parts and by petitioner. Even purchases by local buyers for television
materials, of any and all nature and description”. The company cabinets were by orders for existing models except only for some
kept samples or models of its woodwork on display from where adjustments in sizes and accessories utilized.
its customers may refer to when placing their orders.
The Court finds itself in agreement with CTA and as the CTA did
On March 1979, the examiners from BIR who conducted an not err in holding that private respondent is a “manufacturer,”
investigation on the company’s tax liabilities reported that subject then private respondent is entitled to the tax exemption under
corporation should be considered a contractor and not a See. 202 (d) and (e) now Sec. 167 (d) and (e)] of the Tax Code.
manufacturer since the corporation renders service in the course
of an independent occupation representing the will of his
employer only as to the result of his work, and not as to the ENGINEERING & MACHINERY CORPORATION, petitioner,
means by which it is accomplished. Hence, in the computation of vs. COURT OF APPEALS and PONCIANO L.
the percentage tax, the 3% contractor’s tax should be imposed ALMEDA, respondents.
instead of the 7% manufacturer’s tax. However, responded
company holds that the carpentry shop is a manufacturer and FACTS:
therefore entitled to tax exemption on its gross export sales Almeda and Engineering signed a contract, wherein Engineering
under Section 202 (e) of the National Internal Revenue Code. undertook to fabricate, furnish and install the air-conditioning
CIR rendered its decision classifying the respondent as system in the latter’s building along Buendia Avenue, Makati in
contractor which was in turn reversed by the CTA. consideration of P210,000.00. Petitioner was to furnish the
materials, labor, tools and all services required in order to so
ISSUE: Whether or not the Court of Tax Appeals erred in holding fabricate and install said system. The system was completed in
that private respondent is a manufacturer and not a contractor. 1963 and accepted by private respondent, who paid in full the
COMPILED BY: WIGMORE #WIGMOREFOREVER 18
SALES Case Digest (Atty. Sarona)
Compiled by: Wigmore #wigmoreforever

contract price. the complaint was filed on May 8, 1971, it is clear that the action
has not prescribed.
Almeda learned from the employees of NIDC of the defects of
the air-conditioning system of the building. Almeda spent for the
repair of the air-conditioning system. He now sues Engineering INOCENCIA YU DINO and her HUSBAND doing business
for the refund of the repair. Engineering contends that the under the trade name "CANDY CLAIRE FASHION
contract was of sale and the claim is barred by prescription since GARMENTS", petitioners, vs. COURT OF APPEALS and
the responsibility of a vendor for any hidden faults or defects in ROMAN SIO, doing business under the name "UNIVERSAL
the thing sold runs only for 6 months (Arts 1566, 1567, 1571). TOY MASTER MANUFACTURING", respondents.
Almeda contends that since it was a contract for a piece of work,
hence the prescription period was ten years (Hence Art 1144 FACTS:
should apply on written contracts). Petitioners spouses Dino, doing business under the trade name
"Candy Claire Fashion Garment" are engaged in the business of
RTC found that Engineering failed to install certain parts and manufacturing and selling shirts. Respondent Sio is part owner
accessories called for by the contract, and deviated from the and general manager of a manufacturing corporation doing
plans of the system, thus reducing its operational effectiveness business under the trade name "Universal Toy Master
to achieve a fairly desirable room temperature. Manufacturing." Petitioners and respondent Sio entered into a
contract whereby the latter would manufacture for the petitioners
ISSUE: 20,000 pieces of vinyl frogs and 20,000 pieces of vinyl
mooseheads with the sample approved by the petitioners.
Whether the contract for the fabrication and installation of a Respondent Sio delivered the 40,000 pieces of frogs and
central air-conditioning system in a building, one of “sale” or “for mooseheads. Subsequently, petitioners returned to respondent
a piece of work”? 29,772 pieces of frogs and mooseheads for failing to comply with
the approved sample. Petitioners then demanded from the
RULING: respondent a refund of the purchase price of the returned goods
in the amount of P208,404.00. As respondent Sio refused to pay,
CONTRACT FOR PIECE OF WORK petitioners filed on July 24, 1989 an action for collection of a sum
A contract for a piece of work, labor and materials may be of money in the Regional Trial Court of Manila, Branch 38.The
distinguished from a contract of sale by the inquiry as to whether trial court ruled in favor of the petitioners. Respondent Sio sought
the thing transferred is one not in existence and which would recourse in the Court of Appeals. On January 24, 1994, the
never have existed but for the order, of the person desiring it. In respondent court reversed its decision and dismissed petitioners'
such case, the contract is one for a piece of work, not a sale. On Complaint for having been filed beyond the prescriptive period.
the other hand, if the thing subject of the contract would have
existed and been the subject of a sale to some other person ISSUE:
even if the order had not been given, then the contract is one of Whether or not the contract between the petitioner and the
sale. respondent was a contract for a piece of work.

A contract for the delivery at a certain price of an article which RULING:


the vendor in the ordinary course of his business manufactures
or procures for the general market, whether the same is on hand CONTRACT OF PIECE OF WORK
at the time or not is a contract of sale, but if the goods are to be
manufactured specially for the customer and upon his special Art. 1467. A contract for the delivery at a certain price of an
order, and not for the general market, it is a contract for a piece article which the vendor in the ordinary course of his business
of work . manufactures or procures for the general market, whether the
same is on hand at the time or not, is a contract of sale, but if the
The contract in question is one for a piece of work. It is not goods are to be manufactured specially for the customer and
petitioner’s line of business to manufacture air-conditioning upon his special order, and not for the general market, it is a
systems to be sold “off-the-shelf.” Its business and particular field contract for a piece of work.
of expertise is the fabrication and installation of such systems as "Art. 1713. By the contract for a piece of work the contractor
ordered by customers and in accordance with the particular binds himself to execute a piece of work for the employer, in
plans and specifications provided by the customers. Naturally, consideration of a certain price or compensation. The contractor
the price or compensation for the system manufactured and may either employ only his labor or skill, or also furnish the
installed will depend greatly on the particular plans and material."
specifications agreed upon with the customers.
It was stipulated in the contract that respondent would
2)The original complaint is one for damages arising from breach manufacture upon order of the petitioners 20,000 pieces of vinyl
of a written contract – and not a suit to enforce warranties frogs and 20,000 pieces of vinyl mooseheads according to the
against hidden defects – we here – with declare that the samples specified and approved by the petitioners. Respondent
governing law is Article 1715 (supra). However, inasmuch as this Sio did not ordinarily manufacture these products, but only upon
provision does not contain a specific prescriptive period, the order of the petitioners and at the price agreed upon. Clearly, the
general law on prescription, which is Article 1144 of the Civil contract executed by and between the petitioners and the
Code, will apply. Said provision states, inter alia, that actions respondent was a contract for a piece of work. At any rate,
“upon a written contract” prescribe in ten (10) years. Since the whether the agreement between the parties was one of a
governing contract was executed on September 10, 1962 and contract of sale or a piece of work, the provisions on warranty of
COMPILED BY: WIGMORE #WIGMOREFOREVER 19
SALES Case Digest (Atty. Sarona)
Compiled by: Wigmore #wigmoreforever

title against hidden defects in a contract of sale apply to the case sponsorship for unfunded IPC research projects from
at bar. Article 1567 provides for the remedies available to the international organizations, private foundations and
vendee in case of hidden defects: governmental agencies. However, such sponsorships are subject
to private respondent‘s terms and conditions, among which are,
"Art. 1567. In the cases of Articles 1561, 1562, 1564, 1565 and that the research is confined to topics consistent with the private
1566, the vendee may elect between withdrawing from the respondent‘s academic agenda; that no proprietary or
contract and demanding a proportionate reduction of the price, commercial purpose research is done; and that private
with damages in either case." respondent retains not only the absolute right to publish but also
the ownership of the results of the research conducted by the
By returning the 29,772 pieces of vinyl products to respondent IPC.
and asking for a return of their purchase price, petitioners were in
effect "withdrawing from the contract" as provided in Art.
1567. The prescriptive period for this kind of action is provided in 4. Distinguished from Agency to Sell
Art. 1571 of the New Civil Code which provides that the action
shall be barred after six months from the delivery of the thing
sold. Respondent made the last delivery of the vinyl products to ANDRES QUIROGA vs PARSONS HARDWARE CO.,
petitioners on September 28,1988 and the action to recover G.R. L-11491 August 23, 1918
the purchase price of the goods was filed on July 24, 1989, more
than nine months from the date of last delivery. Clearly, FACTS: On January 24, 1911, a contract was entered into by
Petitioners are barred from claiming a sum of money and between Andres Quiroga and J. Parsons, to whose rights
from respondent. and obligations the present defendant later subrogated itself. The
contract stipulated that Don Andres Quiroga grants the exclusive
right to sell his beds in the Visayan Islands to J. Parsons.
CIR VS CA AND ATENEO (GR NO 115349 APRIL 18, 1997)
Quiroga files a case against Parsons for allegedly violating the
FACTS: ADMU Institute of Philippine Culture is engaged in following stipulations: not to sell the beds at higher prices than
social science studies of Philippine society and culture. those of the invoices; to have an open establishment in Iloilo;
Occasionally, it accepts sponsorships for its research activities itself to conduct the agency; to keep the beds on public
from international organizations, private foundations and exhibition, and to pay for the advertisement expenses for the
government agencies. same; and to order the beds by the dozen and in no other
manner. With the exception of the obligation on the part of the
On July 1983, CIR sent a demand letter assessing the sum of defendant to order the beds by the dozen and in no other
P174,043.97 for alleged deficiency contractor‘s tax. Accdg to manner, none of the obligations imputed to the defendant in the
CIR, ADMU falls under the purview of independent contractor two causes of action are expressly set forth in the contract. But
pursuant to Sec 205 of Tax Code and is also subject to 3% the plaintiff alleged that the defendant was his agent for the sale
contractor‘s tax under Sec 205 of the same code. (Independent of his beds in Iloilo, and that said obligations are implied in a
Contractor means any person whose activity consists essentially contract of commercial agency. The whole question, therefore,
of the sale of all kinds of services for a fee regardless of whether reduced itself to a determination as to whether the defendant,
or not the performance of the service calls for the exercise or use byreason of the contract hereinbefore transcribed, was a
of the physical or mental faculties of such contractors or their purchaser or an agent of the plaintiff for the sale of his beds.
employees.)
ISSUE: WON the acceptance of research projects by the IPC of ISSUE: Whether the contract is a contract of agency or of sale.
ADMU a contract of sale or a contract for a piece of work?
NEITHER. Transactions of Ateneo‘s Institute of Philippine Culture RULING:
cannot be deemed either as a contract of sale or a contract of a In order to classify a contract, due regard must be given to its
piece of work. essential clauses. In the contract in question, what was essential,
as constituting its cause and subject matter, is that the plaintiff
HELD: Records do not show that Ateneo‘s IPC in fact contracted was to furnish the defendant with the beds which the latter might
to sell its research services for a fee. In the first place, the order, at the price stipulated, and that the defendant was to pay
petitioner has presented no evidence to prove its bare contention the price in the manner stipulated. The price agreed upon was
that, indeed, contracts for sale of services were ever entered into the one determined by the plaintiff for the sale of these beds in
by the private respondent. Funds received by the Ateneo de Manila, with a discount of from 20 to 25 per cent, according to
Manila University are technically not a fee. They may however their class. Payment was to be made at the end of sixty days, or
fall as gifts or donations which are tax-exempt. Another fact that before, at the plaintiff's request, or in cash, if the defendant so
supports this contention is that for about 30 years, IPC had preferred, and in these last two cases an additional discount was
continuously operated at a loss, which means that sponsored to be allowed for prompt payment. These are precisely the
funds are less than actual expenses for its research projects. essential features of a contract of purchase and sale. There was
the obligation on the part of the plaintiff to supply the beds, and,
In fact, private respondent is mandated by law to undertake on the part of the defendant, to pay their price. These features
research activities to maintain its university status. In fact, the exclude the legal conception of an agency or order to sell
research activities being carried out by the IPC is focused not on whereby the mandatory or agent received the thing to sell it,
business or profit but on social sciences studies of Philippine and does not pay its price, but delivers to the principal the
society and culture. Since it can only finance a limited number of price he obtains from the sale of the thing to a third person,
IPC‘s research projects, private respondent occasionally accepts
COMPILED BY: WIGMORE #WIGMOREFOREVER 20
SALES Case Digest (Atty. Sarona)
Compiled by: Wigmore #wigmoreforever

and if he does not succeed in selling it, he returns it. By that is the exclusive agent of Starr Piano in the Philippines, not
virtue of the contract between the plaintiff and the defendant, the the agent of Arco. It is out of the ordinary for one to be the agent
latter, on receiving the beds, was necessarily obliged to pay their of both the seller and the buyer. The facts and
price within the term fixed, without any other consideration and circumstances show that Arco entered into a contract of sale
regardless as to whether he had or had not sold the beds. with GPS, the exclusive agent of Starr Piano. As such, it is not
duty bound to reveal the private arrangement it had with
It would be enough to hold, as we do, that the contract by and Starr Piano relative to the 25% discount. Thus, GPS is not bound
between the defendant and the plaintiff is one of purchase and to reimburse Arco for any difference between the cost price
sale, in order to show that it was not one made on the basis of a and the sales price, which represents the profit realized by
commission on sales, as the plaintiff claims it was, for these GPS out of the transaction.
contracts are incompatible with each other. But, besides,
examining the clauses of this contract, none of them is found that
substantially supports the plaintiff's contention. Not a single one KER & CO, LTD. LINGAD
of these clauses necessarily conveys the idea of an agency. The
words commission on sales used in clause (A) of article 1 mean FACTS
nothing else, as stated in the contract itself, than a mere discount Ker and Co, Ltd. was assessed by then Commissioner of Internal
on the invoice price. The word agency, also used in articles 2 and Revenue Domingo the sum of P 20,272.33 as the commercial
3, only expresses that the defendant was the only one that could broker’s percentage tax surcharge and compromise penalty.
sell the plaintiff's beds in the Visayan Islands. With regard to the There was a request on the part of Ker for the cancellation of
remaining clauses, the least that can be said is that they are not such assessment which request was turned down. As a result, it
incompatible with the contract of purchase and sale. filed a petition for review with the Court of Tax Appeals. The CTA
held that Ker is taxable except as to the compromise penalty of P
500, the amount due from it being fixed at P 19, 772.33.

PUYAT & SONS, INC. V. ARCO AMUSEMENT CO. Such liability arose from a contract of Ker with the United States
Rubber International (USRI). The former being referred to as the
FACTS distributor and the latter specifically designated as the company.
Arco Amusement was engaged in the business of operating The contract was to apply to transactions between the former
cinematographs. Gonzalo Puyat & Sons Inc. (GPS) was the and Ker, as distributor from July 1, 1948 to continue in force until
exclusive agent in the Philippines for the Starr Piano terminated by either party giving to the other 60 days’ notice. The
Company. Desiring to equip its cinematograph with sound shipments would cover products for consumption in Cebu, Bohol,
reproducing devices, Arco approached GPS, through its Leyte, Samar, Jolo, Negros Oriental and Mindanao except the
president, GIl Puyat, and an employee named Santos. After province of Davao, Ker as distributor being precluded from
some negotiations, it was agreed between the parties that disposing such products elsewhere than in the above places
GPS would order sound reproducing equipment from Starr unless written consent be obtained from the company. Ker as
Piano Company and that Arco would pay GPS, in distributor is required to exert every effort to have the shipment
addition to the price of the equipment, a 10% commission, plus of the products in the maximum quantity and to promote in every
all expenses such as freight, insurance, etc. When GPS inquired way the sale thereof.
Starr Piano the price (without discount) of the equipment, the
latter quoted such at $1,700 FOB Indiana. Being agreeable to (Crucial stipulation: The company shall form time to time consign
the price (plus 10%commission plus all other expenses), to Ker and Ker will receive, accept and/or hold upon consignment
Arco formally authorized the order. The following year, the products specified under the terms of this agreement in such
both parties agreed for another order of sound reproducing quantities as in the judgment of company may be necessary.
equipment on the same terms as the first at $1,600 plus 10%
plus all other expenses. Three years later, Arco discovered It is further agreed that this agreement does not constitute Ker
that the prices quoted to them by GPS with regard to their first 2 the agent or legal representative of the company for any purpose
orders mentioned were not the net prices, but rather the list whatsoever.)
price, and that it had obtained a discount from Starr Piano.
Moreover, Arco alleged that the equipment were overpriced. ISSUE
Thus, being its agent, GPS had to reimburse the excess W/N the contract between Ker and the USRI is a contract of sale.
amount it received from Arco.
HELD
ISSUE No. By taking the contractual stipulations as a whole and not just
W/N there was a contract of agency, not of sale the disclaimer, it would seem that the contract between them is a
contract of agency. That the petitioner Ker & Co., Ltd. is, by
HELD contractual stipulation, an agent of U.S. Rubber International is
NO. The letters containing Arco's acceptance of the prices borne out by the facts that:
for the equipment are clear in their terms and admit no
other interpretation that the prices are fixed and 1. petitioner can dispose of the products of the Company only to
determinate. While the letters state that GPS was to receive a certain persons or entities and within stipulated limits, unless
10% commission, this does not necessarily mean that it is excepted by the contract or by the Rubber Company;
an agent of Arco, as this provision is only an
additional price which it bound itself to pay, and which 2. it merely receives, accepts and/or holds upon consignment the
stipulation is not incompatible with the contract of sale. It is GPS products, which remain properties of the latter company;
COMPILED BY: WIGMORE #WIGMOREFOREVER 21
SALES Case Digest (Atty. Sarona)
Compiled by: Wigmore #wigmoreforever

mere indentor, it avers that is not liable for the seller’s implied
3. every effort shall be made by petitioner to promote in every warranty against hidden defects, Schmid not having personally
way the sale of the products (Par. 3); that sales made by assumed any such warranty.
petitioner are subject to approval by the company;
ISSUE:
4. on dates determined by the rubber company, petitioner shall 1) WON the second transaction between the parties was a sale
render a detailed report showing sales during the month; or an indent transaction? INDENT TRANSACTION
2) Even is Schmid is merely an indentor, may it still be liable for
5. the rubber company shall invoice the sales as of the dates of the warranty? YES, under its contractual obligations it may be
inventory and sales report (Par. 14); that the rubber company liable. But in this case, Schmid did not warrant the products.
agrees to keep the consigned goods fully insured under
insurance policies payable to it in case of loss; HELD:
An indentor is a middlemen in the same class as commercial
6. upon request of the rubber company at any time, petitioner brokers and commission merchants. A broker is generally
shall render an inventory of the existing stock which may be defined as one who is engaged, for others, on a commission,
checked by an authorized representative of the former negotiating contracts relative to property with the custody of
which he has no concern; the negotiator between other parties,
7. upon termination or cancellation of the Agreement, all goods never acting in his own name but in the name of those who
held on consignment shall be held by petitioner for the account of employed him; he is strictly a middleman and for some purpose
the rubber company until their disposition is provided for by the the agent of both parties. There are 3 parties to an indent
latter transaction, (1) buyer, (2) indentor, and (3) supplier who is
usually a non-resident manufacturer residing in the country
The National Internal Revenue Code defined “Commercial where the goods are to be bought. The chief feature of a
broker” as “all persons, other than importer, manufacturers, commercial broker and a commercial merchant is that in effecting
producers or bona fide employees who, for compensation or a sale, they are merely intermediaries or middle-men, and act in
profit, sell or bring about sales or purchase of merchandise for a certain sense as the agent of both parties to the transaction.
other persons or being proposed buyers and sellers together”
and also includes commission merchants such as Ker in this RJL MARTINEZ admitted that the generators were purchased
case. “through indent order.” RJL admitted in its demand letter
previously sent to SCHMID that 12 of 15 generators “were
The mere disclaimer in a contract that an entity like Ker is not purchased through your company, by indent order and three (3)
“the agent or legal representative for any purpose whatsoever” by direct purchase.” The evidence also show that RJL
does not suffice to yield the conclusion that it is an independent MARTINEZ paid directly NAGATA CO, for the generators, and
merchant if the control over the goods for resale of goods that the latter company itself invoiced the sale and shipped the
consigned is pervasive in character. generators directly to the former. The only participation of
Schmid was to act as an intermediary or middleman between
Nagata and RJL, by procuring an order from RJL and forwarding
SCHMID & OBERLY, INC. vs. RJL MARTINEZ the same to Nagata for which the company received a
G.R. No. 75198 October 18, 1988 commission from Nagata.

FACTS: Sale vs. Indent Transaction:


RJL Martinez Fishing Corporation is engaged in deep-sea The essence of the contract of sale is transfer of title or
fishing. In the course of its business, it needed electrical agreement to transfer it for a price paid or promised. If such
generators for the operation of its business. Schmid and Oberly transfer puts the transferee in the attitude or position of an owner
sells electrical generators with the brand of “Nagata”, a Japanese and makes him liable to the transferor as a debtor for the agreed
product. D. Nagata Co. Ltd. of Japan was Schmid’s supplier. price, and not merely as an agent who must account for the
Schmid advertised the 12 Nagata generators for sale and RJL proceeds of a resale, the transaction is, a sale.
purchased 12 brand new generators. Through an irrevocable line 3 evidences pointing to fact that Schmid is merely an indentor:
of credit, Nagata shipped to the Schmid the generators and RJL a. the Quotation and the General Conditions of Sale on the
paid the amount of the purchase price. (First sale = 3 generators; dorsal side thereof do not necessarily lead to the conclusion that
Second sale = 12 generators). NAGATA CO., was the real seller of the 12 generators.
b. When RJL complained to SCHMID, it immediately asked RJL
Later, the generators were found to be factory defective. RJL to send the defective generators to its shop to determine what
informed the Schmid that it shall return the 12 generators. 3 were was wrong. SCHMID informed NAGATA about the complaint of
returned. Schmid replaced the 3 generators subject of the first RJL. After the generators were found to have factory defects,
sale with generators of a different brand. As to the second sale, 3 SCHMID facilitated the shipment of three (3) generators to Japan
were shipped to Japan and the remaining 9 were not replaced. and, after their repair, back to the Philippines.
c. the letter from NAGATA CO. to SCHMID regarding the repair
RJL sued the defendant on the warranty, asking for rescission of of the generators indicated that the latter was “within the purview
the contract and that Schmid be ordered to accept the of a seller.”
generators and be ordered to pay back the purchase money as
well as be liable for damages. Schmid opposes such liability 2) Even as SCHMID was merely an indentor, there was nothing
averring that it was merely the indentor in the sale between to prevent it from voluntarily warranting that twelve (12)
Nagata Co., the exporter and RJL Martinez, the importer. As generators subject of the second transaction are free from any
COMPILED BY: WIGMORE #WIGMOREFOREVER 22
SALES Case Digest (Atty. Sarona)
Compiled by: Wigmore #wigmoreforever

hidden defects. In other words, SCHMID may be held appointment and act on it, and in the absence of such intent,
answerable for some other contractual obligation, if indeed it had there is generally no agency. One factor which most clearly
so bound itself. As stated above, an indentor is to some extent distinguishes agency from other legal concepts is control; one
an agent of both the vendor and the vendee. As such agent, person - the agent - agrees to act under the control or direction
therefore, he may expressly obligate himself to undertake the of another - the principal. Indeed, the very word "agency" has
obligations of his principal. come to connote control by the principal. The control factor, more
than any other, has caused the courts to put contracts between
Notably, nowhere in the Quotation is it stated therein that principal and agent in a separate category.
SCHMID did bind itself to answer for the defects of the things
sold. Balagtas testified initially that the warranty was in the
receipts covering the sale. Nowhere is it stated in the invoice that 5. Distinguished from Dacion En Pago
SCHMID warranted the generators against defects. He again
changed his mind and asserted that the warranty was given
verbally. Hence, RJL has failed to prove that SCHMID had given PNB vs. PINEDA
a warranty on the 12 generators subject of the second G.R. No. L-46658, May 13, 1991
transaction.
FACTS: In 1963, Ignacio and Lourdes Arroyo (spouses Arroyo)
obtained a loan of P580,000.00 from PNB to purchase 60% of
VICTORIAS MILLING CO. vs CA and CONSOLIDATED the subscribed capital stock, and thereby acquire the controlling
SUGAR CO. interest of private respondent Tayabas Cement Company, Inc.
[G.R. No. 117356. June 19, 2000] (TCC). Spouses Arroyo executed a real estate mortgage over a
parcel of land (La Vista property) as security for the said loan.
FACTS: St. Therese Merchandising (STM), who regularly bought
sugar from Victorias Milling Co. (VMC), was issued Shipping Thereafter, TCC filed with PNB an application and agreement for
List/Delivery Receipts (SLDRs) by the latter as proof of the establishment of an 8 year deferred letter of credit (L/C) for
purchases for bags of sugar. Thereafter, STM sold to $7,000,000.00 in favor of Toyo Menka Kaisha, Ltd. of Tokyo,
Consolidated Sugar Co. (CSC) its rights in one of the SLDRs. Japan, to cover the importation of a cement plant machinery and
CSC communicated to VMC that it had been authorized by STM equipment.
to withdraw the sugar covered by SLDR. Enclosed in the letter
were a copy of SLDR and a letter of authority from STM Upon approval of said application and opening of an L/C by PNB
authorizing CSC "to withdraw for and in our behalf the refined in favor of Toyo Menka Kaisha, Ltd. for the account of TCC, the
sugar covered by SLDR”. CSC surrendered the SLDR to VMC’s Arroyo spouses executed documents (Surety Agreement and
warehouse and was allowed to withdraw sugar but after several Covenant) to secure the loan accommodation.
bags were released, it was later on refused to allow further
withdrawals of sugar. CSC communicated to VMC to allow it to The imported cement plant machinery and equipment arrived
withdraw sugar because the SLDR had been “sold and from Japan and were released to TCC under a trust receipt
endorsed” to it by STM. VMC contended that it could not allow agreement. Toyo Menka Kaisha made the corresponding
any further withdrawals of sugar against SLDR because STM drawings against the L/C as scheduled. TCC, however, failed to
had already withdrawn sugar covered by cleared checks. CSC remit and/or pay the amount covered by the drawings. Thus,
filed complaint against VMC. VMC contended that it had no PNB notified TCC of its intention to repossess the imported
privity of contract with CSC, the dealings between it and STM machinery and equipment for failure of TCC to settle its
were part of a series of transactions involving only one account obligations under the L/C.
or one general contract of sale because CSC was an agent of
STM. CSC countered that the sugar purchases involving SLDR In the meantime, the personal accounts of the spouses Arroyo,
were separate and independent transactions. which included another loan of P160,000.00 secured by a real
estate mortgage over Hacienda Bacon located in Isabela,
ISSUE: Whether or not CSC was an agent of STM. Negros Occidental, had likewise become due. The spouses
HELD: No. CSC was a buyer of the SLDR form, and not an Arroyo failed to satisfy their obligations with PNB and the latter
agent of STM. decided to foreclose the real estate mortgages.

CSC was not subject to STM's control. The question of whether a At the auction sale of the La Vista Property, PNB was the highest
contract is one of sale or agency depends on the intention of the bidder with a bid price of P1,000,001.00. However, when said
parties as gathered from the whole scope and effect of the property was about to be awarded to PNB, the representative of
language employed. That the authorization given to CSC the mortgagor-spouses objected and demanded from the PNB
contained the phrase "for and in our (STM's) behalf" did not the difference between the bid price of P1,000,001.00 and the
establish an agency. CSC communicated to VMC that the SLDR indebtedness of P499,060.25 of the Arroyo spouses on their
had been “sold and endorsed” to it by STM. The use of the words personal account. It was the contention of the spouses Arroyo's
"sold and endorsed" means that STM and CSC intended a representative that the foreclosure proceedings referred only to
contract of sale, and not an agency. the personal account of the mortgagor spouses without reference
to the account of TCC.
The basis of agency is representation. On the part of the
principal, there must be an actual intention to appointor an To remedy the situation, PNB filed a supplemental petition
intention naturally inferable from his words or actions; and on the requesting the Sheriff's Office to proceed with the sale of the
part of the agent, there must be an intention to accept the
COMPILED BY: WIGMORE #WIGMOREFOREVER 23
SALES Case Digest (Atty. Sarona)
Compiled by: Wigmore #wigmoreforever

subject real properties to satisfy not only the amount of LO VS. KJS
P499,060.25 owed by the spouses Arroyos on their personal G.R. No. 149420, October 8, 2003
account but also the amount of P35,019,901.49 owed by said
spouses as sureties of TCC. Said petition was opposed by the FACTS: Respondent KJS ECO-FORMWORK System Phil., Inc.
spouses Arroyo and the other bidder Araneta. is a corporation engaged in the sale of steel scaffoldings, while
petitioner Sonny L. Lo, doing business under Sans Enterprises,
PNB filed a petition for mandamus to compel the sheriff to is a building contractor. Lo ordered scaffolding equipment from
proceed with the foreclosure sale of the mortgaged properties KJS worth P540,425.80. He paid a down payment in the amount
and the petition was granted. However, TCC filed a complaint of P150,000.00. The balance was made payable in ten monthly
against PNB to restrain the foreclosure of the mortgages over the instalments.
said properties as well as a declaration that its obligation with
PNB had been fully paid by reason of the latter's repossession of KJS delivered the scaffoldings to Lo. Lo was able to pay the first
the imported machinery and equipment. two monthly instalments. His business, however, encountered
financial difficulties and he was unable to settle his obligation to
ISSUE: Whether or not TCC's liability has been extinguished by KJS despite oral and written demands made against him.
the repossession of PNB of the imported cement plant machinery
and equipment and whether or not the repossession amounts to Lo and KJS executed a Deed of Assignment whereby Lo
dacion en pago. assigned to KJS his receivables in the amount of P335,462.14
from Jomero Realty Corporation.
HELD: No. It must be remembered that PNB took possession of
the imported cement plant machinery and equipment pursuant to However, when KJS tried to collect the said credit from Jomero
the trust receipt agreement executed by PNB and TCC giving the Realty Corporation, the latter refused to honor the Deed of
former the unqualified right to the possession and disposal of all Assignment because it claimed that Lo was also indebted to it.
property shipped under the Letter of Credit until such time as all Subsequently, KJS sent a letter to Lo demanding payment of his
the liabilities and obligations under said letter had been obligation, but he refused to pay claiming that his obligation had
discharged. been extinguished when they executed the Deed of Assignment.

PNB's possession of the subject machinery and equipment being Consequently, KJS filed an action for recovery of a sum of
precisely as a form of security for the advances given to TCC money against Lo before the RTC. Lo argued that his obligation
under the Letter of Credit, said possession by itself cannot be was extinguished with the execution of the Deed of Assignment
considered payment of the loan secured thereby. Payment would of credit. KJS, for its part, presented the testimony of its
legally result only after PNB had foreclosed on said securities, employee, Almeda Baaga, who testified that Jomero Realty
sold the same and applied the proceeds thereof to TCC's loan refused to honor the assignment of credit because it claimed that
obligation. Mere possession does not amount to foreclosure for Lo had an outstanding indebtedness to it.
foreclosure denotes the procedure adopted by the mortgagee to
terminate the rights of the mortgagor on the property and RTC dismissed the complaint on the ground that the assignment
includes the sale itself. of credit extinguished the obligation. CA reversed the decision. In
finding that the Deed of Assignment did not extinguish the
Neither can said repossession amount to dacion en pago. obligation of the petitioner to the respondent, the CA held that (1)
Dation in payment takes place when property is alienated to petitioner failed to comply with his warranty under the Deed; (2)
the creditor in satisfaction of a debt in money and the same the object of the Deed did not exist at the time of the transaction,
is governed by sales. Dation in payment is the delivery and rendering it void pursuant to Article 1409 of the Civil Code; and
transmission of ownership of a thing by the debtor to the (3) petitioner violated the terms of the Deed of Assignment when
creditor as an accepted equivalent of the performance of the he failed to execute and do all acts and deeds as shall be
obligation. As aforesaid, the repossession of the machinery necessary to effectually enable the respondent to recover the
and equipment in question was merely to secure the collectibles.
payment of TCC's loan obligation and not for the purpose of
transferring ownership thereof to PNB in satisfaction of said ISSUE: Whether or not the deed of assignment extinguished
loan. Thus, no dacion en pago was ever accomplished. Lo’s obligations.

Proceeding from this finding, PNB has the right to foreclose the HELD: No. An assignment of credit is an agreement by virtue of
mortgages executed by the spouses Arroyo as sureties of TCC. which the owner of a credit, known as the assignor, by a legal
A surety is considered in law as being the same party as the cause, such as sale, dacion en pago, exchange or donation, and
debtor in relation to whatever is adjudged touching the obligation without the consent of the debtor, transfers his credit and
of the latter, and their liabilities are interwoven as to be accessory rights to another, known as the assignee, who
inseparable. As sureties, the Arroyo spouses are primarily liable acquires the power to enforce it to the same extent as the
as original promissors and are bound immediately to pay the assignor could enforce it against the debtor.
creditor the amount outstanding.
Corollary thereto, in dacion en pago, as a special mode of
payment, the debtor offers another thing to the creditor who
accepts it as equivalent of payment of an outstanding debt. In
order that there be a valid dation in payment, the following are
the requisites:

COMPILED BY: WIGMORE #WIGMOREFOREVER 24


SALES Case Digest (Atty. Sarona)
Compiled by: Wigmore #wigmoreforever

1.) There must be the performance of the prestation in lieu of favor of Agrifina amounting to P546,459 and that their debtors
payment (animo solvendi) which may consist in the delivery had executed promissory notes in favor of Agrifina. Spouses
of a corporeal thing or a real right or a credit against the third insisted that by virtue of these documents, Agrifina became the
person; new collector of their debts. Agrifina was able to collect the total
2.) There must be some difference between the prestation due amount of P301,000 from Felicdad’s debtors. She tried to collect
and that which is given in substitution (aliud pro alio) the balance of Felicidad and when the latter reneged on her
3.) There must be an agreement between the creditor and promise, Agrifina filed a complaint in the office of the barangay
debtor that the obligation is immediately extinguished by for the collection of P773,000.00. There was no settlement. RTC
reason of the performance of a prestation different from that favored Agrifina. Court of Appeals affirmed the decision with
due. modification ordering defendant to pay the balance of total
indebtedness in the amount of P51,341,00 plus 6% per month.
The undertaking really partakes in one sense of the nature of
sale, that is, the creditor is really buying the thing or property of ISSUE: Whether or not the deeds of assignment in favor of
the debtor, payment for which is to be charged against the petitioner has the effect of payment of the original obligation that
debtor’s debt. As such, the vendor in good faith shall be would partially extinguish the same
responsible, for the existence and legality of the credit at the time
of the sale but not for the solvency of the debtor, in specified RULING: YES. Substitution of the person of the debtor may be
circumstances. affected by delegacion. Meaning, the debtor offers, the creditor
accepts a third person who consent of the substitution and
Hence, it may well be that the assignment of credit, which is in assumes the obligation. It is necessary that the old debtor be
the nature of a sale of personal property, produced the effects of released fro the obligation and the third person or new debtor
a dation in payment which may extinguish the obligation. takes his place in the relation . Without such release, there is no
However, as in any other contract of sale, the vendor or assignor novation. Court of Appeals correctly found that the
is bound by certain warranties. More specifically, the first respondent’s obligation to pay the balance of their account
paragraph of Article 1628 of the Civil Code provides: with petitioner was extinguished pro tanto by the deeds of
credit. CA decision is affirmed with the modification that the
The vendor in good faith shall be responsible for the existence principal amount of the respondents is P33,841.
and legality of the credit at the time of the sale, unless it should
have been sold as doubtful; but not for the solvency of the In its modern concept, what actually takes place in dacion en
debtor, unless it has been so expressly stipulated or unless the pago is an objective novation of the obligation where the thing
insolvency was prior to the sale and of common knowledge. offered as an accepted equivalent of the performance of an
obligation is considered as the object of the contract of sale,
From the above provision, petitioner, as vendor or assignor, is while the debt is considered as the purchase price.
bound to warrant the existence and legality of the credit at the
time of the sale or assignment. When Jomero Realty claimed
that it was no longer indebted to Lo since the latter also had an SSS V CA, 553 SCRA 677 (2008)
unpaid obligation to it, it essentially meant that its obligation to Lo
has been extinguished by compensation. In other words, KJS FACTS: AG&P and Semirara Coal Company proposed to pay its
alleged the non-existence of the credit and asserted its claim to arrears of premiums and loan amortization delinquencies through
Lo’s warranty under the assignment. Therefore, it behooved on dacion en pago which was subsequently accepted by SSS.
Lo to make good its warranty and paid the obligation. Thereafter, SSS directed herein defendant to submit certain
documents necessary for the agreement which AG&P
Furthermore, we find that Lo breached his obligation under the immediately complied with. SSS finally approved the dacion en
Deed of Assignment. Indeed, by warranting the existence of the pago which as of March 2001 amounted to P29, 261,902.45. To
credit, petitioner should be deemed to have ensured the effect said transfer, a Deed of Assignment had to be executed
performance thereof in case the same is later found to be between the two parties which SSS failed to come up. On the
inexistent. He should be held liable to pay to respondent the other hand, defendant continuously submitted drafts to SSS of
amount of his indebtedness. the needed Deed of Assignment. ON 2003, SSS sent to AG&P a
revised copy of the Deed of Assignment, however, the amount
Hence, we affirm the decision of the Court of Appeals ordering went from P29, 261,902.45 to P40, 846,610.64 allegedly
petitioner to pay respondent the sum of P335,462.14 with legal because of the additional interest and penalties. AG&P
interest thereon. requested for the deduction of these interests and penalties for
the delay of the Deed of Assignment was the fault of SSS.

AQUINTEY V. SPOUSES TIBONG Thus, AG&P filed a complaint for the specific performance and
G.R. NO. 166704,DECEMBER 20, 2006 damages against SSS. SSS contended that the court has no
jurisdiction over the case in accordance with R.A. 8282 which
FACTS: On May 6, 1999, petitioner Aquintey filed before RTC provides that any dispute should be filed in the Commission.
Baguio, a complaint for sum of money and damages against RTC ruled in favor of AG&P. upon appeal, the CA held that the
respondents. Agrifina alleged that Felicidad secured loans from court has jurisdiction and that the case be reverted back to the
her on several occasions at monthly interest rates of 6% to 7%. Trial Court for actual proceedings. Thus, SSS appealed to the
Despite demands, spouses Tibong failed to pay their outstanding Court.
loans of P773,000,00 exclusive of interests. However, spouses
Tiong alleged that they had executed deeds of assignment in
COMPILED BY: WIGMORE #WIGMOREFOREVER 25
SALES Case Digest (Atty. Sarona)
Compiled by: Wigmore #wigmoreforever

ISSUE: would depend on the amount of the claim. However, where the
A. WON dacion en pago should be implemented basic issue is something other than the right to recover a sum of
B. Which body has jurisdiction over non-implementation of a money, where the money claim is purely incidental to, or a
dacion en pago agreed by the parties? consequence of, the principal relief sought, this Court has
considered such actions as cases where the subject of the
HELD: litigation may not be estimated in terms of money, and are
1. Yes. The Supreme Court absolutely adopted the CA decision. cognizable exclusively by courts of first instance (now Regional
Dacion en pago is the delivery and transmission of ownership of Trial Courts).
a thing by the debtor to the creditor as an accepted equivalent of
the performance of the obligation. It is a special mode of
payment where the debtor offers another thing to the creditor MAR YUSON vs ATTY. JEREMIAS R. VITAN
who accepts it as equivalent of payment of an outstanding debt. A.C. No. 6955, July 27, 2006
The undertaking really partakes in one sense of the nature of
sale, that is the creditor is really buying the thing or property of FACTS: In October 2002, Mar Yuson who was a taxi driver and
the debtor, payment for which is to be charged against the had 8 children, received a sum of money by way of inheritance.
debtor’s debt. As such, the essential elements of a contract of He and his wife intended to use the money for several purposes.
sale, namely, consent, object certain, and cause or consideration
must be present. In its modern concept, what actually takes When they were able to purchase a secondhand taxi, and Atty.
place in dacion en pago is an objective novation of the obligation Vitan helped him with legal matters regarding the purchase.
where the thing offered as an accepted equivalent of the Unfortunately, Yuson’s other plans were put on hold when Atty.
performance of an obligation is considered as the object of the Vitan borrowed P100, 000 from them in December 2002. To
contract of sale, while the debt is considered as the purchase guarantee payment, Atty. Vitan executed in favor of Yuson
price. In any case, common consent is an essential prerequisite, several postdated checks to over the loaned amount, but
be it sale or novation, to have the effect of totally extinguishing however, these turned out to be worthless.
the debt or obligation.
Yuson maintained that he had repeatedly tried to recover the
From the averments in their complaint, the appellate court debt, but was unsuccessful every time. When no payment was
observed that private respondents are seeking to implement the still made pursuant to the administrative case against Atty. Vitan,
Deed of Assignment which they had drafted and submitted to Yuson demanded a collateral to secure the loan. Thus, in his
SSS pursuant to the approval by SSS. The appellate court thus favor, Atty. Vitan executed a document denominated as a Deed
held that the subject of the complaint is no longer the payment of of Absolute Sale, covering Atty. Vitan’s parcel of land located in
the premium and loan amortization delinquencies, as well as the Sta. Maria, Bulacan. According to Yuson, their intention was to
penalties appurtenant thereto, but the enforcement of the dacion transfer the title of the property to him temporarily, so that he
en pago. Thus, the trial court was ordered to settle the could either sell or mortgage the said land. Further, if it was
controversy. mortgaged, Atty. Vitan would redeem it as partial or full payment
of the loan. Allegedly, the parties executed another Deed of
From the allegations of respondents’ complaint, it readily appears Absolute Sale in favor of Atty. Vitan wherein Yuson was vendor.
that there is no longer any dispute with respect to respondents’ The purpose for this was not explained by either party.
accountability to the SSS. Respondents had, in fact admitted
their delinquency and offered to settle them by way of dacion en Yuson was able to mortgage the property for P30,000 but
pago subsequently approved by the SSS in Resolution No. 270- contrary to their earlier agreement, Atty. Vita did not redeem it
s. 2001. SSS stated in said resolution that “the dacion en pago from the mortgage, sent a letter instead, promising Yuson to pay
proposal of AG&P Co. of Manila and Semirara Coals Corporation on or before July 12, 2004.
to pay their liabilities in the total amount ofP30,652,710.71 as of
31 March 2001 by offering their 5.8 ha. property located in San In the IBP-NCLA, Atty. Vitan averred that he had settled his
Pascual, Batangas, be, as it is hereby, approved..” This obligation through a Deed of Absolute Sale over his residential
statement unequivocally evinces its consent to the dacion en property. The purpose of such was for Yuson to use, mortgage,
pago. or sell the property and return to him the excess of the proceeds
after obtaining his money. Additionally, he called the second
2. The controversy, instead, lies in the non-implementation of the document as a Counter Deed of Sale, executed to be sort of a
approved and agreed dacion en pago on the part of the SSS. As collateral/security for the account of his liaison officer Estur,
such, respondents filed a suit to obtain its enforcement which is, whom he alleged that she was the one who incurred said debts.
doubtless, a suit for specific performance and one incapable of ISSUE: W/N Atty. Vitan’s obligation was extinguished by
pecuniary estimation beyond the competence of the virtue of the first Deed of Absolute Sale
Commission. Pertinently, the Court ruled in Singson v. Isabela
Sawmill, as follows: HELD: NO. Atty. Vitan contends that his obligation was already
extinguished, because he had allegedly sold his Bulacan
In determining whether an action is one the subject matter of property to complainant. Basically, he is asserting that what had
which is not capable of pecuniary estimation this Court has transpired was a dation in payment. Governed by the law on
adopted the criterion of first ascertaining the nature of the sales, it is a transaction that takes place when a piece of
principal action or remedy sought. If it is primarily for the property is alienated to the creditor in satisfaction of a debt in
recovery of a sum of money, the claim is considered capable of money. It involves delivery and transmission of ownership of a
pecuniary estimation, and whether thing -- by the debtor to the creditor -- as an accepted equivalent
jurisdiction in the municipal courts or in the courts of first instance of the performance of the obligation.
COMPILED BY: WIGMORE #WIGMOREFOREVER 26
SALES Case Digest (Atty. Sarona)
Compiled by: Wigmore #wigmoreforever

nonetheless justified in refusing payment since Filinvest is not


However, the records reveal that he did not really intend to sell entitled to recover the same due to the breach of warranty
and relinquish ownership over his property in Sta. Maria, committed by the original vendor-assignor Alexander Lim.
Bulacan, notwithstanding the execution of a Deed of Absolute Additionally, it argues that by virtue of the return, it extinguished
Sale in favor of Yuson. The second Deed of Absolute Sale, which their obligation through dation in payment.
reconveyed the property to respondent, is proof that he had no
such intention. This second Deed, which he referred to as his ISSUE: W/N the return of the mortgaged motor vehicle to
"safety net," betrays his intention to counteract the effects of the Filinvest by virtue of its voluntary surrender by Philippine
first one. Acetylene totally extinguished and/or cancelled its
obligation to Filinvest
Ergo, Atty. Vitan was taking back with his right hand what he had
given with his left. The second Deed of Absolute Sale returned HELD: NO. The mere return of the mortgaged motor vehicle by
the parties right back where they started, as if there were no sale the mortgagor, PhilAcetylene, to the mortgagee, Filinvest, does
in favor of complainant to begin with. In effect, on the basis of the not constitute dation in payment or dacion en pago in the
second Deed of Sale, respondent took back and asserted his absence, express or implied of the true intention of the parties.
ownership over the property despite having allegedly sold it. Dacion en pago, according to Manresa, is the transmission of the
Thus, he fails to convince us that there was a bona fide dation in ownership of a thing by the debtor to the creditor as an accepted
payment or sale that took place between the parties; that is, that equivalent of the performance of obligation.
there was an extinguishment of obligation.
The evidence on the record fails to show that the mortgagee,
It appears that the true intention of the parties was to use the consented, or at least intended, that the mere delivery to, and
Bulacan property to facilitate payment. They only made it appear acceptance by him, of the mortgaged motor vehicle be construed
that the title had been transferred to complainant to authorize as actual payment, more specifically dation in payment or dacion
him to sell or mortgage the property.Atty. Vitan himself admitted en pago. The fact that the mortgaged motor vehicle was
in his letter dated July 30, 2004, that their intention was to delivered to him does not necessarily mean that ownership
convert the property into cash, so that payment could be thereof, as juridically contemplated by dacion en pago, was
obtained by complainant and the excess returned to respondent. transferred from PhilAcetylene to Filinvest. In the absence of
The records, however, do not show that the proceeds derived clear consent of Filinvest, there can be no transfer of ownership
were sufficient to discharge the obligation of the lawyer fully; of the mortgaged motor vehicle from appellant to appellee. Only
thus, he is still liable to the extent of the deficiency. transfer of possession of the mortgaged motor vehicle took
place, for it is quite possible that Filinvest merely wanted to
secure possession to forestall the loss, destruction, fraudulent
FILINVEST CREDIT CORPORATION VS. PHILIPPINE transfer of the vehicle to third persons, or its being rendered
ACETYLENE, CO., INC. valueless if left in the hands of PhilAcetylene.
G.R. No. L-50449, January 30, 1982
The true intention of the parties is furnished by the document
FACTS: On October 30, 1971, the Philippine Acetylene Co., Inc., executed by appellant captioned "Voluntary Surrender with
purchased from Alexander Lim, as evidenced by a Deed of Sale, Special Power of Attorney To Sell". The document reveals that
a Chevrolet 1969 model motor vehicle payable under the terms the possession of the mortgaged motor vehicle was voluntarily
and conditions of the promissory note provided by PhilAcetylene. surrendered by PhilAcetylene to Filinvest authorizing the latter to
As security for the payment, PhilAcetylene executed a chattel look for a buyer and sell the vehicle in behalf of PhilAcetylene
mortgage over the same vehicle in favor of Lim. Subsequently, who retains ownership thereof, and to apply the proceeds of the
Lim assigned to Filinvest Finance all his rights, title, and interests sale to the mortgage indebtedness, with the undertaking of the
in the promissory note and the chattel mortgage which appellant to pay the difference, if any, between the selling price
subsequently assigned it to Filinvest Credit. and the mortgage obligation. With the stipulated conditions as
stated, Filinvest in essence was constituted as a mere agent to
PhilAcetylene failed to comply with the terms in the promissory sell the motor vehicle which was delivered to it, not as its
note and chattel mortgage. With the choice of paying the full property, for if it were, he would have full power of disposition of
amount plus interest and charges or returning the mortgaged the property, not only to sell it as is the limited authority given him
property, PhilAcetylene informed Filinvest Credit that it was in the special power of attorney. Had Filinvest intended to
returning the mortgaged property in full satisfaction of its completely release PhilAcetylene of its mortgage obligation,
indebtedness pursuant to Art. 1484 of the New Civil Code. When there would be no necessity of executing the document.
it was returned to Filinvest, it had with it a document Nowhere in the said document where it states, that the mere
denominated as “Voluntary Surrender with Special Power of surrender of the mortgaged motor vehicle to the appellee
Attorney to Sell.” extinguished appellant’s obligation for the unpaid price.

Filinvest could not however sell the vehicle since there were On PhilAcetylene’s argument that by accepting the delivery of
unpaid taxes on said vehicle. Upon Filinvest’s offer to return the the mortgaged motor vehicle, Filinvest is estopped from
vehicle to PhilAcetylene, the latter refused to accept it. demanding payment of the unpaid obligation, the same is without
merit. As clearly set forth above, Filinvest never accepted the
PhilAectylene contends that Filinvest has no cause of action mortgaged motor vehicle in full satisfaction of the mortgaged
since its obligation was extinguished when it returned the debt.
mortgaged property to Filinvest and assuming however that the
return of the property did not extinguish its obligation, it was Under the law, the delivery of possession of the mortgaged
COMPILED BY: WIGMORE #WIGMOREFOREVER 27
SALES Case Digest (Atty. Sarona)
Compiled by: Wigmore #wigmoreforever

property to the mortgagee, can only operate to extinguish property of the private respondents. This form of agreement has
PhilAcetylene liability if Filinvest had actually caused the been criticized as a lease only in name.
foreclosure sale of the mortgaged property when it recovered
possession thereof. Sellers desirous of making conditional sales of their goods, but
who do not wish openly to make a bargain in that form, for one
reason or another, have frequently resorted to the device of
6. Distinguished from Payment by Cession making contracts in the form of leases either with options to the
buyer to purchase for a small consideration at the end of term,
provided the so-called rent has been duly paid, or with
7. Distinguished from Lease stipulations that if the rent throughout the term is paid, title shall
thereupon vest in the lessee. It is obvious that such transactions
are leases only in name. The so-called rent must necessarily be
FILINVEST CREDIT COROPORATION VS CA, JOSE SY regarded as payment of the price in installments since the due
BANG and ILUMINADA TAN SY BANG payment of the agreed amount results, by the terms of bargain,
G.R. No. 82508, September 29, 1989 in the transfer of title to the lessee.
FACTS: Spouses Jose and Iluminada Sy Bang were engaged in 2.) NO.
the sale of gravel produced from crushed rocks and used for
construction purposes. They engaged the serviced of Mr. Ruben They are alternative. The seller of movable in installments, in
Mercurio of Gemini Motor Sales, to look for a rock crusher. Mr. case the buyer fails to pay 2 or more installments, may elect to
Mercurio then referred them to Rizal Consolidated who had said pursue either of the following remedies: (1) exact fulfillment by
machinery for sale. the purchaser of the obligation; (2) cancel the sale; or (3)
foreclose the mortgage on the purchased property if one was
They applied for financial assistance with Filinvest Credit constituted thereon. It is now settled that the said remedies are
regarding their purchase of the machine. Fiinvest agreed to alternative and not cumulative, and therefore, the exercise of one
extend to the Spouses Sy Bang financial aid on the following bars the exercise of the others. Indubitably, the device – contract
conditions: that the machinery be purchased in the Filinvest's of lease with option to buy – is at times resorted to as a means to
name; that it be leased (with option to purchase upon the circumvent Article 1484, particularly paragraph (3) thereof.
termination of the lease period) to the Spouses Sy Bang; and Through the set-up, the vendor, by retaining ownership over the
that Spouses Sy Bang execute a real estate mortgage as property in the guise of being the lessor, retains, likewise the
security for the amount advanced by Filinvest. Accordingly, on right to repossess the same, without going through the process
May 18,1981, a contract of lease of machinery (with option to of foreclosure, in the event the vendee-lessee defaults in the
purchase) was entered into by the parties whereby the spouses payment of the installments. There arises therefore no need to
agreed to lease from Filinvest the rock crusher for two years constitute a chattel mortgage over the movable sold. More
starting from July 5, 1981 payable as follows: P10,000.00 – first important, the vendor, after repossessing the property and, in
3 months, P23,000.00 – next 6 months, P24,800.00 – next 15 effect, canceling the contract of sale, gets to keep all the
months. It was likewise stipulated that at the end of the two-year installments-cum-rentals already paid.
period, the machine would be owned by the spouses.

The spouses then issued a check for P150,550 as initial rental, 8. Distinguished from Contract to Sell
and 24 postdated checks corresponding to 24 monthly rentals in
favor of Filinvest. They likewise executed a real estate mortgage
over two parcels of land to guarantee their compliance with the JUANA ALMIRA ET AL vs CA and FEDERICO BRIONES
lease contract. The rock crusher was then delivered to the G.R. No. 115966, March 20, 2003
spouses. However, 3 months later, the souses stopped payment
when petitioner had not acted on the complaints of the spouses FACTS: Almira Et al are the wife and children of the late Julio
about the machine. As a consequence, petitioner extra-judicially Garcia who inherited from his mother, Maria Alibudbud, a portion
foreclosed the real estate mortgage. To thwart the impending of a Lot 1642 in Sta. Rosa, Laguna. Lot 1642 was co-owned and
auction, the spouses filed a complaint for rescission of the registered in the names of Vicente de Guzman, Enrique
contract of lease and annulment of the real estate mortgage. Hemedes and Francsisco Alibudbud.

ISSUE: On July 5, 1984, the heirs of Julio Garcia and Federico Briones
1) W/N the nature of the contract is one of a contract entered into a Kasunduan ng Pagbibilhan over the 21,460 sq. m
of sale. portion for the sum of P150,000--- P65,000 was paid at the
2) W/N the remedies of the seller provided for in execution of the contract and the P85,000 was made payable
Article 1484 are cumulative. within 6 months from the date of the execution of the instrument.
At the time of the execution of the document, Briones was
HELD: informed that the title over the property is with their cousin
1.) YES. Conchalina who owns the bigger portion of the land. This
notwithstanding, respondent willingly entered into the Kasunduan
It is apparent here that the intent of the parties to the subject provided that the full payment of the purchase price will be made
contract is for the so-called rentals to be the installment upon delivery to him of the title.
payments. Upon the completion of the payments, then the rock
crusher, subject matter of the contract, would become the Briones took possession of the subject property and made
COMPILED BY: WIGMORE #WIGMOREFOREVER 28
SALES Case Digest (Atty. Sarona)
Compiled by: Wigmore #wigmoreforever

various payments amounting to P58,500.00 but because of the


failure of the heirs of Garcia to deliver to him a separate title to A careful reading of the Kasunduan reveals that it is a contract of
the property, he refused to make further payments. This sale. There was a perfected contract of sale in this case. The
prompted the heirs to file a case for rescission of the Kasunduan parties agreed on the sale of a determinate object which is the
and the return of the possession of the subject land. subject property in this case in the name of Julio Garcia, and
also the price certain therefor, without any reservation of title on
The heirs alleged that they approached Briones several times to the part of the heirs. Ownership was effectively conveyed by
deliver the required title but the latter refused saying that he did petitioners to respondent, who was given possession of the
not have the money to pay the balance of the purchase price. property. The delivery of a separate title in the name of Julio
Garcia was a condition imposed on respondent’s obligation to
The RTC decreed the rescission prayed for and the return of the pay the balance of the purchase price. It was not a condition
possession of the subject property. The CA however reversed the imposed on the perfection of the contract of sale.
lower court’s decision.
As to the rescission prayed for, the Court rules in the negative.
ISSUES: The power to rescind is only given to the injured party. The
(1) W/N payment of the balance of the purchase price is injured party is the party who has faithfully fulfilled his obligation
conditioned upon delivery of a separate title in the or is ready and willing to perform with his obligation. In the case
name of Julio Garcia; at bar, petitioners were not ready, willing and able to comply with
their obligation to deliver a separate title in the name of Julio
(2) W/N the heirs are entitled to rescind the Kasunduan Garcia to respondent. Therefore, they are not in a position to ask
for failure of Briones to complete payment. for rescission of the Kasunduan. Moreover, respondent’s
obligation to pay the balance of the purchase price was made
HELD: subject to delivery by petitioners of a separate title in the name of
(1) YES. Julio Garcia within six (6) months from the time of the execution
of the Kasunduan, a condition with which petitioners failed to
The tenor of the correspondence between the heirs and Briones comply. Failure to comply with a condition imposed on the
shows that the parties intended that a separate title to the performance of an obligation gives the other party the option
property in the name of Julio Garcia shall be delivered to Briones either to refuse to proceed with the sale or to waive that condition
as a condition for the latter’s payment of the balance of the under Article 1545 of the Civil Code. Hence, it is the respondent
purchase price. As such, Briones signified his willingness to pay who has the option either to refuse to proceed with the sale or to
but reminded the heirs of their obligation to deliver title to the waive the performance of the condition imposed on his obligation
property. to pay the balance of the purchase price.

If the parties intended that the heirs deliver TCT No. RT-1076
instead of a separate title in the name of Julio Garcia to Briones, SPOUSES SERRANO VS. CAGUIAT
then there would have been no need for the heirs to ask for 517 SCRA 57
partial sums on the ground that this would be used to pay for the G.R. NO. 139173
processing fee of the title to the property. The heirs only had to FEBRUARY 28, 2007
present the existing title to Briones and demand the balance of
the purchase price, but this they did not do. There is likewise no FACTS: Spouses Serrano are registered owners of a lot located
basis to conclude that insufficiency of funds rather than failure of in Las Pinas. On March 23, 1990, Caguiat offered to buy the lot
the heirs to deliver a separate title in the name of Julio Garcia and the Serranos agreed to sell it at 1,500.00/sqm.
prevented Briones from completing payment of the purchase
price. Caguiat then paid them a partial payment of 100,000.00 as
evidenced by a receipt indicating therein Caguiat’s promise to
That the parties agreed on delivery of a separate title in the pay the remaining balance.
name of Julio Garcia as a condition for respondent’s payment of
the balance of the purchase price is bolstered by the fact that Respondent, after making known his readiness to pay the
there was already an approved subdivision plan of the 21,460 balance, requested from petitioners the preparation of the
square-meter lot years before petitioners filed an action in court necessary Deed of Sale.
for rescission. Unfortunately, the heirs were not able to secure a
separate title in the name of Julio Garcia. Petitioners informed respondent in a letter that Amparo Herrera
would be leaving for abroad on or before April 15, 1990 and they
(2) NO are canceling the transaction and that respondent may recover
the earnest money (100,000) anytime. Petitioners also wrote him
In order to determine if rescission is proper, the Court needed to stating that they already delivered a manager’s check to his
ascertain whether the Kasunduan was a Contract to Sell or a counsel in said amount.
Contract of Sale. In a contract to sell, ownership is, by
agreement, reserved to the vendor and is not to pass until full Respondent thus filed a complaint for specific performance and
payment of the purchase price; whereas, in contract of sale, title damages with the RTC of Makati.
to the property passes to the vendee upon delivery of the thing
sold. Non-payment by the vendee in a contract of sale entitles The trial court relying on Article 1482 of the Civil Code ruled that
the vendor to demand specific performance or rescission of the the payment of 100,000.00 being an earnest money signified
contract, with damages, under Article 1191 of the Civil Code. perfection of the contract of sale and ordered the petitioners to
COMPILED BY: WIGMORE #WIGMOREFOREVER 29
SALES Case Digest (Atty. Sarona)
Compiled by: Wigmore #wigmoreforever

execute a final deed of sale in favor of respondent.

The Court of Appeals denied petitioners’ motion for


reconsideration in affirmation of the lower court’s decision.

ISSUE: WON there was a contract of sale. NO.

HELD: The transaction was a contract to sell.

When petitioners declared in the Receipt of Partial Payment” that


they –
“Received from Mr. Godofredo Caguiat the amount of one
hundred thousand pesos as Partial payment of our lot
situated in Las Pinas…Mr. Caguiat promised to pay the
balance of the purchase price on or before March 23,
1990… And that we will execute and sign the final deed of
sale on this date,”

-- there can be no other interpretation than that they agreed to a


conditional contract of sale, consummation of which is subject
only to the full payment of the purchase price. se pri agreem
ce.

A contract to sell is akin to a conditional sale where the efficacy


or obligatory force of the vendor’s obligation to transfer title is
subordinated to the happening of a future and uncertain event,
so that if the suspensive condition does not take place, the
parties would stand as if the conditional obligation had never
existed. The suspensive condition is commonly full payment of
the purchase price.

In this case, the “Receipt of Partial Payment” shows that the true
agreement between the parties is a contract to sell.

First, ownership of the parcel of land was retained by petitioners


and was not to pass to respondent until full payment of the
purchase price. Second, the agreement between the parties was
not embodied in a deed of sale. The absence of a formal deed
of conveyance is a strong indication that the parties did not
intend immediate transfer of ownership, but only a transfer after
full payment of the purchase price. Third, petitioners retained
possession of the certificate of the lot.

It is true that Article 1482 provides that whenever earnest money


is given in a contract of sale, it shall be considered as part of the
price and proof of the perfection of the contract. However, this
article speaks of earnest money given in a contract of sale. In
this case, the earnest money forms part of the consideration only
if the sale is consummated upon full payment of the purchase
price.

Clearly, respondent cannot compel petitioners to transfer


ownership of the property to him.

NABUS VS.PACSON
G.R. NO. 161318
NOVEMBER 25, 2009

FACin6uof /P </MC14-9(o)n/Lang (en/MC148rETBT/F9 Tf1 0 0 1 28.4 113.8.4 185.8534 165.241.55 Tm[(T)-10(h)11(e60 )-101 0 0 1 28.4 TJETB

COMPILED BY: WIGMORE #WIGMOREFOREVER 30


SALES Case Digest (Atty. Sarona)
Compiled by: Wigmore #wigmoreforever

Conditional Sale" is actually a contract to sell. The contract Petitioner now is seeking the rescission of her contract with
stipulated that "as soon as the full consideration of the sale has respondent for the breach of nonpayment.
been paid by the vendee, the corresponding transfer documents
shall be executed by the vendor to the vendee for the portion RTC: the contract entered into by the parties is a contract to sell
sold." but ruled that the remedy of rescission could not apply because
the respondent’s failure to pay the petitioner the balance of the
Where the vendor promises to execute a deed of absolute sale purchase price in the total amount of ₱805,000.00 was not a
upon the completion by the vendee of the payment of the price, breach of contract, but merely an event that prevented the seller
the contract is only a contract to sell." The aforecited stipulation (petitioner) from conveying title to the purchaser (respondent).
shows that the vendors reserved title to the subject property until CA: Affirmed.
full payment of the purchase price.
ISSUE: W/N the contract entered into by the parties is a contract
Unfortunately for the Spouses Pacson, since the Deed of to sell? YES.
Conditional Sale executed in their favor was merely a contract to
sell, the obligation of the seller to sell becomes demandable HELD: The subject Deed of Conditional Sale with Assumption of
only upon the happening of the suspensive condition. The Mortgage entered into by and among the two parties and FSL
full payment of the purchase price is the positive suspensive Bank is a contract to sell and not a contract of sale. The
condition, the failure of which is not a breach of contract, but nonpayment of the full purchase price cannot give the petitioner
simply an event that prevented the obligation of the vendor to the remedy for rescission since the obligation did not yet exist
convey title from acquiring binding force. since the suspensive condition of payment of the full purchase
price had not taken place.
Thus, for its non-fulfillment, there is no contract to speak of, the
obligor having failed to perform the suspensive condition which A contract to sell may thus be defined as a bilateral contract
enforces a juridical relation. With this circumstance, there can be whereby the prospective seller, while expressly reserving the
no rescission or fulfillment of an obligation that is still non- ownership of the subject property despite delivery thereof to the
existent, the suspensive condition not having occurred as yet. prospective buyer, binds himself to sell the said property
exclusively to the prospective buyer upon fulfillment of the
Emphasis should be made that the breach contemplated in condition agreed upon, that is, full payment of the purchase
Article 1191 of the New Civil Code is the obligor’s failure to price.
comply with an obligation already extant, not a failure of a
condition to render binding that obligation. Based on their contract, the title and ownership of the subject
properties remains with the petitioner until the respondent fully
Therefore, since the Pacsons failed to fulfill the suspensive pays the balance of the purchase price and the assumed
condition, the obligation on the part of the Nabuses to sell them mortgage obligation. Thereafter, FSL Bank shall then issue the
the land never arose, and the Nabuses were well within their corresponding deed of cancellation of mortgage and the
rights when they sold the land to Tolero. The Pacsons, of course, petitioner shall execute the corresponding deed of absolute sale
are entitled to reimbursement. in favor of the respondent.

Accordingly, the petitioner’s obligation to sell the subject


REYES VS.TUPARAN properties becomes demandable only upon the happening of the
G.R. NO. 188064 positive suspensive condition, which is the respondent’s full
JUNE 1, 2011 payment of the purchase price.

FACTS: Petitioner Mila Reyes owns a building which the Without respondent’s full payment, there can be no breach of
respondent Victoria Tuparan was leasing a space for her contract to speak of because petitioner has no obligation yet to
pawnshop business. turn over the title. Respondent’s failure to pay in full the purchase
price is not the breach of contract contemplated under Article
Thereafter, petitioner mortgaged the building to Farmer Savings 1191 of the New Civil Code but rather just an event that prevents
and Loan Bank for P2M. However, the loan reached the petitioner from being bound to convey title to the respondent.
P2,278,078.13. Petitioner then decided to sell her real properties
for P6.5M in order to pay the bank. As a gesture of friendship,
respondent verbally offered to conditionally buy the building for
P4.2M and to assume the bank loan.

It was stipulated however that title to the ownership of the subject


real properties shall remain with the petitioner until full payment
of respondent. And only upon payment of full balance will the
bank issue the Deed of Cancellation of Mortgage and the
petitioner to execute the corresponding Deed of Absolute Sale.
Respondent however defaulted, revealing a balance of
P805,000. She was already able to pay the amountP3.4M and
the bank loan.

COMPILED BY: WIGMORE #WIGMOREFOREVER 31


SALES Case Digest (Atty. Sarona)
Compiled by: Wigmore #wigmoreforever

PART II: ELEMENTS OF A CONTRACT OF SALE since 2/3 belonged to his sisters. Petitioner could not have
given her consent to the contract, being a minor at the time.
2. CONSENT: PARTIES Consent of the contracting parties is among the essential
requisites of a contract, including one of sale, absent which
1) Minors, Insane or Demented Persons, Deaf-Mutes there can be no valid contract. Moreover, petitioner admittedly
did not pay any centavo for the property, which makes the sale
void.
LABAGALA VS. SANTIAGO
371 SCRA 360 Art 1471: If the price is simulated, the sale is void, but the act
G.R. NO. 132305 may be shown to have been in reality a donation, or some other
DECEMBER 4, 2001 act or contract.

FACTS: Jose Santiago owned a parcel of land covered by TCT Even assuming that the deed is genuine, it cannot be a valid
donation. It lacks the acceptance of the donee required by Art
64729. Alleging that Jose had fraudulently registered it in his
name alone, his sisters (Nicolasa and Amanda, herein 725 of the Civil Code. Being a minor, the acceptance of the
respondents) sued Jose for the recovery of 2/3 share of the donation should have been made by her father (Leon Labagala)
property. RTC ruled in favor of the sisters and their names were or mother, or legal representative. No one of those mentioned in
included in the certificate of title. Jose died intestate. the law accepted the “donation” for Ida.

Respondents filed a complaint for the recovery of title, ownership The Court also ruled that petitioner is not the child of Jose
and possession against petitioner Labagala to recover from her Santiago, and cannot inherit from him through succession. No
the 1/3 portion of the said property pertaining to Jose, but which birth certificate was shown, only a baptismal certificate, which is
came into petitioner’s sole possession upon his death. not conclusive proof of filiation.

Respondents alleged that Jose’s share in the property belongs to


PARAGAS vs. HEIRS OF DOMINADOR BALACANO
them by operation of law since they are the only legal heirs of
their brother.
FACTS: Gregorio Balacano, married to Lorenza Sumigcay, was
Respondents’ contentions: the registered owner of Lot 1175-E and Lot 1175-F of the Subd.
The deed of sale was a forgery. The deed showed that Jose Plan Psd-38042. Gregorio and Lorenza had three children,
affixed his thumb mark, but respondents averred that, having namely: Domingo, Catalino and Alfredo, all surnamed Balacano.
been able to graduate from college, Jose never put his thumb Lorenza died on December 11, 1991. Gregorio, on the other
mark on documents and always signed his name in full. hand, died on July 28, 1996. Prior to his death, Gregorio was
admitted on June 28, 1996, transferred hospital in the afternoon
Respondents also pointed out that it is highly improbable for of July 19, 1996 until his death.
petitioner to have paid the supposed consideration of P150,000
for the sale of the property because petitioner was unemployed Gregorio purportedly sold on July 22, 1996, or barely a week
and without any visible means of livelihood at the time of the prior to his death, a portion of Lot 1175-E (15,925 square meters
alleged sale. out of total area of 22,341 square meters) and the whole Lot
1175-F to Spouses Paragas for the total consideration of
It was quite unusual and questionable that petitioner registered P500,000.00. This sale appeared in a deed of absolute sale and
the deed of sale almost 8 years after the execution of the sale. was notarized by Atty. De Guzman. Gregorios certificates of title
were consequently cancelled and new certificates of title were
Petitioner claimed to be the daughter of Jose, and thus entitled to issued in favor of the Spouses Paragas.
his share in the property.
The Spouses Paragas then sold on October 17, 1996 a portion
She argued that the sale was in fact a donation to her, and that of Lot 1175-E consisting of 6,416 square meters to Catalino for
nothing could have precluded Jose from putting his thumb mark the total consideration of P60,000.00.
on the deed of sale instead of his signature.
Domingo’s children filed on October 22, 1996 a complaint for
ISSUE: WON there was a valid sale. NO annulment of sale and partition against Catalino and the
Spouses Paragas. They essentially alleged in asking for the
HELD: The Court agreed with CA that: nullification of the deed of sale that:
This deed is shot through and through with so many intrinsic
defects that a reasonable mind is inevitably led to the conclusion (1) their grandfather Gregorio could not have appeared before
that it is fake. Why hide the nature of the contract in the façade of the notary public on July 22, 1996 at Santiago City because he
a sale? Why did Santiago (fully aware that he owned only 1/3) was then confined at the Veterans Memorial Hospital in Quezon
sell or donate the whole property to Ida? Why did Santiago affix City;
only his thumb mark to a deed that falsely stated that… xxx Ida (2) at the time of the alleged execution of the deed of sale,
was of legal age when she was then only 15 years old? Etc… Gregorio was seriously ill, in fact dying at that time, which vitiated
his consent to the disposal of the property; and
Clearly, there is no valid sale in this case. Jose did not have the (3) Catalino manipulated the execution of the deed and prevailed
right to transfer ownership of the entire property to petitioner upon the dying Gregorio to sign his name on a paper the
contents of which he never understood because of his serious
COMPILED BY: WIGMORE #WIGMOREFOREVER 32
SALES Case Digest (Atty. Sarona)
Compiled by: Wigmore #wigmoreforever

condition. and the Spouses Paragas. He likewise stated that of the stated
P500,000.00 consideration in the deed, Rudy paid Gregorio
Alternatively, they alleged that assuming Gregorio was of sound P450,000.00 in the hospital because Rudy had previously paid
and disposing mind, he could only transfer a half portion of Lots Gregorio P50,000.00. For his part, Antonio added that he was
1175-E and 1175-F as the other half belongs to their asked by Rudy to take pictures of Gregorio signing the deed. He
grandmother Lorenza who predeceased Gregorio they claimed also claimed that there was no entry on the date when he signed;
that Lots 1175-E and 1175-F form part of the conjugal nor did he remember reading Santiago City as the place of
partnership properties of Gregorio and Lorenza. Finally, they execution of the deed. He described Gregorio as still strong but
alleged that the sale to the Spouses Paragas covers only a 5- sickly, who got up from the bed with Julias help.
hectare portion of Lots 1175-E and 1175-F leaving a portion of
6,416 square meters that Catalino is threatening to dispose. The lower court, after trial, rendered the decision declaring null
They asked for the nullification of the deed of sale executed by and void the deed of sale purportedly executed by Gregorio
Gregorio and the partition of Lots 1175-E and 1175-F. They Balacano in favor of the spouses Paragas, noting that at the time
likewise asked for damages. Gregorio executed the deed, Gregorio was ill. Because of the
seriousness of his illness, it is not expected that Gregorio
Plaintiff-appellant Nanette Balacano testified to prove the Balacano would be negotiating a contract of sale. The lower
material allegations of their complaint. On Gregorios medical court also ruled that Lots 1175-E and 1175-F were Gregorios and
condition, she declared that: Lorenzas conjugal partnership properties.
(1) Gregorio, who was then 81 years old, weak and sick, was
brought to the hospital in Bayombong, Nueva Vizcaya on June The Court of Appeals affirmed the Decision of the trial court, with
28, 1996 and stayed there until the afternoon on July 19, 1996; the modification that Lots 1175-E and 1175-F were adjudged as
(2) thereafter, Gregorio, who by then was weak and could no belonging to the estate of Gregorio Balacano.
longer talk and whose condition had worsened, was transferred
in the afternoon of July 19, 1996 to the Veterans Memorial ISSUE:
Hospital in Quezon City where Gregorio died.  WON Gregorio give an intelligent consent to the sale of Lots
1175-E and 1175-F when he signed the deed of sale?
She claimed that Gregorio could not have signed a deed of sale  WON Deed of Sale purportedly executed between
on July 19, 1996 because she stayed at the hospital the whole of petitioners and the late Gregorio Balacano was null and void
that day and saw no visitors. She likewise testified on their
agreement for attorneys fees with their counsel and the litigation RULING: It is not disputed that when Gregorio signed the deed
expenses they incurred. of sale, Gregorio was seriously ill, as he in fact died a week after
the deeds signing. Gregorio died of complications caused by
Defendants posit that Gregorio’s consent to the sale should be cirrhosis of the liver. Gregorios death was neither sudden nor
determined, not at the time Gregorio signed the deed of sale on immediate; he fought at least a month-long battle against the
July 18, 1996, but at the time when he agreed to sell the property disease until he succumbed to death on July 22, 1996. Given
in June 1996 or a month prior to the deeds signing; and in June that Gregorio purportedly executed a deed during the last stages
1996, Gregorio was of sound and disposing mind and his of his battle against his disease, the Court seriously doubt
consent to the sale was in no wise vitiated at that time. They whether Gregorio could have read, or fully understood, the
presented as witnesses Notary Public de Guzman and contents of the documents he signed or of the consequences of
instrumental witness Antonio to prove Gregorios execution of the his act. There was no conclusive evidence that the contents of
sale and the circumstances under the deed was executed. They the deed were sufficiently explained to Gregorio before he affixed
uniformly declared that: his signature. The evidence the defendants-appellants offered to
prove Gregorios consent to the sale consists of the testimonies
(1) on July 18, 1996, they went to the hospital in Bayombong, of Atty. de Guzman and Antonio which the Court did not find
Nueva Vizcaya where Gregorio was confined with Rudy; credible.
(2) Atty. De Guzman read and explained the contents of the deed
to Gregorio; Additionally, the irregular and invalid notarization of the deed is a
(3) Gregorio signed the deed after receiving the money from falsity that raises doubts on the regularity of the transaction itself.
Rudy; While the deed was indeed signed on July 18, 1996 at
(4) Julia and Antonio signed the deed as witnesses. Bayombong, Nueva Vizcaya, the deed states otherwise, as it
shows that the deed was executed on July 22, 1996 at Santiago
Additionally, Atty. De Guzman explained that the execution of the City.
deed was merely a confirmation of a previous agreement
between the Spouses Paragas and Gregorio that was concluded Article 24 of the Civil Code tells us that in all contractual, property
at least a month prior to Gregorios death; that, in fact, Gregorio or other relations, when one of the parties is at a disadvantage
had previously asked him to prepare a deed that Gregorio on account of his moral dependence, ignorance, indigence,
eventually signed on July 18, 1996. He also explained that the mental weakness, tender age or other handicap, the courts must
deed, which appeared to have been executed on July 22, 1996, be vigilant for his protection.
was actually executed on July 18, 1996; he notarized the deed
and entered it in his register only on July 22, 1996. He claimed Gregorio’s consent to the sale of the lots was absent, making the
that he did not find it necessary to state the precise date and contract null and void. Consequently, the spouses Paragas could
place of execution (Bayombong, Nueva Vizcaya, instead of not have made a subsequent transfer of the property to Catalino
Santiago City) of the deed of sale because the deed is merely a Balacano.
confirmation of a previously agreed contract between Gregorio
COMPILED BY: WIGMORE #WIGMOREFOREVER 33
SALES Case Digest (Atty. Sarona)
Compiled by: Wigmore #wigmoreforever

time or by ratification. There are two types of void contracts:


In the case at bar, the Deed of Sale was allegedly signed by
Gregorio on his death bed in the hospital. Gregorio was an I. Those where one of the essential requisites of a valid
octogenarian at the time of the alleged execution of the contract contract as provided for by Art 1318(10) of the NCC is totally
and suffering from liver cirrhosis at that circumstances which wanting; and
raise grave doubts on his physical and mental capacity to freely II. Those declared to be so under Art 14092 (11) of the NCC. By
consent to the contract. Adding to the dubiety of the purported contrast, a voidable or annullable contract is one in which the
sale and further bolstering respondents claim that their uncle essential requisites for validity under Art 1318 are present,
Catalino, one of the children of the decedent, had a hand in the but vitiated by want of capacity, error, violence, intimidation,
execution of the deed is the fact that on 17 October 1996, undue influence or deceit.
petitioners sold a portion of Lot 1175-E consisting of 6,416
square meters to Catalino for P60,000.00. One need not stretch Article 1318 of the Civil Code states that no contract exists
his imagination to surmise that Catalino was in cahoots with unless there is a concurrence of consent of the parties, object
petitioners in maneuvering the alleged sale. certain as subject matter, and cause of the obligation
established. Article 1327 provides that insane or demented
persons cannot give consent to a contract. But, if an insane or
FRANCISCO VS HERRERA demented person does enter into a contract, the legal effect is
that the contract is voidable or annullable as specifically provided
FACTS: Eligio Herrera, Sr., the father of respondent, was the in Article 1390.
owner of two parcels of land, one consisting of 500 sq. m. and
another consisting of 451 sq. m. On January 3, 1991, petitioner An annullable contract may be rendered perfectly valid by
bought from said landowner the first parcel, for the price of ratification, which can be express or implied. Implied ratification
P1,000,000, paid in installments from November 30, 1990 to may take the form of accepting and retaining the benefits of a
August 10, 1991.On March 12, 1991, petitioner bought the contract. This is what happened in this case. Respondents
second parcel, for P750,000. contention that he merely received payments on behalf of his
father merely to avoid their misuse and that he did not intend to
Contending that the contract price for the two parcels of land was concur with the contracts is unconvincing. If he was not
grossly inadequate, the children of Eligio, Sr., namely, Josefina agreeable with the contracts, he could have prevented petitioner
Cavestany, Eligio Herrera, Jr., and respondent Pastor Herrera, from delivering the payments, or if this was impossible, he could
tried to negotiate with petitioner to increase the purchase price. have immediately instituted the action for reconveyance and
have the payments consigned with the court. None of these
When petitioner refused, herein respondent then filed a happened. As found by the trial court and the Court of Appeals,
complaint for annulment of sale. In his complaint, respondent upon learning of the sale, respondent negotiated for the increase
claimed ownership over the second parcel, allegedly by virtue of of the purchase price while receiving the installment payments. It
a sale in his favor since 1973. He likewise claimed that the first was only when respondent failed to convince petitioner to
parcel was subject to the co-ownership of the surviving heirs of increase the price that the former instituted the complaint for
Francisca A. Herrera, the wife of Eligio, Sr., considering that she reconveyance of the properties. Clearly, respondent was
died intestate on April 2, 1990, before the alleged sale to agreeable to the contracts, only he wanted to get more. Further,
petitioner. there is no showing that respondent returned the payments or
made an offer to do so. This bolsters the view that indeed there
Finally, respondent also alleged that the sale of the two lots was was ratification. One cannot negotiate for an increase in the price
null and void on the ground that at the time of sale, Eligio, Sr. in one breath and in the same breath contend that the contract of
was already incapacitated to give consent to a contract because sale is void.
he was already afflicted with senile dementia, characterized by
deteriorating mental and physical condition including loss of
memory. 2) Sale by and Between Spouses

The Regional Trial Court declared the deed of sale null and void.
CA affirmed trial court’s decision. GUIANG VS CA

ISSUE: WON the assailed contracts of sale void or merely FACTS: Plaintiff Gilda Corpuz and defendant Judie Corpuz are
voidable and hence capable of being ratified. legally married spouses. The couple have three children, namely:
Junie 18 years old, Harriet 17 years of age, and Jodie or Joji, the
HELD: It was established that the vendor Eligio, Sr. entered into youngest, who was 15 years of age at the time their mother
an agreement with petitioner, but that the formers capacity to testified in court.
consent was vitiated by senile dementia. Hence, the Court must
rule that the assailed contracts are not void or inexistent per Over the objection of private respondent and while she was in
se; rather, these are contracts that are valid and binding Manila seeking employment, her husband sold to the petitioners-
unless annulled through a proper action filed in court spouses one half of their conjugal property, consisting of their
seasonably. residence and the lot on which it stood.

A void or inexistent contract is one which has no force and effect Sometime on February 14, 1983, the couple Gilda and Judie
from the very beginning. Hence, it is as if it has never been Corpuz, with plaintiff-wife Gilda Corpuz as vendee, bought a 421
entered into and cannot be validated either by the passage of
COMPILED BY: WIGMORE #WIGMOREFOREVER 34
SALES Case Digest (Atty. Sarona)
Compiled by: Wigmore #wigmoreforever

sq. meter lot from Manuel Callejo who signed as vendor through obtained and vitiated through mistake, violence, intimidation,
a conditional deed of sale for a total consideration of P14,735.00. undue influence or fraud. In this instance, private respondents
The consideration was payable in installment, with right of consent to the contract of sale of their conjugal property was
cancellation in favor of vendor should vendee fail to pay three totally inexistent or absent. Gilda Corpuz, on direct examination,
successive installments. Sometime on April 22, 1988, the couple testified thus:
Gilda and Judie Corpuz sold one-half portion of their Lot No. 9,
Block 8, to the defendants spouses Guiang. The latter have ART. 124. The administration and enjoyment of the
since then occupied the one-half portion and built their house conjugal partnership property shall belong to both
thereon. They are thus adjoining neighbors of the Corpuzes. spouses jointly. In case of disagreement, the husbands
decision shall prevail, subject to recourse to the court
Plaintiff Gilda Corpuz left for Manila sometime in June 1989. She by the wife for proper remedy, which must be availed of
was trying to look for work abroad, in the Middle East. within five years from the date of the contract
Unfortunately, she became a victim of an unscrupulous illegal implementing such decision.
recruiter. She was not able to go abroad. She stayed for
sometime in Manila. After his wifes departure for Manila, In the event that one spouse is incapacitated or
defendant Judie Corpuz seldom went home to the conjugal otherwise unable to participate in the administration of
dwelling. Sometime in January 1990, Harriet Corpuz learned that the conjugal properties, the other spouse may assume
her father intended to sell the remaining one-half portion sole powers of administration. These powers do not
including their house, of their home lot to defendants Guiangs. include the powers of disposition or encumbrance
She wrote a letter to her mother informing her. Gilda replied that which must have the authority of the court or the
she was objecting to the sale. Harriet, however, did not inform written consent of the other spouse. In the absence of
her father about this; but instead gave the letter to Mrs. such authority or consent, the disposition or
Luzviminda Guiang so that she Guiang would advise her father. encumbrance shall be void. However, the transaction
shall be construed as a continuing offer on the part of
However, in the absence of his wife Gilda Corpuz, defendant the consenting spouse and the third person, and may
Judie Corpuz pushed through the sale of the remaining one-half be perfected as a binding contract upon the
portion of Lot 9, Block 8. On March 1, 1990, he sold to defendant acceptance by the other spouse or authorization by the
Luzviminda Guiang thru a document known as Deed of Transfer court before the offer is withdrawn by either or both
of Rights the remaining one-half portion of their lot and the house offerors.
standing thereon for a total consideration of P30,000.00 of which
P5,000.00 was to be paid in June , 1990. The legal provision is clear. The disposition or encumbrance is
void. It becomes still clearer if we compare the same with the
Sometime on March 11, 1990, plaintiff returned home. She found equivalent provision of the Civil Code of the Philippines. Under
her children staying with other households. Only Junie was Article 166 of the Civil Code, the husband cannot generally
staying in their house. Harriet and Joji were with Mr. Panes. alienate or encumber any real property of the conjugal
Gilda gathered her children together and stayed at their house. partnership without the wife’s consent. The alienation or
Her husband was nowhere to be found. She was informed by her encumbrance if so made however is not null and void. It is
children that their father had a wife already. merely voidable. The offended wife may bring an action to annul
the said alienation or encumbrance. Thus, the provision of Article
For staying in their house sold by her husband, plaintiff was 173 of the Civil Code of the Philippines, to wit:
complained against by Guiang spouses before the Barangay
authorities for trespassing. On March 16, 1990, the parties Art. 173. The wife may, during the marriage
thereat signed a document known as amicable settlement. and within ten years from the transaction
Believing that she had received the shorter end of the bargain, questioned, ask the courts for the
plaintiff went to the Barangay Captain for the annulment of the annulment of any contract of the husband
settlement. Defendant-spouses Guiang followed thru the entered into without her consent, when
amicable settlement with a motion for the execution of the such consent is required, or any act or
amicable settlement, filing the same with the Municipal Trial contract of the husband which tends to
Court. defraud her or impair her interest in the
conjugal partnership property. Should the
The judgment was rendered for the plaintiff and against the wife fail to exercise this right, she or her
defendants. CA affirmed the trial court’s decision. heirs after the dissolution of the marriage,
may demand the value of property
ISSUE: fraudulently alienated by the husband.(n)
I. WON the contract of sale (Deed of Transfer of Rights) was
merely voidable, and This particular provision giving the wife ten (10) years during
II. WON such contract was ratified by private respondent when the marriage to annul the alienation or encumbrance was not
she entered into an amicable settlement with them carried over to the Family Code. It is thus clear that any
alienation or encumbrance made after August 3, 1988 when
RULING: the Family Code took effect by the husband of the conjugal
(1) The Contract of Sale was not merely voidable but void. partnership property without the consent of the wife is null
The error in petitioners contention is evident. Article 1390, par. 2, and void.
refers to contracts visited by vices of consent, i.e., contracts
which were entered into by a person whose consent was In sum, the nullity of the contract of sale is premised on the
COMPILED BY: WIGMORE #WIGMOREFOREVER 35
SALES Case Digest (Atty. Sarona)
Compiled by: Wigmore #wigmoreforever

absence of private respondents consent. To constitute a valid Pending the appeal, Ignacia died and she was substituted by her
contract, the Civil Code requires the concurrence of the following compulsory heirs. Petitioners contended that they are entitled to
elements: (1) cause, (2) object, and (3) consent, the last element reimbursement of the rentals collected on the apartment built on
being indubitably absent in the case at bar. Lot No. 4349-B-2, while respondent spouses claimed that they
are buyers in good faith.
(2) The amicable settlement did not ratify the contract. Both
the Deed of Transfer of Rights and the amicable settlement Court of Appeals reversed and set aside the decision of the trial
are null and void. court. It ruled that notwithstanding the absence of Ignacia’s
consent to the sale, the same must be held valid in favor of
Art. 1422. A contract which is the direct result of a respondents because they were innocent purchasers for value.
previous illegal contract, is also void and
inexistent. (Civil Code of the Philippines). ISSUE:
(1) What is the status of the sale of Lot No. 4349-B-2 to
The sale of a conjugal property requires the consent of both the respondent spouses?
husband and the wife. The absence of the consent of one (2) Assuming that the sale is annullable, should it be annulled in
renders the sale null and void, while the vitiation thereof makes it its entirety or only with respect to the share of Ignacia?
merely voidable. Only in the latter case can ratification cure the (3) Are respondent spouses purchasers in good faith?
defect.
RULING:
(1) The sale was voidable.
HEIRS OF REYES vs MIJARES Under the regime of the Civil Code, the alienation or
encumbrance of a conjugal real property requires the consent of
FACTS: The controversy stemmed from a dispute over Lot No. the wife. The absence of such consent renders the entire
4349-B-2, approximately 396 square meters and registered in transaction merely voidable and not void. The wife may, during
the name of Spouses Vicente Reyes and Ignacia Aguilar-Reyes. the marriage and within ten years from the transaction
Said lot and the apartments built thereon were part of the questioned, bring an action for the annulment of the contract
spouses’ conjugal properties having been purchased using entered into by her husband without her consent.
conjugal funds from their garments business.
Articles 166 and 173 of the Civil Code, the governing laws at the
Vicente and Ignacia were married in 1960, but had been time the assailed sale was contracted, provide:
separated de facto since 1974. Sometime in 1984, Ignacia
learned that on March 1, 1983, Vicente sold Lot No. 4349-B-2 to Art.166. Unless the wife has been declared a non
respondent Spouses Mijares for P40,000.00. She likewise found compos mentis or a spendthrift, or is under civil
out that Vicente filed a petition for administration and interdiction or is confined in a leprosarium, the husband
appointment of guardian.Vicente misrepresented therein that his cannot alienate or encumber any real property of the
wife, Ignacia, died on March 22, 1982, and that he and their 5 conjugal partnership without the wife’s consent. If she
minor children were her only heirs. On September 29, 1983, the refuses unreasonably to give her consent, the court
court appointed Vicente as the guardian of their minor children. may compel her to grant the same…
Subsequently, in its Order dated October 14, 1983, the court
authorized Vicente to sell the estate of Ignacia. Art. 173. The wife may, during the marriage and within
ten years from the transaction questioned, ask the
On August 9, 1984, Ignacia, through her counsel, sent a letter to courts for the annulment of any contract of the husband
respondent spouses demanding the return of her ½ share in the entered into without her consent, when such consent is
lot. Failing to settle the matter amicably, Ignacia filed on June 4, required, or any act or contract of the husband which
1996 a complaint for annulment of sale against respondent tends to defraud her or impair her interest in the
spouses. The complaint was thereafter amended to include conjugal partnership property. Should the wife fail to
Vicente Reyes as one of the defendants. exercise this right, she or her heirs after the dissolution
of the marriage, may demand the value of property
The court rendered a decision declaring the sale of Lot No. 4349- fraudulently alienated by the husband.
B-2 void with respect to the share of Ignacia. It held that the
purchase price of the lot was P110,000.00 and ordered Vicente Pursuant to the foregoing provisions, the husband could not
to return ½ thereof or P55,000.00 to respondent spouses. alienate or encumber any conjugal real property without the
consent, express or implied, of the wife otherwise, the contract is
Ignacia filed a motion for modification of the decision praying that voidable. This is consistent with Article 173 of the Civil Code
the sale be declared void in its entirety and that the respondents pursuant to which the wife could, during the marriage and within
be ordered to reimburse to her the rentals they collected on the 10 years from the questioned transaction, seek its annulment.
apartments built on Lot No. 4349-B-2 computed from March 1,
1983. In the case at bar, there is no dispute that Lot No. 4349-B-2, is a
conjugal property having been purchased using the conjugal
On May 31, 1990, the trial court modified its decision by funds of the spouses during the subsistence of their marriage. It
declaring the sale void in its entirety and ordering Vicente Reyes is beyond cavil therefore that the sale of said lot to respondent
to reimburse respondent spouses the purchase price of spouses without the knowledge and consent of Ignacia is
P110,000. voidable. Her action to annul the March 1, 1983 sale which was
filed on June 4, 1986, before her demise is perfectly within the 10
COMPILED BY: WIGMORE #WIGMOREFOREVER 36
SALES Case Digest (Atty. Sarona)
Compiled by: Wigmore #wigmoreforever

year prescriptive period under Article 173 of the Civil Code. Even CONCEPCION AINZA ET AL V. ANTONIO AND EUGENIA
if we reckon the period from November 25, 1978 which was the PADUA
date when Vicente and the respondent spouses entered into a JUNE 30, 2005
contract concerning Lot No. 4349-B-2, Ignacia’s action would still
be within the prescribed period. FACTS: In April 1987, Ainza and her daughter Eugenia orally
agreed that Ainza pay P100k in exchange for half of the portion
(2) The trial court correctly annulled the voidable sale of Lot of Eugenia’s undivided conjugal property (a lot located in QC).
No. 4349-B-2 in its entirety. No Deed of Absolute Sale was executed. There was physical
The plain meaning attached to the plain language of the law is delivery of the land through Concepcion’s other daughter
that the contract, in its entirety, executed by the husband without (Natividad) acting as atty-in-fact. Concepcion thereafter allowed
the wife's consent, may be annulled by the wife. Had Congress Natividad and her husband occupy the purchased portion of the
intended to limit such annulment in so far as the contract shall land. In 1994, Antonio caused the division of the lot into three
"prejudice" the wife, such limitation should have been spelled out (two were occupied by the spouses), necessarily displacing
in the statute. Natividad. He also had each subdivision titled. Antonio requested
Natividad to vacate the premises. Antonio averred that his wife
To be underscored here is that upon the provisions of Articles only admitted of selling 1/3 of the property to Concepcion for
161, 162 and 163 of the Civil Code, the conjugal partnership is which a receipt was issued signed by Concepcion. The RTC
liable for many obligations while the conjugal partnership exists. ruled in favor of Concepcion. The CA reversed the RTC ruling.
Not only that. The conjugal property is even subject to the CA explained that the property is conjugal hence the sale should
payment of debts contracted by either spouse before the have been with Antonio’s consent.
marriage, as those for the payment of fines and indemnities
imposed upon them after the responsibilities in Article 161 have ISSUE: Whether or not the contract of sale between Ainza and
been covered (Article 163, par. 3), if it turns out that the spouse Eugenia is valid.
who is bound thereby, "should have no exclusive property or if it
should be insufficient." These are considerations that go beyond RULING: Yes it is valid until annulled (voidable). There was a
the mere equitable share of the wife in the property. perfected contract of sale between Eugenia and Concepcion.
The records show that Eugenia offered to sell a portion of the
(3) Spouses Mijares are not purchasers in good faith. property to Concepcion, who accepted the offer and agreed to
In the instant case, there existed circumstances that should have pay P100,000.00 as consideration. The contract of sale was
placed respondent spouses on guard. The death certificate of consummated when both parties fully complied with their
Ignacia, shows that she died on March 22, 1982. The same respective obligations. Eugenia delivered the property to
death certificate, however, reveals that – (1) it was issued by the Concepcion, who in turn, paid Eugenia the price of P100,000.00,
Office of the Civil Registrar of Lubao Pampanga on March 10, as evidenced by the receipt. Since the land was undivided when
1982; (2) the alleged death of Ignacia was reported to the Office it was sold, Concepcion is entitled to have half of it. Antonio
of the Civil Registrar on March 4, 1982; and (3) her burial or cannot, however, attack the validity of the sale b/n his wife and
cremation would be on March 8, 1982. These obvious flaws in his mom-in-law, either under the Family Code or the Old Civil
the death certificate should have prompted respondents to Code due to prescription. The sale came to his knowledge in
investigate further, especially so that respondent Florentina 1987. He only filed the case is 1999. His right prescribed in 1993
Mijares admitted on cross examination that she asked for the (under the FC [5 years]) and 1997 (under OCC [10 years]).
death certificate of Ignacia because she was suspicious that
Ignacia was still alive.
FUENTES VS ROCA
Respondent spouses cannot deny knowledge that at the time of
the sale in 1978, Vicente was married to Ignacia and that the FACTS: Sabina Tarroza owned a titled 358-square meter lot.
latter did not give her conformity to the sale. This is so because She sold it to her son, Tarciano Roca. Tarciano did not meantime
the 1978 "Agreement" described Vicente as "married" but the have the registered title to his name. Six years later, Tarciano
conformity of his wife to the sale did not appear in the deed. offered to sell the lot to petitioners spouses Fuentes. They
arranged to meet at the office of Atty. Romulo Plagata whom they
Obviously, the execution of another deed of sale in 1983 over the asked to prepare the documents of sale. They later signed an
same Lot No. 4349-B-2, after the alleged death of Ignacia on agreement to sell which agreement stated that it was to take
March 22, 1982, as well as the institution of the special effect in six months.
proceedings were, intended to correct the absence of Ignacia’s
consent to the sale. Even assuming that respondent spouses The agreement required the Fuentes spouses to pay Tarciano a
believed in good faith that Ignacia really died on March 22, 1982, downpayment of P60K for the transfer of the lot’s title to him.
after they purchased the lot, the fact remains that the sale of Lot Within six months, Tarciano was to secure the consent of his
No. 4349-B-2 prior to Ignacia’s alleged demise was without her estranged wife Rosario to the sale.
consent and therefore subject to annulment. The October 14,
1983 order authorizing the sale of the estate of Ignacia, could not In working with the other requirements, Atty. Plagata said that he
have validated the sale of Lot No. 4349-B-2 because said order went to see Rosario in one of his trips in Manila and had her sign
was issued on the assumption that Ignacia was already dead the affidavit of consent. As soon as Tarciano met the other
and that the sale dated March 1, 1983 was never categorically conditions, Atty. Plagata notarized Rosario’s affidavit. A new title
approved in the said order. was issued to the Fuentes spouses and the remaining payment
was given to Tarciano. Tarciano passed away, followed by his
wife.
COMPILED BY: WIGMORE #WIGMOREFOREVER 37
SALES Case Digest (Atty. Sarona)
Compiled by: Wigmore #wigmoreforever

Eight years later, in 1997, the children of Tarciano and Rosario of Spouses Parulan, who have been estranged from one
filed an action for annulment of sale and reconveyance on the another. Real estate broker Atanacio offered the property to
ground that it is void since Rosario did not give her consent to it Spouses Aggabao who upon Atanacio’s insistence prevailed
and her signature has been forged. The Fuentes spouses upon them, so that they and Atanacio met with Ma. Elena
presented Atty. Plagata who testified that he personally saw (Parulan’s wife) at the site of the property. During their meeting,
Rosario sign the document although he admitted otarizing it only Spouses Aggabao paid Ma. Elena earnest money amounting to
4 months after. Besides , the 4-year prescription period for P20,000 which she acknowledged with a handwritten receipt.
annulling sale due to fraud or forgery already lapsed. Then and there, they agreed on the terms of how the buyers will
pay the price of the property.
ISSUE:
1. Whether Rosario’s signature on the document of consent to Spouses Aggabao complied with all the terms with regard to the
her husband Tarciano’s sale of their conjugal land to payment of the properties, but when Ma. Elena already needed
Fuentes spouses were forged? Yes to turn over the owner’s duplicate copies for both lands, she was
2. Whether Rocas’ action for the declaration of nullity of that able to turn over only one (which was successfully transferred to
sale to the spouses already prescribed? No, an action to the name of spouses Aggabao). For the other one, she said that
annul a void contract is imprescriptible it is with a relative in HongKong but she promised to deliver it to
3. Whether only Rosario, the wife, whose consent was not the spouses in a week. Needless to say, she failed to do so and
had, could bring the action to annul that sale? No by doing their own verification, the spouses found out that said
copy of title was in the hands of Dionisio’s brother.
RULING:
1. Atty. Palagata admittedly falsified the jurat of the affidavit of The spouses met with Dionisio’s brother, Atty. Parulan, who told
consent. That jurat declared that Rosario swore to the document them that he is the one with the power to sell the property. He
signed in Zamboanga City in Jan 11, 1989 when, as he testified, demanded P800,000 for said property and gave the spouses
he swore she supposedly signed it about 4 months earlier at her several days to decide. When Atty. Parulan did not hear back
residence in Manila on Sept 15, 1988. from the spouses, he gave them a call, and was then informed
that they have already paid the full amount to Ma. Elena.
While a defective notarization will merely strip the document of
its public character and reduce it into a private instrument, that Subsequently, Dionisio, through Atty. Parulan, commenced an
falsified jurat, taken together with the marks of forgery in the action praying for the declaration of the nullity of the deed of
signature, dooms the document as proof of Rosario’s consent to absolute sale executed by Ma. Elena, and the cancellation of the
the sale of the land. That the Fuentes spouses honestly relied on title issued to the petitioners by virtue thereof.
the notarized affidavit as proof of Rosario’s consent does not
matter. The sale is still void without an authentic consent. ISSUE: Whether or not the sale of conjugal property made by
Ma. Elena, by presenting a special power of attorney to sell
2.) The law that applies to this case is the Family Code, not the (SPA) purportedly executed by respondent husband in her favor
Civil Code. Although Tarciano and Rosario got married in 1950, was validly made to the vendees
Tarciano sold the property in 1989, a few months after the Family
Code took effect. RULING: No, the Court ruled that the sale of conjugal property
without the consent of the husband was not merely voidable but
Art 124. FC: …In the event that one spouse is incapacitated or void; hence, it could not be ratified. Spouses Aggabao also
otherwise unable to participate in the administration of the cannot use the defense that they are buyers in good faith
conjugal properties, the other spouse may assume sole powers because they did not exercise the necessary prudence to inquire
of administration. These powers do not include the powers of into the wife’s authority to sell.
disposition or encumbrance which must have the authority of the
court or the written consent of the other spouse. In the absence The relevant part of Article 124 of the Family Code provides that:
of such authority or consent, the disposition or encumbrance xxx In the event that one spouse is incapacitated or
shall be void. otherwise unable to participate in the administration of the
conjugal properties, the other spouse may assume sole
Art 1410, Civil Code: The action or defense for the declaration of powers of administration. These powers do not include
the inexistence of a contract does not prescribe. disposition or encumbrance without authority of the court or
Hence, in case at bar, the passage of time did not erode the right the written consent of the other spouse. In the absence of
to bring such an action. such authority or consent, the disposition or encumbrance
shall be void. xxx
3.) No, the sale was void from the beginning. Consequently, the
land remained the property of Tarciano’s heirs, namely, the Spouses Aggabao also failed to substantiate their contention that
Rocas. Dionisio, while holding the administration over the property, had
delegated to his brother, Atty. Parulan, the administration of the
property, considering that they did not present in court the SPA
SPOUSES REX AND CONCEPCION AGGABAO versus granting to Atty. Parulan the authority for the administration.
DIONISIO Z. PARULAN, JR. and MA. ELENA PARULAN
G.R. No. 165803 Nonetheless, the Court would like to stress that the power of
administration does not include acts of disposition or
FACTS: Involved in this action are two parcels of land and their encumbrance, which are acts of strict ownership. As such, an
improvements in Parañaque City and registered under the name authority to dispose cannot proceed from an authority to
COMPILED BY: WIGMORE #WIGMOREFOREVER 38
SALES Case Digest (Atty. Sarona)
Compiled by: Wigmore #wigmoreforever

administer, and vice versa, for the two powers may only be contract void ab initio but merely voidable. Said provisions of law
exercised by an agent by following the provisions on agency of provide:
the Civil Code (from Article 1876 to Article 1878). Specifically,
the apparent authority of Atty. Parulan, being a special agency, Art. 166. Unless the wife has been declared a non compos
was limited to the sale of the property in question, and did not mentis or a spendthrift, or is under civil interdiction or is confined
include or extend to the power to administer the property. in a leprosarium, the husband cannot alienate or encumber any
real property of the conjugal property without the wife’s consent.
On the other hand, we agree with Dionisio that the void sale was If she refuses unreasonably to give her consent, the court may
a continuing offer from the petitioners and Ma. Elena that compel her to grant the same.
Dionisio had the option of accepting or rejecting before the offer
was withdrawn by either or both Ma. Elena and the petitioners.
The last sentence of the second paragraph of Article 124 of the ABALOS vs. MACATANGAY (September 30, 2004)
Family Code makes this clear, stating that in the absence of the
other spouse’s consent, the transaction should be construed as a FACTS: Spouses Arturo and Esther Abalos are the registered
continuing offer on the part of the consenting spouse and the owners of a parcel of land with improvements.
third person, and may be perfected as a binding contract upon
the acceptance by the other spouse or upon authorization by the Armed with a Special Power of Attorney dated June 2, 1988,
court before the offer is withdrawn by either or both offerors. purportedly issued by his wife, Arturo executed a Receipt and
Memorandum of Agreement (RMOA) dated October 17, 1989, in
favor of respondent, binding himself to sell to respondent the
PELAYO VS PEREZ subject property and not to offer the same to any other party
JUNE 8, 2005 within thirty (30) days from date. Arturo acknowledged receipt of
a check from respondent in the amount of Five Thousand Pesos
FACTS: David Pelayo husband of Lorenza Pelayo executed on (P5,000.00), representing earnest money for the subject
Jan. 11, 1988 conveyed to Melki Perez two parcels of agricultural property, the amount of which would be deducted from the
rd
land. Lorenza howeer signed only on the 3 page in the space purchase price of One Million Three Hundred Three Hundred
provided for the witness on which, Perez failed to register the Thousand Pesos (P1,300,000.00). Further, the RMOA stated
deed on the registry of deeds. Perez there upon filed a case that full payment would be effected as soon as possession of the
asking specific performance of the spouse. Spouses Pelayo property shall have been turned over to respondent.
responded that the cause of action of Perez is unenforceable
pursuant to RA 6656 which provides in Sec. 6 that contracts Subsequently, Arturo’s wife, Esther, executed a Special Power of
executed prior to its effectivity shall “ be valid only when Attorney dated October 25, 1989, appointing her sister,
registered with the Registry of Deeds within a period of 3 Bernadette Ramos, to act for and in her behalf relative to the
months. Being that Perez was unable to do the same then there transfer of the property to respondent. Ostensibly, a marital
should not be any cause of action. They also said that the said squabble was brewing between Arturo and Esther at the time
contract of sale was only to simulate a sale just so they can and to protect his interest, respondent caused the annotation of
intimidate the illegal occupants of the land since Perez is feared his adverse claim on the title of the spouses to the property on
by many. However, Perez replied that the lot was given to him by November 14, 1989.
spouses in consideration of his services as his attorney-in-fact to
make necessary representation. David Pelayo claimed as well Respondent sent a letter to Arturo and Esther informing them of
that the contract of sale was without his wife’s consent. RTC his readiness and willingness to pay the full amount of the
rendered the deed of sale null and void on the account that there purchase price. The letter contained a demand upon the spouses
was no consent by the wife and that Perez did not possess nor to comply with their obligation to turn over possession of the
pay taxes on the lots and that defendant Pelayo was indedted to property to him. On the same date, Esther, through her attorney-
Perez for services rendered. in-fact, executed in favor of respondent, a Contract to Sell the
property to the extent of her conjugal interest therein.
ISSUE: Whether or not the deed of sale was null and void on the
ground for lack of marital consent. He reiterated his demand upon them to comply with their
obligation to turn over possession of the property. Arturo and
RULING: Petitioners not having questioned the Decision of the Esther failed to deliver the property which prompted respondent
CA dated November 24, 1994 which then attained finality, the to cause the respondent to file a complaint for specific
ruling that the deed of sale subject of this case is not among the performance with damages against petitioners.
transactions deemed as invalid under R.A. No. 6657, is now
immutable. The court declared that the RMOA is a contract to sell because it
signifies a unilateral offer of Arturo to sell the property to
We agree with the CA ruling that petitioner Lorenza, by affixing respondent for a price certain within a period of thirty days. The
her signature to the Deed of Sale on the space provided for RMOA does not impose upon respondent an obligation to buy
witnesses, is deemed to have given her implied consent to the petitioner’s property, as in fact it does not even bear his
contract of sale. signature thereon. It is quite clear that after the lapse of the
thirty-day period, without respondent having exercised his option,
Moreover, under Article 173, in relation to Article 166, both of the Arturo is free to sell the property to another. This shows that the
New Civil Code, which was still in effect on January 11, 1988 intent of Arturo is merely to grant respondent the privilege to buy
when the deed in question was executed, the lack of marital the property within the period therein stated. There is nothing in
consent to the disposition of conjugal property does not make the the RMOA which indicates that Arturo agreed therein to transfer
COMPILED BY: WIGMORE #WIGMOREFOREVER 39
SALES Case Digest (Atty. Sarona)
Compiled by: Wigmore #wigmoreforever

ownership of the land which is an essential element in a contract husband or the wife to one-half of the conjugal assets does not
of sale. vest until the liquidation of the conjugal partnership.

ISSUE: Can the sale be declared valid based on the RMOA? prior to the liquidation of the conjugal partnership, the interest of
each spouse in the conjugal assets is inchoate, a mere
HELD: No. The sale would is not valid. Granting for the sake of expectancy. The right of the husband or wife to one-half of the
argument that the RMOA is a contract of sale, the same would conjugal assets does not vest until the dissolution and liquidation
still be void. Quite glaring is the absence of the signature of of the conjugal partnership, or after dissolution of the marriage,
Esther in the RMOA, which proves that she did not give her when it is finally determined that, after settlement of conjugal
consent to the transaction initiated by Arturo. The husband obligations, there are net assets left which can be divided
cannot alienate any real property of the conjugal partnership between the spouses or their respective heirs.
without the wife’s consent.

However, it was the Contract to Sell executed by Esther through CALIMLIM- CANULLAS vs. FORTUN (June 22, 1984)
her attorney-in-fact which the Court of Appeals made full use of.
Holding that the contract is valid, the appellate court explained FACTS: Petitioner MERCEDES Calimlim-Canullas and
that while Esther did not authorize Arturo to sell the property, her FERNANDO Canullas were married on December 19, 1962.
execution of the SPA authorizing her sister to sell the land to They begot five children. They lived in a small house on the
respondent clearly shows her intention to convey her interest in residential land in question located at Bacabac, Bugallon,
favor of respondent. In effect, the court declared that the lack of Pangasinan. After FERNANDO's father died in 1965,
Esther’s consent to the sale made by Arturo was cured by her FERNANDO inherited the land.
subsequent conveyance of her interest in the property through
her attorney-in-fact. In 1978, FERNANDO abandoned his family and was living with
private respondent Corazon DAGUINES. During the pendency of
This ruling is erroneous. this appeal, they were convicted of concubinage.

The nullity of the RMOA as a contract of sale emanates not only On April 15, 1980, FERNANDO sold the subject property with the
from lack of Esther’s consent thereto but also from want of house thereon to DAGUINES for the sum of P2,000.00. In the
consideration and absence of respondent’s signature thereon. document of sale, FERNANDO described the house as "also
Such nullity cannot be obliterated by Esther’s subsequent inherited by me from my deceased parents."
confirmation of the putative transaction as expressed in the
Contract to Sell. Under the law, a void contract cannot be Unable to take possession of the lot and house, DAGUINES
ratified and the action or defense for the declaration of the initiated a complaint on June 19, 1980 for quieting of title and
inexistence of a contract does not prescribe. A void contract damages against MERCEDES. The latter resisted and claimed
produces no effect either against or in favor of anyone–it cannot that the house in dispute where she and her children were
create, modify or extinguish the juridical relation to which it residing, including the coconut trees on the land, were built and
refers. planted with conjugal funds and through her industry; that the
sale of the land together with the house and improvements to
True, in the Contract to Sell, Esther made reference to the earlier DAGUINES was null and void because they are conjugal
RMOA executed by Arturo in favor of respondent. However, the properties and she had not given her consent to the sale.
RMOA which Arturo signed is different from the deed which
Esther executed through her attorney-in-fact. For one, the first is ISSUE: whether or not the sale of the lot together with the house
sought to be enforced as a contract of sale while the second is and improvements thereon was valid.
purportedly a contract to sell only. For another, the terms and
conditions as to the issuance of title and delivery of possession HELD: No. Not Valid. Second paragraph of Article 158 of the
are divergent. Civil Code, provides:
xxx xxx xxx
The congruence of the wills of the spouses is essential for the Buildings constructed at the expense of the partnership during
valid disposition of conjugal property. Where the conveyance is the marriage on land belonging to one of the spouses also
contained in the same document which bears the conformity of pertain to the partnership, but the value of the land shall be
both husband and wife, there could be no question on the validity reimbursed to the spouse who owns the same.
of the transaction. But when there are two (2) documents on
which the signatures of the spouses separately appear, textual We hold that pursuant to the foregoing provision both the land
concordance of the documents is indispensable. Hence, in this and the building belong to the conjugal partnership but the
case where the wife’s putative consent to the sale of conjugal conjugal partnership is indebted to the husband for the value of
property appears in a separate document which does not, the land. The spouse owning the lot becomes a creditor of the
however, contain the same terms and conditions as in the first conjugal partnership for the value of the lot, which value would
document signed by the husband, a valid transaction could not be reimbursed at the liquidation of the conjugal partnership.
have arisen.
The foregoing premises considered, it follows that FERNANDO
Even on the supposition that the parties only disposed of their could not have alienated the house and lot to DAGUINES since
4
respective shares in the property, the sale, assuming that it MERCEDES had not given her consent to said sale.
exists, is still void for as previously stated, the right of the

COMPILED BY: WIGMORE #WIGMOREFOREVER 40


SALES Case Digest (Atty. Sarona)
Compiled by: Wigmore #wigmoreforever

Further, we find that the contract of sale was null and void for approval of the sale she represented the price to be the best
being contrary to morals and public policy. The sale was made obtainable in the market, she was not entirely truthful. This is one
by a husband in favor of a concubine after he had abandoned his phase to consider.
family and left the conjugal home where his wife and children
lived and from whence they derived their support. That sale was Again, supposing she knew the parcels were actually worth
subversive of the stability of the family, a basic social institution P17,000 then she agreed to sell them to Dr. Ramos at P14,700
which public policy cherishes and protects. and knowing the realty’s value she offered him the next day
P15,000 or P15,500, and got it. Will there be any doubt that she
was recreant to her guardianship, and that her acquisition should
3) Others Relatively Disqualified (Article 1491) be nullified? Even without proof that she had connived with Dr.
Ramos. Remembering the general doctrine that guardianship is
a. Guardians, Agents and Administrators a trust of the highest order, and the trustee cannot be allowed to
have any inducement to neglect his ward’s interest and in line
with the court’s suspicion whenever the guardian acquires the
ward’s property 1 we have no hesitation to declare that in this
THE PHILIPPINE TRUST COMPANY vs. ROLDAN (May 31,
case, in the eyes of the law, Socorro Roldan took by purchase
1956)
her ward’s parcels thru Dr. Ramos, and that Article 1459 of the
Civil Code applies.
FACTS: These 17 parcels located in Guiguinto, Bulacan, were
part of the properties inherited by Mariano L. Bernardo from his
She acted it may be true without malice there may have been no
father, Marcelo Bernardo, deceased. In view of his minority, previous agreement between her and Dr. Ramos to the effect
guardianship proceedings were instituted, wherein Socorro that the latter would buy the lands for her. But the stubborn fact
Roldan was appointed his guardian.
remains that she acquired her protege’s properties, through her
brother-in-law. That she planned to get them for herself at the
Socorro Roldan filed in said guardianship proceedings a motion time of selling them to Dr. Ramos, may be deduced from the very
asking for authority to sell as guardian the 17 parcels for the sum short time between the two sales (one week). The temptation
of P14,700 to Dr. Fidel C. Ramos, the purpose of the sale being
which naturally besets a guardian so circumstanced,
allegedly to invest the money in a residential house, which the
necessitates the annulment of the transaction, even if no actual
minor desired to have on Tindalo Street, Manila. The motion was collusion is proved (so hard to prove) between such guardian
granted. and the intermediate purchaser. This would uphold a sound
principle of equity and justice.
As guardian, Roldan executed the proper deed of sale in favor of
her brother-in-law Dr. Fidel C. Ramos, and on August 12, 1947 Hence, from both the legal and equitable standpoints these three
she asked for, and obtained, judicial confirmation of the sale. On sales should not be sustained the first two for violation of article
August 13, 1947, Dr. Fidel C. Ramos executed in favor of
1459 of the Civil Code and the third because Socorro Roldan
Socorro Roldan, personally, a deed of conveyance covering the
could pass no title to the third buyer.
same seventeen parcels, for the sum of P15,000. And on
October 21, 1947 Socorro Roldan sold four parcels out of the
seventeen to another party, reserving to herself the right to DISTAJO vs. COURT OF APPEALS (August 25, 2000)
repurchase.
FACTS: During the lifetime of Iluminada Abiertas, she
The Philippine Trust Company replaced Socorro Roldan as designated one of her sons, Rufo Distajo, to be the administrator
guardian and seeks to undo what the previous guardian had
of her parcels of land denoted as Lot Nos. 1018, 1046, 1047, and
done. The step-mother in effect, sold to herself, the properties of
1057.
her ward, contends the Plaintiff, and the sale should be annulled
because it violates Article 1459 of the Civil Code prohibiting the Iluminada Abiertas certified to the sale of Lot Nos. 1018, 1046
guardian from purchasing “either in person or through the
and 1047 in favor of Rufo Distajo and other parcels of land to her
mediation of another” the property of her ward.
other kins.
ISSUE: Whether or not the sale should be annulled.
After purchasing the above-mentioned parcels of land, Rufo
Distajo, together with his wife, Lagrimas, took possession of the
HELD: At first glance the resolutions of both courts accomplished
property and paid the corresponding real estate taxes thereon.
substantial justice the minor recovers his properties. But if the
conveyances are annulled as prayed for, the minor will obtain a Consequently, on June 5, 1986, Ricardo Distajo, with the other
better deal he receives all the fruits of the lands from the year
heirs of Iluminada Abiertas, filed a complaint for recovery of
1947 (Article 1303 Civil Code) and will return P14,700, not possession and ownership of those lands sold alleging that Rufo
P15,000. Distajo cannot acquire the subject parcels of land owned by
Iluminada Abiertas because the Civil Code prohibits the
To our minds the first two transactions herein described couldn’t
administrator from acquiring properties under his
be in a better juridical situation than if this guardian had administration. Rufo Distajo merely employed fraudulent
purchased the seventeen parcels on the day following the sale to machinations in order to obtain the consent of his mother to the
Dr. Ramos. Now, if she was willing to pay P15,000 why did she sale, and may have even forged her signature on the deeds of
sell the parcels for less? In one day (or actually one week) the
sale of the parcels of land.
price could not have risen so suddenly. Obviously when, seeking
COMPILED BY: WIGMORE #WIGMOREFOREVER 41
SALES Case Digest (Atty. Sarona)
Compiled by: Wigmore #wigmoreforever

ISSUE: Whether or not the sale transactions are void for having Don Mariano in favor of Antonio Cui on March 2,1946, wherein
been entered into by the administrator of the properties.1âwphi1 the former has constituted the latter as his "true and lawful
attorney" to perform in his name and that of the intestate heirs of
HELD: The sale is valid. Doña Antonia Perales.

The pertinent Civil Code provision provides: ISSUE: WON the sale of the property to Antonio was valid.
"Art. 1491. The following persons cannot acquire by purchase,
even at a public or judicial auction, either in person or through HELD: YES. While under article 1459 of the old Civil Code an
the mediation of another: agent or administrator is disqualified from purchasing property in
xxx his hands for sale or management, and, in this case, the property
(2) Agents, the property whose administration or sale in question was sold to Antonio Cui while he was already the
may have been entrusted to them, unless the consent agent or administrator of the properties of Don Mariano Cui, we
of the principal has been given; however believe that this question cannot now be raised
xxx or invoked.

Under the above article, the prohibition against agents The prohibition of the law is contained in article 1459 of the
purchasing property in their hands for sale or management is not old Civil Code, but this prohibition has already been
absolute. It does not apply if the principal consents to the sale of removed.
the property in the hands of the agent or administrator. In this
case, the deeds of sale signed by Iluminada Abiertas shows that Under the provisions of article 1491, section 2, of the new Civil
she gave consent to the sale of the properties in favor of her son, Code, an agent may now buy property placed in his hands for
Rufo, who was the administrator of the properties. Thus, the sale or administration, provided that the principal gives his
consent of the principal Iluminada Abiertas removes the consent thereto. While the new Code came into effect only on
transaction out of the prohibition contained in Article 1491(2). August 30, 1950, however, since this is a right that is declared for
the first time, the same may be given retroactive effect if no
Petitioner also alleges that Rufo Distajo employed fraudulent vested or acquired right is impaired (Article 2253, new Civil
machinations to obtain the consent of Iluminada Abiertas to the Code). During the lifetime Don Mariano,and particularly on March
sale of the parcels of land. However, petitioner failed to adduce 8, 1946, the herein appellants could not claim any vested or
convincing evidence to substantiate his allegations. acquired right in these properties, for,as heirs, the most they had
was a mere expentancy. We may, therefore, invoke now this
practical and liberal provision of our new Civil Code even if the
JESUS MA. CUI, ET AL., Plaintiff-Appellant, v. ANTONIO MA. sale had taken place before its effectivity.
CUI, ET AL., Defendants-Appellees.
[G.R. No. L-7041. February 21, 1957.]
b. Attorneys
FACTS: Jesus and Antonio are the legitimate children of Don
Mariano Cui and Doña Antonia Perales who died intestate THE DIRECTOR OF LANDS, petitioner, vs. SILVERETRA
in1939. Jesus alleged that during the marriage of Don Mariano ABABA, ET AL., claimants, JUAN LARRAZABAL, MARTA C.
and Dona Antonia, their parents acquired certain properties in DE LARRAZABAL, MAXIMO ABAROQUEZ and ANASTACIA
the City of Cebu, namely, Lots Nos. 2312, 2313 and 2319. Upon CABIGAS, petitioners-appellants, ALBERTO FERNANDEZ,
the death of their mother, the properties were placed under the adverse claimant-appellee.
administration of their dad.that while the latter was 84 years of G.R. No. L-26096 February 27, 1979
age, Antonio by means of deceit, secured the transfer to
themselves the said lots without any pecuniary consideration; FACTS: The adverse claimant Atty. Fernandez was retained as
that in the deed of sale executed on March 8, 1946, Rosario Cui counsel by petitioner (Abarquez) in a civil a case for the
appeared as one of the vendees, but on learning of this fact she annulment of a contract of sale with right of repurchase and for
subsequently renounced her rights under the sale and returned the recovery of the land which was the subject matter thereof.
her portion to Don Mariano Cui by executing a deed of resale in Unable to compensate his lawyer whom he also retained for his
his favor on October 11, 1946; that defendants, fraudulently and appeal, the petitioner executed a document whereby he obliged
with the desire of enriching themselves unjustly at the expense himself to give to his lawyer ½ of whatever he might recover from
of their father, Don Mariano Cui, and of their brothers and co- Lots 5600 and 5602 should the appeal prosper.
heirs,secured a loan of P130,000 from the Rehabilitation
properties, and with the loan thus obtained, defendants The real property sought to be recovered was actually the share
constructed thereon an apartment building of strong materials of petitioner in Lots 5600 and 5602 which were part of the estate
consisting of 14 doors, valued at approximately P130,000 and of his deceased parents and which were partitioned among the
another building on the same parcels of land, which buildings heirs, which included petitioner and his sister.
were leased to some Chinese commercial firms a monthly rental
of P7,600, which defendants have collected and will continue to The case having been resolved and title having been issued to
collect to the prejudice of the plaintiffs;Jesus alleged that the sale petitioner, adverse claimant waited for petitioner to comply with
should be invalidated so far as the portion of the property sold his obligation under the document executed by him by delivering
to Antonio Cui is concerned, for the reason that when that sale the ½ portion of the said parcels of land. Petitioner refused to
was effected, Antonio was then acting as the agent or comply with his obligation and instead offered to sell the whole
administrator of the properties of Don Mariano Cui.Jesus lays parcels of land to spouses Larrazabal. Then, adverse claimant
stress on the power of attorney Exhibit L which was executed by
COMPILED BY: WIGMORE #WIGMOREFOREVER 42
SALES Case Digest (Atty. Sarona)
Compiled by: Wigmore #wigmoreforever

immediately took steps to protect his interest by filing a motion to assisted by Atty. Arsenio Fer. Cabanting, filed a complaint
annotate his attorney’s lien and by notifying the prospective against Paulino for the recovery of possession with damages.
buyers of his claim over the ½ portion of the parcels of land. The Valencias engaged the services of Atty. Dionisio Antiniw.
Atty. Antiniw advised them to present a notarized deed of sale in
The motion was granted. The annotation of adverse claim lieu of the private document written in Ilocano. For this purpose,
appeared on the new transfer certificate of title. This adverse Paulino gave Atty. Antiniw an amount of P200.00 to pay the
claim became the subject of cancellation proceedings filed by person who would falsify the signature of the alleged vendor. A
petitioner-spouses. The trial court resolved the case in favor of "Compraventa Definitiva" as a result thereof.
the adverse claimant. On appeal, petitioners contended that a
contract for a contingent fee violates Article 1491 because it The Court of First Instance of Pangasinan, rendered a decision
involves an assignment of a property subject of litigation. in favor of Serapia. Paulino filed a Petition for Certiorari with
Preliminary Injunction before the CA. While the petition was
ISSUE: WON the contract for a contingent fee as basis of the pending, the TC issued an order of execution stating that "the
interest of Atty. Fernandez is prohibited by Article 1491 of the decision in this case has already become final and executory".
Civil Code.

HELD: NO. The contention is without merit. Article 1491 prohibits On March 20, 1973, Serapia sold 40 square meters of the
only the sale or assignment between the lawyer and his client of litigated lot to Atty. Jovellanos and the remaining portion she sold
property which is the subject of litigation. For the prohibition to to her counsel, Atty. Arsenio Fer. Cabanting, on April 25, 1973.
operate, the sale or assignment of the property must take Paulino filed a disbarment proceeding against Atty. Cabanting on
place during the pendency of the litigation involving the the ground that said counsel allegedly violated Article 1491 of the
property. New Civil Code as well as Article II of the Canons of Professional
Ethics, prohibiting the purchase of property under litigation by a
Likewise, under American Law, the prohibition does not apply to counsel. The appellate court dismissed the petition of Paulino.
“cases where after completion of litigation the lawyer accepts on
account of his fee and interest in the assets realized by the Constancia Valencia, daughter of Paulino, also filed a
litigation. There is clear distinction between such cases and one disbarment proceeding against Atty. Dionisio Antiniw for his
in which the lawyer speculates on the outcome of the matter in participation in the forgery and its subsequent introduction as
which he is employed. evidence for his client; and also, against Attys. Eduardo
Jovellanos and Arsenio Cabanting for purchasing a litigated
Further, a contract for a contingent fee is not covered by Article property allegedly in violation of Article 1491 of the New Civil
1491 because the transfer or assignment of the property in Code; and against the three lawyers, for allegedly rigging the
litigation takes effect only after the finality of a favorable case against her parents.
judgment. In the instant case, the attorney’s fees of Atty.
Fernandez, consisting of ½ of whatever the petitioner might ISSUES:
recover from his share in the lots in question is contingent upon
 Whether or not Atty. Cabanting purchased the subject
the success of the appeal. Hence, the payment of the attorney’s
property in violation of Art. 1491 of the New Civil Code.
fees, that is, the transfer or assignment of ½ of the property in
 Whether or not Attys. Antiniw and Jovellanos are guilty of
litigation will take place only if the appeal prospers. Therefore,
malpractice in falsifying notarial documents.
the transfer actually takes effect after the finality of a favorable
judgment rendered on appeal and not during the pendency of  Whether or not the three lawyers connived in rigging the case
litigation involving the property in question. Consequently, the against spouses Valencia.
contract for a contingent fee is not covered by Article 1491 of the
HELD: Under Article 1491 of the New Civil Code: The following
Civil Code.
persons cannot acquire by purchase, even at a public of judicial
auction, either in person or through the mediation of another: (5)
PAULINO VALENCIA vs. ATTY. ARSENIO FER CABANTING; . . . this prohibition includes the act of acquiring by assignment
CONSTANCIA L. VALENCIA vs. ATTY. DIONISIO C. ANTINIW, and shall apply to lawyers, with respect to the property and rights
ATTY. EDUARDO U. JOVELLANOS and ATTY. ARSENIO FER. which may be the object of any litigation in which they make take
CABANTING part by virtue of their profession. Public policy prohibits the
A.M. Nos. 1302, 1391 and 1543 April 26, 1991 transactions in view of the fiduciary relationship involved. It is
intended to curtail any undue influence of the lawyer upon his
client. Greed may get the better of the sentiments of loyalty and
FACTS: In 1933, complainant Paulino Valencia and his wife disinterestedness. Any violation of this prohibition would
allegedly bought a parcel of land, where they built their house, constitute malpractice and is a ground for suspension. Art. 1491,
from a certain Serapia Raymundo, an heir of Pedro Raymundo prohibiting the sale to the counsel concerned, applies only while
the original owner. However, they failed to register the sale or the litigation is pending.
secure a transfer certificate of title in their names. A conference
was held in the house of Atty. Eduardo Jovellanos to settle the In the case at bar, while it is true that Atty. Arsenio Fer.
land dispute between Serapia and the Valencia spouses. Cabanting purchased the lot after finality of judgment, there was
Serapia was willing to relinquish ownership if the Valencias could still a pending certiorari proceeding. A thing is said to be in
show documents evidencing ownership. Paulino exhibited a litigation not only if there is some contest or litigation over it in
deed of sale written in the Ilocano dialect. However, Serapia court, but also from the moment that it becomes subject to the
claimed that the deed covered a different property. Serapia, judicial action of the judge. Logic indicates,
COMPILED BY: WIGMORE #WIGMOREFOREVER 43
SALES Case Digest (Atty. Sarona)
Compiled by: Wigmore #wigmoreforever

in certiorari proceedings, that the appellate court may either property to Atty. Abecia a violation of the prohibition set forth in
grant or dismiss the petition. Hence, it is not safe to conclude, for Art. 1491 of the NCC.
purposes under Article 1491 that the litigation has terminated
when the judgment of the trial court become final while HELD: Evidence that Daroy has known of such transfer since
a certiorari connected therewith is still in progress. Thus, 1971: 1. In the report of the sheriff who placed them in the actual
purchase of the property by Atty. Cabanting in this case possession of the land he had acquired through action, the
constitutes malpractice in violation of Art. 1491 and the Canons sheriff referred to Nena Abecia as assignee of Daroy.
of Professional Ethics. Clearly, this malpractice is a ground for
suspension. The sale in favor of Atty. Jovellanos does not In a criminal action filed by the oppositors of Daroy in the
constitute malpractice. There was no attorney-client relationship ejectment case, Nena Abecia was impleaded as co-defendant on
between Serapia and Atty. Jovellanos, considering that the latter her capacity as assignee of Daroy. All these fact were never
did not take part as her counsel. The transaction is not covered rebutted by Daroy. The parties were mistaken in thinking that
by Art. 1491 nor by the Canons adverted to. respondent could not validly acquire the land. In Guevara v.
Calalang, on facts similar to those in this case, we held that the
II prohibition in Art. 1491 does not apply to the sale of a parcel
It is asserted by Paulino that Atty. Antiniw asked for and received of land, acquired by a client to satisfy a judgment in his
the sum of P200.00 in consideration of his executing the favor, to his attorney as long as the property was not the
document "Compraventa Definitiva". This charge, Atty. Antiniw subject of the litigation. For indeed, while judges, prosecuting
simply denied. It is settled jurisprudence that affirmative attorneys, and others connected with the administration of justice
testimony is given greater weight than negative testimony. When are prohibited from acquiring “property or rights in litigation or
an individual's integrity is challenged by evidence, it is not levied upon in execution,” the prohibition with respect to
enough that he deny the charges against him; he must meet the attorneys in the case extends only to “property and rights which
issue and overcome the evidence for the relator and show proofs may be the object of any litigation in which they may take part by
that he still maintains the highest degree of morality and integrity virtue of their profession.”
which at all time is expected of him. There is a clear
preponderant evidence that Atty. Antiniw committed falsification The point is, the parties in this case thought the transfer of the
of a deed of sale, and its subsequent introduction in court land to respondent Abecia was prohibited and so they contrived
prejudices his prime duty in the administration of justice as an a way whereby the land would be sold to Jose Gangay, whose
officer of the court. wife Anita is the sister of Mrs. Nena Abecia, and then Gangay
would sell the land to Mrs. Abecia. As Jose Gangay stated in his
III affidavit of March 6, 1985. The sale of the land to Gangay may
There is no evidence on record that the three lawyers involved in be fictitious and, therefore, void, but that complainant Regalado
these administrative cases conspired in executing the falsified Daroy intended to convey the land ultimately to respondent
"Compraventa Definitiva" and rigged the case against spouses Esteban Abecia appears to be the case.
Valencia.

Besides, the camaraderie among lawyers is not proof of FEDERICO N. RAMOS VS PATRICIO A. NGASEO
conspiracy, but a sign of brotherhood among them.
FACTS: Ramos went to Atty. Ngaseo to engage his services as
WHEREFORE, judgment is hereby rendered declaring: 1. counsel in a case involving a piece of land. After the Court of
Dionisio Antiniw DISBARRED from the practice of law, and his Appeals rendered a favorable judgment ordering the land to be
name is ordered stricken off from the roll of attorneys; 2. Arsenio returned to Ramos and his siblings (such decision having been
Fer. Cabanting SUSPENDED; and 3. Admin case against final and executor), Atty. Ngaseo sent a demand letter to Ramos
Attorney Eduardo Jovellanos DISMISSED. asking for the delivery of a piece of land which the complainant
allegedly promised as payment for respondent’s appearance fee.

REGALADO DAROY, complainant, vs. ATTY. ESTEBAN As a result, Ramos filed before the IBP a complaint charging Atty.
ABECIA, respondent. Ngaseo of violation of the CPR for demanding the delivery of a
[A.C. No. 3046. October 26, 1998] parcel of land, which was the subject of litigation. The IBP found
Atty. Ngaseo guilty. Atty. Ngaseo argues that he did not violate
FACTS: (1971) Atty. Abecia was the counsel of Daroy in an Article 1491 CC because when he demanded the delivery of the
ejectment case in which they won. To satisfy the judgement of piece of land, the case has been terminated, when the appellate
damages, one of the properties of their opposition was sold by court ordered the return of the land to the family of Ramos.
the sheriff at an auction to Daroy as the highest bidder. (1984)
Daroy filed a complaint against Atty. Abecia for falsification for ISSUE: Whether or not Atty. Ngaseo violated Art. 1491 CC.
allagedly forging his signature in order to transfer title to said
property first to Jose Gangay and subsequently to Atty. Abcia’s HELD: NO. Under Par. (5), Art. 1491 of the Civil Code, lawyers
wife, Nena Abecia. are prohibited from acquiring either by purchase or assignment
the property or rights involved which are the object of litigation in
He contended that he only knew of such transfer on that year. which they intervene by virtue of their public/judicial sales. The
Moreover, he filed a disbarment proceeding against Atty. Abecia article provides:
for unethical conduct.
Article 1491. The following persons cannot acquire by purchase,
ISSUE: WON the transfer was valid. WON the transfer of the even at a public or judicial action, either in person or through the
COMPILED BY: WIGMORE #WIGMOREFOREVER 44
SALES Case Digest (Atty. Sarona)
Compiled by: Wigmore #wigmoreforever

mediation of another: xxx xxx xxx 31, 1966, spouses Asuncion and spouses Galapon conveyed
their respective shares and interest in Lot 1184-E to "The Traders
(5) Justices, judges, prosecuting attorneys, clerks of superior and Manufacturing and Fishing Industries Inc." with Judge Asuncion
inferior courts, and other officers and employees connected with as the President and Mrs. Asuncion as the secretary.
the administration of justice, the property and rights in litigation or
levied upon an execution before the court within whose Macariola filed on August 9, 1968 the instant complaint dated
jurisdiction or territory they exercise their respective functions; August 6, 1968 alleging that respondent Judge Asuncion violated
this prohibition includes the act of acquiring by assignment and Article 1491, paragraph 5, of the New Civil Code in acquiring by
shall apply to lawyers, with respect to the property and rights purchase a portion of Lot No. 1184-E which was one of those
which may be the object of any litigation in which they may take properties involved in Civil Case No. 3010 decided by him.
part by virtue of their profession.
ISSUE: Whether or not the actuation of Judge Asuncion in
The prohibition in the aforesaid Article applies only to the sale or acquiring by purchase a portion of property in a Civil Case
assignment of the property which is the subject of litigation to the previously handled by him violated the prohibition under the Civil
persons disqualified therein. WE have already ruled that "... for Code?
the prohibition to operate, the sale or assignment of the property
must take place during the pendency of the litigation involving HELD: NO. There is no merit in the contention of Macariola that
the property." respondent Judge Elias B. Asuncion violated Article 1491,
paragraph 5, of the New Civil Code in acquiring by purchase a
Since such prohibition applies only if the sale or assignment of portion of Lot No. 1184-E which was one of those properties
the property takes place during the pendency of the litigation involved in Civil Case No. 3010. 'That Article provides:
involving the client’s property. Consequently, where the property
is acquired after the termination of the case, as in the instant Article 1491. The following persons cannot acquire by purchase,
case, no violation of paragraph 5, article 1491 of Civil Code even at a public or judicial action, either in person or through the
attatches. mediation of another: xxx xxx xxx

In the instant case, there was no actual acquisition of the (5) Justices, judges, prosecuting attorneys, clerks of superior and
property in litigation since the respondent only made a written inferior courts, and other officers and employees connected with
demand for its delivery, which the complainant refused to comply. the administration of justice, the property and rights in litigation or
Mere demand for delivery of the litigated property does not cause levied upon an execution before the court within whose
the transfer of ownership, hence, not a prohibited transaction jurisdiction or territory they exercise their respective functions;
within the contemplation of Article 1491. this prohibition includes the act of acquiring by assignment and
shall apply to lawyers, with respect to the property and rights
Note: (Rationale for prohibition: Public policy disallows the which may be the object of any litigation in which they may take
transactions in view of the fiduciary relationship involved) part by virtue of their profession.

The prohibition in the aforesaid Article applies only to the sale or


c. Judges assignment of the property which is the subject of litigation to the
persons disqualified therein. WE have already ruled that "... for
the prohibition to operate, the sale or assignment of the property
BERNARDITA R. MACARIOLA vs. HONORABLE ELIAS B. must take place during the pendency of the litigation involving
ASUNCION the property."

FACTS: Civil Case No. 3010 of the Court of First Instance of In the case at bar, when the respondent Judge purchased on
Leyte was a complaint for partition filed by Sinforosa R. Bales, March 6, 1965 a portion of Lot 1184-E, the decision in Civil Case
Luz R. Bakunawa, Anacorita Reyes, Ruperto Reyes, Adela No. 3010 which he rendered on June 8, 1963 was already final
Reyes, and Priscilla Reyes, plaintiffs, against Bernardita R. because none of the parties therein filed an appeal within the
Macariola, defendant, concerning the properties left by the reglementary period; hence, the lot in question was no longer
deceased Francisco Reyes, the common father of the plaintiff subject of the litigation. Moreover, at the time of the sale on
and defendant. March 6, 1965, respondent's order dated October 23, 1963 and
the amended order dated November 11, 1963 approving the
On June 8, 1963, a decision was rendered by respondent Judge October 16, 1963 project of partition made pursuant to the June
Asuncion in Civil Case 3010 which became final for lack of an 8, 1963 decision, had long become final for there was no appeal
appeal, and on October 16, 1963, a project of partition was from said orders.
submitted to Judge Asuncion.
Furthermore, respondent Judge did not buy the lot in question on
One of the properties mentioned in the project of partition was March 6, 1965 directly from the plaintiffs in Civil Case No. 3010
Lot 1184 and when the project of partition was approved by the but from Dr. Arcadio Galapon who earlier purchased on July 31,
trial court the adjudicatees caused Lot 1184 to be subdivided into 1964 Lot 1184-E from three of the plaintiffs, namely, Priscilla
five lots denominated as Lot 1184-A to 1184-E. Lot 1184-E was Reyes, Adela Reyes, and Luz R. Bakunawa after the finality of
sold on July 31, 1964 to Dr. Arcadio Galapon. On March 6, 1965, the decision in Civil Case No. 3010. It may be recalled that Lot
Dr. Arcadio Galapon and his wife Sold a portion of Lot 1184-E to 1184 or more specifically one-half thereof was adjudicated in
Judge Asuncion and his wife, Victoria S. Asuncion. On August equal shares to Priscilla Reyes,

COMPILED BY: WIGMORE #WIGMOREFOREVER 45


SALES Case Digest (Atty. Sarona)
Compiled by: Wigmore #wigmoreforever

Adela Reyes, Luz Bakunawa, Ruperto Reyes and Anacorita Public auction sale was executed and Gardner appeared to be
Reyes in the project of partition, and the same was subdivided the highest bidder and was the purchaser of Candida Acabo's
into five lots denominated as Lot 1184-A to 1184-E. As lands and carabaos sold at public auction held in the barrio of
aforestated, Lot 1184-E was sold on July 31, 1964 to Dr. Martelo, Municipality of Tayasan on March 20, 1907.
Galapon for which he was issued TCT No. 2338 by the Register
of Deeds of Tacloban City, and on March 6, 1965 he sold a As Gardner subsequently learned that he was forbidden to
portion of said lot to respondent Judge and his wife who declared purchase, he sold what he had purchased to Faustino Abad,
the same for taxation purposes only. The subsequent sale on Candida Acabo's son.
August 31, 1966 by spouses Asuncion and spouses Galapon of
their respective shares and interest in said Lot 1184-E to the On June 19, 1907, Faustino Abad, for the sum of P375 sold to
Traders Manufacturing and Fishing Industries, Inc., in which Silvino Pabinguit six parcels of land.
respondent was the president and his wife was the secretary,
took place long after thefinality of the decision in Civil Case No. The Court of First Instance of Oriental NEgros rendered
3010 and of the subsequent two aforesaid orders therein judgment in behalf of the plaintiff, Gan Tingco, declaring him the
approving the project of partition. owner of the lands described in the complaint, and ordered the
defendant, Silvino Pabinguit, to restore the plaintiff to their
The fact remains that respondent Judge purchased on March 6, possession.
1965 a portion of Lot 1184-E from Dr. Arcadio Galapon; hence,
after the finality of the decision which he rendered on June 8, The defendant appealed, with the right to a review of the
1963 in Civil Case No. 3010 and his two questioned orders dated evidence.
October 23, 1963 and November 11, 1963. Therefore, the
property was no longer subject of litigation. The appeal was heard by this court, it having been brought it by
bill of exceptions.
Consequently, the sale of a portion of Lot 1184-E to respondent
Judge having taken place over one year after the finality of the The appellant alleges that the trial court erred in holding that,
decision in Civil Case No. 3010 as well as the two orders notwithstanding the sale of the lands in question at public
approving the project of partition, and not during the pendency of auction. Candida Acabo did not cease to be the owner of these
the litigation, there was no violation of paragraph 5, Article 1491 properties, because there were certain irregularities and defects
of the New Civil Code. in the said auction.

Finally, while it is true that respondent Judge did not violate ISSUE: WON Candida Acabo did not cease to be the owner of
paragraph 5, Article 1491 of the New Civil Code in acquiring by the properties despite certain irregularities and defects in the
purchase a portion of Lot 1184-E which was in litigation in his said auction.
court, it was, however, improper for him to have acquired the
same. HELD: The trial court was impressed by the circumstance that in
the public auction the purchaser was the justice of the peace
In conclusion, while respondent Judge Asuncion, now Associate himself. This, in the judge's opinion, was unauthorized, because
Justice of the Court of Appeals, did not violate any law in article 1459, No. 5, of the Civil Code, prohibits judges from
acquiring by purchase a parcel of land which was in litigation in acquring by purchase, even at pub;ic or judicial sale, either in
his court and in engaging in business by joining a private person or by an agent, any property or rights litigated in the court
corporation during his incumbency as judge of the Court of First in the jurisdiction or territory within which they exercise their
Instance of Leyte, he should be reminded to be more discreet in respective duties; this prohibition includes taking of property by
his private and business activities, because his conduct as a assignment.
member of the Judiciary must not only be characterized with
propriety but must always be above suspicion. The appellant alleges that the property purchased by justice of
the peace Gardner was not the subject of litigation in the justice
court; that the action was to recover a certain sum of money, and
GAN TINGCO vs PABINGUIT that he had ordered the property sold on execution.

FACTS: Candida Acabo was the owner of six parcels of land, all This raises, therefore, a question as to the true meaning of
situated in the municipality of Jimalalud, Oriental Negros. These paragraph 5 of article 1459 of the Civil Code.
lands were sold on June 12, 1911 by Candida Acabo, to one Gan
Tingco. But the purchaser Gan Tingco was unable to take The Ley de Bases, in accordance with which the Civil Code was
possession of the six parcels of land sold him by Acabo, for they enacted, provides as follows, in Base No. 26:
were in the possession of Silvino Pabinguit, who alleges certain
rights therein. He claims to have purchased them from Faustino The forms, requirements and conditions of each particular
Abad. Abad had become the owner through purchase from contract shall be determined and defined subject to the general
Henry Gardner. list of obligations and their effects, with the understanding that
the legislation in force and the legal principles evolved therefrom
Prior to the purchase made by Garnder, a judgment has been by judicial decisions, etc., etc., shall serve as basis.
rendered against Ancabo as a result of the complaint filed by
Silvestre Basaltos. Because of Ancabo‘s failure to comply, her One of the bodies of law which conastitute the legislation now in
fixtures and other chattels were levied upon the order of Gardner force in the Novisima Recopilacion. In Law 4, Title 14, Book 5 of
being the justice of peace. the same is found the following provision: "We order that in
COMPILED BY: WIGMORE #WIGMOREFOREVER 46
SALES Case Digest (Atty. Sarona)
Compiled by: Wigmore #wigmoreforever

public auctions held by direction of our alcaldes, neither the latter money and to return the sugar cane to the plaintiff.
nor any person whomsoever in their name shall bid in anything
sold at such public auctions." The word alcaldes means judges. Plaintiff prayed that a writ of preliminary injunction be issued
The caption of Title 14 is " Alcaldes or Provincial Judges," and against the defendant Emiliano J. Valdez his attorneys and
the entire title deals with the exercise of judicial jurisdiction. Prior agents. The preliminary injunction was granted The defendant
to the enactment of the Civil Code, the Penal Code was also in Emiliano J. Valdez, in his amended answer, denied generally and
force. Article 400 of the latter prohibits, under penalty, any judge specifically each and every allegation of the complaint and step
from taking part, either directly, or indirectly, in any operation of up the following defenses:
exchange, trade or porfit with respect to things not the product of
his own property, within the territory over which he exercises (a) That the sugar cane in question had the nature of personal
jurisdiction. Judging from the legal precedents on which the Civil property and was not, therefore, subject to redemption; The
Code is based, it would not seem too much to conclude that the defendant Emiliano J. Valdez filed a counter-claim.
said article of the Civil Code does not make any distinction
between property in litigation. In effect, it appears to be as ISSUE: Whether or not the sugar cane in question had the
delicate a matter for a judge to take part in the sale of property nature of personal property.
that had been the subject of ligitgation in his court, as to
intervene in auction of property which, though not directly HELD: The sugar cane in question had the nature of personal
litigated in his court, is nevertheless levied upon and sold as the property.
result of a writ of execution issued by him. What the law intends
to avoid is the improper interference with an interest of a judge in CIVIL CODE, JURISPRUDENCE
a thing levied upon and sold by his order.
The first question raised by the appeal is, whether the sugar
If under the law Gardner was prohibited from acquiring the cane in question is personal or real property. It is contended that
ownership of Acabo's lands, then he could not have transmitted sugar cane comes under the classification of real property as
to Faustino Abad the right of ownership that he did not possess; "ungathered products" in paragraph 2 of article 334 of the Civil
nor could Abad, to whom this alleged ownership had not been Code. Said paragraph 2 of article 334 enumerates as real
transmitte, have conveyed the same to Pabinguit. What Gardner property the following: Trees, plants, and ungathered products,
should have done in view of the fact that the sale, as he finally while they are annexed to the land or form an integral part of any
acknowledged, was void, was to claim the price that had been immovable property." That article, however, has received in
deposited in court, and the justice of the peace of Guijulngan recent years an interpretation by the Tribunal Supremo de
should have declared the auction void and haveordered a new España, which holds that, under certain conditions, growing
sale to be held, besides correcting the errors that had been crops may be considered as personal property. (Decision of
committed in the proceedings. To the reasons already stated, March 18, 1904, vol. 97, Civil Jurisprudence of Spain.)
there is to be added the additional one, with respect to the sale
made by Faustino Abad to Silvino Pabinguit, that Abad was a However, from the discussion of Manresa it appears (1) that,
minor at the time - a circumstance that deprived him of capacity under Spanish authorities, pending fruits and ungathered
to sell (Civil Code,art. 1263). Abad had no ownership to transmit products may be sold and transferred as personal property; (2)
to anyone and, besides, he had no personality to enable him to that the Supreme Court of Spain, in a case of ejectment of a
contract by himself, on account of his lack of legal age. This lessee of an agricultural land, held that the lessee was entitled to
court finds no reason whatever why it should not affirm the gather the products corresponding to the agricultural year,
judgment appealed from. because said fruits did not go with the land but belonged
separately to the lessee; and (3) that under the Spanish
Mortgage Law of 1909, as amended, the mortgage of a piece of
2. SUBJECT MATTER land does not include the fruits and products existing thereon,
unless the contract expressly provides otherwise.
4. Existing, Future and Contingent
An examination of the decisions of the Supreme Court of
Louisiana may give us some light on the question which we are
discussing. Article 465 of the Civil Code of Louisiana, which
LEON SIBAL vs. EMILIANO J. VALDEZ
corresponds to paragraph 2 of article 334 of our Civil Code,
FACTS: As a first cause of action the plaintiff alleged that the provides:
defendant Vitaliano Mamawal, deputy sheriff of the Province of
"Standing crops and the fruits of trees not gathered, and trees
Tarlac, by virtue of a writ of execution issued by the Court of First
before they are cut down, are likewise immovable, and are
Instance of Pampanga, attached and sold to the defendant
considered as part of the land to which they are attached."
Emiliano J. Valdez the sugar cane planted by the plaintiff and his
tenants on seven parcels of land described in the complaint in
The Supreme Court of Louisiana having occasion to interpret
the third paragraph of the first cause of action; that within one
that provision, held that in some cases "standing crops" may be
year from the date of the attachment and sale the plaintiff offered
considered and dealt with as personal property. In the case of
to redeem said sugar cane and tendered to the defendant Valdez
Lumber Co. vs. Sheriff and Tax Collector (106 La., 418) the
the amount sufficient to cover the price paid by the latter, the
Supreme Court said:
interest thereon and any assessments or taxes which he may
have paid thereon after the purchase, and the interest
"True, by article 465 of the Civil Code it is provided that 'standing
corresponding thereto and that Valdez refused to accept the
crops and the fruits of trees not gathered and trees before they
COMPILED BY: WIGMORE #WIGMOREFOREVER 47
SALES Case Digest (Atty. Sarona)
Compiled by: Wigmore #wigmoreforever

are cut down . . . are considered as part of the land to which they mortgage may contain an agreement stipulating that the
are attached, but the immovability provided for is only one in mortgagor binds himself properly to tend, care for and protect the
abstracto and without reference to rights on or to the crop crop while growing.
acquired by others than the owners of the property to which the
crop is attached. . . . It is clear from the foregoing provisions that Act No. 1508 was
enacted on the assumption that "growing crops" are personal
The existence of a right on the growing crop is a mobilization by property. This consideration tends to support the conclusion
anticipation, a gathering as it were in advance, rendering the hereinbefore stated, that paragraph 2 of article 334 of the Civil
crop movable quoad the right acquired therein. Our Code has been modified by section 450 of Act No. 190 and by
jurisprudence recognizes the possible mobilization of the growing Act No. 1508 in the sense that "ungathered products" as
crop." mentioned in said article of the Civil Code have the nature of
personal property. In other words, the phrase "personal property"
From an examination of the reports and codes of the State of should be understood to include "ungathered products.”
California and other states we find that the settle doctrine
followed in said states in connection with the attachment of We may, therefore, conclude that paragraph 2 of article 334 of
property and execution of judgment is, that growing crops raised the Civil Code has been modified by section 450 of the Code of
by yearly labor and cultivation are considered personal property. Civil Procedure and by Act No. 1508, in the sense that, for the
purpose of attachment and execution, and for the purposes of
Mr. Mechem says that a valid sale may be made of a thing, the Chattel Mortgage Law, "ungathered products" have the
which though not yet actually in existence, is reasonably certain nature of personal property. The lower court, therefore,
to come into existence as the natural increment or usual incident committed no error in holding that the sugar cane in question
of something already in existence, and then belonging to the was personal property and, as such, was not subject to
vendor, and then title will vest in the buyer the moment the thing redemption.
comes into existence.
NOTA BENE: In Sibal v. Valdez, the Court held that pending
Things of this nature are said to have a potential existence. A crops which have potential existence may be the valid subject
man may sell property of which he is potentially and not actually matter of sale, and may be dealt with separately from the land on
possessed. He may make a valid sale of the wine that a vineyard which they grow.
is expected to produce; or the gain a field may grow in a given
time; or the milk a cow may yield during the coming year; or the
wool that shall thereafter grow upon sheep; or what may be LUIS PICHEL VS PRUDENCIO ALONZO (G.R. No. L-36902
taken at the next cast of a fisherman's net; or fruits to grow; or Jan. 30, 1982)
young animals not yet in existence; or the good will of a trade
and the like. The thing sold, however, must be specific and FACTS: Alonzo was awarded by the Government a parcel of
identified. They must be also owned at the time by the vendor. land in Basilan City in accordance with RA 477. The award was
(Hull vs. Hull, 48 Conn., 250 cancelled by the Board of Liquidators on January 27, 1965 on
the ground that, previous thereto, plaintiff was proved to have
[40 Am. Rep., 165].) alienated the land to another, in violation of law. In 1972,
plaintiff's rights to the land were reinstated.
It is contended on the part of the appellee that paragraph 2 of
article 334 of the Civil Code has been modified by section 450 of On August 14, 1968, Alonzo and his wife sold to Luis Pichel all
the Code of Civil Procedure as well as by Act No. 1508, the the fruits of the coconut trees which may be harvested in the land
Chattel Mortgage Law. in question for the period, September 15, 1968 to January 1,
1976, in consideration of P4,200.00. Even as of the date of sale,
Said section 450 enumerates the property of a judgment debtor however, the land was still under lease to Ramon Sua, and it was
which may be subjected to execution. The pertinent portion of the agreement that part of the consideration of the sale, in the
said section reads as follows: "All goods, chattels, moneys, and sum of P3,650.00, was to be paid by Pichel directly to Ramon
other property, both real and personal, * * * shall be liable to Sua so as to release the land from the clutches of the latter.
execution. Said section 450 and most of the other sections of the
Code of Civil Procedure relating to the execution of judgment Pending said payment Alonzo refused to allow the Pichel to
were taken from the Code of Civil Procedure of California. The make any harvest. Later, Pichel for the first time since the
Supreme Court of California, under section 688 of the Code of execution of the deed of sale in his favor, caused the harvest of
Civil Procedure of that state (Pomeroy, p. 424) has held, without the fruit of the coconut trees in the land.
variation, that growing crops were personal property and subject
to execution. Pichel filed for the annulment of the contract on the ground that it
violated the provisions of R.A. 477, which states that lands
CHATTEL MORTGAGE awarded under the said law shall not be subject to encumbrance
or alienation, otherwise the awardee shall no longer be entitled to
Act No. 1508, the Chattel Mortgage Law, fully recognized that apply for another piece of land. RTC ruled that although the
growing crops are personal property. Section 2 of said Act agreement in question is denominated by the parties as a deed
provides: "All personal property shall be subject to mortgage, of sale of fruits of the coconut trees found in the vendor's land, it
agreeably to the provisions of this Act, and a mortgage executed actually is, for all legal intents and purposes, a contract of lease
in pursuance thereof shall be termed a chattel mortgage." of the land itself
Section 7 in part provides: "If growing crops be mortgaged the
COMPILED BY: WIGMORE #WIGMOREFOREVER 48
SALES Case Digest (Atty. Sarona)
Compiled by: Wigmore #wigmoreforever

ISSUE: severed from the trees, to be used, enjoyed, sold or otherwise


1. WON the subject matter of the sale is valid. (YES) disposed of by the owner of the land. Herein respondents, as the
2. WON the sale of the coconut fruits violated RA 477. (NO) grantee of Lot No. 21 from the Government, had the right and
prerogative to sell the coconut fruits of the trees growing on the
HELD: property.
st
1 issue: The Deed of Sale is precisely what it purports to be. It
is a document evidencing the agreement of herein parties for the 4. Licit
sale of coconut fruits of the lot, and not for the lease of the land
itself as found by the lower Court. In clear and express terms, the
document defines the object of the contract thus: "the herein sale BELINDA TANEDO vs CA AND SPOUSES RICARDO AND
of the coconut fruits are for an the fruits on the aforementioned TERESITA TANEDO (G.R. No. 104482 January 22, 1996)
parcel of land during the years ...(from) SEPTEMBER 15, 1968;
up to JANUARY 1, 1976." FACTS: On October 20, 1962, Lazardo Ta edo executed a
notarized deed of absolute sale in favor of his eldest brother,
Moreover, as petitioner correctly asserts, the document in Ricardo Ta edo, and the latter's wife, Teresita Barera, private
question expresses a valid contract of sale. It has the essential respondents herein, whereby he conveyed to the latter in
elements of a contract of sale as defined under Article 1485 of consideration of P1,500.00, "1 hectare of whatever share I shall
the New Civil Code which provides thus: have over Lot No. 191 of the cadastral survey of Gerona,
Province of Tarlac and covered by Title T-13829 of the Register
Art. 1458. By the contract of sale one of the contracting parties of Deeds of Tarlac", the said property being his "future
obligates himself to transfer the ownership of and to deliver a inheritance" from his parents. Upon the death of his father
determinate thing, and the other to pay therefor a price certain in Matias, Lazaro executed an "Affidavit of Conformity" dated Feb
money or its equivalent.A contract of sale may be absolute or 28, 1980 to "re-affirm, respect, acknowledge and validate the
conditional. sale I made in 1962."

The subject matter of the contract of sale in question are the On January 13, 1981, Lazaro executed another notarized deed
fruits of the coconut trees on the land during the years from of sale in favor of private respondents covering his "undivided
September 15, 1968 up to January 1, 1976, which subject matter 1/12 of a parcel of land known as Lot 191". He acknowledged
is a determinate thing. Under Article 1461 of the New Civil Code, therein his receipt of P10,000.00 as consideration therefor. In
things having a potential existence may be the object of the Feb 1981, Ricardo learned that Lazaro sold the same property to
contract of sale. And in Sibal vs. Valdez, 50 Phil. 512, pending his children, petitioners herein, through a deed of sale dated Dec
crops which have potential existence may be the subject matter 29, 1980. Then, private respondents recorded the Deed of Sale
of the sale. in their favor in the Registry of Deeds and the corresponding
entry was made in TCT No. 166451.
According to Mechem, a valid sale may be made of a thing,
which though not yet actually in existence, is reasonably certain Petitioners on July 16, 1982 filed a complaint for rescission (plus
to come into existence as the natural increment or usual incident damages) of the deeds of sale executed by Lazaro in favor of
of something already in existence, and then belonging to the private respondents covering the property inherited by Lazaro
vendor, and the title will vest in the buyer the moment the thing from his father. Petitioners claimed that their father, Lazaro,
comes into existence. executed an "Absolute Deed of Sale" conveying to his 10
nd children his allotted portion tinder the extrajudicial partition
2 issue: The contract was clearly a "sale of the coconut fruits." executed by the heirs of Matias, which deed included the land in
The vendor sold, transferred and conveyed "by way of absolute litigation (Lot 191).
sale, all the coconut fruits of his land," thereby divesting himself
of all ownership or dominion over the fruits during the seven- Petitioners also presented in evidence: (1) a private writing
year period. The possession and enjoyment of the coconut trees purportedly prepared and signed by Matias stating that it was his
cannot be said to be the possession and enjoyment of the land desire that whatever inheritance Lazaro would receive from him
itself because these rights are distinct and separate from each should be given to his (Lazaro's) children; (2) a typewritten
other, the first pertaining to the accessory or improvements document signed by Lazaro in the presence of 2 witnesses,
(coconut trees) while the second, to the principal (the land). A wherein he confirmed that he would voluntarily abide by the
transfer of the accessory or improvement is not a transfer of the wishes of his father, Matias, to give to his (Lazaro's) children all
principal. It is the other way around, the accessory follows the the property he would inherit from the latter; and (3) a letter of
principal. Hence, the sale of the nuts cannot be interpreted nor Lazaro to his daughter, Carmela, stating that his share in the
construed to be a lease of the trees, much less extended further extrajudicial settlement of the estate of his father was intended
to include the lease of the land itself. for his children, petitioners herein.

The grantee of a parcel of land under R.A. No. 477 is not Private respondents presented in evidence a "Deed of
prohibited from alienating or disposing of the natural and/or Revocation of a Deed of Sale", wherein Lazaro revoked the sale
industrial fruits of the land awarded to him. What the law in favor of petitioners for the reason that it was "simulated or
expressly disallows is the encumbrance or alienation of the land fictitious without any consideration whatsoever".
itself or any of the permanent improvements thereon. While
coconut trees are permanent improvements of a land, their nuts Lazaro executed a sworn statement which virtually repudiated
are natural or industrial fruits which are meant to be gathered or
COMPILED BY: WIGMORE #WIGMOREFOREVER 49
SALES Case Digest (Atty. Sarona)
Compiled by: Wigmore #wigmoreforever

the contents of the Deed of Revocation of a Deed of Sale and one in favor of petitioners, ownership would vest in the former
the Deed of Sale in favor of private respondents. However, because of the undisputed fact of registration. On the other hand,
Lazaro testified that he sold the property to Ricardo, and that it petitioners have not registered the sale to them at all.
was a lawyer who induced him to execute a deed of sale in favor
of his children after giving him P5.00 to buy a "drink". Petitioners contend that they were in possession of the property
and that private respondents never took possession thereof. As
The trial court decided in favor of private respondents, holding between two purchasers, the one who registered the sale in his
that petitioners failed "to adduce a preponderance of evidence to favor has a preferred right over the other who has not registered
support (their) claim." On appeal, the CA affirmed the decision of his title, even if the latter is in actual possession of the
the trial court, ruling that the Deed of Sale was valid and that its immovable property.
registration in good faith vested title in said respondents.

ISSUE: Is the sale of a future inheritance valid? NO. MARTINEZ VS CA (G.R. L-31271 April 29, 1974)

HELD: The Court ruled that pursuant to Article 1347 of the Civil FACTS: The spouses Romeo Martinez and Leonor Suarez, are
Code, "no contract may be entered into upon a future inheritance the registered owners of two (2) parcels of land located in Lubao,
except in cases expressly authorized by law." Consequently, said Pampanga. Both parcels of land are fishponds and the property
nd
contract made in 1962 is not valid and cannot be the source of involved in this case is the 2 parcel of land.
any right nor the creator of any obligation between the parties.
The disputed property was originally owned by one Paulino
Hence, the "affidavit of conformity" dated Feb 28, 1980, insofar Montemayor, who secured a "titulo real" over it way back in
as it sought to validate or ratify the 1962 sale, is also useless 1883. After the death of Paulino Montemayor the said property
and, in the words of the respondent Court, "suffers from the passed to his successors-in-interest, Maria Montemayor and
same infirmity." Even private respondents in their memorandum Donata Montemayor, who in turn, sold it, as well as the first
concede this. parcel, to a certain Potenciano Garcia.

However, the documents that are critical to the resolution of this Because Potenciano Garcia was prevented by the then
case are: (a) the deed of sale of January 13, 1981 in favor of municipal president of Lubao from restoring the dikes
private respondents covering Lazaro's undivided inheritance of constructed on the contested property, the former, filed with the
(1/12) share in Lot No. 191, which was subsequently registered CFI against municipal president to restrain the latter in his official
on June 7, 1982; and (b) the deed of sale dated Dec 29, 1980 in capacity from molesting him in the possession of the lot.
favor of petitioners covering the same property. These two
documents were executed after the death of Matias and after a The Court, by decision promulgated the preliminary injunction
deed of extra-judicial settlement of his (Matias') estate was and the dikes around the property in question remained closed
executed, thus vesting in Lazaro actual title over said property. In until a portion thereof was again opened just before the outbreak
other words, these dispositions, though conflicting, were no of the Pacific War. On April 17, 1925. Potenciano Garcia applied
longer infected with the infirmities of the 1962 sale. for the registration of both parcels of land in his name and the
CFI granted the registration. Thereafter, the ownership of these
Petitioners contend that what was sold on Jan 13, 1981 was only properties changed hands until eventually they were acquired by
one- half hectare out of Lot No. 191, citing as authority the trial the herein appellee spouses.
court's decision. As earlier pointed out, what is on review in these
proceedings by this Court is the CA's decision — which correctly To avoid any untoward incident, the petitioners agreed to refer
identified the subject matter of the Jan 13, 1981 sale to be the the matter to the Committee on Rivers and Streams. Said Sub-
entire undivided 1/12 share of Lazaro in Lot No. 191 and which is Committee submitted its report, which said that the subject
the same property disposed of on Dec 29, 1980 in favor of property was not a public river but a private fishpond owned by
petitioners. the herein spouses.

Article 1544 of the CC governs the preferential rights of vendees ISSUE: W/N the subject property belongs to the petitioners. (NO)
in cases of multiple sales, as follows: Art. 1544. If the same thing
should have been sold to different vendees, the ownership shall HELD: IT IS A PROPERTY OF PUBLIC DOMINION
be transferred to the person who may have first taken The ruling of the Court of Appeals that the lot of the petitioners-
possession thereof in good faith, if it should be movable property. appellants is a public stream and that said title should be
cancelled and the river covered reverted to public domain, is
Should it be immovable property, the ownership shall belong to assailed by the petitioners-appellants as being a collateral attack
the person acquiring it who in good faith first recorded it in the on the indefeasibility of the torrens title originally issued in 1925
Registry of Property. Should there be no inscription, the in favor of the petitioners-appellants' predecessor-in-interest,
ownership shall pertain to the person who in good faith was first Potenciano Garcia, which is violative of the rule of res judicata. It
in the possession; and, in the absence thereof, to the person is argued that as the decree of registration issued by the Land
who presents the oldest title, provided there is good faith. Registration Court was not re-opened through a petition for
review filed within one (1) year from the entry of the decree of
The property in question is land, an immovable, and following the title, the certificate of title issued pursuant thereto in favor of the
law, ownership shall belong to the buyer who in good faith appellants for the land covered thereby is no longer open to
registers it first in the registry of property. Thus, although the attack under Section 38 of the Land Registration Act (Act 496)
deed of sale in favor of private respondents was later than the and the jurisprudence on the matter established by this Tribunal.
COMPILED BY: WIGMORE #WIGMOREFOREVER 50
SALES Case Digest (Atty. Sarona)
Compiled by: Wigmore #wigmoreforever

Section 38 of the Land Registration Act cited by appellants So respondent Socco-Beltran filed an application for the
expressly makes a decree of registration, which ordinarily makes purchase of Lot No. 6-B before the DAR, alleging that it was
the title absolute and indefeasible. adjudicated in her favor in the extra-judicial settlement of
ConstanciaSocco‘s estate.
At the time of the enactment of Section 496, one right recognized
or existing under the law is that provided for in Article 339 of the Now, petitioners, the heirs of the late Arturo Reyes, filed their
old Civil Code which reads as follows: protest to respondent‘s petition before the DAR on the ground
that the subject property was sold by respondent‘s brother,
Property of public ownership is: Miguel R. Socco, in favor of their father, Arturo Reyes, as
evidenced by the Contract to Sell, dated 5 September 1954.
1. That destined to the public use, such as roads, canals, rivers, Petitioners averred that they took physical possession of the
torrents, ports, and bridges constructed by the State, and banks subject property in 1954 and had been uninterrupted in their
shores, roadsteads, and that of a similar character. (Par. 1) possession of the said property since then.

The above-mentioned properties are parts of the public domain Investigation was conducted by the legal officer Pinlac, and in
intended for public use, are outside the commerce of men and, the end, the legal officer recommended the approval of
therefore, not subject to private appropriation. respondent‘s petition for issuance of title over the subject
property, ruling that respondent was qualified to own the subject
A simple possession of a certificate of title under the Torrens property pursuant to Article 1091 of the New Civil Code.
system does not necessarily make the possessor a true owner of However, DAR Regional Director Mr. Acosta, dismissed
all the property described therein. If a person obtains title under respondent‘s petition for issuance of title over the subject
the Torrens system which includes by mistake or oversight, lands property on the ground that respondent was not an actual tiller
which cannot be registered under the Torrens system, he does and had abandoned the said property for 40 years. It went up to
not by virtue of said certificate alone become the owner of the the Department Secretary, then to the OP.
land illegally included.
Aggrieved, the petitioners went the to the Court of Appeals but it
It is useless for the appellant now to allege that she has obtained promulgated its decision, affirming the that of the Office of the
certificate of title No. 329 in her favor because the said certificate President. It held that petitioners could not have been actual
does not confer upon her any right to the creek in question, occupants of the subject property, since actual occupancy
inasmuch as the said creek, being of the public domain, is requires the positive act of occupying and tilling the land, not just
included among the various exceptions enumerated in Section the introduction of an unfinished skeletal structure thereon. The
39 of Act 496 to which the said certificate is subject by express Contract to Sell on which petitioners based their claim over the
provision of the law. subject property was executed by Miguel Socco, who was not
the owner of the said property and, therefore, had no right to
Torrens certificate of title does not operate when the land transfer the same.
covered thereby is not capable of registration.
ISSUES:
It is, therefore, clear that the authorities cited by the appellants 1. WON title to the property was transferred to petitioners by
as to the conclusiveness and incontestability of a Torrens virtue of the Contract to Sell executed by Miguel Socco. (NO)
certificate of title do not apply here. The Land Registration Court 2. WON petitioners are in OCEAN possession of the property
has no jurisdiction over non-registerable properties, such as since 1954 (more than 30 years). (NO)
public navigable rivers which are parts of the public domain, and
cannot validly adjudge the registration of title in favor of a private HELD: Petitioner‘s claim over the subject property is anchored
applicant. on the Contract to Sell executed between Miguel Socco and
Arturo Reyes. Petitioners additionally allege that they and their
predecessor-in- interest, Arturo Reyes, have been in possession
HEIRS OF ARTURO REYES VS SOCCO-BELTRAN (G.R. No. of the subject lot since 1954 for an uninterrupted period of more
176474 November 27, 2008) than 40 years.

FACTS: The subject property in this case is a parcel of land Petitioners cannot derive title to the subject property by virtue of
originally identified as Lot No. 6-B (with an area of 360 square the Contract to Sell. It was unmistakably stated in the Contract
meters). It was originally part of a larger parcel of land, and made clear to both parties thereto that the vendor, Miguel R.
measuring 1,022 square metersallocated to the Spouses Socco, was not yet the owner of the subject property and was
Marcelo Laquian and ConstanciaSocco (Spouses Laquian). merely expecting to inherit the same as his share as a co-heir of
Upon their death, they left the original parcel of landto Constancia‘s estate. It was also declared in the Contract itself
Constancia‘ssiblings (Filomena, Isabel, Miguel R. Socco, and that Miguel R. Socco‘s conveyance of the subject to the buyer,
Elena Socco-Beltran). Pursuant to an unnotarized document Arturo Reyes, was a conditional sale. It is, therefore, apparent
entitled ―Extrajudicial Settlement of the Estate of the Deceased that the sale of the subject property in favor of Arturo Reyes was
Constancia R. Socco,‖ executed by Constancia‘s heirs sometime conditioned upon the event that Miguel Socco would actually
in 1965, the parcel of land was partitioned into three lots—Lot inherit and become the owner of the said property. Absent such
No. 6-A, Lot No.6-B, and Lot No. 6-C. The subject property, Lot occurrence, Miguel R. Socco never acquired ownership of the
No. 6-B, was adjudicated to respondent Elena Socco-Beltran, but subject property which he could validly transfer to Arturo Reyes.
no title had been issued in her name.
Under Article 1459 of the Civil Code on contracts of sale, ―The
COMPILED BY: WIGMORE #WIGMOREFOREVER 51
SALES Case Digest (Atty. Sarona)
Compiled by: Wigmore #wigmoreforever

thing must be licit and the vendor must have a right to Ederlina went to Germany to file a divorce however Ederlina had
transfer ownership thereof at the time it is delivered.‖ The not been able to secure a divorce from Klaus. The latter could
law specifically requires that the vendor must have ownership of charge her for bigamy and could even involve Alfred, who himself
the property at the time it is delivered. was still married.

Petitioners claim that the property was constructively delivered to Alfred and Ederlina’s relationship started deteriorating. They lived
them in 1954 by virtue of the Contract to Sell. However, as separately.
already pointed out by this Court, it was explicit in the Contract
itself that, at the time it was executed, Miguel R. Socco was not Alfred filed a Complaint dated October 28, 1985, against
yet the owner of the property and was only expecting to inherit it. Ederlina, with the Regional Trial Court of Quezon City, for
Hence, there was no valid sale from which ownership of the recovery of real and personal properties located in Quezon City
subject property could have transferred from Miguel Socco to and Manila. Alfred alleged, inter alia, that Ederlina, without his
Arturo Reyes. Without acquiring ownership of the subject knowledge and consent, managed to transfer funds from their
property, Arturo Reyes also could not have conveyed the same joint account in HSBC Hong Kong, to her own account with the
to his heirs, herein petitioners. same bank.

Petitioners, nevertheless, insist that they physically occupied the In the meantime, on November 7, 1985, Alfred also filed a
subject lot for more than 30 years and, thus, they gained complaint against Ederlina with the Regional Trial Court, Davao
ownership of the property through acquisitive prescription. City, for specific performance, declaration of ownership of real
and personal properties, sum of money, and damages.
In the case of San Miguel Corporation it was underscored that
open, continuous, exclusive, and notorious occupation of Quezon City Trial Court decided in favor of Alfred but the Davao
property for more than 30 years must be no less than conclusive, Trial Court is in favor of Ederlina. The trial court ruled that based
such quantum of proof being necessary to avoid the erroneous on documentary evidence, the purchaser of the three parcels of
validation of actual fictitious claims of possession over the land subject of the complaint was Ederlina. The court further
property that is being claimed. In the present case, the evidence stated that even if Alfred was the buyer of the properties, he had
presented by the petitioners falls short of being conclusive. Apart no cause of action against Ederlina for the recovery of the same
from their self-serving statement that they took possession of the because as an alien, he was disqualified from acquiring and
subject property, the only proof offered to support their claim was owning lands in the Philippines. The sale of the three parcels of
a general statement made Barangay Captain Carlos Gapero, land to the petitioner was null and void ab initio. Applying the pari
certifying that Arturo Reyes was the occupant of the subject delicto doctrine, the petitioner was precluded from recovering the
property ―since peace time and at present. properties from the respondent.

In contrast, respondent‘s claim over the subject property is CA affirmed the decision of Davao City Court.
backed by sufficient evidence. Her predecessors-in-interest, the
spouses Laquian, have been identified as the original allocatees ISSUE: W/n the lower court erred in applying the in pari delicto
who have fully paid for the subject property. The subject property rule in the case at bar.
was allocated to respondent in the extrajudicial settlement by the
heirs of Constancia‘s estate which its authenticity or legality was HELD: No. Section 14, Article XIV of the 1973 Constitution
never put into question. Moreover, respondent has continuously provides, as follows:
paid for the realty tax due on the subject property, a fact which,
though not conclusive, served to strengthen her claim over the Save in cases of hereditary succession, no private land shall be
property. transferred or conveyed except to individuals, corporations, or
associations qualified to acquire or hold lands in the public
domain.
FRENZEL V. CATITO
Lands of the public domain, which include private lands, may be
FACTS: Petitioner Alfred Fritz Frenzel is an Australian citizen of transferred or conveyed only to individuals or entities qualified to
German descent. He arrived in the Philippines and engaged in acquire or hold private lands or lands of the public domain.
businesses. After two years, he married Teresita Santos, a Aliens, whether individuals or corporations, have been
Filipino citizen. In 1981, Alfred and Teresita separated from bed disqualified from acquiring lands of the public domain. Hence,
and board without obtaining a divorce. they have also been disqualified from acquiring private lands.

Sometime in 1983 he arrived in Sydney and met Ederlina Catito, Even if, as claimed by the petitioner, the sales in question were
a Filipina and a native of Bajada, Davao City. Unknown to Alfred, entered into by him as the real vendee, the said transactions are
she was married to Klaus Muller when she was in Germany. in violation of the Constitution; hence, are null and void ab initio.
A contract that violates the Constitution and the law, is null and
Alfred was so enamored with Ederlina that he persuaded her to void and vests no rights and creates no obligations. It produces
stop working, move to the Philippines and get married. no legal effect at all. The petitioner, being a party to an illegal
contract, cannot come into a court of law and ask to have his
They bought several properties in Manila and Davao using the illegal objective carried out. One who loses his money or
money of Alfred. He also sold all his properties in Australia property by knowingly engaging in a contract or transaction
before moving in the country. They also opened an HSBC which involves his own moral turpitude may not maintain an
Savings Account in Hong Kong in the name of Ederlina. action for his losses. To him who moves in deliberation and
COMPILED BY: WIGMORE #WIGMOREFOREVER 52
SALES Case Digest (Atty. Sarona)
Compiled by: Wigmore #wigmoreforever

premeditation, the law is unyielding. The law will not aid either A contract of sale may be absolute or conditional. As thus
party to an illegal contract or agreement; it leaves the parties defined, the essential elements of sale are the following:
where it finds them.
a) Consent or meeting of the minds, that is, consent to transfer
Under Article 1412 of the New Civil Code, the petitioner cannot ownership in exchange for the price;
have the subject properties deeded to him or allow him to b) Determinate subject matter; and
recover the money he had spent for the purchase thereof. Equity c) Price certain in money or its equivalent.
as a rule will follow the law and will not permit that to be
done indirectly which, because of public policy, cannot be done As shown in the receipt, dated September 29, 1964, the late
directly. Where the wrong of one party equals that of the other, Juan San Andres received P500.00 from respondent as
the defendant is in the stronger position … it signifies that in such "advance payment for the residential lot adjoining his previously
a situation, neither a court of equity nor a court of law will paid lot on three sides excepting on the frontage; the agreed
administer a remedy. The rule is expressed in the maxims: EX purchase price was P15.00 per square meter; and the full
DOLO MALO NON ORITUR ACTIO and IN PARI DELICTO amount of the purchase price was to be based on the results of a
POTIOR EST CONDITIO DEFENDENTIS. survey and would be due and payable in five (5) years from the
execution of a deed of sale.

3. Determinate or At Least Determinable Petitioner's contention is without merit. There is no dispute that
respondent purchased a portion of Lot 1914-B-2 consisting of
345 square meters. This portion is located in the middle of Lot
HEIRS OF JUAN SAN ANDRES V. RODRIGUEZ 1914-B-2, which has a total area of 854 square meters, and is
clearly what was referred to in the receipt as the "previously paid
FACTS: Juan San Andres was the owner of the lot situated in lot." Since the lot subsequently sold to respondent is said to
Liboton, Naga city. The sale was evidenced by a deed of sale. adjoin the "previously paid lot" on three sides thereof, the subject
Upon the death of Juan Andres, Ramon San Andres was lot is capable of being determined without the need of any new
appointed as administrator of the estate, and hired geodetic contract. The fact that the exact area of these adjoining
engineer. Jose Panero prepared a consolidated plan of the residential lots is subject to the result of a survey does not
estate and also prepared a sketch plan of the lot sold to detract from the fact that they are determinate or determinable.
respondent. It was found out that respondent had enlarged the As the Court of Appeals explained:
area which he purchased from Juan. The administrator sent a
letter to the respondent to vacate the said portion in which the Concomitantly, the object of the sale is certain and determinate.
latter refused to do. Under Article 1460 of the New Civil Code, a thing sold is
determinate if at the time the contract is entered into, the thing is
Respondent alleged that apart from the original lot, which had capable of being determinate without necessity of a new or
been sold to him, the latter likewise sold to him the following day further agreement between the parties. Here, this definition finds
the remaining portion of the lot. He alleged that the payment for realization.
such would be affected in 5 years from the execution of the
formal deed of sale after a survey is conducted. He also alleged Thus, all of the essential elements of a contract of sale are
that under the consent of Juan, he took possession of the same present, i.e., that there was a meeting of the minds between the
and introduced improvements thereon. Respondent deposited in parties, by virtue of which the late Juan San Andres undertook to
court the balance of the purchase price amounting to P7,035.00 transfer ownership of and to deliver a determinate thing for a
for the aforesaid 509-square meter lot. price certain in money.

On September 20, 1994, the trial court rendered judgment in As Art. 1475 of the Civil Code provides:
favor of petitioner. It ruled that there was no contract of sale to The contract of sale is perfected at the moment there is a
speak of for lack of a valid object because there was no sufficient meeting of minds upon the thing which is the object of the
indication to identify the property subject of the sale, hence, the contract and upon the price. . . .That the contract of sale is
need to execute a new contract. perfected was confirmed by the former administrator of the
estates, Ramon San Andres, who wrote a letter to respondent on
Respondent appealed to the Court of Appeals, which on April 21, March 30, 1966 asking for P300.00 as partial payment for the
1998 rendered a decision reversing the decision of the trial court. subject lot.
The appellate court held that the object of the contract was
determinable, and that there was a conditional sale with the As the Court of Appeals observed:
balance of the purchase price payable within five years from the Without any doubt, the receipt profoundly speaks of a meeting of
execution of the deed of sale. the mind between San Andres and Rodriguez for the sale.
Evidently, this is a perfected contract of sale on a deferred
ISSUE: Whether or not there was a valid sale. payment of the purchase price. All the pre-requisite elements for
a valid purchase transaction are present.
HELD: YES. The Civil Code provides that By the contract of sale
one of the contracting parties obligates himself to transfer the There is a need, however, to clarify what the Court of Appeals
ownership of and to deliver a determinate thing, and the other to said is a conditional contract of sale. Apparently, the appellate
pay therefor a price certain in money or its equivalent. court considered as a "condition" the stipulation of the parties
that the full consideration, based on a survey of the lot, would be
due and payable within five (5) years from the execution of a
COMPILED BY: WIGMORE #WIGMOREFOREVER 53
SALES Case Digest (Atty. Sarona)
Compiled by: Wigmore #wigmoreforever

formal deed of sale. It is evident from the stipulations in the deed of sale was lot No. 353-A and not lot 535-E, while the land
receipt that the vendor Juan San Andres sold the residential lot in which remained in the possession of Eulogio I, and which was
question to respondent and undertook to transfer the ownership passed to Ladislao was lot No. 353-E and not lot No. 535-A.
thereof to respondent without any qualification, reservation or
condition. On 1960, the heirs of Eulogio II alleging, inter alia, that they
offered to surrender to the possession of lot No. 535-A and
A deed of sale is considered absolute in nature where there is demanded in return the possession of lot No. 535-E, but the
neither a stipulation in the deed that title to the property sold is defendants refused to accept the exchange. The plaintiffs'
reserved in the seller until full payment of the price, nor one insistence is quite understandable, since lot No. 535-E has an
giving the vendor the right to unilaterally resolve the contract the area of 2,612 square meters as compared to the 1,808 square-
moment the buyer fails to pay within a fixed period. meter area of lot No. 535-A.

Applying these principles to this case, it cannot be gainsaid that In their answer to the complaint, the defendants alleged that the
the contract of sale between the parties is absolute, not reference to lot No. 535-E in the deed of sale was an involuntary
conditional. There is no reservation of ownership nor a stipulation error; that the intention of the parties to that sale was to convey
providing for a unilateral rescission by either party. In fact, the the lot correctly identified as lot No. 535-A. On the basis of the
sale was consummated upon the delivery of the lot to foregoing allegations the defendants interposed a counterclaim,
respondent. praying that the plaintiffs be ordered to execute in their favor the
corresponding deed of transfer with respect to Lot No. 535-E.
Thus, Art. 1477 provides that the ownership of the thing sold
shall be transferred to the vendee upon the actual or constructive The trial court rendered judgment in favor of the plaintiffs.
delivery thereof.
ISSUE: Whether or not there has been a valid sale in view of the
The stipulation that the "payment of the full consideration based real intention of the parties.
on a survey shall be due and payable in five (5) years from the
execution of a formal deed of sale" is not a condition which HELD: YES. When one sells or buys real property — a piece of
affects the efficacy of the contract of sale. It merely provides the land, for example — one sells or buys the property as he sees it,
manner by which the full consideration is to be computed and the in its actual setting and by its physical metes and bounds, and
time within which the same is to be paid. But it does not affect in not by the mere lot number assigned to it in the certificate of title.
any manner the effectivity of the contract. Consequently, the In the instant case, the portion correctly referred to as lot No.
contention that the absence of a formal deed of sale stipulated in 535-A was already in the possession of the vendee, Eulogio
the receipt prevents the happening of a sale has no merit. Atilano II, who had constructed his residence therein, even
before the sale in his favor even before the subdivision of the
The claim of petitioners that the price of P7,035.00 is iniquitous is entire lot No. 535 at the instance of its owner, Eulogio Atillano I.
untenable. The amount is based on the agreement of the parties In like manner the latter had his house on the portion correctly
as evidenced by the receipt (Exh. 2). Time and again, we have identified, after the subdivision, as lot No. 535-E, even adding to
stressed the rule that a contract is the law between the parties, the area thereof by purchasing a portion of an adjoining property
and courts have no choice but to enforce such contract so long belonging to a different owner. The two brothers continued in
as they are not contrary to law, morals, good customs or public possession of the respective portions the rest of their lives,
policy. Otherwise, court would be interfering with the freedom of obviously ignorant of the initial mistake in the designation of the
contract of the parties. Simply put, courts cannot stipulate for the lot subject of the 1920 until 1959, when the mistake was
parties nor amend the latter's agreement, for to do so would be discovered for the first time.
to alter the real intentions of the contracting parties when the
contrary function of courts is to give force and effect to the From the facts and circumstances, the object is lot No. 535-A
intentions of the parties. and its designation as lot No. 535-E in the deed of sale was a
simple mistake in the drafting of the document. The mistake did
not vitiate the consent of the parties, or affect the validity and
ATILANO V. ATILANO binding effect of the contract between them. The new Civil Code
provides a remedy by means of reformation of the instrument.
FACTS: In 1916, Eulogio Atilano I acquired lot No. 535 by This remedy is available when, there having been a meeting of
purchase. In 1920, he had the land subdivided into five parts, the minds of the parties to a contract, their true intention is not
identified as lots Nos. 535-A, 535-B, 535-C, 535-D and 535-E, expressed in the instrument purporting to embody the agreement
respectively. After the subdivision had been effected, Eulogio I by reason of mistake, fraud, inequitable conduct or accident
executed a deed of sale covering lot No. 535-E in favor of his
brother Eulogio II. Three other portions, namely, lots Nos. 535-B, In this case, the deed of sale executed in 1920 need no longer
535-C, and 535-D, were likewise sold to other persons. Eulogio I be reformed. The parties have retained possession of their
retained for himself the remaining portions of the land, respective properties conformably to the real intention of the
presumably covered by the title to lot No. 535-A. upon his death, parties to that sale, and all they should do is to execute mutual
the title to this lot passed to Ladislao, in whose name the deed of conveyance.
corresponding certificate was issued.

On 1959, Eulogio II and his children had the land resurveyed so


that it could be properly subdivided. However, they discovered
that the land they were actually occupying on the strength of the
COMPILED BY: WIGMORE #WIGMOREFOREVER 54
SALES Case Digest (Atty. Sarona)
Compiled by: Wigmore #wigmoreforever

MELLIZA V. CITY OF ILOILO Pio Sian Melliza appealed to the Court of Appeals. On 19 May
1965, the CA affirmed the interpretation of the CFI that the
FACTS: Juliana Melliza during her lifetime owned, among other portion of Lot 1214 sold by Juliana Melliza was not limited to the
properties, 3 parcels of residential land in Iloilo City (OCT 3462). 10,788 square meters specifically mentioned but included
Said parcels of land were known as Lots Nos. 2, 5 and 1214. whatever was needed for the construction of avenues, parks and
The total area of Lot 1214 was 29,073 sq. m. the city hall site. Nonetheless, it ordered the remand of the case
for reception of evidence to determine the area actually taken by
On 27 November 1931 she donated to the then Municipality of Iloilo City for the construction of avenues, parks and for city hall
Iloilo, 9,000 sq. m. of Lot 1214, to serve as site for the municipal site.
hall. The donation was however revoked by the parties for the
reason that the area donated was found inadequate to meet the In the present petition, Melliza maintains that only Lots No. 1214-
requirements of the development plan of the municipality, the so- C and 1214-D were included in the sale, and that the purpose of
called “Arellano Plan.” the second paragraph of the deed of sale was only to better
identify the lots. Melliza also argues that the interpretation given
Subsequently, Lot 1214 was divided by Certeza Surveying Co., by the lower courts would render the sale invalid for it lacks an
Inc. into Lots 1214-A and 1214-B. And still later, Lot 1214-B was essential element of a sale, a “determinate (or determinable)”
further divided into Lots 1214-B-1, Lot 1214-B-2 and Lot 1214-B- object. Respondents, however, maintain that the object of the
3. As approved by the Bureau of Lands, Lot 1214-B-1, with 4,562 sale remains determinate, as it could be ascertained what lots
sq. m., became known as Lot 1214-B; Lot 1214-B-2, with 6,653 were needed by the Municipality of Iloilo for the “Arellano Plan” at
sq. m., was designated as Lot 1214-C; and Lot 1214-B-3, with the time of the execution of the sale.
4,135 sq. m., became Lot 1214-D.
ISSUE: W/n there was a determinate object of the sale,
On 15 November 1932, Juliana Melliza executed an instrument rendering the sale valid.
without any caption providing for the absolute sale involving all of
lot 5, 7669 sq. m. of Lot 2 (sublots 2-B and 2-C), and a portion of HELD: YES. The paramount intention of the parties was
10,788 sq. m. of Lot 1214 (sublots 1214-B2 and 1214-B3) in to provide Iloilo municipality with lots sufficient or adequate in
favor of the Municipal Government of Iloilo for the sum of P6,422; area for the construction of the Iloilo City hall site, with its
these lots and portions being the ones needed by the municipal avenues and parks. For this matter, a previous donation for this
government for the construction of avenues, parks and City hall purpose between the same parties was revoked by them,
site according the “Arellano plan.” because of inadequacy of the area of the lot donated. Said
instrument described 4 parcels of land by their lot numbers and
On 14 January 1938, Melliza sold her remaining interest in Lot area; and then it goes on to further describe, not only those lots
1214 to Remedios Sian Villanueva (thereafter TCT 18178). already mentioned, but the lots object of the sale, by stating that
Remedios in turn on 4 November 1946 transferred her rights to said lots were the ones needed for the construction of the city
said portion of land to Pio Sian Melliza (thereafter TCT 2492). hall site, avenues and parks according to the Arellano plan. If the
Annotated at the back of Pio Sian Melliza’s title certificate was parties intended merely to cover the specified lots (Lots 2, 5,
the following “that a portion of 10,788 sq. m. of Lot 1214 now 1214-C and 1214-D), there would scarcely have been any need
designated as Lots 1412-B-2 and 1214-B-3 of the subdivision for the next paragraph, since these lots were already plainly and
plan belongs to the Municipality of Iloilo as per instrument dated very clearly described by their respective lot number and areas.
15 November 1932.” Said next paragraph does not really add to the clear description
that was already given to them in the previous one. It is therefore
On 24 August 1949 the City of Iloilo, which succeeded to the the more reasonable interpretation to view it as describing those
Municipality of Iloilo, donated the city hall site together with the other portions of land contiguous to the lots that, by reference to
building thereon, to the University of the Philippines (Iloilo the Arellano plan, will be found needed for the purpose at hand,
branch). The site donated consisted of Lots 1214-B, 1214-C and the construction of the city hall site.
1214-D, with a total area of 15,350 sq. m., more or less.
Sometime in 1952, the University of the Philippines enclosed the The requirement of the law that a sale must have for its object a
site donated with a wire fence. Pio Sian Melliza thereupon made determinate thing, is fulfilled as long as, at the time the contract
representations, thru his lawyer, with the city authorities for is entered into, the object of the sale is capable of being made
payment of the value of the lot (Lot 1214-B). No recovery was determinate without the necessity of a new or further agreement
obtained, because as alleged by Pio Sian Melliza, the City did between the parties (Art. 1273, old Civil Code; Art. 1460, New
not have funds. The University of the Philippines, meanwhile, Civil Code). The specific mention of some of the lots plus the
obtained Transfer Certificate of Title No. 7152 covering the three statement that the lots object of the sale are the ones needed for
lots, Nos. 1214-B, 1214-C and 1214-D. city hall site; avenues and parks, according to the Arellano plan,
sufficiently provides a basis, as of the time of the execution of the
On 10 December 1955 Pio Sian Melliza filed an action in the CFI contract, for rendering determinate said lots without the need of
Iloilo against Iloilo City and the University of the Philippines for a new and further agreement of the parties.
recovery of Lot 1214-B or of its value. After stipulation of facts
and trial, the CFI rendered its decision on 15 August 1957,
dismissing the complaint. Said court ruled that the instrument NATIONAL GRAINS AUTHORITY V. IAC
executed by Juliana Melliza in favor of Iloilo municipality included
in the conveyance Lot 1214-B, and thus it held that Iloilo City had FACTS: On August 23, 1979, private respondent Leon Soriano
the right to donate Lot 1214-B to UP. offered to sell palay grains to NFA through William Cabal, the
provincial manager in Tuguegarao. The documents submitted
COMPILED BY: WIGMORE #WIGMOREFOREVER 55
SALES Case Digest (Atty. Sarona)
Compiled by: Wigmore #wigmoreforever

were processed, and he was given a quota of 2,640 cavans, cancellation fee of 30% of F.O.B. value, or plaintiff will endorse
which is the maximum number of cavans he may sell to NFA. On the case to its lawyers. Demand letters sent to defendant by
the same day and on the following day, Soriano delivered 630 plaintiff's counsel dated March 22, 1983 and June 9, 1983 were
cavans, which were no rebagged, classified and weighed. When to no avail. Consequently, petitioner filed a complaint for
he demanded payment, he was told that payment will be held in recovery of actual or compensatory damages, unearned profits,
abeyance since Mr. Cabal was still investigating on an interest, attorney's fees and costs against private respondent.
information received that Soriano was not a bona fide farmer.
Instead of withdrawing the palay, Soriano insisted that the palay ISSUE: Whether or not a contract of sale has been perfected
grains be delivered and paid. He filed a complaint for specific between the parties
performance. Petitioners contend that the delivery was merely
made for the purpose of offering it for sale because until the HELD:
grains were rebagged, classified and weighed, they are not Article 1319 of the Civil Code states: "Consent is manifested by
considered sold. the meeting of the offer and acceptance upon the thing and the
cause which are to constitute the contract. The offer must be
ISSUE: Whether there was a perfected sale. certain and the acceptance absolute. A qualified acceptance
constitutes a counter offer." The facts presented to us indicate
HELD: Soriano initially offered to sell palay grains produced in that consent on both sides has been manifested. The offer by
his farmland to NFA. When the latter accepted the offer by noting petitioner was manifested on December 17, 1981 when petitioner
in Soriano's Farmer's Information Sheet a quota of 2,640 cavans, submitted its proposal containing the item number, quantity, part
there was already a meeting of the minds between the parties. number, description, the unit price and total to private
The object of the contract, being the palay grains produced in respondent. On December 24, 1981, private respondent
Soriano's farmland and the NFA was to pay the same depending informed petitioner of his desire to avail of the prices of the parts
upon its quality. The fact that the exact number of cavans of at that time and simultaneously enclosed its Purchase Order. At
palay to be delivered has not been determined does not affect this stage, a meeting of the minds between vendor and vendee
the perfection of the contract. Article 1349 of the New Civil Code has occurred, the object of the contract: being the spare parts
provides: ". . .. The fact that the quantity is not determinate shall and the consideration, the price stated in petitioner's offer dated
not be an obstacle to the existence of the contract, provided it is December 17, 1981 and accepted by the respondent on
possible to determine the same, without the need of a new December 24, 1981.
contract between the parties." In this case, there was no need for
NFA and Soriano to enter into a new contract to determine the
exact number of cavans of palay to be sold. Soriano can deliver 4. Obligation to Transfer Ownership
so much of his produce as long as it does not exceed 2,640
cavans. From the moment the contract of sale is perfected, it is
incumbent upon the parties to comply with their mutual ALCANTARA-DAUS v. SPOUSES DE LEON
obligations or "the parties may reciprocally demand
performance" thereof. FACTS: Spouses De Leon are the owners of a parcel of land
situated in the Municipality of San Manuel, Pangasinan with an
area of Four Thousand Two Hundred Twelve square meters
SCHUBACK & SONS VS. CA more or less. Respondent Hermoso De Leon inherited the said
lot from his father Marcelino De Leon by virtue of a Deed of
FACTS: On October 16, 1981, defendant submitted to plaintiff Extra-Judicial Partition. Said lot is covered by Original Certificate
the list of bus spare parts he wanted to purchase to its of Title No. 22134 of the Land Records of Pangasinan.
counterpart in Hamburg. Plaintiff sent an offer on the items listed.
On December 4, 1981, defendant informed plaintiff that he Sometime 1960s, Spouses De Leon engaged the services of the
preferred genuine to replacement parts, and requested a 15% late Atty. Florencio Juan to take care of the documents of their
discount. On December 17, plaintiff submitted its formal offer. On properties. They were asked to sign voluminous documents by
December 24, defendant submitted a purchase order, and the latter. After the death of Atty. Juan, some documents
submitted the quantity on December 29. Plaintiff immediately surfaced and most revealed that their properties had been
ordered the items from Schuback Hamburg, which thereafter conveyed by sale or quitclaim to Hermoso’s brothers and sisters,
ordered the same from NDK, a supplier in Germany. to Atty. Juan and his sisters, when in truth and in fact, no such
conveyances were ever intended by them. Furthermore,
Plaintiff sent a pro-forma invoice to be used in applying for letter respondent found out that his signature in the Deed of Extra-
of credit. On February 16, 1982, plaintiff reminded defendant to judicial Partition with Quitclaim made in favor of Rodolfo de Leon
open a letter of credit to avoid delay in shipment. Defendant was forged. They discovered that the land in question was sold
mentioned the difficulty he was encountering in procuring the by Rodolfo de Leon to Aurora Alcantara.
same. Plaintiff continued receiving invoices and partial deliveries
from NDK. On October 18, 1982, plaintiff again reminded the Spouses De Leon demanded the annulment of the document
defendant to open a letter of credit. Defendant replied that he did and re-conveyance but defendants refused. Petitioner, Aurora
not make a valid purchase order and that there was no definite Alcantara-Daus averred that she bought the land in question in
contract between him and the plaintiff. Plaintiff sent a rejoinder good faith and for value on December 1975 and that she has
explaining that there is a valid Purchase been in continuous, public, peaceful, open possession over the
same and has been appropriating the produce thereof without
Order and suggesting that defendant either proceed with the objection from anyone.
order and open a letter of credit or cancel the order and pay the
COMPILED BY: WIGMORE #WIGMOREFOREVER 56
SALES Case Digest (Atty. Sarona)
Compiled by: Wigmore #wigmoreforever

The RTC of Urdaneta, Pangasinan rendered its Decision in favor acts of the person signing.
of herein petitioner. It ruled that respondents’ claim was barred
by laches, because more than 18 years had passed since the
land was sold. It further ruled that since it was a notarial CONCHITA NOOL and GAUDENCIO ALMOJERA vs. CA
document, the Deed of Extrajudicial Partition in favor of Rodolfo
de Leon was presumptively authentic. FACTS: One lot formerly owned by Victorio Nool has an area of
1 hectare. Another lot previously owned by Francisco Nool has
ISSUES: an area of 3.0880 hectares. Spouses (plaintiffs) Conchita Nool
 Whether or not the Deed of Absolute executed by Rodolfo De and Gaudencio Almojera alleged that they are the owners of the
Leon over the land in question in favor of petitioner was subject lands. They are in dire need of money, they obtained a
perfected and binding upon the parties therein? loan DBP , secured by a real estate mortgage on said parcels of
 Whether or not the evidentiary weight of the Deed of land, which were still registered in the names of Victorino and
Extrajudicial Partition with Quitclaim, executed by respondent Francisco Nool, at the time, Since the plaintiffs failed to pay the
Hermoso de Leon, Perlita de Leon and Carlota de Leon in said loan, the mortgage was foreclosed; that within the period of
favor of Rodolfo de Leon was overcome by more than a redemption, the plaintiffs contacted Anacleto Nool for the latter to
preponderance of evidence of respondents? redeem the foreclosed properties from DBP, which the latter did;
and as a result, the titles of the 2 parcels of land in question were
HELD: transferred to Anacleto; that as part of their arrangement or
First Issue: understanding, Anacleto agreed to buy from Conchita the 2
NO. It is during the delivery that the law requires the seller to parcels of land , for a total price of P100,000.00, P30,000.00 of
have the right to transfer ownership of the thing sold. In general, which price was paid to Conchita, and upon payment of the
a perfected contract of sale cannot be challenged on the ground balance of P14,000.00, the plaintiffs were to regain possession
of the seller’s non-ownership of the thing sold at the time of the of the 2 hectares of land, which amounts spouses Anacleto Nool
perfection of the contract. and Emilia Nebre failed to pay. Anacleto Nool signed the private
writing, agreeing to return subject lands when plaintiffs have the
Further, even after the contract of sale has been perfected money to redeem the same; defendant Anacleto having been
between the parties, its consummation by delivery is yet another made to believe, then, that his sister, Conchita, still had the right
matter. It is through tradition or delivery that the buyer acquires to redeem the said properties.
the real right of ownership over the thing sold.
ISSUE: Is the purchase of the subject lands to Anacleto valid?
Undisputed is the fact that at the time of the sale, Rodolfo De
Leon was not the owner of the land he delivered to petitioner. HELD: Nono dat quod non habet, No one can give what he does
Thus, the consummation of the contract and the consequent not have; Contract of repurchase inoperative thus void.
transfer of ownership would depend on whether he subsequently
acquired ownership of the land in accordance with Article 1434 of Article 1505 of the Civil Code provides that “where goods are
the Civil Code. Therefore, we need to resolve the issue of the sold by a person who is not the owner thereof, and who does not
authenticity and the due execution of the Extrajudicial Partition sell them under authority or with consent of the owner, the buyer
and Quitclaim in his favor. acquires no better title to the goods than the seller had, unless
the owner of the goods is by his conduct precluded from denying
Second Issue: the seller’s authority to sell.”
NO. As a general rule, the due execution and authenticity of a
document must be reasonably established before it may be Jurisprudence, on the other hand, teaches us that “a person can
admitted in evidence. Notarial documents, however, may be sell only what he owns or is authorized to sell; the buyer can as a
presented in evidence without further proof of their authenticity, consequence acquire no more than what the seller can legally
since the certificate of acknowledgment is prima facie evidence transfer.” No one can give what he does not have — nono dat
of the execution of the instrument or document involved. To quod non habet. In the present case, there is no allegation at all
contradict facts in a notarial document and the presumption of that petitioners were authorized by DBP to sell the property to
regularity in its favor, the evidence must be clear, convincing and the private respondents.
more than merely preponderant.
Further, the contract of repurchase that the parties entered into
The CA ruled that the signature of Hermoso De Leon on the presupposes that petitioners could repurchase the property that
Extrajudicial Partition and Quitclaim was forged. However, this they “sold” to private respondents. As petitioners “sold” nothing, it
factual finding is in conflict with that of the RTC. While normally follows that they can also “repurchase” nothing. In this light, the
this Court does not review factual issues, this rule does not apply contract of repurchase is also inoperative and by the same
when there is a conflict between the holdings of the CA and analogy, void.
those of the trial court, as in the present case.

After poring over the records, the SC finds no reason to reverse HEIRS OF SEVERINA SAN MIGUEL VS. CA
the factual finding of the appellate court. A comparison of the
genuine signatures of Hermoso De Leon with his purported FACTS: This case involves a parcel of land originally claimed by
signature on the Deed of Extrajudicial Partition with Quitclaim will Severina San Miguel (petitioners predecessor-in-interest,
readily reveal that the latter is a forgery. As aptly held by the CA, hereafter, Severina). The land is situated in Panapan, Bacoor,
such variance cannot be attributed to the age or the mechanical Cavite with an area of 632 sq. m., more or less. Without
Severina's knowledge, Dominador managed to cause the
COMPILED BY: WIGMORE #WIGMOREFOREVER 57
SALES Case Digest (Atty. Sarona)
Compiled by: Wigmore #wigmoreforever

subdivision of the land into three (3) lots, to wit: Although a contract is the law between the parties, the provisions
of positive law which regulate contracts are deemed written
LRC Psu 1312 - with an area of 108 square meters; therein and shall limit and govern the relations between the
LRC Psu -1313 - Lot 1, with an area of 299 square meters; parties. The Civil Code provisions on sales state:
LRC Psu -1313 - Lot 2, with an area of 225 square meters.
Article 1458. By the contract of sale one of the contracting
On September 25, 1974, Dominador, et al. filed a petition with parties obligates himself to transfer the ownership of and to
the CFI as a land registration court, to issue title over Lots 1 and deliver a determinate thing, and the other to pay a price certain in
2 of LRC Psu-1313, in their names. money or its equivalent. xxx

On July 19, 1977, the Land Registration Commission (hereafter Article 1459. The thing must be licit and the vendor must have a
LRC) rendered a decision directing the issuance of OCT in the right to transfer the ownership thereof at the time it is delivered.
names of Dominador, et al. Subsequently, Severina filed with the
CFI a petition for review of the decision alleging that the land Article 1495. The vendor is bound to transfer the ownership of
registration proceedings were fraudulently concealed by and deliver, as well as warrant the thing which is the object of
Dominador from her., and the court declared the OCT as null and sale (underscoring ours).
void. The Register of Deeds of Cavite issued a TCT in the names
of Severina and her heirs. True, in contracts of sale, the vendor need not possess title
to the thing sold at the perfection of the contract. However,
The trial court issued an order in favor of Severinas heirs and the vendor must possess title and must be able to transfer
ordered that the writ of possession previously issued in favor of title at the time of delivery. In a contract of sale, title only
Severina be implemented. However, the writ was returned passes to the vendee upon full payment of the stipulated
unsatisfied. Subsequently, the trial court ordered that an alias consideration, or upon delivery of the thing sold.
writ of demolition be issued in favor of petitioners, Severina San
Miguel. Again, the writ was not satisfied. Severinas heirs are not in a position to transfer title. SC noted
that there is no proof of ownership in favor of Severinas heirs. In
Severinas heirs, decided not to pursue the writs of possession fact, it is a certain Emiliano Eugenio who holds a tax declaration
and demolition and entered into a compromise (kasunduan) with over the said land in his name. Though tax declarations do not
Dominador, et al. According to the compromise, Severinas heirs prove ownership of the property, tax declarations and receipts
were to sell the subject lots to Dominador, et al. for P1.5 M with can be strong evidence of ownership of land when accompanied
the delivery of TCT conditioned upon the purchase of another lot by possession for a period sufficient for prescription. Severinas
which was not yet titled at an additional sum of P300k. On the heirs have nothing to counter this document.
same day, on August 9, 1993 pursuant to the kasunduan,
Severinas heirs and Dominador, et al. executed a deed of sale Therefore, to insist that Dominador, et al. pay the price under
designated as kasulatan sa bilihan ng lupa. such circumstances would result in Severinas heirs unjust
enrichment. The essence of a sale is the transfer of title or an
Dominador, et al. filed with the trial court a motion praying that agreement to transfer it for a price actually paid or promised.
Severinas heirs deliver the owners copy of the certificate of title
to them. Severinas heirs opposed the motion stressing that Severinas heirs insist that delivery of the certificate of title is
under the kasunduan, the certificate of title would only be predicated on a condition - payment of P300k to cover the sale of
surrendered upon Dominador, et al's payment of the amount of Lot 3. The condition cannot be honored. Article 1183 of the Civil
P300k within two months from August 6, 1993, which was not Code provides that,
complied with.
Impossible conditions, those contrary to good customs or public
Dominador, et al. admitted non-payment of P300k for the reason policy and those prohibited by law shall annul the obligation,
that Severinas heirs have not presented any proof of ownership which depends upon them. If the obligation is divisible, that part
over the untitled parcel of land. Apparently, the parcel of land is thereof which is not affected by the impossible or unlawful
declared in the name of a third party, a certain Emiliano Eugenio. condition shall be valid. xxx

ISSUE: Whether Dominador, et al. may be compelled to pay the Hence, the non-payment of the P300k is not a valid justification
P300k as agreed upon in the kasunduan (as a pre-requisite for for refusal to deliver the certificate of title. Besides, the certificate
the release of the certificate of title), despite Severinas heirs lack of title that covers Lots 1 and 2 were fully paid for by Dominador,
of evidence of ownership over the parcel of land. et al. Therefore, Severinas heirs are bound to deliver the
certificate of title covering the lots.
HELD: No. Severinas heirs anchor their claim on the kasunduan,
stressing on their freedom to stipulate and the binding effect of
contracts. This argument is misplaced. The Civil Code provides:

Article 1306. The contracting parties may establish such


stipulations, clauses, terms and conditions as they may deem
convenient provided they are not contrary to law, morals, good
customs, public order or public policy.

It is basic that the law is deemed written into every contract.


COMPILED BY: WIGMORE #WIGMOREFOREVER 58
SALES Case Digest (Atty. Sarona)
Compiled by: Wigmore #wigmoreforever

2. PRICE should be asked whether its case is one wherein there is no


consideration, or one with a statement of a false consideration. If
1) Price must be Real the former, it is void and inexistent; if the latter, only voidable,
under the Old Civil Code.
a. Price Simulated There is lack of consideration
As observed earlier, the deed of sale of 1936 stated that it had
for its consideration Five Hundred (P500.00) Pesos. In fact,
MAPALO VS. MAPALO however, said consideration was totally absent. The problem,
therefore, is whether a deed which states a consideration that in
FACTS: The spouses Miguel Mapalo and Candida Quiba were fact did not exist, is a contract without consideration, and
the registered owners of a residential land located in therefore void ab initio, or a contract with a false consideration,
Pangasinan. (1,635 sq. m.) The spouses donated the eastern and therefore, at least under the Old Civil Code, voidable.
half of the land to Miguel’s brother – Maximo Mapalo who was
about to get married. When there is no consideration, the contract is null and void
According to Manresa, what is meant by a contract that states a
However, they were deceived into signing, on October 15, 1936, false consideration is one that has in fact a real consideration but
a deed of absolute sale over the entire land in Maximo’s favor. the same is not the one stated in the document.
Their signatures were procured by fraud because they were
made to believe by Maximo and the lawyer who acted as notary In our view, therefore, the ruling of this Court in Ocejo, Perez &
public who "translated" the document, that the same was a deed Co. vs. Flores, 40 Phil. 921, is squarely applicable herein. In that
of donation in Maximo's favor covering one-half of their land. (It case we ruled that a contract of purchase and sale is null and
must be noted that the spouses are illiterate farmers). Although void and produces no effect whatsoever where the same is
the document of sale stated a consideration of Five Hundred without cause or consideration in that the purchase price which
(P500.00) Pesos, the aforesaid spouses did not receive anything appears thereon as paid has in fact never been paid by the
of value for the land. purchaser to the vendor.

In 1938, Maximo Mapalo, without the consent of the spouse, 2.) No, they were no purchasers in good faith. Aside from the fact
registered the sale in his favor. After thirteen years (1951), he that all the parties in these cases are neighbors, except Maximo
sold the land to the Narcisos. (Evaristo, Petronila Pacifico and Mapalo the foregoing facts are explicit enough and sufficiently
Miguel) who thereafter registered the sale and obtained a title in reveal that the Narcisos were aware of the nature and extent of
their favor. the interest of Maximo Mapalo their vendor, over the above-
described land before and at the time the deed of sale in their
In 1952, the Narcisos filed a complaint with the CFI to be favor was executed.
declared owners of the entire land, for possession of its western
portion; for damages; and for rentals. The Mapalo spouses filed The Narcisos were purchaser-in-value but not purchasers in
a counterclaim seeking cancellation of the Narcisos’ titles as to good faith
the western half of the land. They said that their signatures to What was the necessity, purpose and reason of Pacifico Narciso
the deed of sale of 1936 was procured by fraud and that the in still going to the spouses Mapalo and asked them to permit
Narcisos were buyers in bad faith. their brother Maximo to dispose of the above-described land? To
this question it is safe to state that this act of Pacifico Narciso is
They also filed another complaint wherein they asked the court to a conclusive manifestation that they (the Narcisos) did not only
declare deeds of sale of 1936 and of 1951 over the land in have prior knowledge of the ownership of said spouses over the
question be declared null and void as to the western half of said western half portion in question but that they also have
land. recognized said ownership. It also conclusively shows their prior
knowledge of the want of dominion on the part of their vendor
CFI ruled in favor of the Mapalo spouses. Upon appeal filed by Maximo Mapalo over the whole land and also of the flaw of his
Narcisos, CA reversed the lower court’s ruling solely on the title thereto. Under this situation, the Narcisos may be
ground that the consent of the Mapalo spouses to the deed of considered purchasers in value but certainly not as purchasers in
sale of 1936 having been obtained by fraud, the same was good faith.
voidable, not void ab initio, and, therefore, the action to annul the
same, within four years from notice of the fraud, had long
prescribed. (From March 15, 1938). Hence, this appeal. MODINA VS. CA

ISSUES: FACTS: This case involves parcels of land registered under the
1. Whether or not the deed of sale executed in 1936 was null and name of Ramon Chiang. Chiang theorized that the subject
void. YES properties were sold to him by his wife, Merlinda Plana Chiang
2. Whether or not the Narcisos were purchasers in good faith. as evidenced by a Deed of Sale and were subsequently sold by
NO Chiang to the petitioner Serafin Modina. (Dates of sale: August 3,
1979 and August 24, 1979, respectively.)
HELD:
1.) YES, the sale was void. The Civil Code governs the Modina brought a Complaint for Recovery of Possession with
transaction because it was executed in 1936. Accordingly, since Damages against the private respondents before the RTC. Upon
the deed of sale of 1936 is governed by the Old Civil Code, it learning the institution of the said case, Merlinda presented a
COMPILED BY: WIGMORE #WIGMOREFOREVER 59
SALES Case Digest (Atty. Sarona)
Compiled by: Wigmore #wigmoreforever

Complaint-in-intervention, seeking the declaration of nullity of the Since one of the characteristics of a void or inexistent contract is
Deed of Sale between her husband and MODINA on the ground that it does not produce any effect, MERLINDA can recover the
that the titles of the parcels of land in dispute were never legally property from petitioner who never acquired title thereover.
transferred to her husband. She contended that fraudulent acts
were allegedly employed by her husband to obtain a Torrens Title Records show that in the complaint-in-intervention of
in his favor. However, she confirmed the validity of the lease MERLINDA, she did not aver the same as a ground to nullify
contracts with the other private respondents. subject Deed of Sale. In fact, she denied the existence of the
Deed of Sale in favor of her husband. In the said Complaint, her
MERLINDA also admitted that the said parcels of land were allegations referred to the want of consideration of such Deed of
those ordered sold by the CFI of Iloilo in “Intestate Estate of Sale. She did not put up the defense under Article 1490, to
Nelson Plana” where she was appointed as the administratix, nullify her sale to her husband CHIANG because such a defense
being the widow of the deceased, her first husband. An Authority would be inconsistent with her claim that the same sale was
to Sell was issued by the said Probate Court for the sale of the inexistent.
same properties.
2.) Modina was not a purchaser in good faith
RTC ruled in favor of the wife Merlinda declaring the two sales in
August 1979 as void and inexistent. Upon appeal, the CA There are circumstances which are indicia of bad faith on
affirmed in toto the RTC ruling. Mondina’s part:

ISSUES: (1) He asked his nephew, Placido Matta, to investigate the origin
1. Whether or not the sale of subject lots should be nullified. YES of the property and the latter learned that the same formed part
2. Whether or not petitioner Modina was a purchaser in good of the properties of MERLINDA’s first husband;
faith. NO (2) that the said sale was between the spouses;
(3) that when the property was inspected, MODINA met all the
HELD: lessees who informed that subject lands belong to MERLINDA
1.)The sale of the subject lots should be nullified. and they had no knowledge that the same lots were sold to the
husband.
Prohibition of sale between spouses
Art. 1490. The husband and the wife cannot sell property to It is a well-settled rule that a purchaser cannot close his eyes to
each other, except: facts which would put a reasonable man upon his guard to make
(1) when a separation of property was agreed upon in the the necessary inquiries, and then claim that he acted in good
marriage settlements; or faith. His mere refusal to believe that such defect exists, or his
(2) when there has been a judicial separation of property under wilful closing of his eyes to the possibility of the existence of a
Art. 191. defect in his vendor’s title, will not make him an innocent
purchaser for value, if it afterwards develops that the title was in
The sale between Chiang spouses was null and void. The fact defective, and it appears that he had such notice of the
ownership of the lot did not transfer to Ramon Chiang. Hence, defect as would have led to its discovery had he acted with that
the sale to Modina was null and void. The exception to the rule measure of precaution which may reasonably be required of a
laid down in Art. 1490 of the New Civil Code not having existed prudent man in a like situation.
with respect to the property relations of Ramon Chiang and
Merlinda Plana Chiang, the sale by the latter in favor of the
former of the properties in question is invalid for being prohibited VDA. DE CATINDIG VS. HEIRS OF ROQUE
by law. Not being the owner of subject properties, Ramon
Chiang could not have validly sold the same to plaintiff Serafin FACTS: The subject property in this case is a fishpond which
Modina. The sale by Ramon Chiang in favor of Serafin Modina was part of the Malolos Cadastre and has an area of more than
is, likewise, void and inexistent. Serafin Modina is, likewise, void thirteen hectares. As shown in Original Certificate of Title, it is co-
and inexistent. owned or registered in the names of the different persons. (note:
there are 16/16 shares)
A contract of sale without consideration is a void contract
Under Article 1409 of the New Civil Code, enumerating void The co-owners of the fishpond leased it to Mrs. Catindig for a
contracts, a contract without consideration is one such void term of ten years counted from October 1, 1941 for a total rental
contract. One of the characteristics of a void or inexistent of six thousand pesos. After the termination of the lease on
contract is that it produces no effect. So also, inexistent September 30, 1951, Mrs. Catindig remained in possession of
contracts can be invoked by any person whenever juridical the fishpond because she was negotiating with the co-owners for
effects founded thereon are asserted against him. A transferor the purchase thereof. She wanted to buy it for P52,000.
can recover the object of such contract by accion reivindicatoria
and any possessor may refuse to deliver it to the transferee, who On October 18, 1960 German Ramirez, one of the co-owners,
cannot enforce the transfer. executed a deed wherein he sold his 2/16 share to Mrs. Catindig
for P6,500 The sale was annotated on the title on October 19,
Thus, Modina’s insistence that Merlinda cannot attack subject 1960. Two weeks later, Pedro Villanueva, one of the co-owners,
contract of sale as she was a guilty party thereto is equally learned of the sale executed by German Ramirez. That sale
unavailing. retroacted to April 13, 1950. In 1960 the respondents filed this
action against Mrs. Catindig to compel her to allow them to
Merlinda can recover the property redeem the portion sold by German Ramirez. The respondents
COMPILED BY: WIGMORE #WIGMOREFOREVER 60
SALES Case Digest (Atty. Sarona)
Compiled by: Wigmore #wigmoreforever

amended their complaint by including a prayer for the recovery the receiver until the said share is redeemed by the respondents.
of the possession of the fishpond.
Ruling by the Supreme Court:
The RTC declared void certain documents of sale regarding - The receiver (not Asuncion Meneses Vda. de Catindig) should
portions of the fishpond in litigation. It ordered Mrs. Catindig to deliver the possession of the fishpond to the respondents or their
deliver to the respondents (except German Ramirez) the duly authorized representative, together with 14/16 of the net
possession of the said fishpond and to allow the respondents to earnings of the fishpond from January 15, 1964 up to the time
redeem from Mrs. Catindig the 2/16 portion of the fishpond which the possession is delivered to the respondents.
German Ramirez had sold to her. CA affirmed in toto the RTC - The receiver should deliver to Mrs. Catindig a 2/16 share of the
ruling. CA said that Mrs. Catindig did not pay P52,000 (the net earnings of the fishpond, corresponding to the share of
projected sale) and that it the contract was simulated. Hence, German Ramirez, from January 15, 1964 up to the time the said
this appeal. share is redeemed from her.

ISSUE: Whether or not the sale by German Ramizer to Mrs.


Catindig was null and void. SPOUSES LEQUIN VS. SPS. VIZCONDE
SUNDAY, AUGUST 24, 2014
HELD: YES. The alleged sales were null and void. The
conclusive factual finding of the Appellate Court that the alleged FACTS: In 1995, spouses Ramon and Virginia Lequin, residents
sales on April 13 or 14, 1950 of respondents' shares are bought the subject lot consisting of 10,115 sq. m. from one
simulated and void ab initio renders untenable appellant Carlito de Leon. The sale was negotiated by respondent
Catindig's contentions that the remedies available to the Raymundo Vizconde.
respondents, such as an action for annulment, rescission or
reformation, are barred by prescription or laches. In 1997, spouses Vizconde represented to spouses Lequin that
they had also bought from Carlito de Leon a 1,012 sq. m. lot
The alleged sales were absolutely simulated, fictitious or adjacent to the Lequins and built a house thereon.
inexistent contracts (Arts. 1346 and 1409(2)). "The action or
defense for the declaration of the inexistence of a contract does As later confirmed by de Leon, however, the 1,012 sq. m. lot
not prescribe" (Art. 1410). Mere lapse of time cannot give claimed by the Vizcondes is part of the 10,115 sq. m. lot Lequin
efficacy to a void contract. bought from him.

The CA’s finding that the price was not paid or that the statement With the consent of the Vizcondes, spouses Lequin then
in the supposed contracts of sale as to the payment of the price constructed their house on the 500-square meter half-portion of
was simulated fortifies the view that the alleged sales were void. the lot claimed by respondents, as this was near the road.
"If the price is simulated, the sale is void..." (Art. 1471, Civil
Code). Given this situation where the house of Lequins stood on a
portion of the lot allegedly owned by Vizcondes, the former
A contract of sale with no consideration is void consulted a lawyer, who advised them that the 1,012 sq. m. lot
A contract of sale is void and produces no effect whatsoever be segregated from the subject lot whose title they own and to
where the price, which appears thereon as paid, has in fact make it appear that they are selling to respondents 512 square
never been paid by the purchaser to the vendor. Such a sale is meters thereof.
non-existent or cannot be considered consummated.
This sale was embodied in the February 12, 2000 Kasulatan
Mrs. Catindig cannot demand where it was made to appear that the Vizcondes paid PhP
Mrs. Catindig is not entitled to demand the execution of a 15,000 for the purchase of the 512-square meter portion of the
notarized deed of sale for the 14/16 pro indiviso portion of the subject lot.
fishpond. She is not entitled because, as already held, the
alleged sales in her favor are void. In July 2000, petitioners tried to develop the dried up canal
located between their 500-square meter lot and the public
Reasonable value of the use and occupation of the fishpond road. However, the respondents objected, claiming ownership of
should be limited said dried up canal or sapang patay.
We hold that, as a matter of fairness and equity or to avoid unjust
enrichment, the liability of Mrs. Catindig for the reasonable value This prompted the Liquins to look into the ownership of the dried
of the use and occupation of the fishpond should be limited to the up canal and the lot claimed by the respondents Carlito de Leon
period from October 1, 1951 up to the time in January, 1964 told petitioners that what he had sold to respondents was the
when she turned over the fishpond to the receiver, namely, the dried up canal or sapang patay and that the 1,012-square meter
deputy clerk of court of the Court of First Instance of Bulacan, lot claimed by respondents really belongs to petitioners. In 2001,
Malolos Branch I. petitioners filed a complaint praying for the Kasulatan to be
declared as null and void ab initio.
From the compensation of P6,000 per annum which Mrs.
Catindig is obligated to pay to the respondents, should be The RTC found the Kasulatan allegedly conveying 512 square
deducted the 2/16 portion of said compensation, corresponding meters to respondents to be null and void due to: (1) the vitiated
to the share of German Ramirez, from October 1, 1951 to consent of petitioners in the execution of the simulated contract
January, 1964. Thereafter, Mrs. Catindig is entitled to demand of sale; and (2) lack of consideration, since it was shown that
the 2/16 share in the net fruits or earnings of the fishpond from while petitioners were ostensibly conveying to respondents 512
COMPILED BY: WIGMORE #WIGMOREFOREVER 61
SALES Case Digest (Atty. Sarona)
Compiled by: Wigmore #wigmoreforever

square meters of their property, yet the consideration of PhP a loan nor a forbearance of credit. After finality of decision, the
15,000 was not paid to them and, in fact,they were the ones who amount of PhP 50,000 shall earn interest of 12% per annum until
paid respondents PhP 50,000. fully paid.

Upon appeal by the respondent-spouses, CA reversed the There was vitiated consent on the part of Spouses Lequin.
ruling. There was fraud in the execution of the contract used on
petitioners which affected their consent. Petitioners’ reliance and
ISSUE: WON the Kasulatan was null and void. belief on the wrongful claim by respondents operated as a
But take note, on the issue of consent, the SC said that the concealment of a material fact in their agreeing to and in readily
Kasulatan was merely voidable. But on the issue of executing the contract of sale, as advised and proposed by a
consideration, it was void. Final ruling - void. notary public.

HELD: YES Believing that Carlito de Leon indeed sold a 1,012-square meter
portion of the subject property to respondents, petitioners signed
Re: Lack of Consideration the contract of sale based on respondents’ representations. Had
The contract of sale or Kasulatan states that respondents paid petitioners known, as they eventually would sometime in late
petitioners PhP 15,000 for the 512-square meter portion. On its 2000 or early 2001 when they made the necessary inquiry from
face, the above contract of sale appears to be supported by a Carlito de Leon, they would not have entered or signed the
valuable consideration. We, however, agree with the trial court’s contract of sale, much less pay PhP 50,000 for a portion of the
finding that this is a simulated sale and unsupported by any subject lot which they fully own. Thus, petitioners’ consent was
consideration, for respondents never paid the PhP 15,000 vitiated by fraud or fraudulent machinations of Raymundo. In the
purported purchase price. eyes of the law, petitioners are the rightful and legal owners of
the subject 512 square-meter lot anchored on their purchase
The kasulatan did not express the true intent of the parties thereof from de Leon. This right must be upheld and protected.
Lack of consideration was proved by petitioners’ evidence
aliunde showing that the Kasulatan did not express the true
intent and agreement of the parties. As explained above, said HEIRS OF INTAC VS. CA
sale contract was fraudulently entered into through the TUESDAY, AUGUST 19, 2014
misrepresentations of respondents causing petitioners’ vitiated
consent. FACTS: Ireneo Mendoza, married to Salvacion Fermin, was the
owner of the subject property located in Quezon city which he
There can be no doubt that the contract of sale or Kasulatan purchased in 1954. (TCT No. 242655). Ireneo had two children:
lacked the essential element of consideration. respondents Josefina and Martina (respondents), Salvacion
being their stepmother. When he was still alive, Ireneo, also took
It is a well-entrenched rule that where the deed of sale states care of his niece, Angelina, since she was three years old until
that the purchase price has been paid but in fact has never been she got married.
paid, the deed of sale is null and void ab initio for lack of
consideration. Moreover, Art. 1471 of the Civil Code, which On October 25, 1977, Ireneo, with the consent of Salvacion,
provides that “if the price is simulated, the sale is void,” also executed a deed of absolute sale of the property in favor of
applies to the instant case, since the price purportedly paid as Angelina and her husband, Mario (Spouses Intac). Despite the
indicated in the contract of sale was simulated for no payment sale, Ireneo and his family, including the respondents, continued
was actually made. staying in the premises and paying the realty taxes. After Ireneo
died intestate in 1982, his widow and the respondents remained
The contract is void ab intio in the premises. After Salvacion died, respondents still
Consideration and consent are essential elements in a contract maintained their residence there. Up to the present, they are in
of sale. Where a party’s consent to a contract of sale is vitiated the premises, paying the real estate taxes thereon, leasing out
or where there is lack of consideration due to a simulated price, portions of the property, and collecting the rentals.
the contract is null and void ab initio.
The controversy arose when respondents sought the
The PhP 50,000 paid by petitioners to respondents as cancellation of TCT No. 242655, claiming that the sale was only
consideration for the transfer of the 500-square meter lot to simulated and, therefore, void. The heirs of Ireneo, the
petitioners must be restored to the latter. respondents in this case, alleged that:

Otherwise, an unjust enrichment situation ensues. The facts 1. When Ireneo was still alive, Spouses Intac borrowed the title
clearly show that the 500-square meter lot is legally owned by of the property (TCT No. 106530) from him to be used as
petitioners as shown by the testimony of de Leon; therefore, they collateral for a loan from a financing institution;
have no legal obligation to pay PhP 50,000 therefor. 2. They objected because the title would be placed in the names
of said spouses and it would then appear that the couple owned
Considering that the 512 square-meter lot on which respondents’ the property; that Ireneo, however, tried to appease them, telling
house is located is clearly owned by petitioners, then the Court them not to worry because Angelina would not take advantage of
declares petitioners’ legal ownership over said 512 square-meter the situation considering that he took care of her for a very long
lot. The amount of PhP 50,000 should only earn interest at the time; that during his lifetime, he informed them that the subject
legal rate of 6% per annum from the date of filing of complaint up property would be equally divided among them after his death;
to finality of judgment and not 12% since such payment is neither and
COMPILED BY: WIGMORE #WIGMOREFOREVER 62
SALES Case Digest (Atty. Sarona)
Compiled by: Wigmore #wigmoreforever

3. That respondents were the ones paying the real estate taxes vendees, Spouses Intac. There was simply no consideration and
over said property. no intent to sell it.

Spouses Intac countered, among others, that the subject Evidences to prove that there was no absolute deed of sale
property had been transferred to them based on a valid deed of between the parties
absolute sale and for a valuable consideration; that the action to Critical is the testimony of Marietto, a witness to the execution of
annul the deed of absolute sale had already prescribed; that the the subject absolute deed of sale. He testified that Ireneo
stay of respondents in the subject premises was only by personally told him that he was going to execute a document of
tolerance during Ireneo’s lifetime because they were not yet in sale because Spouses Intac needed to borrow the title to the
need of it at that time; and that despite respondents’ knowledge property and use it as collateral for their loan application. Ireneo
about the sale that took place on October 25, 1977, respondents and Salvacion never intended to sell or permanently transfer the
still filed an action against them. full ownership of the subject property to Spouses Intac. Marietto
was characterized by the RTC as a credible witness.
RTC ruled in favor of the respondents saying that the sale to the
spouses Intac was null and void. The CA also ruled that there Aside from their plain denial, the heirs of Intac failed to present
was no consideration in the sale to the spouses Intac and that any concrete evidence to disprove Marietto’s testimony. They
the contract was one for equitable mortgage. claimed that they actually paid P150,000.00 for the subject
property. They, however, failed to adduce proof, even by
ISSUES: circumstantial evidence, that they did, in fact, pay it. Even for the
WON the Deed of Absolute Sale was a simulated contract or a consideration of P60,000.00 as stated in the contract, petitioners
valid agreement. could not show any tangible evidence of any payment therefor.
WON the Deed of Absolute Sale, dated October 25, 1977, Their failure to prove their payment only strengthened Marietto’s
involving the subject real property in Pagasa, Quezon City, was a story that there was no payment made because Ireneo had no
simulated contract or a valid agreement. intention to sell the subject property.

HELD: Angelina’s story, except on the consideration, was consistent


The deed of sale executed by Ireneo and Salvacion was with that of Marietto. Angelina testified that she and her husband
absolutely simulated for lack of consideration and cause and, mortgaged the subject property sometime in July 1978 to finance
therefore, void. the construction of a small hospital in Sta. Cruz, Laguna.
Angelina claimed that Ireneo offered the property as he was in
Articles 1345 and 1346 of the Civil Code provide: deep financial need.
Art. 1345. Simulation of a contract may be absolute or relative.
The former takes place when the parties do not intend to be The contract of sale was only for the purpose of lending the title
bound at all; the latter, when the parties conceal their true of the property to Spouses Intac to enable them to secure a
agreement. loan.
Their arrangement was only temporary and could not give rise to
Art. 1346. An absolutely simulated or fictitious contract is void. A a valid sale. Where there is no consideration, the sale is null and
relative simulation, when it does not prejudice a third person and void ab initio. The case of Lequin vs. VIzconde was cited in this
is not intended for any purpose contrary to law, morals, good case.
customs, public order or public policy binds the parties to their
real agreement. The fact that Ireneo was still in physical possession of the
subject property after the sale is a strong evidence to prove that
Relatively simulated agreement vs. Absolute simulation there was no valid sale between the parties.
If the parties state a false cause in the contract to conceal their More importantly, Ireneo and his family continued to be in
real agreement, the contract is only relatively simulated and the physical possession of the subject property after the sale in 1977
parties are still bound by their real agreement. Hence, where the and up to the present. They even went as far as leasing the
essential requisites of a contract are present and the simulation same and collecting rentals. If Spouses Intac really purchased
refers only to the content or terms of the contract, the agreement the subject property and claimed to be its true owners, why did
is absolutely binding and enforceable between the parties and they not assert their ownership immediately after the alleged sale
their successors in interest took place? Why did they have to assert their ownership of it only
after the death of Ireneo and Salvacion? One of the most striking
In absolute simulation, there is a colorable contract but it has no badges of absolute simulation is the complete absence of any
substance as the parties have no intention to be bound by it. attempt on the part of a vendee to assert his right of dominion
"The main characteristic of an absolute simulation is that the over the property.
apparent contract is not really desired or intended to produce
legal effect or in any way alter the juridical situation of the As heretofore shown, the contemporaneous and subsequent
parties." "As a result, an absolutely simulated or fictitious acts of both parties in this case, point to the fact that the intention
contract is void, and the parties may recover from each other of Ireneo was just to lend the title to the Spouses Intac to enable
what they may have given under the contract." them to borrow money and put up a hospital in Sta. Cruz,
Laguna. Clearly, the subject contract was absolutely simulated
No valid sale took place between Ireneo and Spouses Intac and, therefore, void.
In the case at bench, the Court is one with the courts below that
no valid sale of the subject property actually took place between The Spouses Intac never became the owners of the property
the alleged vendors, Ireneo and Salvacion; and the alleged despite its registration in their names.
COMPILED BY: WIGMORE #WIGMOREFOREVER 63
SALES Case Digest (Atty. Sarona)
Compiled by: Wigmore #wigmoreforever

It is also of no moment that TCT No. 106530 covering the subject If there is a meeting of the minds of the parties as to the price,
property was cancelled and a new TCT (TCT No. 242655)21 was the contract of sale is valid, despite the manner of payment, or
issued in their names. After all, registration does not vest title. As even the breach of that manner of payment. If the real price is
a logical consequence, petitioners did not become the owners of not stated in the contract, then the contract of sale is valid but
the subject property even after a TCT had been issued in their subject to reformation. If there is no meeting of the minds of the
names. parties as to the price, because the price stipulated in the
contract is simulated, then the contract is void. Article 1471 of the
Civil Code states that if the price in a contract of sale is
BUENAVENTURA VS. CA simulated, the sale is void.
TUESDAY, JULY 1, 2014
It is not the act of payment of price that determines the validity of
FACTS: Defendant spouses Leonardo Joaquin and Feliciana a contract of sale.
Landrito are the parents of plaintiffs Consolacion, Nora, Emma Payment of the price has nothing to do with the perfection of the
and Natividad as well as of defendants Fidel, Tomas, Artemio, contract. Payment of the price goes into the performance of the
Clarita, Felicitas, Fe, and Gavino, all surnamed contract. Failure to pay the consideration is different from lack of
JOAQUIN. (Note: So there are two sets of children here.) consideration. The former results in a right to demand the
fulfillment or cancellation of the obligation under an existing valid
Sought to be declared null and void ab initio are certain deeds of contract while the latter prevents the existence of a valid
sale of real property executed by Leonardo Joaquin and contract.
Feliciana Landrito in favor of their co-defendant children and the
corresponding certificates of title issued in their names. The Petitioners failed to show that the prices in the Deeds of Sale
plaintiffs in this case sought for the declaration of nullity of the six were absolutely simulated.
deeds of sale and certificates of title in favor of the defendants. To prove simulation, petitioners presented Emma Joaquin
They alleged that certain deed of sale were null and void ab initio Valdoz’s testimony stating that their father, respondent Leonardo
because they are simulated. Joaquin, told her that he would transfer a lot to her through a
deed of sale without need for her payment of the purchase price.
They said that: The trial court did not find the allegation of absolute simulation of
price credible.
a. Firstly, there was no actual valid consideration for the deeds of
sale xxx over the properties in litis; Petitioners’ failure to prove absolute simulation of price is
b. Secondly, assuming that there was consideration in the sums magnified by their lack of knowledge of their respondent siblings’
reflected in the questioned deeds, the properties are more than financial capacity to buy the questioned lots. On the other hand,
three-fold times more valuable than the measly sums appearing the Deeds of Sale which petitioners presented as evidence
therein; plainly showed the cost of each lot sold. Not only did
c. Thirdly, the deeds of sale do not reflect and express the true respondents’ minds meet as to the purchase price, but the real
intent of the parties (vendors and vendees); and price was also stated in the Deeds of Sale. As of the filing of the
d. Fourthly, the purported sale of the properties in litis was the complaint, respondent siblings have also fully paid the price to
result of a deliberate conspiracy designed to unjustly deprive the their respondent father.
rest of the compulsory heirs (plaintiffs herein) of their legitime.
2nd issue: The general rule is that inadequacy of
Defendants, on the other hand aver: consideration shall not invalidate a contract.

(1) That plaintiffs do not have a cause of action against them as Articles 1355 of the Civil Code states:
well as the requisite standing and interest to assail their titles Art. 1355. Except in cases specified by law, lesion or
over the properties in litis; inadequacy of cause shall not invalidate a contract, unless there
(2) That the sales were with sufficient considerations and made has been fraud, mistake or undue influence. (Emphasis
by defendants parents voluntarily, in good faith, and with full supplied)
knowledge of the consequences of their deeds of sale; and
(3) That the certificates of title were issued with sufficient factual Article 1470 of the Civil Code further provides:
and legal basis. Art. 1470. Gross inadequacy of price does not affect a contract
of sale, except as may indicate a defect in the consent, or that
RTC ruled in favor of the defendants (respondents in this case) the parties really intended a donation or some other act or
and dismissed the complaint. Upon appeal, the CA upheld RTC’s contract. (Emphasis supplied)
ruling.
Petitioners failed to prove any of the instances mentioned in
ISSUES: Articles 1355 and 1470 of the Civil Code which would invalidate,
1. Whether the Deeds of Sale are void for lack of consideration. or even affect, the Deeds of Sale. Indeed, there is no
NO requirement that the price be equal to the exact value of the
2. Whether the Deeds of Sale are void for gross inadequacy of subject matter of sale. All the respondents believed that they
price. NO received the commutative value of what they gave.

HELD: Ruling: In the instant case, the trial court found that the lots were
1st issue: There was a consideration. sold for a valid consideration, and that the defendant children
actually paid the purchase price stipulated in their respective
COMPILED BY: WIGMORE #WIGMOREFOREVER 64
SALES Case Digest (Atty. Sarona)
Compiled by: Wigmore #wigmoreforever

Deeds of Sale. Actual payment of the purchase price by the mortgage debt contracted by Catalina in favor of Laurelia and
buyer to the seller is a factual finding that is now conclusive upon shall cause transfer of said annotation to the title to be issued in
us. WHEREFORE, we AFFIRM the decision of the Court of her (Purificacion's) name; and furthermore that any and all
Appeals in toto. expenses for segregation survey, re-titling and annotation of said
mortgage shall be shouldered by said Purificacion Arce-Caluza;
Before the agreement could be implemented, Purificacion died.
b. Price is “False” Consequently, another compromise agreement was executed
stating that Corazon and Catalina agreed that title to the
MACAPAGAL vs. CATALINA O. REMORIN, CORAZON southernmost apartment as well as the portion of the lot
CALUZA BAMRUNGCHEEP, and LAURELIA CALUZA- occupied thereby shall be transferred direct to its interested
VALENCIANO buyer with defendant Catalina assuming and paying (from the
proceeds of the sale) her mortgage obligation with Laurelia; any
FACTS: Lots 24 and 25 were registered in the name of Candido and all expenses for segregation survey, re-titling, capital gains
Caluza under Transfer Certificate of Title (TCT) No. 160544. taxes and those connected with the annotation and/or release of
Purificacion Arce-Caluza (Purificacion) is his second wife. said mortgage should now be shouldered by defendant Catalina
Corazon Caluza-Bamrungcheep (Corazon) is his legally adopted O. Remorin. Corazon then sold the subject Lot to Laurelia by
daughter during his first marriage. After Candido died in 1981, virtue of a deed entitled "Sale of Unsegregated Portion of Land."
Corazon and Purificacion executed a Deed of Extrajudicial However, Catalina also sold the same lot to Macapagal claiming
Settlement adjudicating between themselves the properties of to be authorized under the Compromise Agreement. Macapagal
Candido, as the latter's surviving heirs. sought to nullify the sale executed by Corazon in favor of
Laurelia and to declare valid the one executed by Catalina in her
Lots 24 and 25, together with Lot 23, which was registered in favor.
Candido's name, were adjudicated to Corazon. Purificacion got
Candido's land in Bulacan. However, administration of Lots 23, RTC rendered judgment in favor of petitioner. Corazon and
24 and 25 were entrusted to Purificacion by Corazon as she had Laurelia appealed to the Court of Appeals which reversed the
to leave for Thailand after her marriage to a Thai. decision of the trial court.

Unknown to Corazon, Purificacion executed an Affidavit of Loss Macapagal’s contention: the sale executed by Catalina in her
alleging that the TCTs of Lots No 23, 24 and 25 were lost and favor should prevail over the one executed by Corazon in favor
could no longer be found. She filed a petition for the issuance of of Laurelia, as Catalina was the one authorized to sell the
new owner's duplicates of title alleging that she was her disputed property under the Compromise Agreement
deceased husband's sole heir. The petition was granted and new
TCTs were issued in Purificacion's name. Purificacion then sold Respondent’s contention: Corazon, the registered owner of the
the lots to Catalina Remorin (Catalina) and Catalina mortgaged disputed property, did not give Catalina authority to sell the lot. It
Lots 24 and 25 to L & R Lending Corp. was provided in the Agreement that Catalina shall pay off her
mortgage obligation and incidental expenses from the proceeds
Corazon filed a complaint for reconveyance and damages of the sale only to reassure Catalina that her obligation would be
against Purificacion and Catalina upon discovery of sale. Plaintiff paid in the event that Corazon sells the property.
alleged that the two defendants connived with each other in
transferring the three lots in their names through simulated sales. ISSUE: WON Catalina was authorized to sell the land as
Corazon likewise filed a criminal complaint for falsification and provided by their Compromise Agreement.
perjury against the two.
RULING: The Compromise Agreement dated September 9, 1988
Catalina executed a Deed of Transfer, signed by Purificacion as cannot be taken as a waiver of Corazon's authority to sell and
witness, admitting the wrong they did in illegally transferring the grant thereof to Catalina considering that the Agreement merely
lots in their names and acknowledging Corazon to be the rightful provided that Catalina pay off her mortgage obligation and
owner under the Deed of Extrajudicial Settlement. Corazon incidental expenses from the proceeds of the sale. Although it
presented the Deed of Transfer before the Register of Deeds of was imperative, as part of the compromise, that the money come
Quezon City and Catalina's TCT over Lots 24 and 25 was from the proceeds of the sale, it was not expressly stated, nor did
cancelled and a TCT was issued in Corazon's name. it necessarily mean, that Catalina herself be the one to directly
sell the property. Authority to sell must be couched in clear and
Prior thereto, however, Catalina mortgaged Lots 24 and 25 to unmistakable language.
respondent Laurelia Caluza-Valenciano (Laurelia) to pay off her
mortgage indebtedness to L & R Lending Corporation. The Moreover, intent to give Catalina authority to sell may not be
inscription of the mortgage in favor of Laurelia was carried over easily attributed to Corazon considering that the latter had to file
to Corazon's TCT. the reconveyance case as a result of Purificacion's and
Catalina's acts of transferring the disputed lot in their names.
-Corazon, Purificacion, Catalina, and Laurelia executed a
Memorandum of Agreement to settle Civil Case. It stipulated that In contract interpretation, analysis is not to be limited to the
Corazon cedes and grants unto and in favor of Purificacion full words used in the contract, as they may not accurately reflect the
ownership and other real rights over the southernmost apartment parties' true intent. If the words of the contract appear to be
as well as the portion of the lot occupied thereby subject to the contrary to the evident intention as revealed by the
condition that Purificacion shall assume satisfaction of the circumstances, the latter shall prevail over the former.

COMPILED BY: WIGMORE #WIGMOREFOREVER 65


SALES Case Digest (Atty. Sarona)
Compiled by: Wigmore #wigmoreforever

The fact that the deed of sale between respondents Corazon and deed of sale and declared that the Sale dated April 1, 1980, as
Laurelia did not accurately reflect the true consideration thereof valid and enforceable. No appeal having been made, the
is not cause for declaration of its nullity. When the parties decision became final and executory.
intended to be bound by the contract except that it did not reflect
the actual purchase price of the property, there is only a relative Examining the terms and conditions of the "Deed of Sale" dated
simulation of the contract which remains valid and enforceable. It April 1, 1980, the P45,000.00 balance is payable only "after the
cannot be declared null and void since it does not fall under the property covered by T.C.T. No. 135671 has been partitioned and
category of an absolutely simulated or fictitious contract. The subdivided, and title issued in the name of the BUYER" hence,
contract of sale is valid but subject to reformation. vendor Roque cannot demand payment of the balance unless
and until the property has been subdivided and titled in the name
Petition denied. of private respondents. Devoid of any stipulation that “ownership
in the thing shall not pass to the purchaser until he has fully paid
the price,” ownership of the thing shall pass from the vendor to
c. Non-Payment of Price the vendee upon actual or constructive delivery of the thing sold
even if the purchase price has not yet been fully paid. The failure
of the buyer to make good the price does not, in law, cause
CLARA M. BALATBAT vs CA, SPS. REPUYAN ownership to evert to the seller unless the bilateral contract of
G.R. No. 109410, August 28, 1996 sale is first rescinded or resolved pursuant to Article 1191 of the
New Civil Code.
FACTS: A parcel of land was acquired by plaintiff Aurelio Roque
and Maria Mesina during their conjugal union. Maria died on Non-payment only creates a right to demand the fulfillment of the
August 28, 1966. obligation or to rescind the contract.

On June 15, 1977, Aurelio filed a case for partition. The trial court With respect to the non-delivery of the possession of the subject
in that case for partition, held that Aurelio is entitled to the ½ property to the private respondent, suffice it to say that
portion of his share in the conjugal property, and the other half ownership of the thing sold is acquired only from the time of
which formed part of the estate of Maria Mesina, will be divided delivery thereof, actual or constructive.
equally between him and their 4 children receiving 1/5 each. The
decision having become final and executory, the Register of A contract of sale being consensual, it is perfected by the mere
Deeds of Manila issued a transfer certificate of title on October 5, consent of the parties. Delivery of the thing bought or payment of
1979 according to the ruling of the court. the price is not necessary for the perfection of the contract; and
failure of the vendee to pay to price after the execution of the
On April 1, 1980, Aurelio sold his 6/10 shares to spouses Aurora contract does not make the sale null and void for lack of
Tuazon-Repuyan and Jose Repuyan, as evidenced by a deed of consideration but results at most in default on the part of the
absolute sale. On June 21, 1980, Aurora caused the annotation vendee, for which the vendor may exercise his legal remedies.
of her affidavit of adverse claim. However, on August 20, 1980,
Aurelio filed a complaint for rescission of contract grounded on
the buyers’ failure to pay the balance of the purchase price. 2) Must be in Money or its Equivalent

Subsequently, on February 4, 1982, another deed of absolute


sale was executed between Aurelio and his children, and herein IMELDA ONG, ET AL. vs ALFREDO ON ET AL.
petitioner Clara Balatbat, involving the entire lot. Balatbat filed a G.R. No. L-67888, October 8, 1985
motion for the issuance of writ of possession, which was granted
by the court on September 20, 1982, subject to valid rights and FACTS: On February 25, 1976, Imelda Ong for and in
interests of third persons. Balatbat filed a motion to intervene in consideration of One (1.00) Peso and other valuable
the rescission case, but did not file her complaint in intervention. considerations, executed in favor of Sandra Maruzzo, then a
The court ruled that the sale between Aurelio and Aurora is valid. minor, a Quitclaim Deed whereby she transferred, released, and
On March 3, 1987 however, Balatbat filed a notice of lis pendens assigned all her rights and title over a parcel of land in Makati.
before the Register of Deeds regarding the subject property.
However, on November 19, 1980, Imelda Ong revoked the
ISSUES: aforesaid Quitclaim and donated the property to her son Rex.
W/N the alleged sale to Spouses Repuyan was merely
executor Subsequently, Sandra Maruzzo on June 20, 1983, through here
guardian ad litem Alfredo Ong, filed with the RTC an action for
HELD: the recovery of ownership/possession and nullification of the
No. The sale was consummated, hence, valid and enforceable. Deed of Donation in favor of Rex.

Contrary to petitioner's contention that the sale dated April 1, Petitioners claimed that the Quitclaim Deed is null and void
1980 in favor of Spouses Repuyan was merely executory for the inasmuch as it is equivalent to a Deed of Donation, acceptance
reason that there was no delivery of the subject property and that of which by the donee is necessary to give it validity. Further, it is
consideration/price was not fully paid, the Court finds the sale as averred that the donee, Sandra Maruzzo, being a minor, had no
consummated, hence, valid and enforceable. The Court legal personality and therefore incapable of accepting the
dismissed vendor's Aurelio Roque complaint for rescission of the donation.

COMPILED BY: WIGMORE #WIGMOREFOREVER 66


SALES Case Digest (Atty. Sarona)
Compiled by: Wigmore #wigmoreforever

estate of Hilarion which they contend were sold by Hilario


The trial court ruled in favor of Maruzzo and held that the through two deeds of sale where the consideration for the lands
Quitclaim Deed is equivalent to a Deed of Sale and, hence, there was one (1.00) Peso and services rendered, being rendered and
was a valid conveyance in favor of the latter. to be rendered.

Appealing to the IAC, petitioners additionally contends that the Bagnas et al. filed a case against respondents seeking
One (1.00) Peso consideration is not a consideration at all to annulment of the deeds of sale as fictitious, fraudulent or
sustain the ruling that the Quitclaim Deed is equivalent to a sale. falsified, or alternatively, as donations void for want of
The IAC however affirmed the TC. acceptance embodied in a public instrument. In answer to the
complaint, the respondents denied the alleged fictitious or
ISSUE: W/N the quitclaim is equivalent to a deed of sale or fraudulent character of the sales in their favor, asserting that the
to a deed of donation said sales were made for good and valuable consideration.

HELD: The Quitclaim Deed is equivalent to a deed of sale. A ISSUE: W/N the said consideration is valid.
careful perusal of the subject deed reveals that the conveyance
of the one- half (½) undivided portion of the above-described HELD: The Court ruled that the deeds of sale are void and are of
property was for and in consideration of the One (P 1.00) Peso no force and effect.
and the other valuable considerations (emphasis supplied) paid
by private respondent Sandra Maruzzo through her Upon the consideration alone that the apparent gross, not to say
representative, Alfredo Ong, to petitioner Imelda Ong. Stated enormous, disproportion between the stipulated price (in each
differently, the cause or consideration is not the One (P1.00) deed) of P l.00 plus unspecified and unquantified services and
Peso alone but also the other valuable considerations. the undisputably valuable real estate allegedly sold worth at least
P10,500.00 going only by assessments for tax purposes which, it
Although the cause is not stated in the contract it is presumed is well-known, are notoriously low indicators of actual value
that it is existing unless the debtor proves the contrary (Article plainly and unquestionably demonstrates that they state a false
1354 of the Civil Code). One of the disputable presumptions is and fictitious consideration, and no other true and lawful cause
that there is a sufficient cause of the contract. It is a legal having been shown, the Court finds both said deeds, insofar as
presumption of sufficient cause or consideration supporting a they purport to be sales, not merely voidable, but void ab initio.
contract even if such cause is not stated therein (Article 1354,
New Civil Code of the Philippines.) This presumption cannot be The transfers in question being void, it follows as a necessary
overcome by a simple assertion of lack of consideration consequence that the properties purportedly conveyed remained
especially when the contract itself states that consideration was part of the estate of Hilario Mateum, said transfers
given, and the same has been reduced into a public instrument notwithstanding, recoverable by his intestate heirs, the
with all due formalities and solemnities. To overcome the petitioners herein, whose status as such is not challenged.
presumption of consideration the alleged lack of consideration
must be shown by preponderance of evidence in a proper action. Even if the “contract of sale” would be shown as a donation
The execution of a deed purporting to convey ownership of a (apparently, this was the intent of the donor), failure to conform to
realty is in itself prima facie evidence of the existence of a the requirements would not make it a valid donation.
valuable consideration, the party alleging lack of consideration
has the burden of proving such allegation. There is no Contract of Sale for lack of consideration. Likewise,
there is also no valid deed of donation for failure to conform to
Even granting that the Quitclaim deed in question is a donation, the requirements of donation.
Article 741 of the Civil Code provides that the requirement of the
acceptance of the donation in favor of minor by parents of legal
representatives applies only to onerous and conditional REPUBLIC vs PRDC and CA
donations where the donation may have to assume certain G.R. No. L-10141, January 31, 1958
charges or burdens. Donation to an incapacitated donee does
not need the acceptance by the lawful representative if said FACTS: The Republic brought an action against Apostol for the
donation does not contain any condition. In simple and pure collection of sums owing to it for his purchase of Palawan
donation, the formal acceptance is not important for the donor Almaciga and other logs. His total debt amounted to some
requires no right to be protected and the donee neither P34,000. PRDC intervened claiming that Apostol, as President of
undertakes to do anything nor assumes any obligation. The the company, without prior authority, took goods (steel sheets,
Quitclaim now in question does not impose any condition. pipes, bars, etc) from PRDC warehouse and appropriated them
to settle his personal debts in favor of the government. The
Republic opposed the intervention of PRDC, arguing that price is
ISAAC BAGNAS ET AL. vs. CA always paid in money and that payment in kind is no payment at
G.R. No. L-38498, August 10, 1989 all; hence, money and not the goods of PRDC are under dispute.

FACTS: Hilario Mateum died without a will and was survived only ISSUE: W/N payment in kind is equivalent to price paid in
by collateral relatives. Bagnas et al., the petitioners, were his money.
nearest kin. The respondents Retonil et al. on the other hand
were relatives to a farther extent. HELD: Yes. The Government argues that "Price . . . is always
paid in terms of money and the supposed payment being in kind,
Retonil et al. claims ownership of 10 parcels of land from the it is no payment at all, "citing Article 1458 of the new Civil Code.
COMPILED BY: WIGMORE #WIGMOREFOREVER 67
SALES Case Digest (Atty. Sarona)
Compiled by: Wigmore #wigmoreforever

However, the same Article provides that the purchaser may pay settle the conflicting claims, petitioners could have presented the
"a price certain in money or its equivalent," which means that contract of sale. However, it was not presented in evidence.
they meant of the price need not be in money. Whether the G.I. Petitioners aver that even if the (unsigned) deed was not
sheets, black sheets, M. S. Plates, round bars and G. I. pipes produced, Jose “admitted preparing said deed in accordance
claimed by the respondent corporation to belong to it and with their agreement”.
delivered to the Bureau of Prison by Macario Apostol in payment
of his account is sufficient payment therefore, is for the court to We do not agree with petitioners. Assuming arguendo that such
pass upon and decide after hearing all the parties in the case. draft deed existed, it does not necessarily follow that there
Should the trial court hold that it is as to credit Apostol with the was already a definite agreement as to the price. If there was,
value or price of the materials delivered by him, certainly the why then did private respondent Jose de la Cruz not sign it? If
herein respondent corporation would be affected adversely if its indeed the draft deed of sale was that important to petitioners'
claim of ownership of such sheets, plates, bars and pipes is true. cause, they should have shown some effort to procure it. They
could have secured it through a subpoena ducestecum or thru
the use of one of the modes of discovery. But petitioners made
3) Must be Certain or Ascertainable at Perfection no such effort. And even if produced, it would not have
commanded any probative value as it was not signed.
VILLANUEVA VS. CA
267 SCRA 89 The price of the leased land not having been fixed, the essential
G.R. NO. 107624 elements which give life to the contract were lacking. It follows
JANUARY 28, 1997 that the lessee cannot compel the lessor to sell the leased land
to him.
FACTS: Petitioner Gamaliel Villanueva has been a tenant-
occupant of a unit in an apartment building erected on a parcel of The price must be certain; it must be real, not fictitious. It is not
land owned by private respondents dela Cruz. In 1986, Jose dela necessary that the certainty of the price be actual or determined
Cruz offered said land with the apartment building for sale and at the time of executing the contract. The fact that the exact
petitioners (Gamaliel and Irene) showed interest in the property. amount to be paid therefor is not precisely fixed, is no bar to an
action to recover such compensation, provided the contract, by
As initial step, Jose gave Irene a letter of authority for her to its terms, furnishes a basis or measure for ascertaining the
inspect the property. Since the property was in arrears for amount agreed upon. The price could be made certain by the
payment of realty taxes, Jose approached Irene and asked for a application of known factors. A contract of sale is not void for
certain amount to pay for the taxes so that the property would be uncertainty when the price, though not directly stated in terms of
cleared of any encumbrance. Irene gave 10k (5k on 2 pesos and centavos, can be made certain by reference to
occasions). It was agreed by them that the 10k would form part existing invoices identified in the agreement.
of the sale price of 550k.
In the instant case, however, what is dramatically clear from the
Thereafter, Jose went to Irene, bringing with him Mr. Sabio, evidence is that there was no meeting of mind as to the price,
requesting her to allow Sabio to purchase ½ of the property, to expressly or impliedly, directly or indirectly.
which they consented, so they would just purchase the other half
(265k, having paid the 10k). Dela Cruz executed in favor of their
co-defendants (Guido and Felicitas Pile) a Deed of Assignment MORENO, JR. VS. PRIVATE MANAGEMENT OFFICE
of the other ½ portion of the land, wherein Gamaliel’s apartment 507 SCRA 63
unit is situated. This was purportedly as full payment and G.R. NO. 159373
satisfaction of an indebtedness obtained from the Piles. TCT was NOVEMBER 16, 2006
later issued in the name of the Piles.
FACTS: The subject-matter in the civil case is the J. Moreno
Soon, Gamaliel learned about the assignment and issuance of Building (formerly known as the North Davao Mining
new TCT. Petitioners elevated their complaint to the Court Building) – or more specifically, the 2nd, 3rd, 4th, 5th and
(specific performance). They contend that a contract of sale has 6th floors of the building.
been perfected and that the 10k formed part of the purchase
price (necessarily then, there must have been an agreement as Moreno is the owner of the Ground Floor, the 7th Floor and
to the price). They cite Art 1482: Whenever earnest money is the Penthouse of the J. Moreno Building and the lot on which it
given in a contract of sale, it shall be considered as part of the stands.
price and proof of perfection of the contract. On the other hand,
private respondents claim that what was agreed upon was that Private Management Office (formerly, Asset Privatization Trust
the 10k be primarily intended as payment for realty tax, and was or APT) on the other hand, is the owner of the 2nd, 3rd, 4th,
going to for part of the consideration of the sale if the transaction 5th and 6th floors of the building, the subject-matter of this suit.
would finally be consummated. They insist that there was no
clear agreement as to the true amount of consideration. On February 13, 1993, APT called for a conference for the
purpose of discussing Moreno’s right of first refusal over the
ISSUE: Was there a perfected contract of sale? NO floors of the building owned by APT. At said meeting, APT
informed Moreno that the proposed purchase price for said floors
HELD: After a review of the evidence, SC found that there was was P21 Million.
no agreement as to the price (based on the testimonies). To

COMPILED BY: WIGMORE #WIGMOREFOREVER 68


SALES Case Digest (Atty. Sarona)
Compiled by: Wigmore #wigmoreforever

In a letter dated February 22, 1993, APT, informed Moreno price on or before February 26, 1993. The balance will be
that the Board of Trustees (BOT) of APT "is in agreement due within fifteen (15) days after Mr. Moreno receives the
that Mr. Jose Moreno, Jr. has the right of first refusal" and formal notice of approval of the indicative price. xxx
requested Moreno to deposit 10% of the "suggested indicative
price" of P21 million on or before February 26, 1993. The letter clearly states that P21M is merely a "suggested
Moreno paid the P21 million on February 26, 1993. APT issued indicative price" of the subject floors as it was yet to be approved
an OR for the said payment. by the BOT.

But later, APT wrote Moreno that its Legal Department has Before the Board could confirm the suggested indicative price,
questioned the basis for the computation of the indicative price the Committee on Privatization must first approve the terms of
for the said floors. Thus, on April 2, 1993, APT wrote Moreno that the sale or disposition. The imposition of this suspensive
the APT BOT has "tentatively agreed on a settlement price of condition finds basis under Proclamation No. 5022 which
P42,274,702.17" for the said floors. vests in the Committee the power to approve the sale of
government assets, including the price of the asset to be sold
RTC ruled in favor of Moreno, declared that there was a (apparently government pala itong APT, and may procedure na
perfected contract of sale and ordered APT to sell the subject sinusunod sa law).
floors at P21M.
Other discussions that may be relevant:
CA reversed, hence the petition. On Moreno’s argument that the "suggested indicative price" of
P21M is not a proposed price, but the selling price indicative of
ISSUE: WON there was a perfected contract of sale over the the value at which APT was willing to sell.
subject floors at the price of 21 Million. NO
The trial court relied upon the definition of the word "indicative"
HELD: A contract of sale is perfected at the moment there is a under the Webster Comprehensive Dictionary, International
meeting of minds upon the thing which is the object of the Edition. According to Webster, "to indicate" is to point out; direct
contract and upon the price. Consent is manifested by the attention; to indicate the correct page. "Indicative" is merely the
meeting of the offer and the acceptance upon the thing and the adjective of the verb to indicate. xxx
cause which are to constitute the contract. The offer must be
certain and the acceptance absolute. Under the Rules of Court, the terms are presumed to have been
used in their primary and general acceptation, but evidence is
To reach that moment of perfection, the parties must agree on admissible to show that they have a local, technical, or
the same thing in the same sense, so that their minds meet as to otherwise peculiar signification, and were so used and
all the terms. They must have a distinct intention common to both understood in the particular instance, in which case the
and without doubt or difference; until all understand alike, there agreement must be construed accordingly.
can be no assent, and therefore no contract. The minds of
parties must meet at every point; nothing can be left open for The reliance of the trial court in the Webster definition of the term
further arrangement "indicative," as also adopted by Moreno, is misplaced. The
transaction at bar involves the sale of an asset under a
So long as there is any uncertainty or indefiniteness, or future privatization scheme which attaches a peculiar meaning or
negotiations or considerations to be had between the parties, signification to the term "indicative price." Under No. 6.1 of the
there is not a completed contract, and in fact, there is no contract General Bidding Procedures and Rules of respondent, "an
at all. indicative price is a ballpark figure and [respondent] supplies
such a figure purely to define the ball-park." The plain contention
Once there is concurrence of the offer and acceptance of the of Moreno that the transaction involves an "ordinary arms-length
object and cause, the stage of negotiation is finished. This sale of property" is unsubstantiated and leaves much to be
situation does not obtain in the case at bar. The letter of desired. This case sprung from a case of specific performance
February 22, 1993 and the surrounding circumstances clearly initiated by Moreno who has the burden to prove that the case
show that the parties are not past the stage of negotiation, should be spared from the application of the technical terms in
hence there could not have been a perfected contract of sale. the sale and disposition of assets under privatization.

The letter is clear evidence that APT did not intend to sell the He failed to discharge the burden.
subject floors at the price certainofP21M, viz.:
It appears in the case at bar that Moreno’s construction of
(This letter was addressed to Moreno’s Atty.) the letter of February 22, 1993 – that his assent to the
xxx We are pleased to inform you that the Board is in "suggested indicative price" of P21M converted it as the price
agreement that Mr. Jose Moreno, Jr. has the right of first certain, thus giving rise to a perfected contract of sale– is his
refusal. This will be confirmed by our Board during the next own subjective understanding. As such, it is not shared by APT.
board meeting on February 26, 1993. In the meantime, Under American jurisprudence, mutual assent is judged by an
please advise Mr. Moreno that the suggested indicative price objective standard, looking to the express words the parties used
for APT’s five (5) floors of the building in question is P21 in the contract. Under the objective theory of contract,
Million. understandings and beliefs are effective only if shared. Based on
the objective manifestations of the parties in the case at bar,
If Mr. Moreno is in agreement, he should deposit with there was no meeting of the minds.
APT the amount of P2.1 Million equivalent to 10% of the
COMPILED BY: WIGMORE #WIGMOREFOREVER 69
SALES Case Digest (Atty. Sarona)
Compiled by: Wigmore #wigmoreforever

4. Manner of Payment of Price ESSENTIAL ISSUES:


WON there was a perfected contract to repurchase the
NAVARRA VS. PLANTERS foreclosed properties between the petitioners and the
527 SCRA 561 private respondent Planters Development Bank. NO
G.R. NO. 172674 WON the parties never got past the negotiation stage. YES
JULY 12, 2007
HELD:
FACTS: The Navarras obtained a loan of P1,200,000.00 from In general, contracts undergo three distinct stages: negotiation,
Planters Bank and, by way of security therefor, executed a deed perfection or birth, and consummation. Negotiation begins from
of mortgage over their five (5) parcels of land. Unfortunately, the the time the prospective contracting parties manifest their
couple failed to pay their loan obligation. Hence, Planters Bank interest in the contract and ends at the moment of their
foreclosed on the mortgage. The one year redemption period agreement. Perfection or birth of the contract takes place when
expired without the Navarras having redeemed the foreclosed the parties agree upon the essential elements of the contract,
properties. i.e., consent, object and price. Consummation occurs when
the parties fulfill or perform the terms agreed upon in the
On the other hand, co-petitioner RRRC Development contract, culminating in the extinguishment thereof.
Corporation (RRRC) is a real estate company owned by the
parents of Carmelita Bernardo Navarra. RRRC itself obtained a A negotiation is formally initiated by an offer which should
loan from Planters Bank secured by a mortgage over another set be certain with respect to both the object and the cause or
of properties owned by RRRC. The loan having been likewise consideration of the envisioned contract. In order to produce a
unpaid, Planters Bank similarly foreclosed the mortgaged assets contract, here must be acceptance, which may be express or
of RRRC. implied, but it must not qualify the terms of the offer. In other
words, it must be identical in all respects with that of the offer so
Unlike the Navarras, however, RRRC was able to negotiate with as to produce consent or meeting of the minds.
the Bank for the redemption of its foreclosed properties by way
of a concession whereby the Bank allowed RRRC to refer to it Here, the Navarras assert that the following exchange of
would-be buyers of the foreclosed RRRC properties who would correspondence between them and Planters Bank constitutes
remit their payments directly to the Bank, which payments would the offer and acceptance, thus:
then be considered as redemption price for RRRC. Eventually,
the foreclosed properties of RRRC were sold to third persons Letter dated July 18, 1985 of Jorge Navarra:
whose payments therefor, directly made to the Bank, were in
excess by P300,000.00 for the redemption price. This will formalize my request for your kind consideration
in allowing my brother and me to buy back my house and
In the meantime, Jorge Navarra sent a letter {*contents of lot and my restaurant building and lot together with the
the letters are found in the ruling*} to Planters Bank, adjacent road lot. Since my brother, who is working in
proposing to repurchase the five (5) lots earlier auctioned to the Saudi Arabia, has accepted this arrangement only
Bank, with a request that he be given until August 31, 1985 to recently as a result of my urgent offer to him, perhaps it will
pay the down payment of P300,000.00. In response, Planters be safe for us to set August 31, 1985 as the last day for
Bank, thru its Vice-President Ma. Flordeliza Aguenza, wrote back the payment of a P300,000.00 down payment. I hope you
Navarra via a letter dated August 16, 1985. Then, on January 21, will grant us the opportunity to raise the funds within this
1987, Planters Bank sent a letter to Jorge Navarra informing period, which includes an allowance for delays.
him that it could not proceed with the documentation of the
proposed repurchase of the foreclosed properties on account The purchase price, I understand, will be based on the
of his non-compliance with the Bank’s request for the redemption value plus accrued interest at the prevailing
submission of the needed board resolution of RRRC, thus, rate up to the date of our sales contract.
demanding that they surrender and vacate the properties in
question for their failure to exercise their right of redemption. Maybe you can give us a long term payment scheme on the
basis of my brother’s annual savings of roughly
The Navarras filed their complaint for Specific Performance US$30,000.00 everytime he comes home for his home
with Injunction against Planters Bank, alleging that a perfected leave. I realize that this is not a regular transaction but I
contract of sale was made between them and Planters Bank am seeking your favor to give me a chance to reserve
whereby they would repurchase the subject properties for whatever values I can still recover from the properties and
P1,800,000.00 with a down payment of P300,000.00. In its to avoid any legal complications that may arise as a
Answer, Planters Bank asserted that there was no perfected consequence of the total loss of the Balangay lot. I hope that
contract of sale because the terms and conditions for the you will extend to me your favorable action on this grave
repurchase have not yet been agreed upon. matter.

The RTC ruled that there was a perfected contract of sale Letter dated August 16, 1985 of Planters Bank:
between the Navarras and Planters Bank. The CA reversed
the decision citing Article 1319 as basis, declaring that the Regarding your letter dated July 18, 1985, requesting that
acceptance of the offer was not absolute. we give up to August 31, 1985 to buy back your house and
lot and restaurant and building subject to a P300,000.00
downpayment on the purchase price, please be advised that
the Collection Committee has agreed to your request.
COMPILED BY: WIGMORE #WIGMOREFOREVER 70
SALES Case Digest (Atty. Sarona)
Compiled by: Wigmore #wigmoreforever

Please see Mr. Rene Castillo, Head, Acquired Assets Unit, may be formalized.
as soon as possible for the details of the transaction so that
they may work on the necessary documentation. Such statement in the Bank’s letter clearly manifests lack of
agreement between the parties as to the terms of the purported
Given the above, the basic question that comes to mind is: contract of sale/repurchase, particularly the mode of payment of
Was the offer certain and the acceptance absolute enough the purchase price and the period for its payment. The law
so as to engender a meeting of the minds between the requires acceptance to be absolute and unqualified.
parties? Definitely not.
As it is, the Bank’s letter is not the kind which would constitute
While the foregoing letters indicate the amount of P300,000.00 acceptance as contemplated by law for it does not evince any
as down payment, they are, however, completely silent as to categorical and unequivocal undertaking on the part of the Bank
how the succeeding instalment payments shall be made. to sell the subject properties to the Navarras.

At most, the letters merely acknowledge that the down payment The Navarras’ attempt to prove the existence of a perfected
of P300,000.00 was agreed upon by the parties. However, this contract of sale all the more becomes futile in the light of the
fact cannot lead to the conclusion that a contract of sale evidence that there was in the first place no acceptance of their
had been perfected. Quite recently, this Court held that before offer. It should be noted that aside from their first letter dated
a valid and binding contract of sale can exist, the manner of July 18, 1985, the Navarras wrote another letter dated August
payment of the purchase price must first be established since 20, 1985, this time requesting the Bank that the down payment of
the agreement on the manner of payment goes into the price P300,000.00 be instead taken from the excess payment made by
such that a disagreement on the manner of payment is the RRRC in redeeming its own foreclosed properties.
tantamount to a failure to agree on the price.
The very circumstance that the Navarras had to make this new
The Navarras’ letter/offer failed to specify a definite amount of request is a clear indication that no definite agreement has yet
the purchase price for the sale/repurchase of the subject been reached at that point. As we see it, this request constitutes
properties. It merely stated that the "purchase price will be based a new offer on the part of the Navarras, which offer was again
on the redemption value plus accrued interest at the prevailing conditionally accepted by the Bank as in fact it even required the
rate up to the date of the sales contract." Navarras to submit a board resolution of RRRC before it could
proceed with the proposed sale/repurchase.
The ambiguity of this statement only bolsters the uncertainty
of the Navarras’ so-called "offer" for it leaves much rooms for The eventual failure of the spouses to submit the required board
such questions, as: what is the redemption value? What resolution precludes the perfection of a contract of
prevailing rate of interest shall be followed: is it the rate sale/repurchase between the parties.
stipulated in the loan agreement or the legal rate? When will the
date of the contract of sale be based, shall it be upon the time of Evidently, what transpired between the parties was only a
the execution of the deed of sale or upon the time when the last prolonged negotiation to buy and to sell, and, at the most, an
instalment payment shall have been made? To our mind, these offer and a counter offer with no definite agreement having been
questions need first to be addressed, discussed and negotiated reached by them. With the hard reality that no perfected
upon by the parties before a definite purchase price can be contract of sale/repurchase exists in this case, any
arrived at. Significantly, the Navarras wrote in the same letter the independent transaction between the Planters Bank and a third-
following: party, like the one involving the Gatchalian Realty, cannot be
affected.
Maybe you can give us a long-term payment scheme on the
basis of my brother’s annual savings of roughly
US$30,000.00 every time he comes home for his home AMADO VS. SALVADOR
leave. G.R. NO. 171401 DECEMBER 13, 2007

Again, the offer was not clear insofar as concerned the exact FACTS: Judge Amado is the owner of a lot, a portion of which is
number of years that will comprise the long-term payment the subject of the present litigation. It was alleged that sometime
scheme. As we see it, the absence of a stipulated period within in 1979, Judge Amado and Salvador agreed that the latter
which the repurchase price shall be paid all the more adds to the would sell the lot in favor of Salvador at P60/sqm. The
indefiniteness of the Navarras’ offer. Clearly, then, the lack of a payment was to be made in cash or construction material,
definite offer on the part of the spouses could not possibly serve whichever the Judge preferred and to whomever the latter
as the basis of their claim that the sale/repurchase of their wished during his lifetime. The terms of payment, though, were
foreclosed properties was perfected. The reason is obvious: one not stipulated.
essential element of a contract of sale is wanting: the price
certain. Here, what is dramatically clear is that there was no Thereafter, Salvador undertook and the location of the squatters
meeting of minds vis-a-vis the price, expressly or impliedly, in said land and eventually built several structures thereon for his
directly or indirectly. business. Salvador claims that by October 1980, he had already
given Judge Amado total cash advances of P30,310.93 and
Further, the tenor of Planters Bank’s letter reply negates the delivered construction materials amounting to P36,904.45, the
contention of the Navarras that the Bank fully accepted their total of which exceeded the agreed price for the subject
offer. The letter specifically stated that there is a need to property.
negotiate on the other details of the transaction before the sale
COMPILED BY: WIGMORE #WIGMOREFOREVER 71
SALES Case Digest (Atty. Sarona)
Compiled by: Wigmore #wigmoreforever

Petitioner heirs averred that Judge Amado and Salvador were construction materials were payment for the subject property, the
co-borrowers from a bank. A loan agreement was executed by act of partially paying for construction materials would be
them with Capitol City Dev’t bank as lender and the Lot of Judge incongruous to such intention.
Amado was used as collateral. The loaned amount was released
to Salvador and Judge Amado’s share was paid to him in several Thirdly, Salvador himself gave conflicting statements on whether
instalments. Salvador failed to pay his share in the amortization he has completed payment. Other proofs presented gave no
of the lot so that Judge Amado had to pay the loan to avoid weight to respondent’s allegations. The testimony of the witness
foreclosure. presented by Salvador was not given credence. Finally, the act of
Salvador in relocating the squatters is not substantial proof of
Thereafter, Judge Amado demanded Salvador to leave the ownership.
premises and an ejectment case was filed to that effect.
Salvador filed a case for specific performance contending that a
balance of P4,040.62 was not paid to Judge Amado because of BANK OF COMMERCE VS MANALO
the latter’s failure to execute the deed of sale. Salvador
presented several documentary evidence. FACTS: The Xavierville Estate, Inc. was the owner of parcels of
land in Quezon City, known as the Xavierville Estate Subdivision,
RTC dismissed the complaint because Salvador’s evidence does with an area of 42 hectares. XEI caused the subdivision of the
not show that the money and construction materials were property into residential lots, which was then offered for sale to
intended as payment for the subject property. CA reversed the individual lot buyers.
decision on the finding that the construction materials delivered
were not paid for. Sometime in 1972, then XEI president Emerito Ramos, Jr.
contracted the services of Engr. Carlos Manalo, Jr. who was in
ISSUE: WON there was a perfected contract of sale. NO business of drilling deep water wells and installing pumps under
the business name Hurricane Commercial, Inc. For P34,887.66,
HELD: No Convincing Proof as to Manner of Payment Manalo, Jr. installed a water pump at Ramos residence at the
corner of Aurora Boulevard and Katipunan Avenue, Quezon City.
In the present case, Salvador fails to allege the manner of Manalo, Jr. then proposed to XEI, through Ramos, to purchase a
payment of the purchase price on which the parties should lot in the Xavierville subdivision, and offered as part of the
have agreed. No period was set within which the payment must downpayment the P34,887.66 Ramos owed him. XEI, through
be made. Of the purchase price of P66,360.00, which the Ramos, agreed. In a letter dated February 8, 1972, Ramos
parties purportedly agreed upon, the amount which should be requested Manalo, Jr. to choose which lots he wanted to buy so
paid in cash and the amount for construction materials was not that the price of the lots and the terms of payment could be fixed
determined. This means that the parties had no exact notion of and incorporated in the conditional sale. Manalo, Jr. met with
the consideration for the contract to which they supposedly gave Ramos and informed him that he and his wife Perla had chosen
their consent. Thus, such failure is fatal to Salvador’s claim that a Lots 1 and 2 of Block 2 with a total area of 1,740.3 square
sale had been agreed upon by the parties. meters.

Furthermore, after carefully examining the records, serious In a letter dated August 22, 1972 to Perla Manalo, Ramos
doubts became apparent as to whether cash advances and confirmed the reservation of the lots. He also pegged the price of
deliveries of construction materials evidenced by numerous the lots at P200.00 per square meter, or a total of P348,060.00,
statements of accounts and delivery receipts were actually with a 20% down payment of the purchase price amounting to
intended as payment for the land. P69,612.00 less the P34,887.66 owing from Ramos, payable on
or before December 31, 1972; the corresponding Contract of
First of all, the statements of accounts and the delivery receipts Conditional Sale would then be signed on or before the same
do not indicate that the construction materials or the cash date, but if the selling operations of XEI resumed after December
advances were made in connection with the sale of the subject 31, 1972, the balance of the downpayment would fall due then,
property. Any doubt as to the real meaning of the contract must and the spouses would sign the aforesaid contract within 5 days
be resolved against the person who drafted the instrument and is from receipt of the notice of resumption of such selling
responsible for the ambiguity thereof. Since Salvador prepared operations. It was also stated in the letter that, in the meantime,
these statements of accounts and therefore caused the the spouses may introduce improvements thereon subject to the
ambiguity, he cannot benefit from the resulting ambiguity. rules and regulations imposed by XEI in the subdivision. Perla
Salvador is hardly an ignorant and illiterate person; rather, he Manalo conformed to the letter agreement.
is a businessman engaged in manufacturing and distributing
construction materials and operates no less than two branches. It The spouses Manalo took possession of the property on
should have been noted in the statement of accounts, or even in September 2, 1972, constructed a house thereon, and installed a
another document, that the cash advances and deliveries of fence around the perimeter of the lots.
construction materials were made in connection with a
transaction as important as a sale of land. As they are, the The spouses Manalo were notified of the resumption of the
statements of accounts and especially the straightforward selling operations of XEI. However, they did not pay the balance
delivery receipts are insufficient proof that Judge Amado sold his of the downpayment on the lots because Ramos failed to
property to Salvador. prepare a contract of conditional sale and transmit the same to
Manalo for their signature. On August 14, 1973, Perla Manalo
Secondly, one of the delivery receipts presented by Salvador went to the XEI office and requested that the payment of the
was partially paid. If Judge Amado had already agreed that the amount representing the balance of the downpayment be
COMPILED BY: WIGMORE #WIGMOREFOREVER 72
SALES Case Digest (Atty. Sarona)
Compiled by: Wigmore #wigmoreforever

deferred, which, however, XEI rejected. On August 10, 1973, XEI the contract of sale or to sell is not perfected, it cannot, as an
furnished her with a statement of their account as of July 31, independent source of obligation, serve as a binding juridical
1973, showing that they had a balance of P34,724.34 on the relation between the parties.
downpayment of the two lots after deducting the account of
Ramos, plus P3,819.68 interest thereon from September 1, 1972 A definite agreement as to the price is an essential element of a
to July 31, 1973, and that the interests on the unpaid balance of binding agreement to sell personal or real property because it
the purchase price of P278,448.00 from September 1, 1972 to seriously affects the rights and obligations of the parties. Price is
July 31, 1973 amounted to P30,629.28. The spouses were an essential element in the formation of a binding and
informed that they were being billed for said unpaid interests. enforceable contract of sale. The fixing of the price can never be
left to the decision of one of the contracting parties. But a price
Subsequently, XEI turned over its selling operations to OBM, fixed by one of the contracting parties, if accepted by the other,
including the receivables for lots already contracted and those gives rise to a perfected sale.
yet to be sold. Subsequently, the Commercial Bank of Manila
(CBM) acquired the Xavierville Estate from OBM. It is not enough for the parties to agree on the price of the
property. The parties must also agree on the manner of payment
In a letter dated August 5, 1986, the CBM requested Perla of the price of the property to give rise to a binding and
Manalo to stop any on-going construction on the property since it enforceable contract of sale or contract to sell. This is so
(CBM) was the owner of the lot and she had no permission for because the agreement as to the manner of payment goes into
such construction. She agreed to have a conference meeting the price, such that a disagreement on the manner of payment is
with CBM officers where she informed them that her husband tantamount to a failure to agree on the price.
had a contract with OBM, through XEI, to purchase the property.
When asked to prove her claim, she promised to send the In a contract to sell property by installments, it is not enough that
documents to CBM. However, she failed to do so. On September the parties agree on the price as well as the amount of
5, 1986, CBM reiterated its demand that it be furnished with the downpayment. The parties must, likewise, agree on the manner
documents promised, but Perla Manalo did not respond. of payment of the balance of the purchase price and on the other
terms and conditions relative to the sale. Even if the buyer
On July 27, 1987, CBM filed a complaint for unlawful detainer makes a downpayment or portion thereof, such payment cannot
against the spouses with the MTC Court of Quezon City. be considered as sufficient proof of the perfection of any
purchase and sale between the parties.
In the meantime, the CBM was renamed the Boston Bank of the
Philippines. After CBM filed its complaint against the spouses There is no showing, in the records, of the schedule of payment
Manalo, the latter filed a complaint for specific performance and of the balance of the purchase price on the property amounting
damages against the bank before the RTC of Quezon City. to P278,448.00. The said parties confined themselves to
agreeing on the price of the property (P348,060.00), the 20%
Boston Bank, now petitioner, maintains that, as held by the CA, downpayment of the purchase price (P69,612.00), and credited
the records do not reflect any schedule of payment of the 80% respondents for the P34,887.00 owing from Ramos as part of the
balance of the purchase price, or P278,448.00. Petitioner insists 20% downpayment. The determination of the terms of payment
that unless the parties had agreed on the manner of payment of of the P278,448.00 had yet to be agreed upon on or before
the principal amount, including the other terms and conditions of December 31, 1972, or even afterwards, when the parties sign
the contract, there would be no existing contract of sale or the corresponding contract of conditional sale.
contract to sell.
Jurisprudence is that if a material element of a contemplated
ISSUE: WON the manner of payment has been agreed upon and contract is left for future negotiations, the same is too indefinite to
WON it is essential for there to be an existing contract of sale or be enforceable. And when an essential element of a contract is
contract to sell reserved for future agreement of the parties, no legal obligation
arises until such future agreement is concluded.
RULING: No, it was not agreed upon thus, there was no contract
to sell. The Court agrees with petitioners contention that, for a Indeed, the parties are in agreement that there had been no
perfected contract of sale or contract to sell to exist in law, there contract of conditional sale ever executed by XEI, OBM or
must be an agreement of the parties, not only on the price of the petitioner, as vendor, and the respondents, as vendees.
property sold, but also on the manner the price is to be paid by
the vendee. Respondents failed to allege and prove, in the trial court, that, as
a matter of business usage, habit or pattern of conduct, XEI
Under Article 1458 of the New Civil Code, in a contract of sale, granted all lot buyers the right to pay the balance of the purchase
whether absolute or conditional, one of the contracting parties price in installments of 120 months of fixed amounts with pre-
obliges himself to transfer the ownership of and deliver a computed interests, and that XEI and the respondents had
determinate thing, and the other to pay therefor a price certain in intended to adopt such terms of payment relative to the sale of
money or its equivalent. A contract of sale is perfected at the the two lots in question. Habit, custom, usage or pattern of
moment there is a meeting of the minds upon the thing which is conduct must be proved like any other facts.
the object of the contract and the price. From the averment of
perfection, the parties are bound, not only to the fulfillment of As a consequence, respondents and XEI (or OBM for that
what has been expressly stipulated, but also to all the matter) failed to forge a perfected contract to sell the two lots;
consequences which, according to their nature, may be in hence, respondents have no cause of action for specific
keeping with good faith, usage and law. On the other hand, when performance against petitioner.
COMPILED BY: WIGMORE #WIGMOREFOREVER 73
SALES Case Digest (Atty. Sarona)
Compiled by: Wigmore #wigmoreforever

5. Inadequacy of Price Does Not Affect Ordinary Sale more than P60,000. A judicial sale of real property will be set
aside when the price is so inadequate as to shock the
conscience of the court. (National Bank vs. Gonzalez, 45 Phil.,
THE DIRECTOR OF LANDS, vs. ABARCA, ET AL 693.)

FACTS: About fourteen years, the lot now in question was the In the instant case there is another important consideration. In
subject of litigation between Datu Bualan and his co-claimants, fairness and equity, which after all are the true aims of the law,
on the one hand, and Ciriaco Lizada, on the other. Juan A. the amount paid by Datu Bualan and his co-claimants for taxes
Sarenas and Domingo Braganza were the attorneys for Datu and penalties due on the contested property should be credited
Bualan and his co-claimants in that suit, wherein a judgment was on the judgment obtained by Sarenas and Braganza. Such taxes
rendered declaring Datu Bualan and his co-claimants the owners and penalties accrued while the property was in that possession
of the land involved in the litigation. under a claim of ownership. It follows that the error assigned by
Datu Bualan and his co-claimants against the judgment below, to
Subsequently, a controversy arose between the Bagobos and the effect that the lower court erred in subjecting the property
their attorneys as to the amount of fees due the latter, sought to be registered to a lien in favor of Sarenas and
whereupon the attorneys took possession of the property now in Braganza for P877.25 with interests, must be sustained.
question. Action was brought by the Bagobos against their
former attorneys for the recovery of the land. In this action
judgment was rendered ordering the attorneys to return the BRAVO-GUERRERO vs. EDWARD P. BRAVO
property seized by them, and requiring the Bagobos to pay their
former attorneys the sum of P6,000 as fees. As a result of this FACTS: Spouses Mauricio and Simona Bravo owned 2 parcels
judgment Datu Bualan and his co-claimants paid Sarenas and of land measuring 287 and 291 square meters and located in
Braganza the sum of P5,126.13. They also paid to the municipal Makati City, Metro Manila. The Properties are registered under
treasurer of Davao in the name of Sarenas and Braganza, for TCT Nos. 58999 and 59000 issued by the Register of Deeds of
taxes and penalties due on the property in the year 1926, while Rizal on 23 May 1958. The Properties contain a large residential
the same was in the possession of the latter, the sum of dwelling, a smaller house and other improvements.
P1,035.87. The Bagobos assumed that, by these payments
which amounted in all to P6,162, the judgment rendered against Mauricio and Simona had three children - Roland, Cesar and
them for P6,000 together with interests due thereon, was fully Lily, all surnamed Bravo. Cesar died without issue. Lily Bravo
satisfied. married David Diaz, and had a son, David B. Diaz, Jr. ("David
Jr."). Roland had six children, namely, Elizabeth Bravo-Guerrero,
Claiming that the sum paid to the municipal treasurer of Davao Edward, Roland, Senia, Benjamin, and their half-sister, Ofelia.
should not be credited on the amount of the judgment obtained
by them, Sarenas and Braganza caused the clerk of the court to Simona executed a General Power of Attorney ("GPA") on 17
issue a writ of execution on the said judgment. By reason of the June 1966 appointing Mauricio as her attorney-in-fact. In the
writ of execution so issued, the sheriff levied on the property here GPA, Simona authorized Mauricio to "mortgage or otherwise
in question and sold it to Sarenas and Braganza for the sum of hypothecate, sell, assign and dispose of any and all of my
P877.25. Upon the failure of the Bagobos to redeem the property, real, personal or mixed, of any kind whatsoever and
property, they filed their claim in the present cadastral case, wheresoever situated, or any interest therein." Mauricio
alleging that they were the absolute owners of the lot in question. subsequently mortgaged the Properties to the PNB and DBP for
P10,000 and P5,000, respectively.
In view of the evidence presented by the parties, the lower court
dismissed the claim of Sarenas and Braganza, and ordered the On 25 October 1970, Mauricio executed a Deed of Sale with
registration of the lot now in question in the names of Datu Assumption of Real Estate Mortgage conveying the Properties to
Bualan and his co-claimants, subject, however, to a lien in favor vendees Roland A. Bravo, Ofelia A. Bravo and Elizabeth Bravo-
of Sarenas and Braganza for the sum of P877.25, with interest at Guerrero.” The sale was conditioned on the payment of P1,000
the rate of 12 per cent per annum from April 27, 1927. and on the assumption by the vendees of the PNB and DBP
mortgages over the Properties.
In dismissing the claim of Sarenas and Braganza, the lower court
held that the sale by the sheriff of the property in question in As certified by the Clerk of Court of the Regional Trial Court of
favor of said claimants was null and void, because the amount of Manila, the Deed of Sale was notarized by Atty. Victorio Q.
P877.25 paid by Sarenas and Braganza was absolutely Guzman on 28 October 1970 and entered in his Notarial
inadequate. Register. However, the Deed of Sale was not annotated on TCT
Nos. 58999 and 59000. Neither was it presented to PNB and
ISSUE: WON a judicial sale of real property will be set aside DBP. The mortage loans and the receipts for loan payments
when price is adequate issued by PNB and DBP continued to be in Mauricio’s name
even after his death on 20 November 1973. Simona died in
RULING: Yes if the price is so inadequate as to shock the 1977.
conscience of the court.
On 23 June 1997, Edward, represented by his wife, Fatima
The lower court was right in declaring the sheriff's sale null and Bravo, filed an action for the judicial partition of the Properties.
void on the ground of the inadequacy of the price paid. It appears Edward claimed that he and the other grandchildren of Mauricio
that in 1927 the assessed value of the contested property was and Simona are co-owners of the Properties by succession.
Despite this, petitioners refused to share with him the possession
COMPILED BY: WIGMORE #WIGMOREFOREVER 74
SALES Case Digest (Atty. Sarona)
Compiled by: Wigmore #wigmoreforever

and rental income of the Properties. Edward later amended his Respondents argue that P16,000 is still far below the actual
complaint to include a prayer to annul the Deed of Sale, which he value of the Properties The tax declarations placed the assessed
claimed was merely simulated to prejudice the other heirs. value of both Properties at P16,160. Compared to this, the price
of P16,000 cannot be considered grossly inadequate, much less
The trial court upheld Mauricio’s sale of the Properties to the so shocking to the conscience as to justify the setting aside of
vendees. The trial court ruled that the sale did not prejudice the the Deed of Sale.
compulsory heirs, as the Properties were conveyed for valuable
consideration.
PART III: FORMATION OF CONTRACT OF SALE
Citing Article 166 of the Civil Code, the Court of Appeals
reversed trial court’s decision and declared the Deed of Sale void I. POLICITATION
for lack of Simona’s consent. It also found that there was
insufficient proof that the vendees made the mortgage payments 1. Option Contract
on the Properties, since the PNB and DBP receipts were issued
in Mauricio’s name. The appellate court opined that the rental
income of the Properties, which the vendees never shared with TAYAG vs. LACSON
respondents, was sufficient to cover the mortgage payments to
PNB and DBP. FACTS: Respondents Angelica Tiotuyco Vda. de Lacson, and
her children were the registered owners of three parcels of land
ISSUE: WON the sale of the properties was simulated or void for located in Mabalacat, Pampanga. The properties were tenanted
gross inadequacy of price agricultural lands.
RULING: No, the sale of the properties is not void either for On March 17, 1996, a group of original farmers/tillers, Tiamson,
being simulated or for inadequacy of price. et al., individually executed in favor of the petitioner separate
Deeds of Assignment in which the assignees assigned to the
Respondents, however, contend that the sale of the Properties petitioner their respective rights as tenants/tillers of the
was merely simulated. As proof, respondents point to the landholdings possessed and tilled by them for and in
consideration of P1,000 in the Deed of Sale, which respondents consideration of P50.00 per square meter. The said amount was
claim is grossly inadequate compared to the actual value of the made payable "when the legal impediments to the sale of the
Properties. property to the petitioner no longer existed." The petitioner was
also granted the exclusive right to buy the property if and when
Simulation of contract and gross inadequacy of price are distinct the respondents, with the concurrence of the defendants-tenants,
legal concepts, with different effects. When the parties to an agreed to sell the property. In the interim, the petitioner gave
alleged contract do not really intend to be bound by it, the varied sums of money to the tenants as partial payments, and
contract is simulated and void. A simulated or fictitious contract the latter issued receipts for the said amounts.
has no legal effect whatsoever because there is no real
agreement between the parties. On July 24, 1996, the petitioner called a meeting of the
defendants-tenants to work out the implementation of the terms
In contrast, a contract with inadequate consideration may of their separate agreements.
nevertheless embody a true agreement between the parties. A
contract of sale is a consensual contract, which becomes valid However, on August 8, 1996, the defendants-tenants, through
and binding upon the meeting of minds of the parties on the price Joven Mariano, wrote the petitioner stating that they were not
and the object of the sale. The concept of a simulated sale is attending the meeting and instead gave notice of their collective
thus incompatible with inadequacy of price. When the parties decision to sell all their rights and interests, as tenants/lessees,
agree on a price as the actual consideration, the sale is not over the landholding to the respondents Lacson.
simulated despite the inadequacy of the price.
On August 19, 1996, the petitioner filed a complaint against the
Gross inadequacy of price by itself will not result in a void defendants-tenants, as well as the respondents, for the court to
contract. Gross inadequacy of price does not even affect the fix a period within which to pay the agreed purchase price of
validity of a contract of sale, unless it signifies a defect in the P50.00 per square meter to the defendants, as provided for in
consent or that the parties actually intended a donation or some the Deeds of Assignment.
other contract. Inadequacy of cause will not invalidate a contract
unless there has been fraud, mistake or undue influence. In this Respondents as defendants asserted that they never induced
case, respondents have not proved any of the instances that the defendants Tiamson to violate their contracts with the
would invalidate the Deed of Sale. petitioner; and, being merely tenants-tillers, the defendants-
tenants had no right to enter into any transactions involving their
Respondents even failed to establish that the consideration paid properties without their knowledge and consent. They also
by the vendees for the Properties was grossly inadequate. As the averred that the transfers or assignments of leasehold rights
trial court pointed out, the Deed of Sale stipulates that, in made by the defendants-tenants to the petitioner is contrary to
addition to the payment of P1,000, the vendees should assume Presidential Decree (P.D.) No. 27 and Republic Act No. 6657, the
the mortgage loans from PNB and DBP. The consideration for Comprehensive Agrarian Reform Program (CARP).
the sale of the Properties was thus P1,000 in cash and the
assumption of the P15,000 mortgage. The defendants-tenants Tiamson, et al., alleged in their answer
COMPILED BY: WIGMORE #WIGMOREFOREVER 75
SALES Case Digest (Atty. Sarona)
Compiled by: Wigmore #wigmoreforever

with counterclaim for damages, that the money each of them Thereafter, Adelfa Properties expressed interest in buying the
received from the petitioner were in the form of loans, and that western portion of the property from Rosario and Salud.
they were deceived into signing the deeds of assignment. What Accordingly, on 25 November 1989, an “Exclusive Option to
they knew was that they were made to sign a document that will Purchase” was executed between the parties, with the condition
serve as a receipt for the loan granted to them by the plaintiff. that the selling price shall be P2,856,150, that the option money
of P50,000 shall be credited as partial payment upon the
ISSUE: WON the Deeds of Assignment are perfected option consummation of sale, that the balance is to be paid on or before
contracts 30 November 1989, and that in case of default by Adelfa
Properties to pay the balance, the option is cancelled and 50% of
RULING: No, there is no perfected option contract. the option money shall be forfeited and the other 50% refunded
upon the sale of the property to a third party.
The Court does not agree with the contention of the petitioner
that the deeds of assignment executed by the defendants- Before Adelfa Properties could make payment, it received
tenants are perfected option contracts. An option is a contract by summons on 29 November 1989, together with a copy of a
which the owner of the property agrees with another person that complaint filed by the nephews and nieces of Rosario and Salud
he shall have the right to buy his property at a fixed price within a against the latter, Jose and Dominador Jimenez, and Adelfa
certain time. It is a condition offered or contract by which the Properties in the RTC Makati (Civil Case 89-5541), for
owner stipulates with another that the latter shall have the right to annulment of the deed of sale in favor of Household Corporation
buy the property at a fixed price within a certain time, or under, or and recovery of ownership of the property covered by TCT
in compliance with certain terms and conditions, or which gives 309773.
to the owner of the property the right to sell or demand a sale. It
imposes no binding obligation on the person holding the option, As a consequence, in a letter dated 29 November 1989, Adelfa
aside from the consideration for the offer. Until accepted, it is not, Properties informed Rosario and Salud that it would hold
properly speaking, treated as a contract. The second party gets payment of the full purchase price and suggested that the latter
not lands, not an agreement that he shall have the lands, but the settle the case with their nephews and nieces. . Salud Jimenez
right to call for and receive lands if he elects. An option contract refused to heed the suggestion of Adelfa Properties and
is a separate and distinct contract from which the parties may attributed the suspension of payment of the purchase price to
enter into upon the conjunction of the option. “lack of word of honor.”

In this case, the defendants-tenants-subtenants, under the deeds On 14 December 1989, Rosario and Salud sent Francisca
of assignment, granted to the petitioner not only an option but the Jimenez to see Atty. Bernardo, in his capacity as Adelfa
exclusive right to buy the landholding. But the grantors were Properties’ counsel, and to inform the latter that they were
merely the defendants-tenants, and not the respondents, the cancelling the transaction. In turn, Atty. Bernardo offered to pay
registered owners of the property. Not being the registered the purchase price provided that P500,000.00 be deducted
owners of the property, the defendants-tenants could not legally therefrom for the settlement of the civil case. This was rejected
grant to the petitioner the option, much less the "exclusive right" by Rosario and Salud. On 22 December 1989, Atty. Bernardo
to buy the property. As the Latin saying goes, "NEMO DAT wrote Rosario and Salud on the same matter but this time
QUOD NON HABET." reducing the amount from P500,000.00 to P300,000.00, and this
was also rejected by the latter. On 23 February 1990, the RTC
dismissed Civil Case 89-5541.
ADELFA PROPERTIES VS. CA
[G.R. NO. 111238. JANUARY 25, 1995.] On 16 April 1990, Atty. Bernardo wrote Rosario and Salud
informing the latter that in view of the dismissal of the case
PARTIES: Roasrio and Salud Jimenez – Seller against them, Adelfa Properties was willing to pay the purchase
Adelfa Properties – Buyer price, and he requested that the corresponding deed of absolute
Subject:: western portion of a parcel of land 8855 sq. ms. sale be executed. This was ignored by Rosario and Salud. On 27
Covered by TCT 309773 situated in Barrio Culasi, Las Pinas, July 1990, Jimenez’ counsel sent a letter to Adelfa Properties
Metro Manila enclosing therein a check for P25,000.00 representing the refund
of 50% of the option money paid under the exclusive option to
FACTS: Rosario Jimenez-Castaneda, Salud Jimenez and their purchase. Rosario and Salud then requested Adelfa Properties
brothers, Jose and Dominador Jimenez, were the registered co- to return the owner’s duplicate copy of the certificate of title of
owners of a parcel of land consisting of 17,710 sq. ms Salud Jimenez. Adelfa Properties failed to surrender the
(TCT 309773) situated in Barrio Culasi, Las Piñas, Metro certificate of title.
Manila. On 28 July 1988, Jose and Dominador Jimenez sold
their share consisting of 1/2 of said parcel of land, specifically the Rosario and Salud Jimenez filed Civil Case 7532 in the RTC
eastern portion thereof, to Adelfa Properties pursuant to a Pasay City (Branch 113) for annulment of contract with
“Kasulatan sa Bilihan ng Lupa.” Subsequently, a “Confirmatory damages, praying, among others, that the exclusive option to
Extrajudicial Partition Agreement” was executed by the purchase be declared null and void; that Adelfa Properties be
Jimenezes, wherein the eastern portion of the subject lot, with an ordered to return the owner’s duplicate certificate of title; and that
area of 8,855 sq. ms. was ADJUDICATED TO JOSE AND the annotation of the option contract on TCT 309773 be
DOMINADOR JIMENEZ, WHILE THE WESTERN PORTION cancelled.
WAS ALLOCATED TO ROSARIO AND SALUD Jimenez.
RTC: On 5 September 1991, the trial court rendered judgment
holding that the agreement entered into by the parties was
COMPILED BY: WIGMORE #WIGMOREFOREVER 76
SALES Case Digest (Atty. Sarona)
Compiled by: Wigmore #wigmoreforever

merely an option contract, and declaring that the suspension of or two, and words in context not words standing alone.
payment by Adelfa Properties constituted a counter-offer which, Moreover, judging from the subsequent acts of the parties which
therefore, was tantamount to a rejection of the option. It likewise will hereinafter be discussed, it is undeniable that the intention of
ruled that Adelfa Properties could not validly suspend payment in the parties was to enter into a contract to sell. In addition, the title
favor of Rosario and Salud on the ground that the vindicatory of a contract does not necessarily determine its true nature.
action filed by the latter’s kin did not involve the western portion Hence, the fact that the document under discussion is entitled
of the land covered by the contract between the parties, but the “Exclusive Option to Purchase” is not controlling where the text
eastern portion thereof which was the subject of the sale thereof shows that it is a contract to sell.
between Adelfa Properties and the brothers Jose and Dominador
Jimenez. The trial court then directed the cancellation of the Test to determine contract as a “contract of sale or
exclusive option to purchase. purchase” or mere “option”. The test in determining whether a
contract is a “contract of sale or purchase” or a mere “option” is
On appeal, whether or not the agreement could be specifically enforced.
RTC: the Court of appeals affirmed in toto the decision of the There is no doubt that Adelfa’s obligation to pay the purchase
court a quo. That Article 1590 of the Civil Code on suspension of price is specific, definite and certain, and consequently binding
payments applies only to a contract of sale or a contract to sell, and enforceable. Had the Jimenezes chosen to enforce the
but not to an option contract which it opined was the nature of contract, they could have specifically compelled Adelfa to pay the
the document subject of the case at bar. balance of P2,806,150.00. This is distinctly made manifest in the
contract itself as an integral stipulation, compliance with which
Hence, the petition for review on certiorari. could legally and definitely be demanded from petitioner as a
consequence.
Adelfa properties posits that the contract is a Contract of Sale
and not an Option Contract or Contract to Sell, making the Adelfa Properties justified in suspending payment of
suspension of payment applicable in the case. balance by reason of vindicatory action filed against it. In
Civil Case 89-5541, it is easily discernible that, although the
ISSUE: Whether or not the contract is a Contract of Sale , Option complaint prayed for the annulment only of the contract of sale
Contract or Contract to Sell. executed between Adelfa Properties and the Jimenez brothers,
the same likewise prayed for the recovery of therein Jimenez’
SC: The Supreme Court affirmed the assailed judgment of the share in that parcel of land specifically covered by TCT 309773.
Court of Appeals in CA-GR CV 34767, with modificatory In other words, the Jimenezes were claiming to be co-owners of
premises. the entire parcel of land described in TCT 309773, and not only
of a portion thereof nor did their claim pertain exclusively to the
Agreement between parties a contract to sell and not an eastern half adjudicated to the Jimenez brothers. Therefore,
option contract or a contract of sale Adelfa Properties was justified in suspending payment of the
The alleged option contract is a contract to sell, rather than a balance of the purchase price by reason of the aforesaid
contract of sale. The distinction between the two is important for vindicatory action filed against it. The assurance made by the
in contract of sale, the title passes to the vendee upon the Jimenezes that Adelfa Properties did not have to worry about the
delivery of the thing sold; whereas in a contract to sell, by case because it was pure and simple harassment is not the kind
agreement the ownership is reserved in the vendor and is not to of guaranty contemplated under the exceptive clause in Article
pass until the full payment of the price. In a contract of sale, the 1590 wherein the vendor is bound to make payment even with
vendor has lost and cannot recover ownership until and unless the existence of a vindicatory action if the vendee should give a
the contract is resolved or rescinded; whereas in a contract to security for the return of the price.
sell, title is retained by the vendor until the full payment of the
price Thus, a deed of sale is considered absolute in nature Jimenezes may no longer be compelled to sell and deliver
where there is neither a stipulation in the deed that title to the subject property. Be that as it may, and the validity of the
property sold is reserved in the seller until the full payment of the suspension of payment notwithstanding, the Jimenezes may no
price, nor one giving the vendor the right to unilaterally resolve longer be compelled to sell and deliver the subject property to
the contract the moment the buyer fails to pay within a fixed Adelfa Properties for two reasons, that is, Adelfa’s failure to duly
period. effect the consignation of the purchase price after the
disturbance had ceased; and, secondarily, the fact that the
That the parties really intended to execute a contract to sell is contract to sell had been validly rescinded by the Jimenezes.
bolstered by the fact that the deed of absolute sale would have
been issued only upon the payment of the balance of the Rescission in a contract to sell. Article 1592 of the Civil Code
purchase price, as may be gleaned from Adelfa Properties’ letter which requires rescission either by judicial action or notarial act
dated 16 April 1990 wherein it informed the vendors that it “is is not applicable to a contract to sell. Furthermore, judicial action
now ready and willing to pay you simultaneously with the for rescission of a contract is not necessary where the contract
execution of the corresponding deed of absolute sale.” provides for automatic rescission in case of breach, as in the
contract involved in the present controversy. By Adelfa’s failure
Contract interpreted to ascertain intent of parties; Title not to comply with its obligation, the Jimenezes elected to resort to
controlling if text shows otherwise. The important task in and did announce the rescission of the contract through its letter
contract interpretation is always the ascertainment of the to Adelfa dated 27 July 1990. That written notice of rescission is
intention of the contracting parties and that task is to be deemed sufficient under the circumstances.
discharged by looking to the words they used to project that
intention in their contract, all the words not just a particular word WHEREFORE, on the foregoing modificatory premises, and
COMPILED BY: WIGMORE #WIGMOREFOREVER 77
SALES Case Digest (Atty. Sarona)
Compiled by: Wigmore #wigmoreforever

considering that the same result has been reached by The respondent appellate court failed to give due consideration
respondent Court of Appeals with respect to the relief awarded to to petitioners' evidence which shows that in 1969 the Villamor
private respondents by the court a quo which we find to be spouses bough an adjacent lot from the brother of Macaria
correct, its assailed judgment in CA-G.R. CV No. 34767 is Labing-isa for only P18.00 per square meter which the private
hereby AFFIRMED. respondents did not rebut. Thus, expressed in terms of money,
the consideration for the deed of option is the difference
between the purchase price of the 300 square meter portion
i. Meaning of Separate Consideration of the lot in 1971 (P70.00 per sq.m.) and the prevailing
reasonable price of the same lot in 1971. Whatever it is,
(P25.00 or P18.00) though not specifically stated in the deed
SPOUSES JULIO D. VILLAMOR AND MARINA VILLAMOR vs of option, was ascertainable. Petitioner's allegedly paying
THE HON. COURT OF APPEALS AND SPOUSES MACARIA P52.00 per square meter for the option may, as opined by the
LABINGISA REYES AND ROBERTO REYES appellate court, be improbable but improbabilities does not
G.R. No. 97332 October 10, 1991 invalidate a contract freely entered into by the parties.

FACTS: Macaria sold 300 square meters from her 600 square The "deed of option" entered into by the parties in this case had
meter lot to the spouses Villamor which is located at Baesa, unique features. Ordinarily, an optional contract is a privilege
Caloocan City, for the total amount of P21,000.00. existing in one person, for which he had paid a consideration and
which gives him the right to buy, for example, certain
Earlier, Macaria borrowed P2,000.00 from the spouses which merchandise or certain specified property, from another person,
amount was deducted from the total purchase price of the 300 if he chooses, at any time within the agreed period at a fixed
square meter lot sold. price (Enriquez de la Cavada v. Diaz, 37 Phil. 982). If We look
closely at the "deed of option" signed by the parties, We will
Macaria executed a "Deed of Option" in favor of Villamor in which notice that the first part covered the statement on the sale of the
the remaining 300 square meter portion (TCT No. 39934) of the 300 square meter portion of the lot to Spouses Villamor at the
lot would be sold to Villamor under the conditions stated therein. price of P70.00 per square meter "which was higher than the
actual reasonable prevailing value of the lands in that place at
Included in the Deed of Option is: that time (of sale)."
That the only reason why the Spouses-vendees Julio Villamor
and Marina V. Villamor, agreed to buy the said one-half portion at The second part stated that the only reason why the Villamor
the above-stated price of about P70.00 per square meter, is spouses agreed to buy the said lot at a much higher price is
because I, and my husband Roberto Reyes, have agreed to sell because the vendor (Reyeses) also agreed to sell to the
and convey to them the remaining one-half portion still owned by Villamors the other half-portion of 300 square meters of the land.
me and now covered by TCT No. 39935 of the Register of Deeds Had the deed stopped there, there would be no dispute that the
for the City of Caloocan, whenever the need of such sale arises, deed is really an ordinary deed of option granting the Villamors
either on our part or on the part of the spouses (Julio) Villamor the option to buy the remaining 300 square meter-half portion of
and Marina V. Villamor, at the same price of P70.00 per square the lot in consideration for their having agreed to buy the other
meter, excluding whatever improvement may be found the half of the land for a much higher price. But, the "deed of option"
thereon; went on and stated that the sale of the other half would be made
"whenever the need of such sale arises, either on our (Reyeses)
In 1984, when the husband of Macaria retired, wanted to part or on the part of the Spouses Julio Villamor and Marina V.
repurchased the said 300 square meter of lot from the Villamor. It appears that while the option to buy was granted to
petitioners. However, petitioners rejected the said offer and the Villamors, the Reyeses were likewise granted an option to
expresses their desire to purchase the remaining half of the lot. sell. In other words, it was not only the Villamors who were
granted an option to buy for which they paid a consideration. The
Trial Court rendered judgment in favor of Villamor. Reyeses as well were granted an option to sell should the need
for such sale on their part arise.
Court of Appeals reversed the decision premised on the finding
of respondent court that the Deed of Option is void for lack of A contract of sale is, under Article 1475 of the Civil Code,
consideration. "perfected at the moment there is a meeting of minds upon the
thing which is the object of the contract and upon the price. From
ISSUE: Whether there is a Separate Consideration for the that moment, the parties may reciprocally demand perform of
Option Contract? YES contracts." Since there was, between the parties, a meeting of
minds upon the object and the price, there was already a
RULING: As expressed in Gonzales v. Trinidad, 67 Phil. 682, perfected contract of sale. What was, however, left to be done
consideration is "the why of the contracts, the essential reason was for either party to demand from the other their respective
which moves the contracting parties to enter into the contract." undertakings under the contract. It may be demanded at any
The cause or the impelling reason on the part of private time either by the private respondents, who may compel the
respondent executing the deed of option as appearing in the petitioners to pay for the property or the petitioners, who may
deed itself is the petitioner's having agreed to buy the 300 square compel the private respondents to deliver the property.
meter portion of private respondents' land at P70.00 per square
meter "which was greatly higher than the actual reasonable However, the Deed of Option did not provide for the period within
prevailing price." which the parties may demand the performance of their
respective undertakings in the instrument. The parties could not
COMPILED BY: WIGMORE #WIGMOREFOREVER 78
SALES Case Digest (Atty. Sarona)
Compiled by: Wigmore #wigmoreforever

have contemplated that the delivery of the property and the indicate that her aforementioned agreement, promise and
payment thereof could be made indefinitely and render uncertain undertaking is supported by a consideration "distinct from the
the status of the land. The failure of either parties to demand price" stipulated for the sale of the land.
performance of the obligation of the other for an unreasonable
length of time renders the contract ineffective. There is no question that under article 1479 of the new Civil
Code "an option to sell," or "a promise to buy or to sell," as used
Under Article 1144 (1) of the Civil Code, actions upon written in said article, to be valid must be "supported by a consideration
contract must be brought within ten (10) years. The Deed of distinct from the price." This is clearly inferred from the context of
Option was executed on November 11, 1971. The acceptance, said article that a unilateral promise to buy or to sell, even if
as already mentioned, was also accepted in the same accepted, is only binding if supported by consideration. In other
instrument. The complaint in this case was filed by the petitioners words, "an accepted unilateral promise can only have a binding
on July 13, 1987, seventeen (17) years from the time of the effect if supported by a consideration which means that the
execution of the contract. Hence, the right of action had option can still be withdrawn, even if accepted, if the same is not
prescribed. There were allegations by the petitioners that they supported by any consideration. It is not disputed that the option
demanded from the private respondents as early as 1984 the is without consideration. It can therefore be withdrawn
enforcement of their rights under the contract. Still, it was beyond notwithstanding the acceptance of it by appellee.
the ten (10) years period prescribed by the Civil Code.
In other words, since there may be no valid contract without a
cause or consideration, the promisor is not bound by his promise
ii. No Separate Consideration and may, accordingly, withdraw it. Pending notice of its
withdrawal, his accepted promise partakes, however, of the
nature of an offer to sell which, if accepted, results in a perfected
NICOLAS SANCHEZ VS. SEVERINA RIGOS contract of sale.
45 SCRA 368 JUNE 1972
Indeed, the presumption is that, in the process of drafting the
FACTS: In an instrument entitled "Option to Purchase," Code, its author has maintained a consistent philosophy or
executed on April 3, 1961, defendant-appellant Severina Rigos position. Moreover, the decision in Southwestern Sugar &
"agreed, promised and committed ... to sell" to plaintiff-appellee Molasses Co. v. Atlantic Gulf & Pacific Co., holding that Art. 1324
Nicolas Sanchez for the sum of P1,510.00 within two (2) years is modified by Art. 1479 of the Civil Code, in effect, considers the
from said date, a parcel of land situated in the barrios of Abar latter as an exception to the former, and exceptions are not
and Sibot, San Jose, Nueva Ecija. It was agreed that said option favored, unless the intention to the contrary is clear, and it is not
shall be deemed "terminated and elapsed," if “Sanchez shall fail so, insofar as said two (2) articles are concerned. What is more,
to exercise his right to buy the property" within the stipulated the reference, in both the second paragraph of Art. 1479 and Art.
period. On March 12, 1963, Sanchez deposited the sum of 1324, to an option or promise supported by or founded upon a
Pl,510.00 with the CFI of Nueva Ecija and filed an action for consideration, strongly suggests that the two (2) provisions
specific performance and damages against Rigos for the latter’s intended to enforce or implement the same principle.
refusal to accept several tenders of payment that Sanchez made
to purchase the subject land. In the present case the trial court found that the "Plaintiff (Nicolas
Sanchez) had offered the sum of Pl,510.00 before any
Defendant Rigos contended that the contract between them was withdrawal from the contract has been made by the Defendant
only “a unilateral promise to sell, and the same being (Severina Rigos)." Since Rigos' offer sell was accepted by
unsupported by any valuable consideration, by force of the New Sanchez, before she could withdraw her offer, a bilateral
Civil Code, is null and void." Plaintiff Sanchez, on the other hand, reciprocal contract — to sell and to buy — was generated.
alleged in his compliant that, by virtue of the option under
consideration, "defendant agreed and committed to sell" and "the
plaintiff agreed and committed to buy" the land described in the PERCELINO DIAMANTE vs. HON. COURT OF APPEALS and
option. The lower court rendered judgment in favor of Sanchez GERARDO DEYPALUBUS
and ordered Rigos to accept the sum Sanchez judicially
consigned, and to execute in his favor the requisite deed of FACTS: A fishery lot, encompassing an area of 9.4 hectares and
conveyance. The Court of Appeals certified the case at bar to the designated as Lot No. 518-A of the Cadastral Survey of
Supreme Court for it involves a question purely of law. Dumangas, Iloilo, was previously covered by Fishpond Permit
No. F-2021 issued in the name of Anecita Dionio. Upon Anecita's
ISSUE: Was there a contract to buy and sell between the parties death, her heirs, petitioner Diamante and Primitivo Dafeliz,
or only a unilateral promise to sell? inherited the property which they later divided between
themselves; petitioner got 4.4. hectares while Dafeliz got 5
RULING: The option did not impose upon plaintiff the obligation hectares. It is the petitioner's share that is the subject of the
to purchase defendant's property. Annex A is not a "contract to present controversy. Primitivo Dafeliz later sold his share to
buy and sell." It merely granted plaintiff an "option" to buy. And private respondent.
both parties so understood it, as indicated by the caption, "Option
to Purchase," given by them to said instrument. Under the On 21 May 1959, petitioner sold to private respondent his
provisions thereof, the defendant "agreed, promised and leasehold rights over the property in question for P8,000.00 with
committed" herself to sell the land therein described to the the right to repurchase the same within three (3) years from said
plaintiff for P1,510.00, but there is nothing in the contract to date.

COMPILED BY: WIGMORE #WIGMOREFOREVER 79


SALES Case Digest (Atty. Sarona)
Compiled by: Wigmore #wigmoreforever

On 16 August 1960, private respondent filed an application with contract to sell and to buy is created, and the offeree ipso
the Bureau of Fisheries, dated 12 July 1960, for a fishpond facto assumes the obligations of a purchaser; the offeror, on the
permit and a fishpond lease agreement over the entire lot, other hand, would be liable for damages if he fails to deliver the
submitting therewith the deeds of sale executed by Dafeliz and thing he had offered for sale.
the petitioner.
The contract of option is a separate and distinct contract from the
Pressed by urgent financial needs, petitioner, on 17 October contract which the parties may enter into upon the
1960, sold all his remaining rights over the property in question consummation of the option, and a consideration for an optional
to the private respondent for P4,000.00. contract is just as important as the consideration for any other
kind of contract. Thus, a distinction should be drawn between
On 25 October 1960, private respondent, with his wife's consent, the consideration for the option to repurchase, and the
executed in favor of the petitioner an Option to Repurchase the consideration for the contract of repurchase itself.
property in question within ten (10) years from said date, with a
ten-year grace period.
Even if the promise was accepted, private respondent was not
bound thereby in the absence of a distinct consideration.
Private respondent submitted to the Bureau of Fisheries the
definite deed of sale; he did not, however, submit the Option to
Repurchase.
BIBLE BAPTIST CHURCH and PASTOR REUBEN
BELMONTE vs CA and SPOUSES VILLANUEVA
Thereafter, on 2 August 1961, the Bureau of Fisheries issued to G.R. No. 126454
private respondent Fishpond Permit November 26, 2004

On 11 December 1963, petitioner, contending that he has a valid FACTS: On June 7, 1985, the Bible Baptist Church entered into
twenty-year option to repurchase the subject property, requested a contract of lease with Spouses Villanueva over a property
the Bureau of Fisheries to respondent’s permit insofar as the located in Malate, Manila. The pertinent portions of the contract
said property is concerned. On 18 December 1964, his letter- are:
complaint was dismissed. Petitioner then sought a
reconsideration of the dismissal; the same was denied on 29 xxx
April 1965. His appeal to the Secretary of the DANR was likewise
dismissed on 30 October 1968. Again, on 20 November 1968, 2. That lease shall take effect on June 7, 1985 and shall be for a
petitioner sought for a reconsideration; this time, however, he period of 15 years.
was successful.
xxx
ISSUE: Was there a valid Option? No
4. That upon signing of the LEASE AGREEMENT, the Baptist
RULING: It is settled by this Court that "an agreement to Church shall pay the sum of P 84,000.00. Said sum shall be paid
repurchase becomes a promise to sell when made after the sale, directly to the Rural Bank of Bulacan for the purpose of
because when the sale is made without such an agreement, the redemption of said property, which was mortgaged by the
purchaser acquires the thing sold absolutely, and if he afterwards Spouses.
grants the vendor the right to repurchase, it is a new contract
entered into by the purchaser, as absolute owner already of the xxx
object. In that case the vendor has not reserved to himself the
right to repurchase." 8. That Bible Baptist has the option to buy the leased
property during the 15 years of the lease. If Baptist Church
decides to purchase the premises the terms will be:
Hence, the Option to Repurchase executed by private 1. A selling price of 1.8 M;
respondent in the present case, was merely a promise to sell, 2. A down payment agreed upon by both parties;
which must be governed by Article 1479 of the Civil Code. 3. The balance may be paid at the rate of P 120T per year.

A copy of the so-called Option to Repurchase is neither attached These stipulations of the lease contract are the subject of the
to the records nor quoted in any of the pleadings of the parties. present controversy for it is now the contention of Baptist Church
This Court cannot, therefore, properly rule on whether the that the option contract is founded upon a separate consideration
promise was accepted and a consideration distinct from the that is the P 84 T paid by them upon the signing of the lease
price, supports the option. Undoubtedly, in the absence of either agreement.
or both acceptance and separate consideration, the promise to
sell is not binding upon the promissor (private respondent). ISSUE: Whether or not the option to buy given to the Baptist
Church is founded upon a consideration.
A unilateral promise to buy or sell is a mere offer, which is not
HELD: No. Article 1479 of the Civil Code provides:
converted into a contract except at the moment it is accepted.
Acceptance is the act that gives life to a juridical obligation,
because, before the promise is accepted, the promissor may “A promise to buy and sell a determinate thing for a price certain
withdraw it at any time. Upon acceptance, however, a bilateral is reciprocally demandable.

COMPILED BY: WIGMORE #WIGMOREFOREVER 80


SALES Case Digest (Atty. Sarona)
Compiled by: Wigmore #wigmoreforever

An accepted unilateral promise to buy or to sell a determinate two slipways within the first 10 years of the lease with a total
thing for a price certain is binding upon the promissor if the value of not less than 450 T.
promise is supported by a consideration distinct from the price.”
On March 14, 1973 the property was mortgaged to China
Baptist Church cannot insist that the 84 T they paid in order to Banking Corporation (CBC) as a security for a loan availed by
release the Spouses’ property from the mortgage should be two of Carmen’s children, Mariano and Gabriel. The owner’s
deemed a separate consideration to support the option contract. duplicate copy was now with CBC.
It must be pointed out that the said amount was in fact
apportioned into monthly rentals spread over a period of one On December 31, 1974 Carmen executed a Deed of Absolute
year, at 7 T per month. Thus for the entire period of June 1985 to Sale with Assumption of Mortgage in which she as the vendor
May 1986, Baptist Church’s monthly rent had already been paid conveyed the property to her children Serafin, Mariano, Rogelio,
for, such that it only commenced paying rentals in June 1986. Carmencita and Mary Carmela for the purchase price of 350 T.
Therefore, the amount of 84 T has been fully exhausted and Mariano wrote a letter to CBC requesting them to conform to the
utilized by their occupation of the premises and there is no sale however CBC refused.
separate consideration to speak of which could support the
option. On June 27, 1977 Mariano presented the deed to the ROD for
registration purposes. They requested the ROD to compel CBC
Baptist Church insists that a consideration need not be a to transmit the owner’s duplicate copy of the title for annotation.
separate sum of money. They posit that their act of advancing CBC informed them that they were just following the instruction
the money to “rescue” the property from the mortgage and of Carmen not to surrender the owner’s duplicate.
impending foreclosure should be enough consideration to
support the option. In Villamor vs CA the court defined In the meantime the balance of the loan was fully paid and on
consideration as “the why of the contracts, the essential reason June 29, 1977 CBC executed a Cancellation of Real Estate
which moves the contracting parties to enter into a contract.” Mortgage however the deed was not presented to the ROD for
registration. On the same date Mariano, on behalf of his siblings,
This would illustrate that the consideration need not be monetary. executed an Affidavit of Adverse claim asserting their rights as
Actual cash need not be exchanged for the prion. However, by vendees of the property.
the very nature of an option contract the same is an onerous
contract for which the consideration must be something of value, On June 30, 1977 Carmen and NIC executed a Supplementary
although its kind may vary. The Villamor case is distinct from this Lease Agreement extending the lease period to October 2005.
case because: NIC was also granted the option to buy the property for 1.6
M.
The Court cannot find that the Baptist Church parted
with anything of value aside from the 84 T; Mariano et. al. was able to have the sale registered and a new
There is no document that contains an agreement title was issued in their name. Thereafter, they have informed
between the parties that Baptist Church’s supposed NIC to vacate the property, as they are now its new owners,
rescue of the mortgaged property was the consideration however, NIC refused.
which the parties contemplated in support of the option
clause in the contract; Meanwhile, Carmen filed a case against NIC anent the
Supplementary Lease Agreement purportedly executed by her as
To summarize the rules, an option contract needs to be lessor and NIC as lessee. She averred that NIC took advantage
supported by a separate consideration. The consideration need of the animosity between her and her children by inserting
not be monetary but could consist of other things or therein blatantly unfair provisions.
undertakings. However, if the consideration is not monetary,
these must be things or undertakings of value, in view of the On June 30, 1990 Mariano et. al informed NIC that they will be
onerous nature of the contract of option. Furthermore, when a no longer renew the contract and that as far as they were
consideration for an option contract is not monetary, said concerned the Supplementary Lease Agreement was null and
consideration must be clearly specified as such in the option void.
contract or clause.
ISSUE: Whether or not there is a perfected option contract.

NAVOTAS INDUSTRIAL CORPORATION, represented by HELD: No. Article 1479, paragraph 1, provides that, “A promise
DANIEL BAUTISTA vs GERMAN CRUZ, et. al. to buy and sell a determinate thing for a price certain is
G.R. No. 159212 reciprocally demandable.” NIC relies on this provision contending
September 12, 2005 that what was entered between them and Carmen was a mutual
promise to buy and sell. However, be it noted, that as early as
FACTS: Carmen Vda. De Cruz (Carmen) was the owner of a 1977 they were already informed of the sale made by Carmen in
parcel of land in Navotas with an area of 13999 square meters. favor of her children and that by virtue of the annotation made by
Mariano on June 30, 1977 NIC was constructively notified
On October 5, 1966, Carmen and Navotas Industrial Corporation thereof. There is therefore a presumption of knowledge of the
(NIC) entered into a contract of lease covering one half portion of sale between Carmen and her children.
the property. The lease was for October 1, 1966 to October 1,
1990. The property was to be used for shipyard slipways and Considering that Carmen was no longer the owner when the
NIC’s other allied businesses. The NIC obliged itself to construct Supplementary Lease Agreement was executed NIC’s claim that
COMPILED BY: WIGMORE #WIGMOREFOREVER 81
SALES Case Digest (Atty. Sarona)
Compiled by: Wigmore #wigmoreforever

it had the option to buy the property or to compel the heirs to sell On September 21, 1964, the spouses Vallejera sold the lot the
the property to it has no legal and factual basis. spouses Vasquez for the amount of 9 T. On the same day and
along with the execution of the Deed of Sale, a separate
Paragraphs 4 and o 5 of the Supplementary Lease Agreement instrument, denominated as “Right to Repurchase” was executed
provides: by the parties granting the Vallejeras the right to repurchase the
lot for 12 T.
4. The LESSEE is hereby granted an exclusive option to buy the
property including all improvements already made by the By virtue of the Deed of Sale the spouses Vasquez secured a
LESSEE (slipways and camarines) subject matter of this contract title in their name. However, on January 2, 1969, the Vallejeras
comprising SIX THOUSAND NINE HUNDRED FORTY-NINE sold the lot to Benito Derrama after securing the spouse
Point FIVE Square Meters (6,949.5) which is one-half portion of Vasquez’ title for 12 T. Upon the protestation of the spouses
the area covered by TCT No. 81574 and same property subject Vasquez the sale was cancelled after payment of 12 T to
matter of this contract should also be equally divided with one- Derrama.
half frontage along M. Naval Street and along the Navotas River
Bank shoreline during the period of the lease. The price of the The spouses Vasquez resisted the action for redemption on the
property is agreed to be fixed for the duration of the Option premise that the deed of “Right to Repurchase” is just an option
to Buy at a flat sum of ONE MILLION SIX HUNDRED to buy since it is not embodied in the same document of sale but
THOUSAND PESOS (P1,600,000.00), Philippine Currency, in a separate document and since such option is not supported
payable over a period to be mutually agreed upon. Should by a consideration distinct from the price, said deed is not
the LESSEE exercise the option to buy during the lifetime of the binding upon them.
LESSOR, the LESSEE will continue to pay the monthly rental to
the LESSOR during her lifetime. The spouses Vazquez insist that they can not be compelled to
resell the subject property for the nature of the sale over the said
5. The LESSEE shall pay to the LESSOR the sum of FORTY- lot between them and the Vallejeras can only be either an option
TWO THOUSAND (P42,000.00) PESOS upon signing of this to buy or a mere promise on their part to resell the property.
contract as consideration thereof, to be applied as against the Spouses Vasquez opined that since the “Right to Repurchase”
rental for the period from October 1, 1990 to September 30, was not supported by any consideration distinct from the
1991. purchase price it is not valid and binding upon the spouses
Vasquez pursuant to Article 1479.
It must be stressed that an option contract is a contract granting
a privilege to buy and sell within an agreed time and at a ISSUE: Whether or not the spouse Vallejera has a right to
determined price. Such contract is a separate and distinct repurchase under the contract.
contract from the time the parties may enter into upon the
construction of the option. An option contract is a preparatory HELD: No. The Court made reference to the earlier case of
contract in which one party grants to the other for a fixed period Sanchez vs. Rigos (Sanchez doctrine), stating that an option
and under specified conditions the power to decide, whether or contract without a separate consideration from the purchase
not to enter into a principal contract. Therefore, it is only when price is void, as a contract, but would still constitute as a valid
the option is exercised may a sale be perfected. An option offer; so that if the option is exercised prior to its withdrawal, that
NEEDS TO BE SUPPORTED by a separate consideration. is equivalent to an offer being accepted prior to withdrawal and
would give rise to a valid and binding sale.
In the present case, there was no given period for the petitioner
to exercise its option; it had yet to be determined and fixed at a The Sanchez doctrine also dictates that the burden of proof to
future time by the parties, subsequent to the execution of the show that the option contract was supported by a separate
Supplementary Lease Agreement. There was, likewise, no consideration is with the party seeking to show it. No reliance
consideration for the option. The amount of P42,000.00 paid by can be placed upon the provisions of Article 1354 which
the petitioner to Carmen Cruz on July 30, 1977 was payment for presumes the existence of a consideration in every contract,
rentals from October 1, 1990 to September 30, 1991, and not as since in the case of an option contract, Article 1479 being the
a consideration for the option granted to the petitioner. specific provision, requires such separate consideration for an
option to be valid.

iii. There Must be Acceptance of Option Offer In an option contract, the offeree has the burden of proving that
the option is supported by a separate consideration, it also held
SPOUSES CIPRIANO VASQUEZ and VALERIANA GAYANELO that the Sanchez doctrine (That upon the option contract not
vs CA and SPOUSES MARTIN VALLEJERA supported by a separate consideration; is void as contract, but
G.R. No. 83759 valid as an offer), can only apply if the option has been accepted
JULY 12, 1991 and such acceptance is communicated to the offeror. It held that
not even the annotation of the option contract on the title of the
FACTS: A certain property in Himamaylan, Negros Occidental property can be considered a proper acceptance of the option.
was registered in the name of Spouse Vallejera. On October
1959, they leased the property to the Spouses Vasquez. After the Neither can the signature of the spouses Vasquez in the
execution of the lease, the Vasquez’ took possession of the lot document called "right to repurchase" signify acceptance of the
and devoted the same to the cultivation of sugar. right to repurchase. The Vallejeras did not sign the offer.
Acceptance should be made by the promisee, in this case, the
Vallejeras and not the promises, spouses Vasquez herein. It
COMPILED BY: WIGMORE #WIGMOREFOREVER 82
SALES Case Digest (Atty. Sarona)
Compiled by: Wigmore #wigmoreforever

would be absurd to require the promisor of an option to buy to ISSUE: Whether or not GHRC’s right of first refusal was violated.
accept his own offer instead of the promisee to whom the option
to buy is given. HELD: Yes. The pertinent portion of the second contract of lease
provides that: Lessee shall also have the option to purchase the
area leased, the price to be negotiated and determined at the
iv. Proper Exercise of Option Contract time the option to purchase is exercised.

An option is a contract by which the owner of the property agrees


2. Right of First Refusal with another person that the latter shall have the right to buy the
former’s property at a fixed price within a certain time. It is a
condition offered or contract by which the owner stipulates with
POLYTECHNIC UNIVERSITY OF THE PHILIPPINES vs another that the latter shall have the right to buy the property at a
GOLDEN HORIZON REALTY CORP. fixed price within a certain time, or under, or in compliance with
x-----------------------------------------------x certain terms and conditions; or which gives to the owner of the
NATIONAL DEVELOPMENT AUTHORITY vs GOLDEN property the right to sell or demand a sale. It binds the party, who
HORIZON REALTY CORP. has given the option, not to enter into the principal contract with
G.R. No. 183612; G.R. No. 184260 any other person during the period designated, and, within that
MARCH 15, 2010 period, to enter into such contract with the one to whom the
option was granted, if the latter should decide to use the option.
(This is case related to the case of PUP vs. CA and Firestone
Ceramics) Upon the other hand, a right of first refusal is a contractual grant,
not of the sale of a property, but of the first priority to buy the
FACTS: National Development Corp. (NDC) had in its disposal a property in the event the owner sells the same. As distinguished
10-hectare property located at Sta. Mesa, Manila. The estate from an option contract, in a right of first refusal, while the object
was popularly known as NDC Compound. might be made determinate, the exercise of the right of first
refusal would be dependent not only on the owner’s eventual
On September 7, 1977 NDC entered into a Contract of Lease intention to enter into a binding juridical relation with another but
with Golden Horizon Realty Corp. (GHRC) over a portion of the also on terms, including the price, that are yet to be firmed up.
property with an area of 2,407 sq. m. for a period of 10 years,
renewable for another 10 years with mutual consent of the
When a lease contract contains a right of first refusal, the lessor
parties.
has the legal duty to the lessee not to sell the leased property to
anyone at any price until after the lessor has made an offer to
On May 4, 1978, a second Contract of Lease was executed by
sell the property to the lessee and the lessee had failed to accept
NDC and GHRC covering 3,222 sq. m., also renewable upon the
it. Only after the lessee has failed to exercise his right of first
mutual consent after the expiration of the 10-year lease period.
priority could the lessor sell the property to other buyers under
In addition, GHRC was granted the “option to purchase the area
the same terms and conditions offered to the lessee, or under
leased, the price to be negotiated and determined at the time the
terms and conditions more favorable to the lessor.
option to purchase is exercised.
NDC contended that the ruling of the Court in PUP vs CA and
On June 13, 1988, before the expiration of the 10-year period
Firestone cannot be applied in this case because the lease
under the second contract, GHRC wrote a letter to NDC
contract of firestone had not yet expired while in this case
indicating its exercise of the option to renew the lease for another
GHRC’s lease contract have already expired. This is untenable.
10 years. NDC gave no response to the said letter.
The reckoning point of the offer of sale to a third party was not
In September of the same year, GHRC discovered that NDC had
the issuance of Memorandum Order No. 214 on January 6, 1989
decided to secretly dispose the property to a third party, PUP.
but the commencement of such negotiations as early as July
This led to the filing of cases before the trial court.
1988 when GHRC’s right of first refusal was still subsisting and
the lease contracts still in force. NDC did not bother to respond
In the meantime President Aquino issued Memo. Order No. 214
to GHRC’s letter of June 13, 1988 informing it of GHRC’s
dated January 6, 1989 ordering the transfer of the whole NDC
exercise of the option to renew and requesting to discuss further
Compound to the National Government, which in turn would
the matter with NDC, nor to the subsequent letter of August 12,
convey the said property in favor of PUP at acquisition cost.
1988 reiterating the request for renewing the lease for another
ten (10) years and also the exercise of the option to purchase
PUP then contended that GHRC’s right to exercise the option to
under the lease contract. NDC had dismissed these letters as
purchase had expired with the termination of the original contract
"mere informative in nature, and a request at its best."
of lease and was not carried over to the subsequent implied new
lease between GHRC and NDC. Moreover, the contracts clearly
GHRC is similarly situated with Firestone such that it was also
state that GHRC is granted the option to “renew for another 10
prejudiced by NDC’s sale to PUP. Therefore, GHRC is entitled to
years with mutual consent of both parties.” As regards the
exercise its option to purchase until October 1988 in as much as
continued receipt of rentals by NDC and possession by GHRC of
the May 4, 1978 contract embodied the option to renew the lease
the leased premises, the impliedly renewed lease was only
contract for another 10 years upon mutual consent and giving
month-to-month and not 10 years since the rentals are being
GHRC the option to purchase the leased premises for a price to
paid on monthly basis.
COMPILED BY: WIGMORE #WIGMOREFOREVER 83
SALES Case Digest (Atty. Sarona)
Compiled by: Wigmore #wigmoreforever

be negotiated and determined at the time such option was considering the mercurial and uncertain forces in our market
exercised by GHRC. It to be noted that MO 214 itself declared economy today, the same right of first refusal to herein
that the transfer is “subject to such liens/leases existing on the plaintiffs/appellants in the event that the subject property is sold
subject property.” for a price in excess of Eleven Million pesos or more had
become final to the effect

ANG YU ASUNCION vs. THE HON. COURT OF APPEALS The owners were ordered to execute the necessary Deed of Sale
G.R. No. 109125 December 2, 1994 of the property in litigation in favor of plaintiffs Ang Yu Asuncion,
Keh Tiong and Arthur Go for the consideration of P15 Million
FACTS: Ang Yu Asuncion and Keh Tiong, et al., are tenants or pesos in recognition of plaintiffs' right of first refusal and that a
lessees of residential and commercial spaces owned by Cu new Transfer Certificate of Title be issued in favor of the buyer.
Unjieng, Rose Cu Unjieng and Jose Tan. They have occupied All previous transactions involving the same property
said spaces since 1935 and have been religiously paying the notwithstanding the issuance of another title to Buen Realty
rental and complying with all the conditions of the lease contract. Corporation, is hereby set aside as having been executed in bad
On several occasions before October 9, 1986, the owners faith.
informed Ang Yu’s party that they are offering to sell the
premises and are giving them priority to acquire the same. A writ of execution was subsequently issued.
During the negotiations, Bobby Cu Unjieng offered a price of P6-
million while Asuncion and Keh Tiong made a counter offer of
P5-million. They thereafter asked the owners to put their offer in ISSUE: Whether or not a Writ of Execution may be decreed on a
writing to which request they acceded; that in reply to judgment recognizing the right of first refusal.
defendant's letter, plaintiffs wrote them on October 24, 1986
asking that they specify the terms and conditions of the offer to HELD: NO. Writ of Execution is not a remedy. In the law on
sell; that when Asuncion did not receive any reply, they sent sales, the so-called "right of first refusal" is an innovative juridical
another letter dated January 28, 1987 with the same request. relation. Needless to point out, it cannot be deemed a perfected
Since the owners failed to specify the terms and conditions of the contract of sale under Article 1458 of the Civil Code. Neither can
offer to sell and because of information received that the owners the right of first refusal, understood in its normal concept, per
were about to sell the property, Ang Yu Asuncion and Keh Tiong se be brought within the purview of an option under the second
were compelled to file the complaint to compel defendants to sell paragraph of Article 1479, aforequoted, or possibly of an offer
9
the property to them. under Article 1319 of the same Code.

10
The trial court found that Cu Unjieng, Rose Cu Unjieng and Jose An option or an offer would require, among other things, a
Tan offer to sell was never accepted by the Ang Yu Asuncion clear certainty on both the object and the cause or consideration
and Keh Tiong, et al., for the reason that the parties did not of the envisioned contract. In a right of first refusal, while the
agree upon the terms and conditions of the proposed sale, object might be made determinate, the exercise of the right,
hence, there was no contract of sale at all. Nonetheless, the however, would be dependent not only on the grantor's eventual
lower court ruled that should the defendants subsequently offer intention to enter into a binding juridical relation with another but
their property for sale at a price of P11-million or below, plaintiffs also on terms, including the price, that obviously are yet to be
will have the right of first refusal. later firmed up. Prior thereto, it can at best be so described as
merely belonging to a class of preparatory juridical relations
Aggrieved by the decision that there was no contract of sale at governed not by contracts (since the essential elements to
all, the lessees brought a petition for review on certiorari to the establish the vinculum juris would still be indefinite and
Supreme Court. The Supreme Court denied the appeal on May inconclusive) but by, among other laws of general application,
6, 1991. the pertinent scattered provisions of the Civil Code on human
conduct.
On November 15, 1990, while the case filed by the lessees was
pending consideration, the Cu Unjieng spouses executed a Deed Even on the premise that such right of first refusal has been
of Sale, transferring the property in question to Buen Realty and decreed under a final judgment, like here, its breach cannot
Development Corporation for 15M. justify correspondingly an issuance of a writ of execution under a
judgment that merely recognizes its existence, nor would it
sanction an action for specific performance without thereby
As a consequence of the sale, TCT No. 105254/T-881 in the negating the indispensable element of consensuality in the
name of the Cu Unjieng spouses was cancelled and, in lieu 11
perfection of contracts. It is not to say, however, that the right
thereof, TCT No. 195816 was issued in the name of petitioner on of first refusal would be inconsequential for, such as already
December 3, 1990. intimated above, an unjustified disregard thereof, given, for
12
instance, the circumstances expressed in Article 19 of the Civil
On July 1, 1991, petitioner as the new owner of the subject Code, can warrant a recovery for damages.
property wrote a letter to the lessees demanding that the latter
vacate the premises. The final judgment in Civil Case No. 87-41058, it must be
stressed, has merely accorded a "right of first refusal" in favor of
The decision that should the the owners decide to offer the petitioners. The consequence of such a declaration entails no
property for sale for a price of P11 Million or lower, and more than what has heretofore been said. In fine, if, as it is here
COMPILED BY: WIGMORE #WIGMOREFOREVER 84
SALES Case Digest (Atty. Sarona)
Compiled by: Wigmore #wigmoreforever

so conveyed to us, petitioners are aggrieved by the failure of Subsequently the property was offered for sale to plaintiff by the
private respondents to honor the right of first refusal, the remedy defendant for the sum of FIFTEEN MILLION (P15,000,000.00)
is not a writ of execution on the judgment, since there is none to PESOS. Plaintiff was given ten (10) days to make good of the
execute, but an action for damages in a proper forum for the offer.
purpose.
On May 8, 1989, before the period given in the letter offering the
Furthermore, whether private respondent Buen Realty properties for sale expired, plaintiff's counsel wrote counsel of
Development Corporation, the alleged purchaser of the property, defendant Santos offering to buy the properties for FIVE
has acted in good faith or bad faith and whether or not it should, MILLION PESOS.
in any case, be considered bound to respect the registration of
the lis pendens in Civil Case No. 87-41058 are matters that must On May 15, 1989, before they replied to the offer to purchase,
be independently addressed in appropriate proceedings. Buen another deed of sale was executed by defendant Santos (in favor
Realty, not having been impleaded in Civil Case No. 87-41058, of) defendant Raymundo for a consideration of NINE MILLION
cannot be held subject to the writ of execution issued by PESOS.
respondent Judge, let alone ousted from the ownership and
possession of the property, without first being duly afforded its
day in court. Defendant Santos violated again paragraph 9 of the contract of
lease by executing a second deed of sale to defendant
Raymundo.
PARAÑAQUE KINGS ENTERPRISES, INC. vs. COURT OF
APPEALS ISSUE: Whether or not there is a violation on the right of first
G.R. No. 111538 February 26, 1997 refusal.

Catalina L. Santos is the owner of eight (8) parcels of land HELD. Yes. In order to have full compliance with the contractual
located at Parañaque, Metro Manila. On November 28, 1977, a right granting petitioner the first option to purchase, the sale of
certain Frederick Chua leased the property from defendant the properties for the amount of P9 million, the price for which
Catalina L. Santos. Subsequently, Chua assigned all his rights they were finally sold to respondent Raymundo, should have
and interest and participation in the leased property to Lee Ching likewise been first offered to petitioner.
Bing, by virtue of a deed of assignment and with the conformity
of defendant Santos, the said assignment was also registered. The basis of the right of first refusal* must be the current offer to
sell of the seller or offer to purchase of any prospective buyer.
Ching Bing also assigned all his rights and interest in the leased Only after the optionee fails to exercise its right of first priority
property to Parañaque Kings Enterprises, Incorporated by virtue under the same terms and within the period contemplated, could
of a deed of assignment and with the conformity of defendant the owner validly offer to sell the property to a third person,
Santos. again, under the same terms as offered to the optionee.

Paragraph 9 of the assigned leased contract provides among Deed of Assignment include the option to purchase
others that:
On the contention of respondent Santos that the assignment of
"9. That in case the properties subject of the lease the lease contract to petitioner did not include the option to
agreement are sold or encumbered, Lessors shall impose purchase. The provisions of the deeds of assignment with regard
as a condition that the buyer or mortgagee thereof shall to matters assigned were very clear. Under the first assignment
recognize and be bound by all the terms and conditions of between Frederick Chua as assignor and Lee Ching Bing as
this lease agreement and shall respect this Contract of assignee, it was expressly stated that:
Lease as if they are the LESSORS thereof and in case of
sale, LESSEE shall have the first option or priority to buy . . . . the ASSIGNOR hereby CEDES, TRANSFERS and
the properties subject of the lease;" ASSIGNS to herein ASSIGNEE, all his rights, interest and
participation over said premises afore-described, . . . .
On September 21, 1988, defendant Santos sold the eight parcels
of land subject of the lease to defendant David Raymundo for a And under the subsequent assignment executed between Lee
consideration of FIVE MILLION PESOS. The said sale was in Ching Bing as assignor and the petitioner, represented by its
contravention of the contract of lease, for the first option or Vice President Vicenta Lo Chiong, as assignee, it was likewise
priority to buy was not offered by defendant Santos to Parañaque expressly stipulated that;
Kings Enterprises, Incorporated (plaintiff).
. . . . the ASSIGNOR hereby sells, transfers and assigns all his
Upon learning of this fact plaintiff's representative wrote a letter rights, interest and participation over said leased premises, . . . .
to defendant Santos, requesting her to rectify the error and
consequently realizing the error, she had it reconveyed to her for One of such rights included in the contract of lease and,
the same consideration of FIVE MILLION (P5,000,000.00) therefore, in the assignments of rights was the lessee's right of
PESOS. first option or priority to buy the properties subject of the lease,

COMPILED BY: WIGMORE #WIGMOREFOREVER 85


SALES Case Digest (Atty. Sarona)
Compiled by: Wigmore #wigmoreforever

as provided in paragraph 9 of the assigned lease contract. The the payments. Thereafter, they received a letter from Eufrocina
deed of assignment need not be very specific as to which rights de Leon offering to sell to them the property they were leasing for
and obligations were passed on to the assignee. It is understood P2,000,000.00. xxx.
in the general provision aforequoted that all specific rights
and obligations contained in the contract of lease are those The lessees offered to buy the property from de Leon for the
referred to as being assigned. Needless to state, respondent amount of P1,000,000.00. De Leon told them that she will be
Santos gave her unqualified conformity to both assignments of submitting the offer to the other heirs. Since then, no answer was
rights. given by de Leon as to their offer to buy the property. However,
in November 1990, Rene Joaquin came to the leased premises
introducing himself as its new owner.
ROSENCOR DEVELOPMENT CORPORATION vs. PATERNO
INQUING In January 1991, the lessees again received another letter from
G.R. No. 140479 March 8, 2001 Atty. Aguila demanding that they vacate the premises. A month
thereafter, the lessees received a letter from de Leon advising
FACTS: This is a petition for review on certiorari under Rule 45 them that the heirs of the late spouses Tiangcos have already
1
of the Rules of Court seeking reversal of the Decision of the sold the property to Rosencor. The following month Atty. Aguila
Court of Appeals dated June 25, 1999 in CA-G.R. CV No. 53963. wrote them another letter demanding the rental payment and
The Court of Appeals decision reversed and set aside the introducing herself as counsel for Rosencor/Rene Joaquin, the
2
Decision dated May 13, 1996 of Branch 217 of the Regional new owners of the premises.
Trial Court of Quezon City in Civil Case No. Q-93-18582.
The lessees requested from de Leon why she had disregarded
The case was originally filed on December 10, 1993 by Paterno the pre-emptive right she and the late Tiangcos have promised
Inquing, Irene Guillermo and Federico Bantugan, herein them. They also asked for a copy of the deed of sale between
respondents, against Rosencor Development Corporation her and the new owners thereof but she refused to heed their
(hereinafter "Rosencor"), Rene Joaquin, and Eufrocina de Leon. request. In the same manner, when they asked Rene Joaquin a
Originally, the complaint was one for annulment of absolute deed copy of the deed of sale, the latter turned down their request and
of sale but was later amended to one for rescission of absolute instead Atty. Aguila wrote them several letters demanding that
deed of sale. A complaint-for intervention was thereafter filed by they vacate the premises. The lessees offered to tender their
respondents Fernando Magbanua and Danna Lizza Tiangco. rental payment to de Leon but she refused to accept the same.
The complaint-in-intervention was admitted by the trial court in
3
an Order dated May 4, 1994. In April 1992 before the demolition can be undertaken by the
Building Official. It was at this instance that the lessees were
The facts of the case, as stated by the trial court and adopted by furnished with a copy of the Deed of Sale and discovered that
the appellate court, are as follows: they were deceived by de Leon since the sale between her and
Rene Joaquin/Rosencor took place in September 4, 1990 while
"This action was originally for the annulment of the Deed of de Leon made the offer to them only in October 1990 or after the
Absolute Sale dated September 4, 1990 between defendants sale with Rosencor had been consummated. The lessees also
Rosencor and Eufrocina de Leon but later amended (sic) praying noted that the property was sold only for P726,000.00.
for the rescission of the deed of sale.
The lessees offered to reimburse de Leon the selling price of
Paterno Inquing, Irene Guillermo and Federico Bantugan averred P726,000.00 plus an additional P274,000.00 to complete their
that they are the lessees since 1971 of a two-story residential P1,000.000.00 earlier offer. When their offer was refused, they
apartment owned by spouses Faustino and Cresencia Tiangco. filed the present action praying for the following: a) rescission of
The lease was not covered by any contract. The lessees were the Deed of Absolute Sale between de Leon and Rosencor dated
renting the premises then for P150.00 a month and were September 4, 1990; b) the defendants Rosencor/Rene Joaquin
allegedly verbally granted by the lessors the pre-emptive right to be ordered to reconvey the property to de Leon; and c) de Leon
purchase the property if ever they decide to sell the same. be ordered to reimburse the plaintiffs for the repairs of the
property, or apply the said amount as part of the price for the
4
purchase of the property in the sum of P100,000.00."
Upon the death of the spouses Tiangcos in 1975, the
management of the property was adjudicated to their heirs who
were represented by Eufrocina de Leon. The lessees were The trial court held that the right of redemption on which the
allegedly promised the same pre-emptive right by the heirs of complaint was based was merely an oral one and as such, is
Tiangcos since the latter had knowledge that this right was unenforceable under the law.
extended to the former by the late spouses Tiangcos.
ISSUE: Whether or not a right of first refusal is indeed covered
In June 1990, the lessees received a letter from Atty. Erlinda by the provisions of the New Civil Code on the statute of frauds.
Aguila demanding that they vacate the premises so that the
demolition of the building be undertaken. They refused to leave RULING: NO. It is not covered by the statute of frauds.
the premises. In that same month, de Leon refused to accept the
lessees’ rental payment claiming that they have run out of
receipts and that a new collector has been assigned to receive
COMPILED BY: WIGMORE #WIGMOREFOREVER 86
SALES Case Digest (Atty. Sarona)
Compiled by: Wigmore #wigmoreforever

A right of first refusal is not among those listed as unenforceable respondents or by the heirs of the spouses Tiangco. It is
under the statute of frauds. Furthermore, the application of axiomatic that good faith is always presumed unless contrary
Article 1403, par. 2(e) of the New Civil Code presupposes the evidence is adduced.
existence of a perfected, albeit unwritten, contract of sale. A right
of first refusal, such as the one involved in the instant case, is not On this point, we hold that the evidence on record fails to show
by any means a perfected contract of sale of real property. At that petitioners acted in bad faith in entering into the deed of sale
best, it is a contractual grant, not of the sale of the real property over the disputed property with the heirs of the spouses Tiangco.
involved, but of the right of first refusal over the property sought Respondents failed to present any evidence that prior to the sale
to be sold. of the property on September 4, 1990, petitioners were aware or
had notice of the oral right of first refusal.
It is thus evident that the statute of frauds does not contemplate
cases involving a right of first refusal. As such, a right of first
refusal need not be written to be enforceable and may be proven
by oral evidence. VAZQUEZ vs. AYALA CORPORATION
G.R. No. 149734 November 19, 2004
The next question to be ascertained is whether or not FACTS: On April 23, 1981, spouses Daniel Vasquez and Ma.
respondents have satisfactorily proven their right of first refusal Luisa M. Vasquez (hereafter, Vasquez spouses) entered into a
over the property subject of the Deed of Absolute Sale dated Memorandum of Agreement (MOA) with Ayala Corporation
September 4, 1990 between petitioner Rosencor and Eufrocina (hereafter, AYALA) with AYALA buying from the Vazquez
de Leon. spouses, all of the latter's shares of stock in Conduit
Development, Inc. (hereafter, Conduit). The main asset of
Respondents have adequately proven the existence of their right Conduit was a 49.9 hectare property in Ayala Alabang,
of first refusal. Federico Bantugan, Irene Guillermo, and Paterno Muntinlupa, which was then being developed by Conduit under a
Inquing uniformly testified that they were promised by the late development plan where the land was divided into Villages 1, 2
spouses Faustino and Crescencia Tiangco and, later on, by their and 3 of the "Don Vicente Village." The development was then
heirs a right of first refusal over the property they were currently being undertaken for Conduit by G.P. Construction and
leasing should they decide to sell the same. Moreover, Development Corp.
respondents presented a letter20 dated October 9, 1990 where
Eufrocina de Leon, the representative of the heirs of the spouses Under the MOA, Ayala was to develop the entire property, less
Tiangco, informed them that they had received an offer to buy what was defined as the "Retained Area" consisting of 18,736
the disputed property for P2,000,000.00 and offered to sell the square meters. This "Retained Area" was to be retained by the
same to the respondents at the same price if they were Vazquez spouses. The area to be developed by Ayala was called
interested. Verily, if Eufrocina de Leon did not recognize the "Remaining Area". In this "Remaining Area" were 4 lots
respondents’ right of first refusal over the property they were adjacent to the "Retained Area" and Ayala agreed to offer these
leasing, then she would not have bothered to offer the property lots for sale to the Vazquez spouses at the prevailing price at the
for sale to the respondents. It must be noted that petitioners did time of purchase. Among the relevant provisions of the MOA on
not present evidence before the trial court contradicting the this point is:
existence of the right of first refusal of respondents over the
disputed property. 5.15. The BUYER agrees to give the SELLERS a first option to
purchase four developed lots next to the "Retained Area" at the
The final question to be resolved is May a contract of sale prevailing market price at the time of the purchase."
entered into in violation of a third party’s right of first refusal be
rescinded in order that such third party can exercise said right? Taking the position that Ayala was obligated to sell the 4 lots
adjacent to the "Retained Area" within 3 years from the date of
the MOA, the Vasquez spouses sent several "reminder" letters of
The prevailing doctrine, as enunciated in some cited cases, is the approaching so-called deadline. However, no demand after
that a contract of sale entered into in violation of a right of first April 23, 1984, was ever made by the Vasquez spouses for Ayala
refusal of another person, while valid, is rescissible. to sell the 4 lots. On the contrary, one of the letters signed by
their authorized agent, Engr. Eduardo Turla, categorically stated
There is, however, a circumstance which prevents the that they expected "development of Phase 1 to be completed by
application of this doctrine in the case at bench. In some cases, February 19, 1990, three years from the settlement of the legal
the Court ordered the rescission of sales made in violation of a problems with the previous contractor."
right of first refusal precisely because the vendees therein did not
act in good faith as they were aware or should have been aware By early 1990 Ayala finished the development of the vicinity of
of the right of first refusal granted to another person by the the 4 lots to be offered for sale. The four lots were then offered to
vendors therein. be sold to the Vasquez spouses at the prevailing price in 1990.
This was rejected by the Vasquez spouses who wanted to pay at
In the instant case was an oral one given to respondents by the 1984 prices, thereby leading to a suit.
deceased spouses Tiangco and subsequently recognized by
The court ordered Ayala to sell to the Vazquez the relevant lots
their heirs. As such, in order to hold that petitioners were in bad
faith, there must be clear and convincing proof that petitioners described in the Complaint in the Ayala Alabang Village at the
were made aware of the said right of first refusal either by the price of P460.00 per square meter amounting to P1,349,540.00

COMPILED BY: WIGMORE #WIGMOREFOREVER 87


SALES Case Digest (Atty. Sarona)
Compiled by: Wigmore #wigmoreforever

In its decision, the court a quo concluded that the option to Consequently, the "offer" may be withdrawn anytime by
purchase the 4 lots is valid because it was supported by communicating the withdrawal to the other party.
consideration as the option is incorporated in the MOA where the
parties had prestations to each other. In this case, Ayala Corporation offered the subject lots for sale to
petitioners at the price of P6,500.00/square meter, the prevailing
ISSUE: whether or not paragraph 5.15 of the MOA can properly market price for the property when the offer was made on June
be construed as an option contract or a right of first refusal. 18, 1990. Insisting on paying for the lots at the prevailing market
price in 1984 of P460.00/square meter, petitioners rejected the
HELD: Paragraph 5.15 of the MOA is a mere right of first offer. Ayala Corporation reduced the price to P5,000.00/square
refusal. meter but again, petitioners rejected the offer and instead made
a counter-offer in the amount of P2,000.00/square meter. Ayala
The Court has clearly distinguished between an option contract Corporation rejected petitioners' counter-offer. With this rejection,
and a right of first refusal. An option is a preparatory contract in petitioners lost their right to purchase the subject lots.
which one party grants to another, for a fixed period and at a
determined price, the privilege to buy or sell, or to decide It cannot, therefore, be said that Ayala Corporation breached
whether or not to enter into a principal contract. It binds the party petitioners' right of first refusal and should be compelled by an
who has given the option not to enter into the principal contract action for specific performance to sell the subject lots to
with any other person during the period designated, and within petitioners at the prevailing market price in 1984.
that period, to enter into such contract with the one to whom the
option was granted, if the latter should decide to use the option.
It is a separate and distinct contract from that which the parties TANAY RECREATION CENTER AND DEVELOPMENT CORP.
may enter into upon the consummation of the option. It must be vs. CATALINA MATIENZO FAUSTO
supported by consideration. G.R. No. 140182. April 12, 2005

In a right of first refusal, on the other hand, while the object might FACTS: Petitioner Tanay Recreation Center and Development
be made determinate, the exercise of the right would be Corp. (TRCDC) is the lessee of a 3,090-square meter property
dependent not only on the grantor's eventual intention to enter located in Sitio Gayas, Tanay, Rizal, owned by Catalina Matienzo
into a binding juridical relation with another but also on terms, Fausto, under a Contract of Lease. On this property stands the
including the price, that are yet to be firmed up. Tanay Coliseum Cockpit operated by petitioner. The lease
contract provided for a 20-year term, subject to renewal within
Applied to the instant case, paragraph 5.15 is obviously a mere sixty days prior to its expiration. The contract also provided that
right of first refusal and not an option contract. Although the should Fausto decide to sell the property, petitioner shall have
paragraph has a definite object, i.e., the sale of subject lots, the the “priority right” to purchase the same.
period within which they will be offered for sale to petitioners and,
necessarily, the price for which the subject lots will be sold are On June 17, 1991, petitioner wrote Fausto informing her of its
not specified. The phrase "at the prevailing market price at the intention to renew the lease. However, it was Fausto’s daughter,
time of the purchase" connotes that there is no definite period respondent Anunciacion F. Pacunayen, who replied, asking that
within which Ayala Corporation is bound to reserve the subject petitioner remove the improvements built thereon, as she is now
lots for petitioners to exercise their privilege to purchase. Neither the absolute owner of the property. It appears that Fausto had
is there a fixed or determinable price at which the subject lots will earlier sold the property to Pacunayen and title has already been
be offered for sale. The price is considered certain if it may be transferred in her name. Petitioner filed an Amended Complaint
determined with reference to another thing certain or if the for Annulment of Deed of Sale, Specific Performance with
determination thereof is left to the judgment of a specified person Damages, and Injunction.
or persons.
In her Answer, respondent claimed that petitioner is estopped
Further, paragraph 5.15 was inserted into the MOA to give from assailing the validity of the deed of sale as the latter
petitioners the first crack to buy the subject lots at the price which acknowledged her ownership when it merely asked for a renewal
Ayala Corporation would be willing to accept when it offers the of the lease. According to respondent, when they met to discuss
subject lots for sale. It is not supported by an independent the matter, petitioner did not demand for the exercise of its option
consideration. As such it is not governed by Articles 1324 and to purchase the property, and it even asked for grace period to
1479 of the Civil Code, viz: vacate the premises.

Art. 1324. When the offeror has allowed the offeree a certain ISSUE: The contention in this case refers to petitioner’s priority
period to accept, the offer may be withdrawn at any time before right to purchase, also referred to as the right of first refusal.
acceptance by communicating such withdrawal, except when the
option is founded upon a consideration, as something paid or HELD: When a lease contract contains a right of first refusal, the
promised. lessor is under a legal duty to the lessee not to sell to anybody at
any price until after he has made an offer to sell to the latter at a
Art. 1479. A promise to buy and sell a determinate thing for a certain price and the lessee has failed to accept it. The lessee
price certain is reciprocally demandable. has a right that the lessor's first offer shall be in his favor.
Petitioner’s right of first refusal is an integral and indivisible part
An accepted unilateral promise to buy or to sell a determinate of the contract of lease and is inseparable from the whole
thing for a price certain is binding upon the promissor if the contract. The consideration for the lease includes the
promise is supported by a consideration distinct from the price. consideration for the right of first refusal and is built into the
COMPILED BY: WIGMORE #WIGMOREFOREVER 88
SALES Case Digest (Atty. Sarona)
Compiled by: Wigmore #wigmoreforever

reciprocal obligations of the parties. ISSUE: Whether or not the sale of the subject lot by Cornelio to
his sons is invalid for (1) violating the prohibitory clause in the
It was erroneous for the CA to rule that the right of first refusal lease agreement between Cornelio, as lessor-owner, and
does not apply when the property is sold to Fausto’s relative. Orlando, as lessee; and (2) contravening the right of first refusal
When the terms of an agreement have been reduced to writing, it of Orlando over the subject lot.
is considered as containing all the terms agreed upon. As such,
there can be, between the parties and their successors in HELD: No. Sale was valid. Under Article 1311 of the Civil Code,
interest, no evidence of such terms other than the contents of the the heirs are bound by the contracts entered into by their
written agreement, except when it fails to express the true intent predecessors-in-interest except when the rights and obligations
and agreement of the parties. In this case, the wording of the therein are not transmissible by their nature, by stipulation or by
stipulation giving petitioner the right of first refusal is plain and provision of law. A contract of lease is generally transmissible to
unambiguous, and leaves no room for interpretation. It simply the heirs of the lessor or lessee. It involves a property right and
means that should Fausto decide to sell the leased property the death of a party does not excuse non-performance of the
during the term of the lease, such sale should first be offered to contract. The rights and obligations pass to the heirs of the
petitioner. The stipulation does not provide for the qualification deceased and the heir is bound to respect the period of the
that such right may be exercised only when the sale is made to lease.
strangers or persons other than Fausto’s kin. Thus, under the
terms of petitioner’s right of first refusal, Fausto has the legal The parties expressly stipulated in the March 31, 1978
duty to petitioner not to sell the property to anybody, even her Agreement that Romeo, as lessee, shall transfer all his rights
relatives, at any price until after she has made an offer to sell to and interests under the lease contract with option to renew “in
petitioner at a certain price and said offer was rejected by favor of the party of Orlando, the latter’s heirs, successors and
petitioner. assigns” indicating the clear intent to allow the transmissibility of
all the rights and interests of Orlando under the lease contract
unto his heirs, successors or assigns. The rights and obligations
ESTATE OF LLENADO VS EDUARDO LLENADO ET AL under the lease contract with option to renew were transmitted
MARCH 4, 2009 from Orlando to his heirs upon his death. It does not follow,
G.R. No. 145736 however, that the lease subsisted at the time of the sale of the
subject.
FACTS: The subject of this controversy is a parcel of land
denominated as Lot 249-D-1 (subject lot) registered in the names The election of the option to renew the lease in this case cannot
of Eduardo and Jorge Llenado. The subject lot once formed part be inferred from petitioner Wenifreda’s continued possession of
of Lot 249-D owned by and registered in the name of their father, the subject lot. It was incumbent upon Wenifreda with the burden
Cornelio Llenado. of proof during the trial below to establish by some positive act
that Orlando or his heirs exercised the option to renew the lease.
Cornelio leased Lot 249-D-1 to his nephew, Romeo. On March SC held that there was no evidence presented before the trial
31, 1978, Cornelio, Romeo and the latter’s cousin Orlando court to prove that Orlando or his heirs exercised the option to
executed an Agreement whereby Romeo assigned all his rights renew prior to or at the time of the expiration of the lease. As a
to Orlando. The parties further agreed that Orlando shall have result, there was no obstacle to the sale of the subject lot by
the option to renew the lease contract and that during the period Cornelio to respondents Eduardo and Jorge as the prohibitory
that the agreement is enforced, the property cannot be sold, clause under the lease contract was no longer in force.
transferred, alienated or conveyed in whatever manner to any
third party. Orlando died and his wife, Wenifreda, took over the On the issue on the right of first refusal of Orlando and his heirs,
operation of the gasoline station. Cornelio sold Lot 249-D to his SC held that no testimonial evidence was presented to prove the
children through a deed of sale, denominated as “Kasulatan sa existence of said right. The claims based on this alleged right of
Ganap Na Bilihan,” for the sum of P160k. Lot 249-D-1 was sold first refusal cannot be sustained for its existence has not been
to Eduardo and Jorge. duly established.

Eduardo informed Wenifreda of his desire to take over the


subject lot, but the latter refused to vacate the premises despite II. PERFECTION STAGE
repeated demands. Thus, Eduardo filed a complaint for unlawful
detainer against Wenifreda. After Eduardo instituted the unlawful 1. Absolute Acceptance of a Certain Offer
detainer case, Wenifreda instituted a complaint for annulment of
deed of conveyance, title and damages against Eduardo and HEIRS OF IGNACIO VS. HOME BANKERS SAVINGS AND
Jorge. TRUST CO.
G.R. NO. 177783 , JANUARY 23, 2013
Petitioner alleged that the transfer and conveyance of the subject
lot was fraudulent and in bad faith considering that the subject lot FACTS: The case sprang from a real estate mortgage of two
was transferred and conveyed to his sons when the lease was in parcels of land in August 1981. Fausto C. Ignacio mortgaged the
full force and effect making the sale null and void; that Cornelio properties to Home Bankers Savings and Trust Company (Bank)
verbally promised Orlando that Orlando or his heirs shall have as security for a loan extended by the Bank. After Ignacio
first priority or option to buy the subject lot. Respondents claimed defaulted in the payment of the loan, the property was foreclosed
that they bought the subject lot from their father for value and in and subsequently sold to the Bank in a public auction.Ignacio
good faith. offered to repurchase the property. Universal Properties Inc.
COMPILED BY: WIGMORE #WIGMOREFOREVER 89
SALES Case Digest (Atty. Sarona)
Compiled by: Wigmore #wigmoreforever

(UPI), the bank’s collecting agent sent Ignacio a letter on March Cervantes entered into several negotiations with Villonco for sale
22, 1984 which contained the terms of the repurchase. However, of the Buendia property. Cervantes made a written offer of
Ignacio annotated in the letter new terms and conditions. He P400/sqm with a downpayment of P100,000 to serve as earnest
claimed that these were verbal agreements between himself and money. The offer also made the consummation of the sale
the Bank’s collection agent, UPI.No repurchase agreement was dependent upon the acquisition by Bormaheco of a Sta. Ana
finalized between Ignacio and the Bank. Thereafter the Bank property. Villonco made a counter-offer stating that the earnest
sold the property to third parties. Ignacio then filed an action for money was to earn 10% interest p.a. The check was enclosed
specific performance against the Bank for the reconveyance of with the reply letter. Cervantes accepted and cashed the check.
the properties after payment of the balance of the purchase The Sta. Ana Property was awarded to Bormaheco; the transfer
price. He argued that there was implied acceptance of the was also duly approved. However, Cervantes sent the check
counter-offer of the sale through the receipt of the terms by back to Villonco with the interest thereon—stating that he was no
representatives of UPI. The Bank denied that it gave its consent longer interested in selling the property. He also claims that no
to the counter-offer of Ignacio. It countered that it did not approve contract was perfected; Villonco sues for specific performance.
the unilateral amendments placed by Ignacio.
ISSUE: W/N there was a perfected contract of sale
ISSUE: Whether or not the negotiations between Ignacio and
UPI is binding on the Bank. HELD: YES. There was a perfected contract of sale. The
contract of sale is perfected at the moment there is a meeting of
HELD: A contract of sale is consensual in nature and is perfected minds upon the thing which is the object of the contract and upon
upon mere meeting of the minds. When there is merely an offer the price. From that moment, the parties may reciprocally
by one party without acceptance of the other, there is no demand performance, subject to the provisions of the law
contract. When the contract of sale is not perfected, it cannot, as governing the form of contracts.” (Art. 1475 Ibid).
an independent source of obligation, serve as a binding juridical
relation between the parties. Consent is manifested by the meeting of the offer and the
acceptance upon the thing and the cause which are to constitute
A contract of sale is perfected only when there is consent validly the contract. The offer must be certain and the acceptance
given. There is no consent when a party merely negotiates a absolute. A qualified acceptance constitutes a counter-offer” (Art.
qualified acceptance or a counter-offer. An acceptance must 1319, Civil Code). “An acceptance may be express or implied”
reflect all aspects of the offer to amount to a meeting of the (Art. 1320, Civil Code).
minds between the parties.In this case, while it is apparent that
Ignacio proposed new terms and conditions to the repurchase A contract is formed if offer is accepted, whether request for
agreement, there was no showing that the Bank approved the changes in terms is granted or not; Change does not amount to
modified offer. rejection of offer or a counter-offer. An acceptance may contain a
request for certain changes in the terms of the offer and yet be a
In the absence of conformity or acceptance by properly binding acceptance. So long as it is clear that the meaning of the
authorized bank officers of petitioner's counter-proposal, no acceptance is positively and unequivocally to accept the offer,
perfected repurchase contract was born out of the talks or whether such request is granted or not, a contract is formed.
negotiations between petitioner and Mr. Lazaro and Mr. Fajardo.
Petitioner therefore had no legal right to compel respondent bank The vendor’s change in a phrase of the offer to purchase, which
to accept the P600,000 being tendered by him as payment for change does not essentially change the terms of the offer, does
the supposed balance of repurchase price. not amount to a rejection of the offer and the tender or a counter-
offer.” (The alleged changes made in the counter-offer are
The negotiations between Ignacio and UPI, the collection agent, immaterial and are mere clarifications. The changes of the words
were merely preparatory to the repurchase agreement and, “Sta. Ana property” to another property as well as the insertion of
therefore, was not binding on the Bank. Ignacio could not compel the number “12” in the date, and the words “per annum” in the
the Bank to accede to the repurchase of the property. interest are trivial. There is no incompatibility in the offer and
counter-offer. Cervantes assented to the interest and he, in fact,
A corporation may only give valid acceptance of an offer of sale paid the same. Also, earnest money constitutes prood of the
through its authorized officers or agents. Specifically, a counter- perfection of the contract of sale and forms part of the
offer to repurchase a property will not bind a corporation by mere consideration. The condition regarding the acquisition of the Sta.
acceptance of an agent in the absence of evidence of authority Ana property was likewise fulfilled; there is thus no ground for the
from the corporation’s board of directors. refusal of Cervantes to consummate the sale.

2. When “Deviation” Allowed: 3. Sale by Auction

VILLONCO REALTY COMPANY vs BORMAHECO, INC.,


FRANCISCO N. CERVANTES and ROSARIO N. CERVANTES 4. Earnest Money
G.R. No. L-26872 July 25, 1975

FACTS: Cervantes and his wife owned 3 parcels of land along


Buendia where he buildings of Bormaheco Inc were situated.
Beside their property were lots owned by Villonco Realty.

COMPILED BY: WIGMORE #WIGMOREFOREVER 90


SALES Case Digest (Atty. Sarona)
Compiled by: Wigmore #wigmoreforever

5. Difference Between Earnest Money and Option Money terms of the contract. Doubtless, the agreement is not a mere
unilateral promise to sell, but, indeed, it is a Contract to Sell as
OESMER VS. PARAISO both the trial court and the appellate court declared in their
Decisions.
FACTS: Petitioners Rizalino, Ernesto, Leonora, Bibiano, Jr.,
Librado, Enriqueta, Adolfo, and Jesus, all surnamed Oesmer
together with Adolfo Oesmer (Adolfo) and Jesus Oesmer 6. Sale Deemed Perfected Where Offer Was Made
(Jesus), are brothers and sisters, and the co-owners of undivided
shares of two parcels of agricultural and tenanted land. Both lots
are unregistered and originally owned by their parents, Bibiano FORMAL REQUIREMENTS OF SALE
Oesmer and Encarnacion Durumpili. When the spouses Oesmer
died, petitioners, together with Adolfo and Jesus, acquired the 1. Form not Important for Validity of Sale
lots as heirs of the former by right of succession.

Respondent Paraiso Development Corporation is engaged in the NARANJA VS. CA


real estate business. In March 1989, one Rogelio Paular, brought
along petitioner Ernesto to meet with a certain Sotero Lee, FACTS: Roque Naranja was the registered owner of a parcel of
President of respondent Paraiso Development Corporation. The land, Bacolod. Roque was also a co-owner of an adjacent lot (Lot
said meeting was for the purpose of brokering the sale of No. 2) which he co-owned with his brothers, Gabino and Placido
petitioners’ properties to Respondent Corporation. Naranja.

Pursuant to the said meeting, a Contract to Sell was drafted by When Placido died, his one-third share was inherited by his
the Executive Assistant of Lee. On 1 April 1989, petitioners children, Nenita, Nazareto, Nilda, Naida and Neolanda, all
Ernesto and Enriqueta signed the aforesaid Contract to Sell. A surnamed Naranja, herein petitioners. The adjacent lot is
check in the amount of P100,000.00, payable to Ernesto, was covered by TCT No. T-18762 in the names of Roque, Gabino
given as option money. and the said children of Placido. TCT No. T-18762 remained
even after Gabino died. The other petitioners — Serafin Naranja,
Sometime thereafter, Rizalino, Leonora, Bibiano, Jr., and Librado Raul Naranja, and Amelia Naranja-Rubinos — are the children of
also signed the said Contract to Sell. However, two of the Gabino.
brothers, Adolfo and Jesus, did not sign the document.
The two lots were being leased by Esso Standard Eastern, Inc.
Petitioners, through a letter, informed the respondent company of for 30 years from 1962-1992. For his properties, Roque was
their intention to rescind the Contract to Sell and to return the being paid P200.00 per month by the company.
amount of P100,000.00 given by respondent as option money.
Respondent did not respond to the aforesaid letter. Roque had no other source of income except for the P200.00
Subsequently, the petitioners, together with Adolfo and Jesus, monthly rental of his two properties. To show his gratitude to
filed a Complaint for Declaration of Nullity or for Annulment of Belardo, Roque sold Lot No. 4 and his one-third share in Lot No.
Option Agreement or Contract to Sell with Damages. 2 to Belardo on August 21, 1981, through a Deed of Sale of Real
Property which was duly notarized by Atty. Eugenio Sanicas.
ISSUE:
(1) WON the supposed Contract to Sell is really a unilateral Roque’s copies of TCT No. T-18764 and TCT No. T-18762 were
promise to sell without consideration distinct from the price, and entrusted to Atty. Sanicas for registration of the deed of sale and
hence, void. (NO, it is indeed a Contract to Sell.) transfer of the titles to Belardo. But the deed of sale could not be
(2) WON the consideration of P100K paid is an option money. (It registered because Belardo did not have the money to pay for
is an earnest money.) the registration fees.

HELD: In the instant case, the consideration of P100,000.00 paid Belardo’s only source of income was her store and coffee shop.
by respondent to petitioners was referred to as "option money." Sometimes, her children would give her money to help with the
However, a careful examination of the words used in the contract household expenses, including the expenses incurred for
indicates that the money is not option money but earnest money. Roque’s support. At times, she would also borrow money from
Margarita Dema-ala, a neighbor. When the amount of her loan
"Earnest money" and "option money" are not the same but reached P15,000.00, Dema-ala required a security.
distinguished thus: (a) earnest money is part of the purchase
price, while option money is the money given as a distinct Roque executed a deed of sale in favor of Dema-ala, covering
consideration for an option contract; (b) earnest money is given his two properties in consideration of the P15,000.00 outstanding
only where there is already a sale, while option money applies to loan and an additional P15,000.00, for a total ofP30,000.00.
a sale not yet perfected; and, (c) when earnest money is given, Dema-ala explained that she wanted Roque to execute the deed
the buyer is bound to pay the balance, while when the would-be of sale himself since the properties were still in his name.
buyer gives option money, he is not required to buy, but may Belardo merely acted as a witness. The titles to the properties
even forfeit it depending on the terms of the option. The sum of were given to Dema-ala for safekeeping.
P100,000.00 was part of the purchase price. Although the same
was denominated as "option money," it is actually in the nature of Three days later, Roque died of influenza. The proceeds of the
earnest money or down payment when considered with the other loan were used for his treatment while the rest was spent for his

COMPILED BY: WIGMORE #WIGMOREFOREVER 91


SALES Case Digest (Atty. Sarona)
Compiled by: Wigmore #wigmoreforever

burial. are now the registered owners of the parcels of land

In 1985, Belardo fully paid the loan secured by the second deed The CA reversed the RTC Decision. The CA held that the
of sale. Dema-ala returned the certificates of title to Belardo, unregisterability of a deed of sale will not undermine its validity
who, in turn, gave them back to Atty. Sanicas. and efficacy in transferring ownership of the properties to private
respondent. The CA noted that the records were devoid of any
Unknown to Belardo, petitioners, the children of Placido and proof evidencing the alleged vitiation of Roque’s consent to the
Gabino Naranja, executed an Extrajudicial Settlement Among sale; hence, there is no reason to invalidate the sale.
Heirs on October 11, 1985, adjudicating among themselves Lot Registration is only necessary to bind third parties, which
No. 4. On February 19, 1986, petitioner Amelia Naranja-Rubinos, petitioners, being the heirs of Roque Naranja, are not. The trial
accompanied by Belardo, borrowed the two TCTs, together with court erred in applying Article 1544 of the Civil Code to the case
the lease agreement with Esso Standard Eastern, Inc., from Atty. at bar since petitioners are not purchasers of the said properties.
Sanicas on account of the loan being proposed by Belardo to Hence, it is not significant that private respondent failed to
her. Thereafter, petitioners had the Extrajudicial Settlement register the deed of sale before the extrajudicial settlement
Among Heirs notarized on February 25, 1986. With Roque’s among the heir.
copy of TCT No. T-18764 in their possession, they succeeded in
having it cancelled and a new certificate of title, TCT No. T- ISSUE: Whether or not the deed of sale must contain a technical
140184, issued in their names. description of the subject property in order to be valid

In 1987, Belardo decided to register the Deed of Sale dated HELD: The Court does not agree with petitioners’ contention that
August 21, 1981. With no title in hand, she was compelled to file a deed of sale must contain a technical description of the subject
a petition with the RTC to direct the Register of Deeds to property in order to be valid. Petitioners anchor their theory on
annotate the deed of sale even without a copy of the TCTs. In an Section 127 of Act No. 496, which provides a sample form of a
Order dated June 18, 1987, the RTC granted the petition. But deed of sale that includes, in particular, a technical description of
she only succeeded in registering the deed of sale in TCT No. T- the subject property.
18762 because TCT No. T-18764 had already been cancelled.
To be valid, a contract of sale need not contain a technical
On December 11, 1989, Atty. Sanicas prepared a certificate of description of the subject property. Contracts of sale of real
authorization, giving Belardo’s daughter, Jennelyn P. Vargas, the property have no prescribed form for their validity; they follow the
authority to collect the payments from Esso Standard Eastern, general rule on contracts that they may be entered into in
Inc. But it appeared from the company’s Advice of Fixed whatever form, provided all the essential requisites for their
Payment that payment of the lease rental had already been validity are present. The requisites of a valid contract of sale
transferred from Belardo to Amelia Naranja-Rubinos because of under Article 1458 of the Civil Code are: (1) consent or meeting
the Extrajudicial Settlement Among Heirs. of the minds; (2) determinate subject matter; and (3) price certain
in money or its equivalent.
On June 23, 1992, Belardo, through her daughter and attorney-
in-fact, Rebecca Cordero, instituted a suit for reconveyance with The failure of the parties to specify with absolute clarity the
damages. The complaint prayed that judgment be rendered object of a contract by including its technical description is of no
declaring Belardo as the sole legal owner of Lot No. 4, declaring moment. What is important is that there is, in fact, an object that
null and void the Extrajudicial Settlement Among Heirs, and TCT is determinate or at least determinable, as subject of the contract
No. T-140184, and ordering petitioners to reconvey to her the of sale. The form of a deed of sale provided in Section 127 of Act
subject property and to pay damages. No. 496 is only a suggested form. It is not a mandatory form that
must be strictly followed by the parties to a contract.
Subsequently, petitioners also filed a case against respondent for
annulment of sale and quieting of title with damages, praying, In the instant case, the deed of sale clearly identifies the subject
among others, that judgment be rendered nullifying the Deed of properties by indicating their respective lot numbers, lot areas,
Sale, and ordering the Register of Deeds of Bacolod City to and the certificate of title covering them. Resort can always be
cancel the annotation of the Deed of Sale on TCT No. T-18762. made to the technical description as stated in the certificates of
title covering the two properties.
The RTC rendered a Decision in the consolidated cases in favor
of petitioners. The trial court noted that the Deed of Sale was
defective in form since it did not contain a technical description of DALION VS. CA
the subject properties but merely indicated that they were Lot No.
4, covered by TCT No. T-18764 consisting of 136 square meters, FACTS: This is a petition to annul and set aside the decision of
and one-third portion of Lot No. 2 covered by TCT No. T-18762. the Court of Appeals rendered on May 26, 1987, upholding the
validity of the sale of a parcel of land by petitioner Segundo
The trial court held that, being defective in form, the Deed of Sale Dalion (hereafter, "Dalion") in favor of private respondent
did not vest title in private respondent. Full and absolute Ruperto Sabesaje, Jr. (hereafter, "Sabesaje").
ownership did not pass to private respondent because she failed
to register the Deed of Sale. She was not a purchaser in good On May 28, 1973, Sabesaje sued to recover ownership of a
faith since she acted as a witness to the second sale of the parcel of land, based on a private document of absolute sale,
property knowing that she had already purchased the property dated July 1, 1965, allegedly executed by Dalion, who, however
from Roque. Whatever rights private respondent had over the denied the fact of sale, contending that the document sued upon
properties could not be superior to the rights of petitioners, who is fictitious, his signature thereon, a forgery, and that subject land
COMPILED BY: WIGMORE #WIGMOREFOREVER 92
SALES Case Digest (Atty. Sarona)
Compiled by: Wigmore #wigmoreforever

is conjugal property, which he and his wife acquired in 1960 from On March 1, 1960, plaintiff-appellee Soledad Biona obtained a
Saturnina Sabesaje as evidenced by the "Escritura de Venta loan from defendant-appellant in the amount of P1,000 and as
Absoluta". security therefore, the subject property was mortgaged. It was
further agreed upon by the contracting parties that for a period of
The spouses denied claims of Sabesaje that after executing a two years until the debt is paid, defendant-appellant shall occupy
deed of sale over the parcel of land, they had pleaded with the land in dispute and enjoy the usufruct thereof.
Sabesaje, their relative, to be allowed to administer the land The two-year period elapsed but Soledad Biona was not able to
because Dalion did not have any means of livelihood. They pay her indebtedness. Defendant-appellant continued occupying
admitted, however, administering since 1958, five (5) parcels of and cultivating the subject property without protest from plaintiffs-
land in Sogod, Southern Leyte, which belonged to Leonardo appellees.
Sabesaje, grandfather of Sabesaje, who died in 1956.
On July 3, 1962, defendant-appellant paid the sum of P1,400.00
They never received their agreed 10% and 15% commission on to the Development Bank of the Philippines to cancel the
the sales of copra and abaca, respectively. Sabesaje's suit, they mortgage previously constituted by the Biona spouses on June
countered, was intended merely to harass, preempt and forestall 3, 1953.
Dalion's threat to sue for these unpaid commissions. Dalion
nonetheless still impugns the validity of the sale on the ground Thereafter, and for a period of not less than twenty-five years,
that the same is embodied in a private document, and did not defendant-appellant continued his peaceful and public
thus convey title or right to the lot in question since "acts and occupation of the property, declaring it in his name for taxation
contracts which have for their object the creation, transmission, purposes, paying real estate property taxes thereon, and causing
modification or extinction of real rights over immovable property the same to be tenanted.
must appear in a public instrument."
On June 19, 1985, plaintiffs-appellees, filed a complaint for
ISSUE: Whether or not the sale is valid? recovery of ownership, possession, accounting and damages,
with a prayer for a writ of preliminary mandatory injunction and/
HELD: Yes. The provision of Art. 1358 on the necessity of a or restraining order against defendant-appellant alleging, among
public document is only for convenience, not for validity or others, that the latter had unlawfully been depriving them of the
enforceability. It is not a requirement for the validity of a contract use, possession and enjoyment of the subject property; that the
of sale of a parcel of land that this be embodied in a public entire parcel of land, which was devoted and highly suited to
instrument. A contract of sale is a consensual contract, which palay and corn, was yielding three harvests annually, with an
means that the sale is perfected by mere consent. No particular average of one hundred twenty (120) sacks of corn and eighty
form is required for its validity. Upon perfection of the contract, cavans of rice per hectare; that plaintiffs-appellees were deprived
the parties may reciprocally demand performance (Art. 1475, of its total produce amounting to P150,000.00.
NCC), i.e., the vendee may compel transfer of ownership of the
object of the sale, and the vendor may require the vendee to pay One of the claims of defendant-appellant was that by virtue of his
the thing sold (Art. 1458, NCC). continuous and peaceful occupation of the property from the time
of its sale and for more than twenty- five years thereafter,
The trial court thus rightly and legally ordered Dalion to deliver to defendant possesses a better right thereto subject only to the
Sabesaje the parcel of land and to execute corresponding formal rights of the tenants whom he had allowed to cultivate the land
deed of conveyance in a public document. Under Art. 1498, under the Land Reform Program of the government; and that
NCC, when the sale is made through a public instrument, the plaintiffs alleged right, if any, is barred by the statutes of fraud.
execution thereof is equivalent to the delivery of the thing.
Delivery may either be actual (real) or constructive. Thus delivery ISSUE: Whether or not the deed of sale was valid and if it
of a parcel of land may be done by placing the vendee in control effectively conveyed to the private respondents the subject
and possession of the land (real) or by embodying the sale in a property
public instrument (constructive).
HELD: YES but with regard only to Soledad’s share (7/12). But
since the daughters of Biona failed to assert their rights and
HEIRS OF BIONA VS. CA allowed defendant Hilajos to occupy the land in peace for more
than 30 years, they are now stopped due to laches.
FACTS: On October 23, 1953, the late Ernesto Biona, married to
plaintiff-appellee Soledad Biona, was awarded Homestead All the requisites for a valid contract of sale are present in the
Patent over the property subject of this suit, a parcel of instant case. For a valuable consideration of P4,500.00, Soledad
agricultural land, located in Bo. 3, Banga, Cotabato, Biona agreed to sell and actually conveyed the subject property
to private respondent. The fact that the deed of sale was not
On June 3, 1954, Ernesto and Soledad Biona obtained a loan notarized does not render the agreement null and void and
from the then Rehabilitation Finance Corporation (now the without any effect. The provision of Article 1358 of the Civil
Development Bank of the Philippines) and put up as collateral Code9 on the necessity of a public document is only for
the subject property. On June 12, 1956, Ernesto Biona died convenience, and not for validity or enforceability.10 The
leaving as his heirs herein plaintiffs-appellees, namely, his wife, observance of which is only necessary to insure its efficacy, so
Soledad Estrobillo Vda. De Biona, and five daughters, Editha B. that after the existence of said contract had been admitted, the
Blancaflor, Marianita B. de Jesus, Vilma B. Blancaflor, Elsie B. party bound may be compelled to execute the proper
Ramos and Perlita B. Carmen. document.11 Undeniably, a contract has been entered into by
Soledad Biona and the private respondent. Regardless of its
COMPILED BY: WIGMORE #WIGMOREFOREVER 93
SALES Case Digest (Atty. Sarona)
Compiled by: Wigmore #wigmoreforever

form, it was valid, binding and enforceable between the parties. balance of P4,500,000.00 by imposing upon the private
respondents to pay same amount within thirty (30) days from
Under Art. 1356 of the Civil Code, contracts shall be obligatory in execution of the contract instead of the former term of ninety (90)
whatever form they may have been entered into provided all the days.
essential requisites for their necessary elements for a valid
contract of sale were met when Soledad Biona agreed to sell and Ruling of the lower court and the respondent judge: The statute
actually conveyed Lot 177 to defendant-appellant who paid the does not require a formal contract drawn up with technical
amount of P4,500.00 therefore. The deed of sale (Exh. 2) is not exactness for the language of Par. 2 of Art. 1403 of the Philippine
made ineffective merely because it is not notarized or does not Civil Code is '... an agreement...or some note or memorandum
appear in a public document. thereof,' thus recognizing a difference between the contract itself
and the written evidence which the statute require. ... The
contract of sale sued upon in this case is supported by letters
2. When Form Important in Sale and telegrams annexed to the complaint. The private
respondents having alleged that the contract is backed up by
letters and telegrams, and the same being sufficient
memorandum, the complaint states a cause of action and they
a. To Bind Third Parties should be given their day in court and allowed to substantiate
their allegations.

b. For Enforceability between the Parties: Statute of Frauds ISSUES: Whether or not there is a perfected contract of sale
between the parties. (NO) and
YUVIENGCO VS Hon. DACUYCUY and Dely Rodriguez,
Felipe Cruz, Constancia Nogar, et al. (GR No. L-55048 May Whether or not the claim for specific performance of respondents
27, 1981) is enforceable under the Statute of Frauds. (NO)

FACTS: Petitioners own a property in Tacloban City which they HELD:


intend to sell for 6.5M. They gave the private respondents the 1st issue: There was no perfected contract of sale yet because
right to purchase the property only until July 31, 1978. Private both parties are still under negotiation and hence, no meeting of
respondents replied that they agree to buy the property and they the minds. Mr. Gamboa even went to the private respondents to
will negotiate for details. Petitioner sent another telegram negotiate for the sale. Even though there was an agreement on
informing respondents that their proposal is accepted and a the terms of payment, there was no absolute acceptance
contract will be prepared. because respondents still insisted on further details.
nd
Lawyer of the petitioners, Mr.Gamboa, arrived bringing a contact 2 issue: The conclusion is inescapable that the claim of private
with an altered mode of payment which says that the balance respondents that petitioners have unjustifiably refused to
payment should be paid within 30 days instead of the former 90 proceed with the sale to them of the property in question is
days. The original terms of the parties was: respondents will pay unenforceable under the Statute of Frauds.
2M upon execution, and the remaining 4.5m after 90 days.
It is nowhere alleged in said paragraphs 8 to 12 of the complaint
In essence, the theory of petitioners is that while it is true that that there is any writing or memorandum, much less a duly
they did express willingness to sell to private respondents the signed agreement to the effect that the price of P6,500,000 fixed
subject property (land and building) for P6,500,000.00 provided by petitioners for the real property herein involved was agreed to
the latter made known their own decision to buy it not later than be paid not in cash but in installments as alleged by
July 31, 1978, the respondents' reply that they were agreeable respondents.
was not absolute, so much so that when ultimately petitioners'
representative went to Cebu City with a prepared and duly The only documented indication of the non-wholly-cash payment
signed contract for the purpose of perfecting and consummating extant in the record is that stipulated, the deeds already signed
the transaction, respondents and said representative found by the petitioners and taken to Tacloban by Atty. Gamboa for the
variance between the terms of payment stipulated in the signatures of the respondents.
prepared document and what respondents had in mind, hence
the bank draft which respondents were delivering to the In other words, the 90-day term for the balance of P4.5 M
representative was returned and the document remained insisted upon by respondents choices not appear in any note,
unsigned by respondents. writing or memorandum signed by either the petitioners or any of
them, not even by Atty. Gamboa. Hence, looking at the pose of
Hence, the action for specific performance filed by the private private respondents that there was a perfected agreement of
respondents. However, the respondents, in their complaint, purchase and sale between them and petitioners under which
contended ―That on August 1, 1978 Pedro Gamboa arrived they would pay in installments of P2 M down and P4.5 M within
Tacloban City bringing with him the prepared contract to 90 days afterwards, it is evident that such oral contract involving
purchase and to sell referred to in his telegram dated July 27, the "sale of real property" comes squarely under the Statute of
1978 for the purpose of closing the transactions referred to in Frauds.
paragraphs 8 and 9 hereof, however, to the complete surprise of
private respondents, the petitioner without giving notice to Respondent judge assumed that the requirement of perfection of
plaintiffs, changed the mode of payment with respect to the such kind of contract under Article 1475 of the Civil Code which
provides that "the contract of sale is perfected at the moment
COMPILED BY: WIGMORE #WIGMOREFOREVER 94
SALES Case Digest (Atty. Sarona)
Compiled by: Wigmore #wigmoreforever

there is a meeting of the minds upon the thing which is the object decision of the trial court 7. Hence, this petition
of the contract and upon the price", the Statute would no longer
apply as long as the total price or consideration is mentioned in ISSUE: Whether or not a contract of sale of land may be proven
some note or memorandum and there is no need of any orally. (NO)
indication of the manner in which such total price is to be paid.
HELD: The rule of thumb is that a sale of land, once
Thus, the SC held that in any sale of real property on consummated, is valid regardless of the form it may have been
installments, the Statute of Frauds read together with the entered into. For nowhere does law or jurisprudence prescribe
perfection requirements of Article 1475 of the Civil Code must be that the contract of sale be put in writing before such contract
understood and applied in the sense that the idea of payment on can validly cede or transmit rights over a certain real property
installments must be in the requisite of a note or memorandum between the parties themselves.
therein contemplated. Stated otherwise, the inessential
elements" relied upon by respondent judge must be deemed to However, in the event that a third party, as in this case, disputes
include the requirement just discussed when it comes to the ownership of the property, the person against whom that
installment sales. claim is brought cannot present any proof of such sale and
hence has no means to enforce the contract. Thus the Statute of
For the essence and thrust of the said monograph refers only to Frauds was precisely devised to protect the parties in a contract
the form of the note or memorandum which would comply with of sale of real property so that no such contract is enforceable
the Statute, and no doubt, while such note or memorandum need unless certain requisites, for purposes of proof, are met. The
not be in one single document or writing and it can be in just provisions of the Statute of Frauds pertinent to the present
sufficiently implicit tenor, imperatively the separate notes must, controversy, state:
when put together', contain all the requisites of a perfected
contract of sale. Art. 1403 (Civil Code). The following contracts are
unenforceable, unless they are ratified:
To put it the other way, under the Statute of Frauds, the contents
of the note or memorandum, whether in one writing or in xxx xxx xxx
separate ones merely indicative for an adequate understanding
of all the essential elements of the entire agreement, may be 2) Those that do not comply with the Statute of Frauds as set
said to be the contract itself, except as to the form. forth in this number. In the following cases, an agreement
hereafter made shall be unenforceable by action unless the
same, or some note or memorandum thereof, be in writing, and
CLAUDEL VS CA and HEIRS OF MACARIO (GR No 85240 subscribed by the party charged, or by his agent; evidence,
July 12, 1991) therefore, of the agreement cannot be received without the
writing, or a secondary evidence of its contents:
FACTS: As early as December 28, 1922, Basilio also known as xxx xxx xxx
"Cecilio" Claudel, acquired from the Bureau of Lands, Lot No. e) An agreement for the leasing for a longer period than one
1230 of the Muntinlupa Estate Subdivision; he secured Transfer year, or for the sale of real property or of an interest therein;
Certificate of Title (TCT) No. 7471 issued by the Registry of xxx xxx xxx
Deeds for the Province of Rizal in 1923; he also declared the lot (Emphasis supplied.)
in his name. He dutifully paid the real estate taxes thereon until
his death in 1937. Thereafter, his widow "Basilia" and later, her The purpose of the Statute of Frauds is to prevent fraud and
son Jose, one of the herein petitioners, paid the taxes. The same perjury in the enforcement of obligations depending for their
piece of land purchased by Cecilio would, however, become the evidence upon the unassisted memory of witnesses by requiring
subject of protracted litigation thirty-nine years after his death. certain enumerated contracts and transactions to be evidenced
in Writing.
Two branches of Cecilio's family contested the ownership over
the land-on one hand the children of Cecilio, namely, Modesto, The provisions of the Statute of Frauds originally appeared under
Loreta, Jose, et al. and on the other, the brother and sisters of the old Rules of Evidence. However when the Civil Code was re-
Cecilio, namely, Macario, Esperidiona, Raymunda, and Celestina written in 1949 (to take effect in 1950), the provisions of the
et. al. In 1972, the HEIRS OF CECILIO partitioned this lot among Statute of Frauds were taken out of the Rules of Evidence in
themselves. order to be included under the title on Unenforceable Contracts
in the Civil Code. The transfer was not only a matter of style but
Four years later, on December 7, 1976, private respondents to show that the Statute of Frauds is also a substantive law.
SIBLINGS OF CECILIO, filed Civil Case No. 5276-P as already
adverted to at the outset, with the then Court of First Instance of Therefore, except under the conditions provided by the Statute of
Rizal, a "Complaint for Cancellation of Titles and Reconveyance Frauds, the existence of the contract of sale made by Cecilio with
with Damages," alleging that 46 years earlier, or sometime in his siblings 13 cannot be proved.
1930, their parents had purchased from the late Cecilio Claudel
several portions of Lot No. 1230 for the sum of P30.00. They
admitted that the transaction was verbal. However, as proof of
the sale, the SIBLINGS OF CECILIO presented a subdivision
plan of the said land, dated March 25, 1930, indicating the
portions allegedly sold to the SIBLINGS OF CECILIO. The Lower
Court dismissed the case. The Court of Appeals reversed the
COMPILED BY: WIGMORE #WIGMOREFOREVER 95
SALES Case Digest (Atty. Sarona)
Compiled by: Wigmore #wigmoreforever

SPOUSES ALFREDO vs SPOUSES BORRAS (GR No 144225 c. For Validity: Sale of Realty through Agent, Authority
June 17, 2003) must be in Writing

FACTS: The Alfredo Spouses mortgaged the subject land


situated in Brgy. Culis, Mabiga, Hermosa, Bataan, to the DBP for III. CONSUMMATION
P7,000.00, and in order to pay their debt, the Alfredo Spouses
sold the subject land to the Borras Spouses for P15,000.00. The
A. Obligations of Seller
Borras paid the loan and its interest and the balance is to be paid
by the Alfredos, and they (Alfredos) delivered the Owner's
Duplicate Copy of OCT No. 284 to them (Borras).
SANTOS VS SANTOS (GR No 133895 October 2, 2001)
Later, Borras discovered that the Alfredos had re-sold portiions of
the land to several persons. Borras filed an adverse claim with FACTS: Petitioner Zenaida M. Santos is the widow of Salvador
the Register of Deeds of Bataan, and later they found out that Santos. Salvador Santos is a brother of private respondents
the Alfredos had secured a duplicate copy of OCT No. 284, the Calixto, Alberto, Antonio, and Rosa Santos-Carreon. The
tax declaration and the receipts of the realty. The Alfredos filed a spouses Jesus and Rosalia Santos are the parents of the 5
complaint for Specific Performance, they claimed that the sale, siblings.
not being in writing, is unenforceable under the Statute of
Frauds. They owned a parcel of land registered under TCT No. 27571
with an area of 154 square meters, located at Sta. Cruz Manila.
ISSUE: W/N the contract of sale is unenforceable under the On it was a four-door apartment administered by Rosalia who
Statute of Frauds. (NO) rented them out. The spouses had five children, Salvador,
Calixto, Alberto, Antonio and Rosa.
HELD: NO. The Statute of Frauds provides that a contract for the
sale of real property shall be unenforceable unless the contract On January 19, 1959 Jesus and Rosalia executed a deed of sale
or some note or memorandum of the sale is in writing and of the properties in favor of their children Salvador and Rosa.
subscribed by the party charged or his agent. The existence of TCT No. 27571 became TCT No. 60819.
the receipt dated 11 March 1970, which is a memorandum of the
sale, removes the transaction from the provisions of the Statute On November 20, 1973 Rosa in turn sold her share to Salvador
of Frauds. which resulted in the issuance of a new TCT No. 113221.
Despite the transfer of the property to Salvador, Rosalia, their
The Statute of Frauds applies only to executory contracts and mother, continued to lease and receive rentals from the
not to contracts either partially or totally performed. Thus, where apartment units.
one party has performed one‘s obligation, oral evidence will be
admitted to prove the agreement. In the instant case, the parties November 1, 1979, Jesus died. January 9, 1985, Salvador died.
have consummated the sale of the Subject Land, with both After a month (Feb 1985), Rosalia died.
sellers and buyers performing their respective obligations under
the contract of sale. In addition, a contract that violates the Shortly after, petitioner Zenaida, claiming to be Salvador's heir
Statute ofFrauds is ratified by the acceptance of benefits under (specifically, as Salvador’s widow), demanded the rent from
the contract. Antonio Hombrebueno, a tenant of Rosalia. When the latter
refused to pay, Zenaida filed and ejectment suit against him with
Alfredo spouses benefited from the contract because they paid the Metropolitan Trial Court of Manila, Branch 24, which
their DBP loan and secured the cancellation of their mortgage eventually decided in Zenaida's favor.
using the money given by Borras. Alfredo also accepted payment
of the balance of the purchase price. On January 5, 1989 - private respondents instituted an action for
reconveyance of property with preliminary injunction against
Alfredo spouses cannot invoke the Statute of Frauds to deny the petitioner in RTC of Manila, where they alleged that the two
existence of the verbal contract of sale because they have deeds of sale executed on January 19, 1959 and November 20,
performed their obligations, and have accepted benefits, under 1973 were simulated for lack of consideration. They were
the verbal contract. The Borras spouses have also performed executed to accommodate Salvador in generation funds for his
their obligations under the verbal contract. Clearly, both the business and providing him with greater business flexibility.
sellers and the buyers have consummated the verbal contract of
sale of the Subject Land. The Statute of Frauds was enacted to Zenaida argued that Salvador was the registered owner of the
prevent fraud. This law cannot be used to advance the very evil property, which could only be subjected to encumbrances or
the law seeks to prevent. liens annotated on the title; that the respondents' right to
reconveyance was already barred by prescription and laches;
and that the complaint state no cause of action.

Ruling of the lower court: RTC decided in favour of private


respondents:
a) Declaring the deed of sale executed by Rosalia Santos and
Jesus Santos on January 19, 1959, as entirely null and void for
being fictitious or stimulated and inexistent
b) Declaringthe deed of sale executed by Rosa Santos in favor of
COMPILED BY: WIGMORE #WIGMOREFOREVER 96
SALES Case Digest (Atty. Sarona)
Compiled by: Wigmore #wigmoreforever

Salvador Santos on November 20, 1973, also as entirely null and


void for being likewise fictitious or stimulated and inexistent Salvador was never placed in control of the property. The original
c) Directing ROD of Manila to cancel TCT#113221 registered in sellers retained their control and possession. Therefore, there
the name of Salvador Santos, as well as, TCT# 60819 in the was no real transfer of ownership.
names of Salvador Santos, Rosa Santos and the Transfer
Certificate of Title No. T-27571 registered in the name of Rosalia In Norkis Distributors, Inc. vs. CA, the SC held that the critical
A. Santos, married to Jesus Santos, the same to be partitioned factor in the different modes of effecting delivery, which gives
by the heirs of the said registered owners in accordance with law. legal effect to the act is the actual intention of the vendor to
deliver, and its acceptance by the vendee. Without that intention,
The trial court reasoned that notwithstanding the deeds of sale there is no tradition. In the instant case, although the spouses
transferring the property to Salvador, the spouses Rosalia and Jesus and Rosalia executed a deed of sale, they did not deliver
Jesus continued to possess the property and to exercise rights of the possession and ownership of the property to Salvador and
ownership not only by receiving the monthly rentals, but also by Rosa. They agreed to execute a deed of sale merely to
paying the realty taxes. Also, Rosalia kept the owner's duplicate accommodate Salvador to enable him to generate funds for his
copy of the title even after it was already in the name of business venture.
Salvador. Further, the spouses had no compelling reason in 1959
to sell the property and Salvador was not financially capable to
purchase it. The deeds of sale were therefore fictitious. Hence,
the action to assail the same does not prescribe.

The CA affirmed the decision of the RTC. It held that in order for DY, JR. V. CA, GELAC TRADING INC., AND ANTONIO V.
the execution of a public instrument to effect tradition, as GONZALES
provided in Article 1498 of the Civil Code, the vendor shall have
had control over the thing sold, at the moment of sale. It was not FACTS: Wilfredo Dy purchased a truck and a farm tractor
enough to confer upon the purchaser the ownership and the right through LIBRA which was also mortgaged with the latter, as a
of possession. The thing sold must be placed in his control. The security to the loan.
subject deeds of sale did not confer upon Salvador the
ownership over the subject property, because even after the Petitioner, expresses his desire to purchase his brother’s tractor
sale, the original vendors remained in dominion, control, and in a letter to LIBRA which also includes his intention to shoulder
possession thereof. its mortgaged. LIBRA approved the request. At the time that
Wilfredo Dy executed a deed of absolute sale in favor of
ISSUE: WON there was DELIVERY by the Seller petitioner, the tractor and truck were in the possession of LIBRA
for his failure to pay the amortization.
HELD: There’s CONSTRUCTIVE DELIVERY but it was NOT
EFFECTED. When petitioner finally fulfilled its obligation to pay the tractor,
LIBRA would only release the same only if he would also pay for
Petitioner in her memorandum invokes Article 1477 of the Civil the truck. In order to fulfill LIBRA’s condition, petitioner convinced
Code which provides that ownership of the thing sold is his sister to pay for the remaining truck, to which she released a
transferred to the vendee upon its actual or constructive delivery. check amounting to P22, 000. LIBRA however, insisted that the
Article 1498, in turn, provides that when the sale is made through check must be first cleared before it delivers the truck and
a public instrument, its execution is equivalent to the delivery of tractor.
the thing subject of the contract. Petitioner avers that applying
said provisions to the case, Salvador became the owner of the Meanwhile, another case penned “Gelac Trading Inc vs. Wilfredo
subject property by virtue of the two deeds of sale executed in Dy” was pending in Cebu as a case to recover for a sum of
his favor. money (P12, 269.80). By a writ of execution the court in Cebu
ordered to seize and levy the tractor which was in the premise of
Nowhere in the Civil Code, however, does it provide that LIBRA, it was sold in a public auction to which it was purchased
execution of a deed of sale is a conclusive presumption of by GELAC. The latter then sold the tractor to Antonio Gonzales.
delivery of possession. The Code merely said that the execution
shall be equivalent to delivery. The presumption can be rebutted RTC rendered in favor of petitioner. CA dismissed the case,
by clear and convincing evidence. Presumptive delivery can be alleging that it still belongs to Wilfredo Dy.
negated by the failure of the vendee to take actual possession of
the land sold. ISSUE: Whether or not there was a consummated sale between
Petitioner and LIBRA?
In Danguilan vs. IAC, 168 SCRA 22, 32 (1988), we held that for
the execution of a public instrument to effect tradition, the HELD: NO. The payment of the check was actually intended to
purchaser must be placed in control of the thing sold. When extinguish the mortgage obligation so that the tractor could be
there is no impediment to prevent the thing sold from converting released to the petitioner. It was never intended nor could it be
to tenancy of the purchaser by the sole will of the vendor, considered as payment of the purchase price because the
symbolic delivery through the execution of a public instrument is relationship between Libra and the petitioner is not one of sale
sufficient. But if, notwithstanding the execution of the instrument, but still a mortgage. The clearing or encashment of the check
the purchaser cannot have the enjoyment and material tenancy which produced the effect of payment determined the full
nor make use of it himself or through another in his name, then payment of the money obligation and the release of the chattel
delivery has not been effected. mortgage. It was not determinative of the consummation of the
COMPILED BY: WIGMORE #WIGMOREFOREVER 97
SALES Case Digest (Atty. Sarona)
Compiled by: Wigmore #wigmoreforever

sale. The transaction between the brothers is distinct and apart of the thing and make use of it himself or through another in his
from the transaction between Libra and the petitioner. The name, because such are opposed by a third person’s will, then
contention, therefore, that the consummation of the sale the delivery has not been effected. In the case at bar, therefore, it
depended upon the encashment of the check is untenable. is evident, that the mere execution of the instrument was not a
fulfillment of the vendor's obligation to deliver the thing sold, and
that from such non-fulfillment arises the purchaser's right to
ADDISON V. FELIX demand, as she has demanded, the rescission of the sale and
the return of the price.
FACTS: The defendants-appellees spouses Maciana Felix and
Balbino Tioco purchased from plaintiff-appellant A.A. Addison
four parcels of land to which Felix paid, at the time of the SPOUSES SANTOS V. CA
execution of the deed, the sum of P3,000 on account of the
purchase price. She likewise bound herself to the remainder in FACTS: Spouses Santos owned the house and lot in Better
installments, the first of P,2000 on July 15, 1914, the second of Living Subdivision, Paranaque, Metro Manila. The land together
P5,000 thirty days after the issuance to her of a certificate of title with the house, was mortgaged with the Rural Bank of Salinas,
under the Land Registration Act, and further, within ten years Inc., to secure a loan of P150K. The bank sent Rosalinda Santos
from the date of such title, P10 for each cocoanut tree in bearing a letter demanding payment of P16K in unpaid interest and other
and P5 for each such tree not in bearing that might be growing charges. Since the Santos couple had no funds, Rosalinda
on said parcels of land on the date of the issuance of title to her, offered to sell the house and lot to Carmen Caseda. After
with the condition that the total price should not exceed P85,000. inspecting the real property, Carmen and her husband agreed.
It was further stipulated that Felix was to deliver to the Addison Carmen and Rosalinda signed a document, involving the sale of
25% of the value of the products that she might obtain from the the house – P350K as full amount, P54K as downpayment.
four parcels "from the moment she takes possession of them Among other condition set is that Caseda will pay the balance of
until the Torrens certificate of title be issued in her favor," and the mortgage in the bank, real estate taxes and the electric and
that within 1 year from the date of the certificate of title in her water bills.
favor, Marciana Felix may rescind the contract of purchase and
sale. The Casedas complied with the bank mortgage and the bills. The
Santoses, seeing that the Casedas lacked the means to pay the
In January 1915, Addison, filed suit in the CFI of Manila to remaining installments and/or amortization of the loan,
compel Felix to pay the first installment of P2,000, demandable, repossessed the property. The Santoses then collected the
in accordance with the terms of the contract of sale. The rentals from the tenants. Carmen approached petitioners and
defendants Felix and her husband Tioco contended that Addison offered to pay the balance of the purchase price for the house
had absolutely failed to deliver the lands that were the subject and lot. The parties, however, could not agree, and the deal
matter of the sale, notwithstanding the demands they made upon could not push through because the Santoses wanted a higher
him for this purpose. The evidence adduced shows Addison was price.
able to designate only two of the four parcels, and more than
two-thirds of these two were found to be in the possession of one Carmen is now praying that the Santoses execute the final deed
Juan Villafuerte, who claimed to be the owner of the parts he so of conveyance over the property.
occupied. The trial court held the contract of sale to be rescinded
and ordered Addison to return to Felix the P3,000 paid on ISSUE: WON there was a perfected contract of sale? NO
account of the price, together with interest thereon at the rate of
10% per annum. HELD: A contract is what the law defines it to be, taking into
consideration its essential elements, and not what the
ISSUE: Was there a delivery made and, therefore, a transfer of contracting parties call it. Article 1458 expressly obliges the
ownership of the thing sold? vendor to transfer ownership of the thing sold as an essential
element of a contract of sale. This is because the transfer of
HELD: The Supreme Court affirmed the decision of the lower ownership in exchange for a price paid or promised is the very
court, with modification that the interest thereon will be at the rate essence of a contract of sale.
of 6% (instead of 10%) per annum from the date of the filing of
the complaint until payment. There was no transfer of ownership simultaneously with the
delivery of the property purportedly sold. The records clearly
The thing is considered to be delivered when it is placed "in the show that, notwithstanding the fact that the Casedas first took
hands and possession of the vendee." It is true that the same then lost possession of the disputed house and lot, the title to the
article declares that the execution of a public instrument is property has remained always in the name of Rosalinda Santos.
equivalent to the delivery of the thing which is the object of the Although the parties had agreed that the Casedas would assume
contract, but, in order that this symbolic delivery may produce the the mortgage, all amortization payments made by Carmen
effect of tradition, it is necessary that the vendor shall have had Caseda to the bank were in the name of Rosalinda Santos. The
such control over the thing sold that, at the moment of the sale, foregoing circumstances categorically and clearly show that no
its material delivery could have been made. Symbolic delivery valid transfer of ownership was made by the Santoses to the
through the execution of a public instrument is sufficient when Casedas. Absent this essential element, their agreement cannot
there is no impediment whatever to prevent the thing sold be deemed a contract of sale.
passing into the tenancy of the purchaser by the sole will of the
vendor. But if, notwithstanding the execution of the instrument, It was a contract to sell. Ownership is reserved by the vendor
the purchaser cannot have the enjoyment and material tenancy and is not to pass until full payment of the purchase price. This
COMPILED BY: WIGMORE #WIGMOREFOREVER 98
SALES Case Digest (Atty. Sarona)
Compiled by: Wigmore #wigmoreforever

we find fully applicable and understandable in this case, given


that the property involved is a titled realty under mortgage to a In this case, no constructive delivery of the land transpired upon
bank and would require notarial and other formalities of law the execution of the deed of sale since it was not the spouses
before transfer thereof could be validly effected. Villamor, Sr. but the respondents who had actual possession of
the land. The presumption of constructive delivery is inapplicable
The CA cannot order rescission. If the vendor should eject the and must yield to the reality that the petitioners were not placed
vendee for failure to meet the condition precedent, he is in possession and control of the land.
enforcing the contract and not rescinding it. When the petitioners
in the instant case repossessed the disputed house and lot for A purchaser in good faith is one who buys property without notice
failure of private respondents to pay the purchase price in full, that some other person has a right to or interest in such property
they were merely enforcing the contract and not rescinding it. and pays its fair price before he has notice of the adverse claims
and interest of another person in the same property. However,
where the land sold is in the possession of a person other than
SPOUSES SANTIAGO V. VILLAMOR the vendor, the purchaser must be wary and must investigate the
rights of the actual possessor; without such inquiry, the buyer
FACTS: Spouses Domingo Villamor, Sr. and Trinidad Villamor cannot be said to be in good faith and cannot have any right over
(spouses Villamor, Sr.) executed a deed of sale covering a parcel the property.
of land in favor of petitioners Spouses Erosto and Nelsie
Santiago (spouses Santiago). The land in dispute was occupied
by spouses Villamor, Sr.s children, herein respondents Mancer
Villamor, Carlos Villamor, and Domingo Villamor, Jr. (Villamor LA FUERZA, INC., vs. THE HON. COURT OF APPEALS
children)
FACTS: The plaintiff (Associated Engineering, Co., Inc.) is a
Spouses Santiago demanded the Villamor children to vacate the corporation engaged in the manufacture and installation of flat
property but the latter refused to do so. Villamor children argued belt conveyors. The defendant (La Fuerza, Inc.) is also a
that they are the lawful owners of the land since they acquired corporation engaged in the manufacture of wines.
the same from San Jacinto Bank. Thus, spouses Santiago filed
an action for quieting of title before the RTC. The RTC ruled in Sometime in the month of January, 1960, Antonio Co, the
favor of spouses Santiago. On appeal, the CA reversed the manager of the plaintiff corporation called the office of the
RTCs decision on the ground that spouses Santiago failed to defendant and offered his services to manufacture and install a
prove their legal or equitable title to the land. conveyor system which, according to him, would increase
production and efficiency of his business.
ISSUE: Whether or not the action to quiet title filed by Spouses
Santiago should prosper? The president of the defendant corporation then expressed his
conformity to the offer made in Exhibit A by writing at the foot
HELD: The petition lacks merit. thereof under the word "confirmation" his signature. He caused,
however, to be added to this offer at the foot a note which reads:
CIVIL LAW: quieting of title; constructive delivery; buyer in good "All specifications shall be in strict accordance with the approved
faith plan made part of this agreement hereof."

Quieting of title is a common law remedy for the removal of any A few days later, Antonio Co made the demand for the down
cloud, doubt or uncertainty affecting title to real property. The payment of P5,000.00 which was readily delivered by the
plaintiffs must show not only that there is a cloud or contrary defendant in the form of a check for the said amount. After that
interest over the subject real property, but that they have a valid agreement, the plaintiff started to prepare the premises for the
title to it. installations of the conveyor system . It seems that the work was
completed during the month of May, 1960. Trial runs were made
Article 1477 of the Civil Code recognizes that the "ownership of in the presence of the president and general manager of the
the thing sold shall be transferred to the vendee upon the actual defendant corporation, Antonio Co, the technical manager of the
or constructive delivery thereof." Related to this article is Article plaintiff, and some other people.
1497 which provides that "the thing sold shall be understood as
delivered, when it is placed in the control and possession of the As a result of this trial or experimental runs, it was discovered,
vendee." according to the defendant's general manager, that the conveyor
system did not function to their satisfaction as represented by the
With respect to incorporeal property, Article 1498 of the Civil technical manager of the plaintiff Antonio Co for the reason that,
Code lays down the general rule: the execution of a public when operated several bottles collided with each other, some
instrument "shall be equivalent to the delivery of the thing which jumping off the conveyor belt and were broken, causing
is the object of the contract, if from the deed the contrary does considerable damage.
not appear or cannot clearly be inferred." However, the execution
of a public instrument gives rise only to a prima facie After the last trial run made in the month of July and defects
presumption of delivery, which is negated by the failure of the indicated by the said president and general manager of the
vendee to take actual possession of the land sold. A person who defendant had not been remedied with the result that when the
does not have actual possession of the thing sold cannot transfer plaintiff billed the defendant for the balance of the contract price,
constructive possession by the execution and delivery of a public the latter refused to pay for the reason that according to the
instrument. defendant the conveyor system installed by the plaintiff did not
COMPILED BY: WIGMORE #WIGMOREFOREVER 99
SALES Case Digest (Atty. Sarona)
Compiled by: Wigmore #wigmoreforever

serve the purpose for which the same was manufactured and In the exercise of this right of election, La Fuerza had chosen to
installed at such a heavy expense. withdraw from the contract, by praying for its rescission; but the
action therefor — in the language of Art. 1571 — "shall be barred
On March 22, 1961, the contractor commenced the present after six months, from the delivery of the thing sold." The period
action to recover the sums of P8,250, balance of the stipulated of four (4) years, provided in Art. 1389 of said Code, for "the
price of the aforementioned conveyors, and P2,000, as attorney's action to claim rescission," applies to contracts, in general , and
fees, in addition to the costs. must yields, in the instant case, to said Art. 1571, which refers to
sales in particular.
La Fuerza maintains that plaintiff is deemed not to have
delivered the conveyors, within the purview of Art. 1571, until it Indeed, in contracts of the latter type, especially when goods,
shall have complied with the conditions or requirements of the merchandise, machinery or parts or equipment thereof are
contract between them — that is to say, until the conveyors shall involved, it is obviously wise to require the parties to define their
meet La Fuerza's "need of a conveyor system that would position, in relation thereto, within the shortest possible time.
mechanically transport empty bottles from the storage room to
the bottle workers in the production room thus increasing the Public interest demands that the status of the relations between
production and efficiency" of its business-and La Fuerza had the vendor and the vendee be not left in a condition of
accepted said conveyors. uncertainty for an unreasonable length of time, which would be
the case, if the lifetime of the vendee's right of rescission were
ISSUE: WON there was delivery. YES four (4) years.

RULING: Upon the completion of the installation of the B. Delivery/Special Rules


conveyors, in May, 1960, particularly after the last trial run, in
July 1960, La Fuerza was in a position to decide whether or not it DAVID VS MISAMIS OCCIDENTAL
was satisfied with said conveyors, and, hence, to state whether
the same were a accepted or rejected. The failure of La Fuerza FACTS: Petitioner Virgilio S. David (David) was the owner or
to express categorically whether they accepted or rejected the proprietor of VSD Electric Sales, a company engaged in the
conveyors does not detract from the fact that the same were business of supplying electrical hardware including transformers
actually in its possession and control; that, accordingly, the for rural electric cooperatives like respondent Misamis Occidental
conveyors had already been delivered by the plaintiff; and that, II Electric Cooperative, Inc. (MOELCI), with principal office
the period prescribed in said Art. 1571 had begun to run. located in Ozamis City.

With respect to the second point raised by La Fuerza, Art. 1571 To solve its problem of power shortage affecting some areas
of the Civil Code provides: within its coverage, MOELCI expressed its intention to purchase
a 10 MVA power transformer from David. For this reason, its
Actions arising from the provisions of the preceding ten articles General Manager, Engr. Reynaldo Rada (Engr. Rada), went to
shall be barred after six months, from the delivery of the thing meet David in the latter’s office in Quezon City. David agreed to
sold. supply the power transformer provided that MOELCI would
secure a board resolution because the item would still have to be
Xx imported.

Among the "ten articles" referred to in this provision, are Articles The board resolution was thereafter attached to the proposal. As
1566 and 1567, reading: stated in the proposal, the subject transformer, together with the
basic accessories, was valued at P5,200,000.00. It was also
Art. 1566. The vendor is responsible to the vendee for any stipulated therein that 50% of the purchase price should be paid
hidden faults or defects in the thing sold, even though he was not as downpayment and the remaining balance to be paid upon
aware thereof. ."This provision shall not apply if the contrary has delivery. Freight handling, insurance, customs duties, and
been stipulated, and the vendor was not aware of the hidden incidental expenses were for the account of the buyer.
faults or defects in the thing sold.
The Board Resolution, on the other hand, stated that the
Art. 1567. In the cases of articles 1561, 1562, 1564, 1565 and purchase of the said transformer was to be financed through a
1566, the vendee may elect between withdrawing from the loan from the National Electrification Administration (NEA). As
contract and demanding a proportionate reduction of the price, there was no immediate action on the loan application, Engr.
with damages in either case. Rada returned to Manila in early December 1992 and requested
David to deliver the transformer to them even without the
xxx xxx xxx required downpayment. David granted the request provided that
MOELCI would pay interest at 24% per annum. Engr. Rada
Pursuant to these two (2) articles, if the thing sold has hidden acquiesced to the condition. On December 17, 1992, the goods
faults or defects — as the conveyors are claimed to have — the were shipped to Ozamiz City via William Lines. In the Bill of
vendor — in the case at bar, the plaintiff — shall be responsible Lading, a sales invoice was included which stated the agreed
therefor and the vendee — or La Fuerza, in the present case — interest rate of 24% per annum.
"may elect between withdrawing from the contract and
demanding a proportional reduction of the price, with damages in When no payment was made after several months, Medina was
either case." constrained to send a demand letter, dated September 15, 1993,

COMPILED BY: WIGMORE #WIGMOREFOREVER 100


SALES Case Digest (Atty. Sarona)
Compiled by: Wigmore #wigmoreforever

which MOELCI duly received. Engr. Rada replied in writing that the purchase. Then, when the loan that MOELCI was relying
the goods were still in the warehouse of William Lines again upon to finance the purchase was not forthcoming, MOELCI,
reiterating that the loan had not been approved by NEA. This through Engr. Rada, convinced David to do away with the 50%
prompted Medina to head back to Ozamiz City where he found downpayment and deliver the unit so that it could already
out that the goods had already been released to MOELCI address its acute power shortage predicament, to which David
evidenced by the shipping company’s copy of the Bill of Lading acceded when it made the delivery, through the carrier William
which was stamped "Released," and with the notation that the Lines, as evidenced by a bill of lading.
arrastre charges in the amount of P5,095.60 had been paid. This
was supported by a receipt of payment with the corresponding Second, the document specified a determinate subject matter
cargo delivery receipt issued by the Integrated Port Services of which was one (1) Unit of 10 MVA Power Transformer with
Ozamiz, Inc. corresponding KV Line Accessories. And third, the document
stated categorically the price certain in money which was
On February 17, 1994, David filed a complaint for specific P5,200,000.00 for one (1) unit of 10 MVA Power Transformer and
performance with damages with the RTC. In response, MOECLI P2,169,500.00 for the KV Line Accessories.
moved for its dismissal on the ground that there was lack of
cause of action as there was no contract of sale, to begin with, or In sum, since there was a meeting of the minds, there was
in the alternative, the said contract was unenforceable under the consent on the part of David to transfer ownership of the power
Statute of Frauds. MOELCI argued that the quotation letter could transformer to MOELCI in exchange for the price, thereby
not be considered a binding contract because there was nothing complying with the first element. Thus, the said document cannot
in the said document from which consent, on its part, to the just be considered a contract to sell but rather a perfected
terms and conditions proposed by David could be inferred. David contract of sale.
knew that MOELCI’s assent could only be obtained upon the
issuance of a purchase order in favor of the bidder chosen by the Second issue: MOELCI, in denying that the power transformer
Canvass and Awards Committee. was delivered to it, argued that the Bill of Lading which David
was relying upon was not conclusive. It argued that although the
ISSUE: bill of lading was stamped "Released," there was nothing in it
Whether or not there was a perfected contract of sale. that indicated that said power transformer was indeed released
Whether or not there was a delivery that consummated the to it or delivered to its possession. For this reason, it is its
contract. position that it is not liable to pay the purchase price of the 10
MVA power transformer.
RULING: The Court finds merit in the petition.
To begin with, among the terms and conditions of the proposal to
First issue: The elements of a contract of sale are, to wit: a) which MOELCI agreed stated:
Consent or meeting of the minds, that is, consent to transfer
ownership in exchange for the price; b) Determinate subject 2. Delivery – Ninety (90) working days upon receipt of your
matter; and c) Price certain in money or its equivalent.9 It is the purchase order and downpayment.
absence of the first element which distinguishes a contract of
sale from that of a contract to sell. C&F Manila, freight, handling, insurance, custom duties and
incidental expenses shall be for the account of MOELCI II. 13
An examination of the alleged contract to sell, "Exhibit A," despite (Emphasis supplied)
its unconventional form, would show that said document, with all
the stipulations therein and with the attendant circumstances On this score, it is clear that MOELCI agreed that the power
surrounding it, was actually a Contract of Sale. The rule is that it transformer would be delivered and that the freight, handling,
is not the title of the contract, but its express terms or stipulations insurance, custom duties, and incidental expenses shall be
that determine the kind of contract entered into by the parties.12 shouldered by it.
First, there was meeting of minds as to the transfer of ownership
of the subject matter. The letter (Exhibit A), though appearing to On the basis of this express agreement, Article 1523 of the Civil
be a mere price quotation/proposal, was not what it seemed. It Code becomes applicable.1âwphi1 It provides:
contained terms and conditions, so that, by the fact that Jimenez,
Chairman of the Committee on Management, and Engr. Rada, Where, in pursuance of a contract of sale, the seller is authorized
General Manager of MOELCI, had signed their names under the or required to send the goods to the buyer delivery of the goods
word "CONFORME," they, in effect, agreed with the terms and to a carrier, whether named by the buyer or not, for the purpose
conditions with respect to the purchase of the subject 10 MVA of transmission to the buyer is deemed to be a delivery of the
Power Transformer. As correctly argued by David, if their goods to the buyer, except in the cases provided for in Article
purpose was merely to acknowledge the receipt of the proposal, 1503, first, second and third paragraphs, or unless a contrary
they would not have signed their name under the word intent appears. (Emphasis supplied)
"CONFORME."
Thus, the delivery made by David to William Lines, Inc., as
Besides, the uncontroverted attending circumstances bolster the evidenced by the Bill of Lading, was deemed to be a delivery to
fact that there was consent or meeting of minds in the transfer of MOELCI. David was authorized to send the power transformer to
ownership. To begin with, a board resolution was issued the buyer pursuant to their agreement. When David sent the item
authorizing the purchase of the subject power transformer. Next, through the carrier, it amounted to a delivery to MOELCI.
armed with the said resolution, top officials of MOELCI visited
David’s office in Quezon City three times to discuss the terms of Furthermore, in the case of Behn, Meyer & Co. (Ltd.) v.
COMPILED BY: WIGMORE #WIGMOREFOREVER 101
SALES Case Digest (Atty. Sarona)
Compiled by: Wigmore #wigmoreforever

Yangco,14 it was pointed out that a specification in a contract On the other hand, if the seller is to pay the freight, the inference
relative to the payment of freight can be taken to indicate the is equally so strong that the duty of the seller is to have the
intention of the parties with regard to the place of delivery. So goods transported to their ultimate destination and that title to
that, if the buyer is to pay the freight, as in this case, it is property does not pass until the goods have reached their
reasonable to suppose that the subject of the sale is transferred destination.
to the buyer at the point of shipment. In other words, the title to
the goods transfers to the buyer upon shipment or delivery to the c.i.f. means Cost, Insurance and Freight = CFI is paid by the
carrier. seller.
The letters "c.i.f." found in British contracts stand for cost,
Of course, Article 1523 provides a mere presumption and in insurance, and freight. They signify that the price fixed covers not
order to overcome said presumption, MOELCI should have only the cost of the goods, but the expense of freight and
presented evidence to the contrary. The burden of proof was insurance to be paid by the seller.
shifted to MOELCI, who had to show that the rule under Article
1523 was not applicable. In this regard, however, MOELCI failed. F.O.B. stands for Free on Board = seller bear all expenses
There being delivery and release, said fact constitutes partial until goods are delivered.
performance which takes the case out of the protection of the In this case, in addition to the letters "c.i.f.," has the word
Statute of Frauds. It is elementary that the partial execution of a following, "Manila." In mercantile contracts of American origin the
contract of sale takes the transaction out of the provisions of the letters "F.O.B." standing for the words "Free on Board," are
Statute of Frauds so long as the essential requisites of consent frequently used. The meaning is that the seller shall bear all
of the contracting parties, object and cause of the obligation expenses until the goods are delivered where they are to be
concur and are clearly established to be present.15 "F.O.B."

According as to whether the goods are to be delivered "F.O.B." at


BEHN MEYER VS. YANGCO the point of shipment or at the point of destination determines the
time when property passes. However, both the terms "c.i.f." and
FACTS: A sale of 80 drums of caustic soda was agreed between "F.O.B." merely make rules of presumption which yield to proof of
Behn, Meyer & Co. and Teodoro Yanco. The merchandise was contrary intention.
shipped from New York to Manila.
Delivery was to be made at Manila.
However, the ship carrying the cargo was detained at Penang Hence, we believe that the word Manila in conjunction with the
and the 71 of the 80 drums were removed. Respondent Yangco letters "c.i.f." must mean that the contract price, covering costs,
also refused to accept the 9 remaining and also refused to insurance, and freight, signifies that delivery was to made at
accept the offer of Behn Meyer to have the products substituted Manila. If petitioner Behn Meyer has seriously thought that the
with other merchandise, which however were different from what place of delivery was New York and Not Manila, it would not have
was ordered. gone to the trouble of making fruitless attempts to substitute
goods for the merchandise named in the contract, but would
It must be noted that the contract provided for "c.i.f. Manila, have permitted the entire loss of the shipment to fall upon the
pagadero against delivery of documents." defendant.

Yanco filed an action seeking for damages for alleged breach of Behn Meyer failed to prove that it performed its part in the
contract. contract.
In this case, the place of delivery was Manila and plaintiff (Behn
ISSUE: WON Behn, Meyer & Co. should bear the burden of the Meyer) has not legally excused default in delivery of the specified
loss of the merchandise? YES merchandise at that place. In resume, we find that the plaintiff
has not proved the performance on its part of the conditions
RULING: Rule as to delivery of goods by a vendor via a precedent in the contract.
common carrier (If contract is silent – delivery of seller to
common carrier transfer ownership to buyer). For breach of warranty, the buyer (Yanco) may demand
Determination of the place of delivery always resolves itself into rescission of the contract of sale.
a question of act. If the contract be silent as to the person or The warranty — the material promise — of the seller to the buyer
mode by which the goods are to be sent, delivery by the vendor has not been complied with. The buyer may therefore rescind the
to a common carrier, in the usual and ordinary course of contract of sale because of a breach in substantial particulars
business, transfers the property to the vendee. going to the essence of the contract. As contemplated by article
1451 of the Civil Code, the vendee can demand fulfillment of the
Payment of freight by the buyer = acquires ownership at the contract, and this being shown to be impossible, is relieved of his
point of shipment. obligation. There thus being sufficient ground for rescission, the
A specification in a contact relative to the payment of freight can defendant is not liable.
be taken to indicate the intention of the parties in regard to the
place of delivery. If the buyer is to pay the freight, it is reasonable
to suppose that he does so because the goods become his at
the point of shipment.

Payment of freight by the seller = title of property does not


pass until the goods have reached their destination.
COMPILED BY: WIGMORE #WIGMOREFOREVER 102
SALES Case Digest (Atty. Sarona)
Compiled by: Wigmore #wigmoreforever

D. Double Sales
It is essential to distinguish between a contract to sell and a
CORONEL vs CA conditional contract of sale specially in cases where the subject
property is sold by the owner not to the party the seller
FACTS: The petition involves a complaint for specific contracted with, but to a third person, as in the case at bench. In
performance to compel petitioners to consummate the sale of a a contract to sell, there being no previous sale of the property, a
parcel of land with its improvements located along Roosevelt third person buying such property despite the fulfillment of the
Avenue in Quezon City entered into by the parties sometime in suspensive condition such as the full payment of the purchase
January 1985 for the price of P1,240,000.00. price, for instance, cannot be deemed a buyer in bad faith and
the prospective buyer cannot seek the relief of reconveyance of
On January 19, 1985, defendants-appellants Romulo Coronel, et the property. There is no double sale in such case. Title to the
al. (Coronels) executed a document entitled "Receipt of Down property will transfer to the buyer after registration because there
Payment" in favor of plaintiff Ramona Patricia Alcaraz is no defect in the owner-seller's title per se, but the latter, of
(hereinafter referred to as Ramona) course, may be used for damages by the intending buyer.

Clearly, the conditions appurtenant to the sale are the following: In a conditional contract of sale, however, upon the fulfillment of
1. Ramona will make a down payment P50,000.00 upon the suspensive condition, the sale becomes absolute and this will
execution of the document aforestated; 2. The Coronels will definitely affect the seller's title thereto. In fact, if there had been
cause the transfer in their names of the title of the property previous delivery of the subject property, the seller's ownership
registered in the name of their deceased father upon receipt of or title to the property is automatically transferred to the buyer
the P50,000.00 down payment; 3. Upon the transfer in their such that, the seller will no longer have any title to transfer to any
names of the subject property, the Coronels will execute the third person. Applying Article 1544 of the Civil Code, such
deed of absolute sale in favor of Ramona and the latter will pay second buyer of the property who may have had actual or
the former the whole balance of P1,190,000.00. constructive knowledge of such defect in the seller's title, or at
On the same date (January 15, 1985), Concepcion D. Alcaraz least was charged with the obligation to discover such defect,
(Concepcion), mother of Ramona, paid the down payment of cannot be a registrant in good faith. Such second buyer cannot
P50,000.00. defeat the first buyer's title. In case a title is issued to the second
buyer, the first buyer may seek reconveyance of the property
On February 6, 1985, the property originally registered in the subject of the sale.
name of the Coronels' father was transferred in their names
under TCT No. 327043. The agreement could not have been a contract to sell because
the sellers herein made no express reservation of ownership or
On February 18, 1985, the Coronels sold the property covered title to the subject parcel of land . Furthermore, the circumstance
by TCT No. 327043 to intervenor-appellant Catalina B. Mabanag which prevented the parties from entering into an absolute
(Catalina) for P1,580,000.00 after the latter has paid contract of sale pertained to the sellers themselves (the
P300,000.00. For this reason, Coronels canceled and rescinded certificate of title was not in their names) and not the full payment
the contract with Ramona by depositing the down payment paid of the purchase price. Under the established facts and
by Concepcion in the bank in trust for Ramona Patricia Alcaraz . circumstances of the case, the Court may safely presume that,
had the certificate of title been in the names of petitioners-sellers
On February 22, 1985, Concepcion, et al., filed a complaint for at that time, there would have been no reason why an absolute
specific performance against the Coronels and caused the contract of sale could not have been executed and
annotation of a notice of lis pendens at the back of TCT No. consummated right there and then.
327403.
Thus, the parties did not merely enter into a contract to sell
On April 2, 1985, Catalina caused the annotation of a notice of where the sellers, after compliance by the buyer with certain
adverse claim covering the same property with the Registry of terms and conditions, promised to sell the property to the latter.
Deeds of Quezon City (Exh. "F"; Exh. "6"). What may be perceived from the respective undertakings of the
parties to the contract is that petitioners had already agreed to
On April 25, 1985, the Coronels executed a Deed of Absolute sell the house and lot they inherited from their father, completely
Sale over the subject property in favor of Catalina to which a new willing to transfer full ownership of the subject house and lot to
title over the subject property was issued in her name. the buyer if the documents were then in order. It just happened,
however, that the transfer certificate of title was then still in the
ISSUE: WON the "Receipt of Down Payment" embodied a name of their father. It was more expedient to first effect the
perfected contract of sale, which perforce, they seek to enforce change in the certificate of title so as to bear their names. That is
by means of an action for specific performance or signified only a why they undertook to cause the issuance of a new transfer of
mere executory contract to sell, subject to certain suspensive the certificate of title in their names upon receipt of the down
conditions/ WON double sale applies. payment in the amount of P50,000.00. As soon as the new
certificate of title is issued in their names, petitioners were
RULING: The parties (Coronel and Alcaraz) had agreed to a committed to immediately execute the deed of absolute sale.
conditional contract of sale, consummation of which is subject Only then will the obligation of the buyer to pay the remainder of
only to the successful transfer of the certificate of title from the the purchase price arise.
name of petitioners' father, Constancio P. Coronel, to their
names. What is clearly established by the plain language of the subject
document is that when the said "Receipt of Down Payment" was
COMPILED BY: WIGMORE #WIGMOREFOREVER 103
SALES Case Digest (Atty. Sarona)
Compiled by: Wigmore #wigmoreforever

prepared and signed by petitioners Romeo A. Coronel, et al., the 1985. At the time of registration, therefore, petitioner Mabanag
parties had agreed to a conditional contract of sale, knew that the same property had already been previously sold to
consummation of which is subject only to the successful transfer private respondents, or, at least, she was charged with
of the certificate of title from the name of petitioners' father, knowledge that a previous buyer is claiming title to the same
Constancio P. Coronel, to their names. property. Petitioner Mabanag cannot close her eyes to the defect
in petitioners' title to the property at the time of the registration of
The Court significantly notes this suspensive condition was, in the property.
fact, fulfilled on February 6, 1985 (Exh. "D"; Exh. "4"). Thus, on
said date, the conditional contract of sale between petitioners Thus, the sale of the subject parcel of land between petitioners
and private respondent Ramona P. Alcaraz became obligatory, and Ramona P. Alcaraz, perfected on February 6, 1985, prior to
the only act required for the consummation thereof being the that between petitioners and Catalina B. Mabanag on February
delivery of the property by means of the execution of the deed of 18, 1985, was correctly upheld by both the courts below.
absolute sale in a public instrument, which petitioners
unequivocally committed themselves to do as evidenced by the
"Receipt of Down Payment." SAN LORENZO DEVELOPMENT CORPORATION VS. CA
G.R. NO. 124242, January 21, 2005
With the foregoing conclusions, the sale to the other petitioner,
Catalina B. Mabanag, gave rise to a case of double sale where FACTS: On 20 August 1986, the Spouses Lu purportedly sold
Article 1544 of the Civil Code will apply, to wit: the two parcels of land to respondent Pablo Babasanta for the
price of P15 per square meter. The latter made a downpayment
Art. 1544. If the same thing should have been sold to different of P50,000.00 as evidenced by a memorandum receipt issued by
vendees, the ownership shall be transferred to the person who Pacita Lu of the same date. Several other payments totaling
may have first taken possession thereof in good faith, if it should P200,000.00 were made by Babasanta.
be movable property. Should if be immovable property, the
ownership shall belong to the person acquiring it who in good Babasanta demanded the execution of a Final Deed of Sale in
faith first recorded it in Registry of Property. Should there be no his favor so he may effect full payment of the purchase price and
inscription, the ownership shall pertain to the person who in good notified the spouses about having received information that the
faith was first in the possession; and, in the absence thereof to spouses sold the same property to another without his
the person who presents the oldest title, provided there is good knowledge and consent. He demanded that the second sale be
faith. cancelled and that a final deed of sale be issued in his favor.

The above-cited provision on double sale presumes title or In response, Pacita Lu wrote a letter to Babasanta wherein she
ownership to pass to the first buyer, the exceptions being: (a) acknowledged having agreed to sell the property, but reminded
when the second buyer, in good faith, registers the sale ahead of Babasanta that when the balance of the purchase price became
the first buyer, and (b) should there be no inscription by either of due, he requested for a reduction of the price and when she
the two buyers, when the second buyer, in good faith, acquires refused, Babasanta backed out of the sale. Pacita added that
possession of the property ahead of the first buyer. Unless, the she returned P50,000.00 to Babasanta through Eugenio Oya.
second buyer satisfies these requirements, title or ownership will Thus, Babasanta filed a case for Specific Performance and
not transfer to him to the prejudice of the first buyer. Damages.

Petitioner point out that the notice of lis pendens in the case at On 19 January 1990, herein petitioner San Lorenzo
bar was annoted on the title of the subject property only on Development Corporation (SLDC) filed a Motion for Intervention
February 22, 1985, whereas, the second sale between and alleged that it had legal interest in the subject matter under
petitioners Coronels and petitioner Mabanag was supposedly litigation because on 3 May 1989, the two parcels of land
perfected prior thereto or on February 18, 1985. The idea involved had been sold to it in a Deed of Absolute Sale with
conveyed is that at the time petitioner Mabanag, the second Mortgage. It alleged that it was a buyer in good faith and for
buyer, bought the property under a clean title, she was unaware value and therefore it had a better right over the property in
of any adverse claim or previous sale, for which reason she is litigation.
buyer in good faith.
Respondent Babasanta argued that the latter had no legal
We are not persuaded by such argument. interest in the case because the two parcels of land involved had
already been conveyed to him by the Spouses Lu and hence, the
In a case of double sale, what finds relevance and materiality is vendors were without legal capacity to transfer or dispose of the
not whether or not the second buyer was a buyer in good faith two parcels of land to the intervenor.
but whether or not said second buyer registers such second sale
in good faith, that is, without knowledge of any defect in the title ISSUES:
of the property sold. 1. Who between SLDC and Babasanta has a better right
over the two parcels of land subject of the instant case
As clearly borne out by the evidence in this case, petitioner in view of the successive transactions executed by the
Mabanag could not have in good faith, registered the sale Spouses Lu. - SLDC
entered into on February 18, 1985 because as early as February 2. Whether or not the agreement between Babasanta and
22, 1985, a notice of lis pendens had been annotated on the Spouses Lu was a contract to sell or a contract of sale. -
transfer certificate of title in the names of petitioners, whereas Contract to Sell
petitioner Mabanag registered the said sale sometime in April,
COMPILED BY: WIGMORE #WIGMOREFOREVER 104
SALES Case Digest (Atty. Sarona)
Compiled by: Wigmore #wigmoreforever

3. Whether or not there was a double sale. - No double deed of sale where she sold the northern portion with an area of
sale 32,325 square meters to respondents for P1,500.00 and the
southern portion consisting of 8,754 square meters to Agaton
HELD: An analysis of the facts obtaining in this case, as well as Pagaduan for P500.00. (FIRST SALE)
the evidence presented by the parties, irresistibly leads to the
conclusion that the agreement between Babasanta and the Later, on June 5, 1962, Eugenia executed another deed of sale,
Spouses Lu is a contract to sell and not a contract of sale. this time conveying the entire parcel of land, including the
southern portion, in respondent’s favor (SECOND SALE). Thus,
The receipt signed by Pacita Lu merely states that she accepted TCT No. T-1221 was cancelled and in lieu thereof TCT No. T-
the sum of fifty thousand pesos (P50,000.00) from Babasanta as 5425 was issued in the name of respondents. On June 27, 1989,
partial payment of 3.6 hectares of farm lot. While there is no respondents subdivided the land into two lots.
stipulation that the seller reserves the ownership of the property
until full payment of the price which is a distinguishing feature of On July 26, 1989, petitioners instituted a complaint for
a contract to sell, the subsequent acts of the parties convince us reconveyance of the southern portion with an area of 8,754
that the Spouses Lu never intended to transfer ownership to square meters, with damages, against respondents before the
Babasanta except upon full payment of the purchase price. RTC of Olongapo City.

Babasanta’s letter dated 22 May 1989 was quite telling. He RTC decided in petitioners’ favor; a constructive trust over the
stated therein that despite his repeated requests for the property was created in petitioners’ favor.
execution of the final deed of sale in his favor so that he could
effect full payment of the price, Pacita Lu allegedly refused to do CA reversed decision; while the registration of the southern
so. In effect, Babasanta himself recognized that ownership of portion in the name of respondents had created an implied trust
the property would not be transferred to him until such time as he in favor of Agaton Pagaduan, petitioners, however, failed to show
shall have effected full payment of the price. Doubtlessly, the that they had taken possession of the said portion.
receipt signed by Pacita Lu should legally be considered as a
perfected contract to sell. ISSUE: Whether or not there was a double sale.
The perfected contract to sell imposed upon Babasanta the HELD: In this case, there was a double sale. Article 1544 should
obligation to pay the balance of the purchase price. There being apply.
an obligation to pay the price, Babasanta should have made the
proper tender of payment and consignation of the price in court ART. 1544. If the same thing should have been sold to different
as required by law. Glaringly absent from the records is any vendees, the ownership shall be transferred to the person who
indication that Babasanta even attempted to make the proper may have first possession thereof in good faith, if it should be
consignation of the amounts due, thus, the obligation on the part movable property.
of the sellers to convey title never acquired obligatory force.
Should it be immovable property, the ownership shall belong to
There was no double sale in this case because the contract in the person acquiring it who in good faith first recorded it in the
favor of Babasanta was a mere contract to sell; hence, Art. 1544 Registry of Property.
is not applicable. There was neither actual nor constructive
delivery as his title is based on a mere receipt. Based on this Should there be no inscription, the ownership shall pertain to the
alone, the right of SLDC must be preferred. person who in good faith was first in possession; and, in the
absence thereof; to the person who presents the oldest title,
provided there is good faith.
PAGADUAN VS. SPOUSES OCUMA
G.R. 176308, May 8, 2009 Where it is an immovable property that is the subject of a double
sale, ownership shall be transferred: (1) to the person acquiring it
FACTS: The subject lot used to be part of a big parcel of land who in good faith first recorded it in the Registry of Property; (2)
that originally belonged to Nicolas Cleto. The big parcel of land in default thereof, to the person who in good faith was first in
was the subject of two separate lines of dispositions. The first possession; and (3) in default thereof, to the person who
line of disposition: Cleto sold land to Antonio Cereso on May 11, presents the oldest title, provided there is good faith. The
1925. Cereso in turn sold the land to the siblings with the requirement of the law then is two-fold: acquisition in good faith
surname Antipolo on September 23, 1943. The Antipolos sold the and registration in good faith.
property to Agaton Pagaduan, father of petitioners, on March 24,
1961. All the dispositions in this line were not registered and did DOUBLE SALE: first sale by Eugenia Reyes to Agaton
not result in the issuance of new certificates of title in the name Pagaduan and a second sale by Eugenia Reyes to the
of the purchasers. respondents.

The second line of disposition: started on January 30, 1954, after For a second buyer like the respondents to successfully invoke
Cleto’s death, when his widow Ruperta Asuncion as his sole heir the second paragraph, Article 1544 of the Civil Code, it must
and new owner of the entire tract, sold the same to Eugenia possess goodvfaith from the time of the sale in its favor until the
Reyes. This resulted in the issuance Transfer Certificate of Title registration of the same. Respondents sorely failed to meet this
(TCT) No. T-1221 in the name of Eugenia Reyes in lieu of TCT requirement of good faith since they had actual knowledge of
No. T-1220 in the name of Ruperta Asuncion. Eugenia’s prior sale of the southern portion property to the
petitioners, a fact antithetical to good faith. This cannot be denied
On November 26, 1961, Eugenia Reyes executed a unilateral by respondents since in the same deed of sale that Eugenia sold
COMPILED BY: WIGMORE #WIGMOREFOREVER 105
SALES Case Digest (Atty. Sarona)
Compiled by: Wigmore #wigmoreforever

them the northern portion to the respondents for P1,500.00, February 8, 1995: Knowing that the sale to Infante has not been
Eugenia also sold the southern portion of the land to Agaton registered, Carbonell filed an adverse claim.
Pagaduan for P500.00.
February 12, 1995: The deed of sale was registered but it has an
It is to be emphasized that the Agaton Pagaduan never parted annotation of the adverse claim of Carbonell.
with the ownership and possession of that portion of Lot No. 785
which he had purchased from Eugenia Santos. Hence, the Thereafter, Emma Infante took possession of the lot, built a
registration of the deed of sale by respondents was ineffectual house and introduced some improvements.
and vested upon them no preferential rights to the property in
derogation of the rights of the petitioners. In June 1995, Carbonell filed a complaint praying that she be
declared the lawful owner of the land, that the subsequent sale to
Knowledge gained by respondents of the first sale defeats their spouses Infante be declared null and void, and that Jose Poncio
rights even if they were first to register the second sale. be ordered to execute the corresponding deed of conveyance of
Knowledge of the first sale blackens this prior registration with said land in her favor.
bad faith. Good faith must concur with the registration. Therefore,
because the registration by the respondents was in bad faith, it RTC ruled that the sale to spouses Infante was null and void.
amounted to no registration at all. As the respondents gained no After re-trial, it reversed its ruling. CA ruled in favor of Carbonell
rights over the land, it is petitioners who are the rightful owners, but after a MfR, it reversed its ruling and ruled in favor of the
having established that their successor-in-interest. Agaton Infantes.
Pagaduan had purchased the property from Eugenia Reyes on
November 26, 1961 and in fact took possession of the said ISSUE: WON Carbonell has a superior right over Emma Infante.
property. YES

HELD: Article 1544 provides that for double sale of an


immovable property, the ownership shall belong to the person
who first acquired it in good faith and recorded it in the Registry
CARBONELL VS. CA of Property
TUESDAY, SEPTEMBER 9, 2014 Article 1544, New Civil Code, which is decisive of this case,
recites:
FACTS: Respondent Jose Poncio was the owner of the parcel of
land located in Rizal. (Area – more or less 195 sq. m.) The said If the same thing should have been sold to different vendees, the
lot was subject to mortgage in favor of the Republic Savings ownership shall be transferred to the person who may have first
Bank for the sum of P1,500.00. taken possession thereof in good faith, if it should movable
property.
Carbonell and respondent Emma Infante offered to buy the said
lot from Poncio. Poncio offered to sell his lot to Carbonell Should it be immovable property, the ownership shall belong to
excluding the house on which he and his family stayed. the person acquiring it who in good faith first recorded it in the
Carbonell accepted the offer and proposed the price of P9.50/sq. Registry of Property.
m.. Poncio accepted the price on the condition that from the
purchase pric would come the money to be paid to the bank. Should there be no inscription, the ownership shall pertain to the
person who in good faith was first in the possession; and, in the
January 27, 1995: The parties executed a document in the absence thereof, to the person who presents the oldest title,
Batanes dialect which is translated as: CONTRACT FOR ONE provided there is good faith.
HALF LOT WHICH I (Poncio) BOUGHT FROM.
The buyer must act in good faith in registering the deed of sale
Carbonell asked a lawyer to prepare the deed of sale and It is essential that the buyer of realty must act in good faith in
delivered the document, together with the balance of P400, to registering his deed of sale to merit the protection of the second
Jose Poncio. (Note: Carbonell already paid P200 for the paragraph of said Article 1544.
mortgage debt of Poncio + obligated herself to pay the remaining
installments.) However, when she went to Poncio, the latter Unlike the first and third paragraphs of said Article 1544, which
informed her that he could no longer proceed with the sale as the accord preference to the one who first takes possession in good
lot was already sold to Emma Infante and that he could not faith of personal or real property, the second paragraph directs
withdraw with the sale. Poncio admitted that on January 30, that ownership of immovable property should be recognized in
1995, Mrs. Infante improved her offer and he agreed to sell the favor of one "who in good faith first recorded" his right. Under the
land and its improvements to her for P3,535.00. first and third paragraph, good faith must characterize the act of
anterior registration.
In a private memorandum agreement, Poncio bound to sell to
Infante the lot for the sum of P2,357.52, with Infante still Rule when there is inscription or not
assuming the mortgage debt of P1,177.48. (Note: The full If there is no inscription, what is decisive is prior possession in
amount of mortgage debt was already paid by the Infantes) good faith. If there is inscription, as in the case at bar, prior
registration in good faith is a pre-condition to superior title.
February 2, 1995: A deed of sale was executed between Poncio
and Infante. Carbonell was in good faith when she bought the lot
When Carbonell bought the lot from Poncio on January 27, 1955,
COMPILED BY: WIGMORE #WIGMOREFOREVER 106
SALES Case Digest (Atty. Sarona)
Compiled by: Wigmore #wigmoreforever

she was the only buyer thereof and the title of Poncio was still in the property to Carbonell especially that it can be shown that he
his name solely encumbered by bank mortgage duly annotated was aware of the offer made by Carbonell.
thereon. Carbonell was not aware — and she could not have
been aware — of any sale of Infante as there was no such sale Poncio alleged in his answer that Mrs. Infante and Mrs.
to Infante then. Carbonell offered to buy the lot at P15/sq. m. which offers he
rejected as he believed that his lot is worth at least P20.00/sq. m.
Hence, Carbonell's prior purchase of the land was made in good It is therefore logical to presume that Infante was told by Poncio
faith. Her good faith subsisted and continued to exist when she and consequently knew of the offer of Carbonell which fact
recorded her adverse claim four (4) days prior to the registration likewise should have put her on her guard and should have
of Infantes's deed of sale. compelled her to inquire from Poncio whether or not he had
already sold the property to Carbonell
Carbonell’s good faith did not cease when she was informed by
Poncio about the sale to Emma Infante The existence of prior sale to Carbonell was duly established
After learning about the second sale, Carbonell tried to talk to the From the terms of the memorandum, it tends to show that the
Infantes but the latter refused. sale of the property in favor of Carbonell is already an
accomplished act. As found by the trial court, to repeat the said
(Exact words of the SC: With an aristocratic disdain unworthy of memorandum states "that Poncio is allowed to stay in the
the good breeding of a good Christian and good neighbor, property which he had sold to the plaintiff ..., it tends to show that
Infante snubbed Carbonell like a leper and refused to see her.) the sale of the property in favor of the plaintiff is already an
accomplished act..."
So Carbonell did the next best thing to protect her right — she
registered her adversed claim on February 8, 1955. Under the There was an adequate consideration or price for the sale in
circumstances, this recording of her adverse claim should be favor of Carbonell
deemed to have been done in good faith and should emphasize Poncio agreed to sell the same to Carbonell at P9.50 per square
Infante's bad faith when she registered her deed of sale four (4) meter, on condition that Carbonell:
days later on February 12, 1955.
The Infantes were in bad faith (5 indications of bad faith listed 1. Should pay (a) the amount of P400.00 to Poncio and the
below) arrears in the amount of P247.26 to the bank
Bad faith arising from previous knowledge by Infante of the prior 2. Should assume his mortgage indebtedness.
sale to Carbonell is shown by the following facts: The bank president agreed to the said sale with assumption of
mortgage in favor of Carbonell an Carbonell accordingly paid the
1. Mrs. Infante refused to see Carbonell. arrears of P247.26.
Her refusal to talk to Carbonell could only mean that she did not
want to listen to Carbonell's story that she (Carbonell) had It is evident therefore that there was ample consideration, and
previously bought the lot from Poncio. not merely the sum of P200.00, for the sale of Poncio to
2. Carbonell was already in possession of mortgage passbook Carbonell of the lot in question.
and copy of the mortgage contract. (Not Poncio’s saving deposit
passbook.) The subject property was identified and described
The court has arrived at the conclusion that there is sufficient
Infante naturally must have demanded from Poncio the delivery description of the lot referred to in Exh. As none other than the
to her of his mortgage passbook and mortgage contract so that parcel of lot occupied by the defendant Poncio and where he has
the fact of full payment of his bank mortgage will be entered his improvements erected. The Identity of the parcel of land
therein; and Poncio, as well as the bank, must have inevitably involved herein is sufficiently established by the contents of the
informed her that said mortgage passbook could not be given to note Exh. 'A'.
her because it was already delivered to Carbonell.

3. Emma Infante did not inquire why Poncio was no longer in SPOUSES ROQUE vs. AGUADO, et.al.
possession of the mortgage passbook and why it was in G.R. No. 193787 April 7, 2014
Carbonell’s possession.
PONENTE: Perlas-Bernabe, J.
The fact that Poncio was no longer in possession of his TOPIC: Contract of conditional sale, contract to sell, double sale
mortgage passbook and that the said mortgage passbook was
already in possession of Carbonell, should have compelled FACTS: On July 21, 1977, petitioners-spouses Roque and the
Infante to inquire from Poncio why he was no longer in original owners of the then unregistered Lot 18089 – namely,
possession of the mortgage passbook and from Carbonell why Rivero, et al. executed the 1977 Deed of Conditional Sale over a
she was in possession of the same. 1,231-sq. m. portion of Lot 18089 for a consideration of
P30,775.00. The parties agreed that Sps. Roque shall make an
4. Emma Infante registered the sale under her name after initial payment of P15,387.50 upon signing, while the remaining
Carbonell filed an adverse claim 4 days earlier. balance of the purchase price shall be payable upon the
registration of Lot 18089, as well as the segregation and the
Here she was again on notice of the prior sale to Carbonell. Such concomitant issuance of a separate title over the subject portion
registration of adverse claim is valid and effective. in their names. After the deed’s execution, Sps. Roque took
possession and introduced improvements on the subject portion
5. Infante failed to inquire to Poncio WON he had already sold which they utilized as a balut factory.
COMPILED BY: WIGMORE #WIGMOREFOREVER 107
SALES Case Digest (Atty. Sarona)
Compiled by: Wigmore #wigmoreforever

HELD: It is a CONTRACT TO SELL. The Court held that where


Pertinent provision of the 1977 Deed of Conditional Sale: the seller promises to execute a deed of absolute sale upon the
completion by the buyer of the payment of the purchase price,
DEED OF CONDITIONAL SALE OF REAL PROPERTY the contract is only a contract to sell even if their agreement is
KNOW ALL MEN BY THESE PRESENTS: denominated as a Deed of Conditional Sale, as in this case. This
xxx treatment stems from the legal characterization of a contract to
That for and in consideration of the sum of THIRTY THOUSAND sell, that is, a bilateral contract whereby the prospective seller,
SEVEN HUNDRED SEVENTY FIVE PESOS (P30,775.00), while expressly reserving the ownership of the subject property
Philippine Currency, payable in the manner hereinbelow despite delivery thereof to the prospective buyer, binds himself to
specified, the VENDORS do hereby sell, transfer and convey sell the subject property exclusively to the prospective buyer
unto the VENDEE, or their heirs, executors, administrators, or upon fulfillment of the condition agreed upon, such as, the full
assignors, that unsegregated portion of the above lot, x x x. payment of the purchase price. Elsewise stated, in a contract to
That the aforesaid amount shall be paid in two installments, the sell, ownership is retained by the vendor and is not to pass to the
first installment which is in the amount of __________ vendee until full payment of the purchase price.
(P15,387.50) and the balance in the amount of __________
(P15,387.50), shall be paid as soon as the described portion of In contracts to sell the obligation of the seller to sell becomes
the property shall have been registered under the Land demandable only upon the happening of the suspensive
Registration Act and a Certificate of Title issued accordingly; condition, that is, the full payment of the purchase price by the
That as soon as the total amount of the property has been paid buyer. It is only upon the existence of the contract of sale that the
and the Certificate of Title has been issued, an absolute deed of seller becomes obligated to transfer the ownership of the thing
sale shall be executed accordingly; sold to the buyer. Prior to the existence of the contract of sale,
xxx the seller is not obligated to transfer the ownership to the buyer,
even if there is a contract to sell between them.
On August 12, 1991, Sabug, Jr, applied for a free patent over the
entire Lot 18089 and was eventually issued OCT No. M-59558 in Final installment not paid thus no perfected contract of sale
his name on October 21, 1991. On June 24, 1993, Sabug, Jr. Here, it is undisputed that Sps. Roque have not paid the final
and Rivero, in her personal capacity and in representation of installment of the purchase price. As such, the condition which
Rivero, et al., executed the 1993 Joint Affidavit, acknowledging would have triggered the parties’ obligation to enter into and
that the subject portion belongs to Sps. Roque and expressed thereby perfect a contract of sale in order to effectively transfer
their willingness to segregate the same from the entire area of the ownership of the subject portion from the sellers (i.e., Rivero
Lot 18089. et al.) to the buyers (Sps. Roque) cannot be deemed to have
been fulfilled. Consequently, the latter cannot validly claim
On December 8, 1999, however, Sabug, Jr., through the 1999 ownership over the subject portion even if they had made an
Deed of Absolute Sale, sold Lot 18089 to Aguado for initial payment and even took possession of the same.
P2,500,000.00, who, in turn, caused the cancellation of OCT No.
M-5955 and the issuance of TCT No. M-96692 dated December Conditional contract of sale and contract to sell in relation
17, 199911 in her name. to double sale
It is essential to distinguish between a contract to sell and a
Thereafter, Aguado obtained an P8,000,000.00 loan from the conditional contract of sale specially in cases where the subject
Land Bank secured by a mortgage over Lot 18089. When she property is sold by the owner not to the party the seller
failed to pay her loan obligation, Land Bank commenced extra- contracted with, but to a third person, as in the case at bench.
judicial foreclosure proceedings and eventually tendered the
highest bid in the auction sale. Upon Aguado’s failure to redeem In a contract to sell, there being no previous sale of the
the subject property, Land Bank consolidated its ownership, and property, a third person buying such property despite the
TCT No. M-11589513 was issued in its name on July 21, 2003. fulfillment of the suspensive condition such as the full
payment of the purchase price, for instance, cannot be
On June 16, 2003, Sps. Roque filed a complaint for deemed a buyer in bad faith and the prospective buyer
reconveyance, annulment of sale, deed of real estate mortgage, cannot seek the relief of reconveyance of the property.
foreclosure, and certificate of sale, and damages before the
RTC. There is no double sale in such case. Title to the property will
transfer to the buyer after registration because there is no defect
Aguado: innocent purchaser for value in the owner-seller’s title per se, but the latter, of course, may
be sued for damages by the intending buyer.
Landbank: no knowledge of Sps. Claim. At the time when the
loan was taken out, Lot 18089 was registered in Aguado’s name On the matter of double sales, suffice it to state that Sps.
64 65
and no lien was annotated on COT. Roque’s reliance on Article 1544 of the Civil Code has been
misplaced since the contract they base their claim of ownership
RTC: dismissed complaint of spouses roque and NCCP. on is, as earlier stated, a contract to sell, and not one of sale. In
66
Cheng v. Genato, the Court stated the circumstances which
CA: affirmed RTC ruling. must concur in order to determine the applicability of Article
1544, none of which are obtaining in this case, viz.:
ISSUE: Whether or not the 1977 Deed of Conditional Sale is a
conditional contract of sale or a contract to sell. (a) The two (or more) sales transactions in issue must pertain to
exactly the same subject matter, and must be valid sales
COMPILED BY: WIGMORE #WIGMOREFOREVER 108
SALES Case Digest (Atty. Sarona)
Compiled by: Wigmore #wigmoreforever

transactions; obtained, seized and impounded the car, but it was delivered
(b) The two (or more) buyers at odds over the rightful ownership back to Jimenez upon his filing of a counter-bond.
of the subject matter must each represent conflicting interests;
and The lower court held that Jimenez had the right of ownership and
(c) The two (or more) buyers at odds over the rightful ownership possession over the car.
of the subject matter must each have bought from the same
seller. ISSUE: W/N Jimenez was a purchaser in good faith and thus
entitled to the ownership and possession of the car. YES
The action for reconveyance shall fail.
HELD: It must be noted that Tagatac was not unlawfully deprived
of his car
D. Obligations of Buyer In this case, there is a valid transmission of ownership from true
owner [Tagatac] to the swindler [Feist], considering that they had
a contract of sale (note: but such sale is voidable for the fraud
PART IV: DOCUMENTS OF TITLE and deceit by Feist).

The disputable presumption that a person found in possession of


A. Types of Documents of Title and its Warranties a thing taken in the doing of a recent wrongful act is the taker
and the doer of the whole act does NOT apply in this case
because the car was not stolen from Tagatac, and Jimenez came
B. Delivery through Carrier
into possession of the car two months after Feist swindled
Tagatac.

Jimenez was a purchaser in good faith for he was not aware of


any flaw invalidating the title from the seller of the car
In addition, when Jimenez acquired the car, he had no
PART V: SALE BY NON-OWNER OR BY ONE HAVING knowledge of any flaw in the title of the person from whom he
VOIDABLE TITLE acquired it. It was only later that he became fully aware that there
were some questions regarding the car, when he filed a petition
to dissolve Tagatac’s search warrant which had as its subject the
TAGATAC VS JIMENEZ car in question.
53 OG 3792
The contract between Feist and Tagactac was a voidable
FACTS: Trinidad Tagatac bought a car for $4,500 in the US. After contract, it can be annulled or ratified
7 months, she brought the car to the Philippines. Warner Feist, . . . The fraud and deceit practiced by Warner L. Feist earmarks
who pretended to be a wealthy man, offered to buy Trinidad’s car this sale as a voidable contract (Article 1390). Being a voidable
for P15,000, and Tagatac was amenable to the idea. Hnece, a contract, it is susceptible of either ratification or annulment. (If
deed of sale was exceuted. the contract is ratified, the action to annul it is extinguished -
Article 1392) and the contract is cleansed from all its defects
Feist paid by means of a postdated check, and the car was (Article 1396); if the contract is annulled, the contracting parties
delivered to Feist. However, PNB refused to honor the checks are restored to their respective situations before the contract and
and told her that Feist had no account in said bank. mutual restitution follows as a consequence (Article 1398).

Tagatac notified the law enforcement agencies of the estafa Being a voidable contract, it remains valid and binding until
committed by Feist, but the latter was not apprehended and the annulled.
car disappeared.
However, as long as no action is taken by the party entitled,
Meanwhile, Feist managed succeeded in having the car’s either that of annulment or of ratification, the contract of sale
registration certificate (RC) transferred in his name. He sold the remains valid and binding. When plaintiff-appellant Trinidad C.
Tagatac delivered the car to Feist by virtue of said voidable
car to Sanchez, who was able to transfer the registration
contract of sale, the title to the car passed to Feist. Of course,
certificate to his name.
the title that Feist acquired was defective and voidable.
Sanchez then offered to sell the car to defendant Liberato
Nevertheless, at the time he sold the car to Felix Sanchez, his
Jimenez, who bought the car for P10,000 after investigating in
title thereto had not been avoided and he therefore conferred a
the Motor Vehicles Office.
good title on the latter, provided he bought the car in good faith,
Tagatac discovered that the car was in California Car Exchange’s for value and without notice of the defect in Feist's title (Article
1506, N.C.C.). There being no proof on record that Felix
(place where Jimenez displayed the car for sale), so she
Sanchez acted in bad faith, it is safe to assume that he acted in
demanded from the manager for the delivery of the car, but the
good faith.
latter refused.
NB: ART. 1506. Where the seller of goods has a voidable title
Tagatac filed a suit for the recovery of the car’s possession, and
thereto, but his title has not been avoided at the time of the sale,
the sheriff, pursuant to a warrant of seizure that Tagatac the buyer acquires a good title to the goods provided he buys
COMPILED BY: WIGMORE #WIGMOREFOREVER 109
SALES Case Digest (Atty. Sarona)
Compiled by: Wigmore #wigmoreforever

them in good faith, for value, and without notice of the seller’s impair the title acquired by the private respondents to the books.
defect of title.
Leonor Santos took care to ascertain first that the books
belonged to Cruz before she agreed to purchase them. The
EDCA PUBLISHING VS SPS. SANTOS EDCA invoice Cruz showed her assured her that the books had
G.R. No. 80298, April 26, 1990 been paid for on delivery. By contrast, EDCA was less than
cautious — in fact, too trusting in dealing with the impostor.
FACTS: On October 5, 1981, a person identifying himself as
Prof. Jose Cruz ordered 406 books from EDCA Publishing. Although it had never transacted with him before, it readily
EDCA Subsequently prepared the corresponding invoice and delivered the books he had ordered (by telephone) and as
delivered the books as ordered, for which Cruz issued a personal readily accepted his personal check in payment. It did not verify
check covering the purchase price of said books. Subsequently his identity although it was easy enough to do this. It did not wait
on October 7, 1981, Cruz sold 120 of the books to Leonor to clear the check of this unknown drawer. Worse, it indicated in
Santos who, after verifying the seller’s ownership from the the sales invoice issued to him, by the printed terms thereon, that
invoice he showed her, paid him P1,700. the books had been paid for on delivery, thereby vesting
ownership in the buyer.
Upon verification by EDCA, it was discovered that Cruz was not
employed as professor by De La Salle College and that he had Santos did not need to go beyond that invoice to satisfy herself
no more account or deposit with Phil. Amanah Bank, the bank that the books being offered for sale by Cruz actually belonged to
where he allegedly drawn the payment check. Upon arrest of him; yet she still did. Although the title of Cruz was presumed
Cruz by the police, it was revealed that his real name was Tomas under Article 559 by his mere possession of the books, these
dela Pena and that there was a further sale of 120 books to Sps. being movable property, Leonor Santos nevertheless demanded
Santos. more proof before deciding to buy them.

EDCA, through the assistance of the police forced their way into NB: Law on Property
the store of Sps. Santos and threatened Leonor with prosecution
for buying stolen property. The 120 books were seized and were Art. 559. The possession of movable property acquired in good
later turned over to EDCA. faith is equivalent to a title. Nevertheless, one who has lost any
movable or has been unlawfully deprived thereof may recover it
This resulted to Sps. Santos filing a case for recovery of the from the person in possession of the same.
books after their demand for the return of the books was
rejected. If the possessor of a movable lost or of which the owner has
been unlawfully deprived has acquired it in good faith at a public
ISSUE: W/N EDCA may retrieve the books from Santos. NO sale, the owner cannot obtain its return without reimbursing the
(W/N EDCA has been unlawfully deprived of the books because price paid therefor.
the heck issued by Cruz in payment thereof was dishonored.
NO.)
AZNAR VS. YAPDIANGCO
HELD: EDCA argues that because Cruz, the impostor acquired 13 SCRA 486
no title to the books, the latter could not have validly transferred G.R. No. L-18536
such to Sps. Santos. Its reason is that as the payment check March 31, 1965
bounced for lack of funds, there was a failure of consideration
that nullified the contract of sale between it and Cruz. Remember this case was discussed already before. Yung may
nagbenta ng kanyang car tapos inutusan nya ang son nya na
However, upon perusal of the provisions on the Law on Sales, a sumama dun sa gusting bumili then pumunta sila sa isang house
contract of sale is consensual and is perfected once agreement tapos nung paglabas ng anak wala na yung kotse and yung
is reached between the parties on the subject matter and the pamangkin ng bibili.
consideration. As provided in Art. 1478- Ownership in the thing
sold shall not pass to the buyer until full payment of the purchase FACTS: Teodoro Santos was selling his FORD FAIRLANE 500.
only if there is a stipulation to that effect. Otherwise, the rule is One day, a certain L. De Dios, claiming to be a nephew of
that such ownership shall pass from the vendor to the vendee Vicente Marella, said that his uncle Vicente wants to buy the car.
upon the actual or constructive delivery of the thing sold even if
the purchase price has not yet been paid. Marella agreed to buy the car for P14,700.00 on the
understanding that the price would be paid only after the car had
Non-payment only creates a right to demand payment or to been registered in his name.
rescind the contract, or to criminal prosecution in the case of
bouncing checks. But absent the stipulation above noted, The DOS for the car was executed in Marella's favor. Afterwhich,
delivery of the thing sold will effectively transfer ownership to the the car in was registered Marella's name. Up to this stage of the
buyer who can in turn transfer it to another. transaction, the purchased price had not been paid.

Actual delivery of the books having been made, Cruz acquired Teodoro Santos gave the registration papers and a copy of the
ownership over the books which he could then validly transfer to DOS to his son Irineo and instructed him not to part with them
the private respondents. The fact that he had not yet paid for until Marella has given the full payment for the car. Irineo Santos
them to EDCA was a matter between him and EDCA and did not and L. De Dios then proceeded to meet Marella and Ireneo
COMPILED BY: WIGMORE #WIGMOREFOREVER 110
SALES Case Digest (Atty. Sarona)
Compiled by: Wigmore #wigmoreforever

demanded the payment. Marella said that the amount he had on of it, he has a right to recover it, not only from the finder,
hand then was short by some P2,000.00 and begged off to be thief or robber, but also from third persons who may have
allowed to secure the shortage from a sister supposedly living acquired it in good faith from such finder, thief or robber.
somewhere on Azcarraga Street, also in Manila. Thereafter, he The said article establishes two exceptions to the general rule
ordered L. De Dios to go to the said sister and suggested that of IRREVINDICABILITY: when the owner (1) has lost the thing,
Irineo go with him. At the same time, he requested the or (2) has been unlawfully deprived thereof. In these cases, the
registration papers and the DOS from Irineo on the pretext that possessor cannot retain the thing as against the owner, who may
he would like to show them to his lawyer. Trusting the good faith recover it without paying any indemnity, except when the
of Marella, Irineo handed over the same to the latter and possessor acquired it in a public sale.
thereupon, in the company of L. De Dios and another
unidentified person, proceeded to the alleged house of Marella's
sister. It was there that the car, L. De Dios and the unidentified PART VI: LOSS, DETERIORATION, FRUITS AND OTHER
companion disappeared. That very same day, Marella was able BENEFITS
to sell the car in question Jose B. Aznar, for P15,000.00.

Aznar claims ownership over the vehicle. Trial court awarded the CHRYSLER VS. CA
vehicle to Santos. 133 SCRA 567
G.R. No. 55684
ISSUE: Between Santos and Aznar, who has a better right to the December 19, 1984
possession of the disputed automobile? - SANTOS
FACTS: Petitioner Chrysler is a domestic corporation engaged in
HELD: Aznar accepts that the car in question originally belonged the assembling and sale of motor vehicles and other automotive
to and was owned by Santos, and that the latter was unlawfully products. Respondent Sambok is a general partnership and was
deprived of the same by Marella. However, Aznar contends that a dealer for automotive products.
upon the facts of this case, the applicable provision of the CC is
Article 1506 and not Article 559 as was held by the decision Chrysler filed with CFI a complaint for damages against Allied
under review. Article 1506 provides: Brokerage Corp, Negros Navigation Co, and Sambok, alleging
that:
ART. 1506. Where the seller of goods has a voidable title 1. On Oct 2, 1970, Sambok Bacolod ordered from petitioner
thereto, but his, title has not been voided at the time of the various automotive products worth P30,909.61 payable in 45
sale, the buyer acquires a good title to the goods, provided days,
he buys them in good faith, for value, and without notice of 2. That on Nov 25, 1970, Chrysler delivered said products to its
the seller's defect of title. forwarding agent Allied Brokerage for delivery to Sambok
(Allied loaded the goods through vessel of Negros Nav),
The contention is clearly unmeritorious. Under the aforequoted 3. That when Chrysler tried to collect from Sambok Bacolod
provision, it is essential that the seller should have a the amount of P31,037.56 (price of spare parts plus handling
voidable title at least. It is very clearly inapplicable where, as in charges), Sambok refused to pay, claiming that it had not
this case, the seller had no title at all. received the merchandise,
4. That Chrysler also demanded the return of the merchandise
Ownership is not transferred by contract merely but by tradition or their value from Allied and Negros Navigation, but both
or delivery. Contracts only constitute titles or rights to the denied any liability.
transfer or acquisition of ownership, while delivery or tradition
is the mode of accomplishing the same. For the legal acquisition Sambok Bacolod denied having received the automotive
and transfer of ownership and other property rights, the thing products and professed no knowledge of having ordered from
transferred must be delivered, inasmuch as, according to settled petitioner the said articles.
jurisprudence, the tradition of the thing is a necessary and
indispensable requisite in the acquisition of said ownership by Trial Court dismissed the complaint against Allied and Negros
virtue of contract. So long as property is not delivered, the Navigation, but found Sambok liable for damages “in refusing to
ownership over it is not transferred by contract merely but by take delivery of the shipment for no justifiable reason. The
delivery. Contracts only constitute titles or rights to the transfer or decision was reversed by CA after finding that Chrysler had not
acquisition of ownership, while delivery or tradition is the method performed its part of the obligation under the contract by not
of accomplishing the same, the title and the method of acquiring delivering the goods at Sambok, Iloilo, the place designated in
it being different in our law. the Parts Order Form. In other words, CA found that there was
misdelivery.
Vicente Marella did not have any title to the property under
litigation because the same was never delivered to him. He ISSUE: W/N Sambok Bacolod should be liable for damages. NO
sought ownership or acquisition of it by virtue of the contract.
Vicente Marella could have acquired ownership or title to the HELD: To our minds, the matter of misdelivery is not the decisive
subject matter thereof only by the delivery or tradition of the car factor for relieving Sambok, Bacolod, of liability herein. While it
to him. may be that the Parts Order Form specifically indicated Iloilo as
the destination, as testified to by Ernesto Ordonez, Parts Sales
The lower court was correct in applying Article 559 of the CC to Representative of petitioner, Sambok, Bacolod, and Sambok,
the case at bar, for under it, the rule is to the effect that if the Iloilo, are actually one. In fact, admittedly, the order for spare
owner has lost a thing, or if he has been unlawfully deprived
COMPILED BY: WIGMORE #WIGMOREFOREVER 111
SALES Case Digest (Atty. Sarona)
Compiled by: Wigmore #wigmoreforever

parts was made by the President of Sambok, Pepito Ng, token of goodwill it sent to Tabora free of charge volumes 75, 76,
through its marketing consultant. 77 and 78 of the Philippine Reports.

Notwithstanding, upon receipt of the Bill of Lading, Sambok, As Tabora failed to pay monthly installments, the company
Bacolod, initiated, but did not pursue, steps to take delivery as commenced the present action for the recovery of the balance of
they were advised by Negros Navigation that because some the obligation.
parts were missing. They would just be informed as soon as
the missing parts were located. It was provided in the contract that "title to and ownership of the
books shall remain with the seller until the purchase price shall
It was only four years later however, or in 1974, when a have been fully paid. Loss or damage to the books after delivery
warehouseman of Negros Navigation, Severino Aguarte, found in to the buyer shall be borne by the buyer." The total price of the
their off-shore bodega, parts of the shipment in question, but books, including the cost of freight, amounts to P1,682.40.
already deteriorated and valueless. Appellant only made a down payment of P300.00 thereby leaving
a balance of P1,382.40.
Under the circumstances, Sambok, Bacolod, cannot be faulted
for not accepting or refusing to accept the shipment from Defendant, in his answer, pleaded force majeure as a defense,
Negros Navigation four years after shipment. The evidence is saying that since the loss was due to force majeure he cannot be
clear that Negros Navigation could not produce the merchandise held responsible for the loss.
nor ascertain its whereabouts at the time Sambok, Bacolod, was
ready to take delivery. Where the seller delivers to the buyer a ISSUE: WON Tabora is absolved from liability on the ground of
quantity of goods less than he contracted to sell, the buyer may force majeure
reject them.
RULING: No. He is liable for the loss. It is true that in the
From the evidentiary record, Negros Navigation was the party contract entered into between the parties the seller agreed that
negligent in failing to deliver the complete shipment either to the ownership of the books shall remain with it until the purchase
Sambok, Bacolod, or to Sambok, Iloilo, but as the Trial Court price shall have been fully paid, but such stipulation cannot make
found, petitioner failed to comply with the conditions precedent to the seller liable in case of loss not only because such was
the filing of a judicial action. Thus, in the last analysis, it is agreed merely to secure the performance by the buyer of his
petitioner that must shoulder the resulting loss. The general obligation but in the very contract it was expressly agreed
rule that before, delivery, the risk of loss is home by the that the "loss or damage to the books after delivery to the
seller who is still the owner, under the principle of "res perit buyer shall be borne by the buyer."
domino", is applicable in petitioner's case.
Article 1504 of our Civil Code, which in part provides:
In sum, the judgment of respondent Appellate Court, will have to (1) Where delivery of the goods has been made to the buyer
be sustained not on the basis of misdelivery but on non-delivery or to a bailee for the buyer, in pursuance of the contract and
since the merchandise was never placed in the control and the ownership in the goods has been retained by the seller
possession of Sambok, Bacolod, the vendee. Decision of CA merely to secure performance by the buyer of his
affirmed. obligations under the contract, the goods are at the buyer's
risk from the time of such delivery.

LAWYER’S COOP VS TABORA The rule that an obligor should be held exempt from liability when
the loss occurs thru a fortuitous event should not apply because
FACTS: Perfecto Tabora bought from the Lawyers Cooperative it only holds true when the obligation consists in the delivery of a
Publishing Company one complete set of American determinate thing and there is no stipulation holding him liable
Jurisprudence consisting of 48 volumes with 1954 pocket parts, even in case of fortuitous event. Here these qualifications are not
plus one set of American Jurisprudence, General Index, present. The obligation does not refer to a determinate thing, but
consisting of 4 volumes, for a total price of P1,675.50 which, in is pecuniary in nature, and the obligor bound himself to assume
addition to the cost of freight of P6.90, makes a total of the loss after the delivery of the goods to him. In other words, the
P1,682.40. obligor agreed to assume any risk concerning the goods from the
time of their delivery, which is an exception to the rule provided
Tabora made a partial payment of P300.00, leaving a balance of for in Article 1262 of our Civil Code.
P1,382.40. The books were delivered and receipted by Tabora in
his law office.
PART ViI: REMEDIES IN CASE OF BREACH
In the midnight of the same date, however, a big fire broke out in
that locality which destroyed and burned all the buildings
standing on one whole block including at the law office and A. Ordinary Remedies of Seller
library of Tabora As a result, the books bought from the company
as above stated, together with Tabora's important documents
and papers, were burned during the conflagration.

This unfortunate event was immediately reported by Tabora to B. Special Remedies of Unpaid Seller of Goods
the company by sending a letter. The company replied and as a

COMPILED BY: WIGMORE #WIGMOREFOREVER 112


SALES Case Digest (Atty. Sarona)
Compiled by: Wigmore #wigmoreforever

1) Possessory Lien however, one on installments, but on straight term, in which the
balance, after payment of the initial sum, should be paid in its
totality at the time specified in the promissory note. The
transaction is not, therefore, the one contemplated in Act No.
2) Stoppage in Transitu 4122 and accordingly the mortgagee is not bound by the
prohibition therein contained as to the right to the recovery of the
unpaid balance.

3) Special Right of Resale The suggestion that the cash payment made in this case should
be considered as an installment in order to bring the contract
sued upon under the operation of the law, is completely
untenable. A cash payment cannot be considered as a payment
4) Special Right to Rescind by installment, and even if it can be so considered, still the law
does not apply, for it requires non-payment of two or more
installments in order that its provisions may be invoked. Here,
only one installment was unpaid.
C. Remedies of Buyer

DELTA MOTOR SALES CORP. v NIU KIM DUAN

FACTS: Niu Kim Duan purchased from Delta Motors 3 air


D. Sale of Movable on Installments – Article 1484 (Recto La) conditioning units. Niu paid the down payment, the balance
(also 1485, 1486) payable in 24 instalments. Title to the property remained with
Delta until the payment of the full purchase price.

LEVY VS. GERVACIO Under the agreement, failure to pay 2monthly instalments makes
the obligation entirely due and demandable. The units were
FACTS: Levy Hermanos, Inc., sold to defendant Lazaro Blas delivered, Niu failed to pay. Thus, Delta filed a complaint for
Gervacio, a Packard car. Defendant, after making the initial Replevin and applied the installments paid by Niu as rentals.
payment, executed a promissory note for the balance of P2,400,
payable on or before June 15, 1937, with interest at 12 % per Niu contends that the contractual stipulations are
annum, to secure the payment of the note, he mortgaged the car unconscionable.
to the plaintiff.
ISSUE: W/N the remedy Delta availed of was unconscionable
Defendant failed to pay the note it its maturity. Thus, Levy
foreclosed the mortgage and the car was sold at public auction, HELD: NO. A stipulation in the contract treating installments as
at which plaintiff was the highest bidder for P1,800. The present rentals in case of failure to pay is VALID — so long as they are
action is for the collection of the balance of P1,600 and interest. not unconscionable. The provision in this case is reasonable.

ISSUE: WON plaintiff still may collect the balance and interest An unpaid seller has 3 alternative (not cumulative) remedies:
after it has already foreclosed the mortgage and sold it at public (1) to exact fulfilment of the obligation;
auction (2) to cancel the sale for default in 2 installments; and
(3) to foreclose the chattel mortgage.
RULING: Yes it can still collect the balance.
Article 1454-A of the Civil Code reads as follows: If the creditor chooses one remedy, he cannot avail himself of
In a contract for the sale of personal property payable in the other two.
installments shall confer upon the vendor the right to cancel
the sale or foreclose the mortgage if one has been given on It is not disputed that the plaintiff-appellee had taken possession
the property, without reimbursement to the purchaser of the of the three air-conditioners, through a writ of replevin when
installments already paid, if there be an agreement to this defendants-appellants refused to extra-judicially surrender the
effect. same. This was done pursuant to paragraphs 5 and 7 of its Deed
of Conditional Sale when defendants-appellants failed to pay at
However, if the vendor has chosen to foreclose the least two (2) monthly installments, so much so that as of January
mortgage he shall have no further action against the 6, 1977, the total amount they owed plaintiff-appellee, inclusive
purchaser for the recovery of any unpaid balance owing by of interest, was P12,920.08. 12 The case plaintiff-appellee filed
the same and any agreement to the contrary shall be null was to seek a judicial declaration that it had validly rescinded the
and void. Deed of Conditional Sale.

In Macondray and Co. vs. De Santos, the Court held that "in Clearly, plaintiff-appellee chose the second remedy of Article
order to apply the provisions of article 1454-A of the Civil 1484 in seeking enforcement of its contract with defendants-
Code it must appear that there was a contract for the sale of appellants. This is shown from the fact that the computation of
personal property payable in installments and that there has the outstanding account of defendants-appellants as of October
been a failure to pay two or more installments." The contract, 3, 1977 took into account "the value of the units repossessed."
in the instant case, while a sale of personal property, is not, Having done so, it is barred from exacting payment from
COMPILED BY: WIGMORE #WIGMOREFOREVER 113
SALES Case Digest (Atty. Sarona)
Compiled by: Wigmore #wigmoreforever

defendants-appellants of the balance of the price of the three air- owner of the car and sentencing Elisco to pay for actual
conditioning units which it had already repossessed. damages caused to the private respondents, thus this petition.

ISSUE: WON the lease with option to buy is in reality an


TAJANLANGIT v SOUTHERN MOTORS installment sale so as to apply the Recto Law under Art. 1484.

FACTS: Tajanlangit bought 2 tractors and a thresher from WON Elisco is entitled to any of the remedies under Art. 1484.
Southern Motors. They executed a promissory note in payment
thereof; it contained an acceleration clause. Tajanlang it failed to HELD:
pay any of the stipulated installments. Thus, Southern Motors 1. Yes.
sued him on the PN. The sheriff levied upon the properties of
Tajanlangit (same machineries) and sold them at a public auction The agreement between Elisco and the Lantans is in reality an
to satisfy the debt. Southern Motors now prayed for execution. installment sale of personal property. However, the remedies
Tajanlangit sought to annul the writ of execution — claiming that under Article 1484 are alternative, not cumulative.
since Southern Motors repossessed the machineries
(mortgaged), he was therefore relieved from liability on the 2. No.
balance of the purchase price.
There was already full payment. In the case at bar, although the
ISSUE: W/N Tajanlangit is relieved from his obligation topay agreement provides for the payment of monthly rentals, it also
provides the option to purchase upon the payment of the 60th
HELD: NO. While it is true that the foreclosure on the chattel monthly rental and that all monthly rentals shall be applied to the
mortgage on the thing sold bars further action for the recovery of payment of the full purchase price of the car. Clearly the
the balance of the purchase price, this does not apply in this transaction is a lease in name only and so Articles 1484 and
case since Southern did not foreclose on the mortgage but 1485 apply.
instead sued based on the PNs exclusively. That being the case,
it is not limited to the proceeds of the sale on execution of the It is noteworthy that the remedies provided for in Art. 1484 are
mortgaged goods and may claim the balance from Tajanlangit. alternative, not cumulative. The exercise of one bars the exercise
of the others. It was held that in choosing to deprive the
defendant of possession of the leased vehicles, the plaintiff
ELISCO TOOL MANUFACTURING vs. COURT OF APPEALS, waived its right to bring an action to recover unpaid rentals on
ROLANDO AND RINA LANTAN the said vehicles.
G.R. No. 109966
May 31, 1999 Furthermore, both the trial court and the CA correctly ruled that
Elisco is not entitled to any of the remedies under Art. 1484 as
FACTS: Private respondent Rolando Lantan was employed at there has already been full payment.
the Elisco Tool Mfg. Corp. On Jan. 1980, he entered into a car
plan with the company, which constitutes a lease with option to
The agreement does not provide for the payment of interest on
buy for a period of 5 years. The agreement provides that Lantan
unpaid monthly "rentals" or installments. The 2% surcharge is not
shall pay a monthly rental of P 1010.65 to be deducted from his
provided for in the agreement. Consequently, the total amount of
salary or a total of P60, 639.00 at the end of 5 years. The
P 61, 070.94 already paid is more than sufficient to cover the full
agreement provides that at the 60th month of payment he may
purchase price of the car which only amounts to P 60, 639.
exercise his option to buy and all monthly rentals shall be applied
to the payment of the full purchase price of the car. In 1981
Elisco Tool ceased operations, and Rolando Lantan was laid off.
Nonetheless, as of December 4, 1984, private respondent was E. Remedies in case of Immovables
able to make payments for the car in the total amount of P61,
070.94.

On 1986 Elisco filed a complaint for replevin plus sum of money F. Sale of Real Estate on Installments – RA 6552 (Maceda
against Rolando Lantan for the latter‘s alleged failure to pay the Law)
monthly rentals as of May 1986. Elisco prayed for the following:
ABELARDO VALARAO, GLORIOSA VALARAO, CARLOS
1. The payment of Lantan of the sum of the monthly rentals due VALARAO vs. COURT OF APPEALS AND MEDEN
as of May 1986 plus legal interest; ARELLANO
G.R. No. 130347
2. The issuance of writ of replevin to gain possession of the car; March 3, 1999
and
FACTS: Spouses Valarao, thru their son, Carlos, sold to Arellano
3. On the alternative, should the delivery of the car not be a parcel of land situated in Diliman, Quezon City for the sum of
possible, that Lantan be ordered to pay the actual value of the 3.225 M embodied under a Deed of Conditional Sale.
car in the amount of 60,000 plus the accrued monthly rentals
thereof with interest until fully paid. 
 Both the trial court and the It was further stipulated upon that should Arellano fail to pay
CA decided in favor of Lantan, declaring the latter the lawful three (3) successive monthly installments or any one year-end
COMPILED BY: WIGMORE #WIGMOREFOREVER 114
SALES Case Digest (Atty. Sarona)
Compiled by: Wigmore #wigmoreforever

lump sum payment within the period stipulated, the sale shall be spouses accepted the payment, she would have paid all three
considered automatically rescinded without the necessity of monthly installments. In other words, there was no deliberate
judicial action and all payments made by Arellano shall be failure on Arellano’s part to meet her responsibility to pay.
forfeited in favor of the spouses by way of rental for the use and
occupancy of the property and as liquidated damages. All 2. Yes. Sec. 3, RA 6552 provides:
improvements introduced by Arellano to the property shall belong
to the spouses without any right of reimbursement. Sec. 3. In all transactions or contracts involving the sale or
financing of real estate on installment payments, including
Arellano alleged that as of September 1990 he was already able residential condominium apartments but excluding
to pay the sum of 2.028 M although she admitted that she failed industrial lots, commercial buildings and sales to tenants
to pay for the installments due in October and November 1990. under Republic Act. Numbered Thirty-eight hundred Forty-four as
Arellano tried to pay but was turned down by the spouses thru amended by Republic Act Numbered Sixty-three hundred eighty-
their maid. Arellano avers that the same maid was the on who nine, where the buyer has paid at least two years of
received payments tendered by her. It appears that the maid installments, the buyer is entitled to the following rights in case
refused to receive the payment allegedly on orders of her he defaults in the payment of succeeding installments:
employees who were not at home. This prompted Arellano to
seek the help of barangay officials. Efforts to settle before the (a) To pay, without additional interest, the unpaid
barangay was unavailing, as the spouses never appeared in installments due within the total grace period earned by him,
meetings. which is hereby fixed at the rate of one month grace period
for every year of installment payments made: Provided, That
Arellano sought judicial action by filing a petition for consignation this right shall be exercised by the buyer only once in every five
on January 4, 1991. years of the life of the contract and its extensions, if any.

Spouses Valarao, thru counsel, sent Arellano a letter dated 4 (b) If the contract is cancelled, the seller shall refund to the
January 1991 notifying her that they were enforcing the provision buyer the cash surrender value on the payments on the
on automatic rescission as a consequence of which the Deed of property equivalent to fifty percent of the total payments
Conditional Sale was deemed null and void, and xxx all made and, after five years of installments, an additional five
payments made, as well as the improvements introduced on the percent every year but not to exceed ninety percent of the
property, were thereby forfeited. The letter also made a formal total payments made: Provided, That the actual cancellation of
demand on Arellano to vacate the property should she not heed the contract shall take place after thirty days from receipt by the
the demand of the spouses to sign a contract of lease for her buyer of the notice of cancellation or the demand for rescission
continued stay in the property. of the contract by a notarial act and upon full payment of the
cash surrender value to the buyer.
The RTC ruled against Arellano but the Court of Appeals
reversed the decision of the trial court hence this petition. Down payments, deposits or options on the contract shall be
included in the computation of the total number of installments
ISSUE: made.
1. WON the automatic forfeiture clause is enforceable.
2. WON RA 6552 is applicable. Therefore, Arellano is entitled to a one-month grace period for
every year of installment paid, which means that she had a total
grace period of three months from December 31, 1990. Indeed,
HELD: to rule in favor of the spouses would result in patent injustice and
1. Yes. As a general rule, a contract is the law between the unjust enrichment.
parties. Thus, "from the moment the contract is perfected, the
parties are bound not only to the fulfillment of what has been
expressly stipulated but also to all consequences which, PAGTALUNAN vs. VDA. DE MANZANO
according to their nature, may be in keeping with good faith, G.R. No. 147695 September 13, 2007
usage and law." Also, "the stipulations of the contract being the
law between the parties, courts have no alternative but to FACTS: On July 19, 1974, Patricio Pagtalunan (Patricio),
enforce them as they were agreed [upon] and written, there petitioner’s stepfather and predecessor-in-interest, entered into a
being no law or public policy against the stipulated forfeiture of Contract to Sell with respondent Rufina dela Cruz Vda. De
payments already made." However, it must be shown that Manzano, whereby the former agreed to sell, and the latter to
Arellano failed to perform her obligation, thereby giving spouses buy, a house and lot which formed half of a parcel of land for a
the right to demand the enforcement of the contract. consideration of P17,800. The parties agreed that it shall be paid
in the following manner: P1,500 as downpayment upon
We concede the validity of the automatic forfeiture clause, which execution of the Contract to Sell, and the balance to be paid in
deems any previous payments forfeited and the contract equal monthly installments of P150 on or before the last day of
automatically rescinded upon the failure of the vendee to pay each month until fully paid.
three successive monthly installments or any one-yearend lump
sum payment. However, the spouses failed to prove the It was also stipulated in the contract that respondent could
conditions that would warrant the implementation of this clause. immediately occupy the house and lot; that in case of default in
the payment of any of the installments for 90 days after its due
Based on the facts of the case, the spouses were not justified in date, the contract would be automatically rescinded without need
refusing the tender of payment made by Arellano. Had the of judicial declaration, and that all payments made and all
COMPILED BY: WIGMORE #WIGMOREFOREVER 115
SALES Case Digest (Atty. Sarona)
Compiled by: Wigmore #wigmoreforever

improvements done on the premises by respondent would be Petitioner contends that that his demand letter dated February
considered as rentals for the use and occupation of the property 24, 1997 should be considered as the notice of cancellation or
or payment for damages suffered, and respondent was obliged demand for rescission by notarial act.
to peacefully vacate the premises and deliver the possession
thereof to the vendor. The Court, however, finds that the letter dated February 24,
1997, which was written by petitioner’s counsel, merely made
Petitioner claimed that respondent paid only P12,950. She formal demand upon respondent to vacate the premises in
allegedly stopped paying after December 1979 without any question.
justification or explanation. Petitioner asserted that when
respondent ceased paying her installments, her status of buyer Clearly, the demand letter is not the same as the notice of
was automatically transformed to that of a lessee. Therefore, she cancellation or demand for rescission by a notarial act required
continued to possess the property by mere tolerance of Patricio by R.A No. 6552. Petitioner cannot rely on Layug v. Intermediate
and, subsequently, of petitioner. Appellate Court to support his contention that the demand letter
was sufficient compliance since the seller therein filed an action
Respondent did not deny that she still owed Patricio P5,650, but for annulment of contract, which is a kindred concept of
claimed that she did not resume paying her monthly installment rescission by notarial act. Evidently, the case of unlawful detainer
because of the unlawful acts committed by Patricio, as well as filed by petitioner does not exempt him from complying with the
the filing of the ejectment case against her. said requirement.

Patricio and his wife died on September 17, 1992 and on In addition, Sec. 3 (b) of R.A. No. 6552 requires refund of the
October 17, 1994, respectively. Petitioner became their sole cash surrender value of the payments on the property to the
successor-in-interest pursuant to a waiver by the other heirs. On buyer before cancellation of the contract. The provision does not
March 5, 1997, respondent received a letter from petitioner’s provide a different requirement for contracts to sell which allow
counsel dated February 24, 1997 demanding that she vacate the possession of the property by the buyer upon execution of the
premises within five days on the ground that her possession had contract like the instant case. Hence, petitioner cannot insist on
become unlawful. Respondent ignored the demand. The Punong compliance with the requirement by assuming that the cash
Barangayfailed to settle the dispute amicably. surrender value payable to the buyer had been applied to rentals
of the property after respondent failed to pay the installments
On April 8, 1997, petitioner filed a Complaint for unlawful due.
detainer against respondent.
There being no valid cancellation of the Contract to Sell, the CA
ISSUE: Whether or not the cancellation of the contract complied correctly recognized respondent’s right to continue occupying the
with what is required under the Maceda law. property subject of the Contract to Sell and affirmed the
dismissal of the unlawful detainer case by the RTC.
HELD: No. R.A. No. 6552, otherwise known as the "Realty
Installment Buyer Protection Act," recognizes in conditional sales SC DISPOSITION: Considering that the Contract to Sell was not
of all kinds of real estate (industrial, commercial, residential) the cancelled by the vendor, the Court agrees with the CA that it is
right of the seller to cancel the contract upon non-payment of an only right and just to allow respondent to pay her arrears and
installment by the buyer, which is simply an event that prevents settle the balance of the purchase price.
the obligation of the vendor to convey title from acquiring binding
force. The Court agrees with petitioner that the cancellation of the
Contract to Sell may be done outside the court particularly when
the buyer agrees to such cancellation.
SPOUSES GARCIA, SPOUSES vs. COURT OF APPEALS
However, the cancellation of the contract by the seller must be in G.R. No. 172036 April 23, 2010
accordance with Sec. 3 (b) of R.A. No. 6552, which requires a
notarial act of rescission and the refund to the buyer of the full FACTS: On May 28, 1993, plaintiffs spouses Faustino and
payment of the cash surrender value of the payments on the Josefina Garcia and spouses Meliton and Helen Galvez (herein
property. Actual cancellation of the contract takes place after 30 appellees) and defendant Emerlita dela Cruz (herein appellant)
days from receipt by the buyer of the notice of cancellation or the entered into a Contract to Sell wherein the latter agreed to sell to
demand for rescission of the contract by a notarial act and upon the former, for P3,170,220.00, five (5) parcels of land. At the time
full payment of the cash surrender value to the buyer. of the execution of the said contract, three of the subject lots,
were registered in the name of one Angel Abelida from whom
Based on the records of the case, the Contract to Sell was not defendant allegedly acquired said properties by virtue of a Deed
validly cancelled or rescinded under Sec. 3 (b) of R.A. No. 6552. of Absolute Sale dated March 31, 1989.

First, Patricio, the vendor in the Contract to Sell, died on As agreed upon, plaintiffs shall make a down payment of
September 17, 1992 without canceling the Contract to Sell. P500,000.00 upon signing of the contract. The balance of
P2,670,220.00 shall be paid in three installments.
Second, petitioner also failed to cancel the Contract to Sell in
accordance with law. On its due date, December 31, 1993, plaintiffs failed to pay the
last installment in the amount of One P1,670,220.00. Sometime
in July 1995, plaintiffs offered to pay the unpaid balance, which
had already been delayed by one and a half year, which
COMPILED BY: WIGMORE #WIGMOREFOREVER 116
SALES Case Digest (Atty. Sarona)
Compiled by: Wigmore #wigmoreforever

defendant refused to accept. On September 23, 1995, defendant either case. He may also seek rescission, even after he has
sold the same parcels of land to intervenor Diogenes G. chosen fulfillment, if the latter should become impossible.
Bartolome for P7,793,000.00.
The Court shall decree the rescission claimed, unless there be
Plaintiffs filed before the RTC a complaint for specific just cause authorizing the fixing of a period.
performance to compel defendant to accept plaintiffs’ payment
and, thereafter, execute the necessary document of transfer.
G.R. NO. 195619, SEPTEMBER 5, 2012
In their complaint, plaintiffs alleged that they discovered the PLANTERS DEVELOPMENT BANK, VS JULIE CHANDUMAL
infirmity of the Deed of Absolute Sale covering those 3 lots,
between their former owner Angel Abelida and defendant, the FACTS: BF Homes and Julie Chandumal entered into a contract
same being spurious because the signature of Angel Abelida and to sell a parcel of land located in Las Pinas. Later, BF Homes
his wife were falsified. Due to their apprehension regarding the sold to PDB all its rights over the contract.
authenticity of the document, they withheld payment of the last
installment. They tendered payment of the unpaid balance Chandumal paid her monthly amortizations until she defaulted in
sometime in July 1995, after Angel Abelida ratified the sale made her payments. So, PDB sent a notice to Chandumal with a
in favor of defendant, but defendant refused to accept their demand to vacate the land within 30days, otherwise all of her
payment for no jusitifiable reason. rights will be extinguished and the contract will be terminated and
deemed rescinded. In spite of the demand, Chandumal failed to
In her answer, defendant denied the allegation that the Deed of settle her account.
Absolute Sale was spurious and argued that plaintiffs failed to
pay in full the agreed purchase price on its due date despite PDB filed an action for judicial confirmation of notarial rescission
repeated demands; that the Contract to Sell contains a proviso and delivery of possession but still Chandumal refused to do so.
that failure of plaintiffs to pay the purchase price in full shall Summons were then issued and served by deputy sheriff Galing
cause the rescission of the contract and forfeiture of 1/2 of the but its was unavailing as she was always out of her house on the
total amount paid to defendant; that a notarized letter stating the dates the summons were served.
indended rescission of the contract to sell and forfeiture of
payments was sent to plaintiffs at their last known address but it RTC then issued an order granting the motion of PDB.
was returned with a notation "insufficient address." Chandumal filed an urgent motion to set aside order of default
and to admit attached answer. Chandumal said that she did not
ISSUE: Whether or not Maceda Law is applicable in this case. receive the summons and was not notified of the same and her
failure to file an answer within the reglementary period was due
HELD: No. Not applicable. It is clear from the above-quoted to fraud. RTC denied Chandumal’s motion to set aside the order
provisions that the parties intended their agreement to be a of default.
Contract to Sell: Dela Cruz retains ownership of the subject lands
and does not have the obligation to execute a Deed of Absolute Chandumal appealed to the CA. CA nullified the RTC’s decision.
Sale until petitioners’ payment of the full purchase price.
ISSUE:
The Maceda Law applies to contracts of sale of real estate on (1) Whether there was valid substituted service of summons?
installment payments, including residential condominium (2) Whether Chandumal voluntarily submitted to the jurisdiction
apartments but excluding industrial lots, commercial buildings of the RTC?
and sales to tenants. The subject lands, comprising five (5) (3) Whether there was proper rescission by notarial act of the
parcels and aggregating 69,028 square meters, do not comprise contract to sell?
residential real estate within the contemplation of the Maceda
Law. Moreover, even if we apply the Maceda Law to the present HELD:
case, petitioners’ offer of payment to Dela Cruz was made a year (1) Correctly ruled that the sheriff’s return failed to justify a resort
and a half after the stipulated date. This is beyond the sixty-day to substituted service of summons. According to the CA, the
grace period under Section 4 of the Maceda Law. Petitioners still Return of Summons does not specifically show or indicate in
cannot use the second sentence of Section 4 of the Maceda Law detail the actual exertion of efforts or any positive step taken by
against Dela Cruz for Dela Cruz’s alleged failure to give an the officer or process server in attempting to serve the summons
effective notice of cancellation or demand for rescission because personally to the defendant.
Dela Cruz merely sent the notice to the address supplied by
petitioners in the Contract to Sell. (2) The Court notes that aside from the allegation that she did
not receive any summons, Chandumal’s motion to set aside
The applicable provision of law in instant case is Article 1191 of order of default and to admit attached answer failed to positively
the New Civil Code which provides as follows: assert the trial court lack of jurisdiction. In fact, what was set forth
therein was the substantial claim that PDB failed to comply with
Art. 1191. The power to rescind obligations is implied in the requirements of R.A. No. 6552 on payment of cash surrender
reciprocal ones, in case one of the obligors should not comply value, which already delves into the merits of PDB’s cause of
with what is incumbent upon him. action. In addition, Chandumal even appealed the RTC decision
to the CA, an act which demonstrates her recognition of the trial
The injured party may choose between the fulfillment and the court’s jurisdiction to render said judgment.
rescission of the obligation, with the payment of damages in

COMPILED BY: WIGMORE #WIGMOREFOREVER 117


SALES Case Digest (Atty. Sarona)
Compiled by: Wigmore #wigmoreforever

(3) R.A. No. 6552 recognizes the right of the seller to cancel the payment of the purchase price in a contract to sell is a
contract but any such cancellation must be done in conformity suspensive condition, the non-fulfillment of which prevents the
with the requirements therein prescribed. In addition to the prospective seller’s obligation to convey title from becoming
notarial act of rescission, the seller is required to refund to the effective, as in this case.
buyer the cash surrender value of the payments on the property.
The actual cancellation of the contract can only be deemed to Further, it is significant to note that given that the Contract to Sell
take place upon the expiry of a thirty (30)-day period following in this case is one which has for its object real property to be sold
the receipt by the buyer of the notice of cancellation or demand on an installment basis, the said contract is especially governed
for rescission by a notarial act and the full payment of the cash by — and thus, must be examined under the provisions of — RA
surrender value. 6552, or the “Realty Installment Buyer Protection Act”, which
provides for the rights of the buyer in case of his default in the
Petition is denied. payment of succeeding instalments.

Given the nature of the contract of the parties, the respondent


OPTIMUM DEVELOPMENT BANK vs. SPOUSES BENIGNO V. court correctly applied Republic Act No. 6552. Known as the
JOVELLANOS and LOURDES R. JOVELLANOS Maceda Law, R.A. No. 6552 recognizes in conditional sales of all
G.R. No. 189145 December 4, 2013 kinds of real estate (industrial, commercial, residential) the right
of the seller to cancel the contract upon non-payment of an
FACTS: On April 26, 2005, Sps. Jovellanos entered into a installment by the buyer, which is simply an event that prevents
6
Contract to Sell with Palmera Homes, Inc. (Palmera Homes) for the obligation of the vendor to convey title from acquiring binding
the purchase of a residential house and lot situated in Block 3, force. It also provides the right of the buyer on installments in
Lot 14, Villa Alegria Subdivision, Caloocan City (subject property) case he defaults in the payment of succeeding installments, viz.:
for a total consideration of P1,015,000.00. Pursuant to the
contract, Sps. Jovellanos took possession of the subject property (1) Where he has paid at least two years of installments,
upon a down payment of P91,500.00, undertaking to pay the (a) To pay, without additional interest, the unpaid installments
remaining balance of the contract price in equal monthly due within the total grace period earned by him, which is hereby
installments of P13,107.00 for a period of 10 years starting June fixed at the rate of one month grace period for every one year of
12, 2005. installment payments made:

On August 22, 2006, Palmera Homes assigned all its rights, title Provided, That this right shall be exercised by the buyer only
and interest in the Contract to Sell in favor of petitioner Optimum once in every five years of the life of the contract and its
Development Bank (Optimum) through a Deed of Assignment of extensions, if any. (b) If the contract is cancelled, the seller shall
even date. refund to the buyer the cash surrender value of the payments on
the property equivalent to fifty per cent of the total payments
On April 10, 2006, Optimum issued a Notice of Delinquency and made and, after five years of installments, an additional five per
Cancellation of Contract to Sell for Sps. Jovellanos’s failure to cent every year but not to exceed ninety per cent of the total
pay their monthly installments despite several written and verbal payments made:
notices.
Provided, That the actual cancellation of the contract shall take
In a final Demand Letter dated May 25, 2006, Optimum required place after cancellation or the demand for rescission of the
Sps. Jovellanos to vacate and deliver possession of the subject contract by a notarial act and upon full payment of the cash
property within seven (7) days which, however, remained surrender value to the buyer.
unheeded. Hence, Optimum filed, on November 3, 2006, a
complaint for unlawful detainer before the MeTC, docketed as Down payments, deposits or options on the contract shall be
Civil Case No. 06-28830. Despite having been served with included in the computation of the total number of installments
summons, together with a copy of the complaint, Sps. Jovellanos made.
failed to file their answer within the prescribed reglementary
period, thus prompting Optimum to move for the rendition of (2) Where he has paid less than two years in installments, Sec.
judgment. 4. x x x the seller shall give the buyer a grace period of not less
than sixty days from the date the installment became due. If the
Thereafter, Sps. Jovellanos filed their opposition with motion to buyer fails to pay the installments due at the expiration of the
admit answer, questioning the jurisdiction of the court, among grace period, the seller may cancel the contract after thirty days
others. Further, they filed a Motion to Reopen and Set the Case from receipt by the buyer of the notice of cancellation or the
for Preliminary Conference, which the MeTC denied. demand for rescission of the contract by a notarial act.
(Emphasis and underscoring supplied)
ISSUE: w/n there was a valid and effective cancellation of the
Contract to Sell in accordance with Section 4 of RA 6552 Pertinently, since Sps. Jovellanos failed to pay their stipulated
monthly installments as found by the MeTC, the Court examines
RULING: YES. Verily, in a contract to sell, the prospective seller Optimum’s compliance with Section 4 of RA 6552, as above-
binds himself to sell the property subject of the agreement quoted and highlighted, which is the provision applicable to
exclusively to the prospective buyer upon fulfillment of the buyers who have paid less than two (2) years-worth of
condition agreed upon which is the full payment of the purchase installments. Essentially, the said provision provides for three (3)
price but reserving to himself the ownership of the subject requisites before the seller may actually cancel the subject
property despite delivery thereof to the prospective buyer.The full contract: first, the seller shall give the buyer a 60-day grace
COMPILED BY: WIGMORE #WIGMOREFOREVER 118
SALES Case Digest (Atty. Sarona)
Compiled by: Wigmore #wigmoreforever

period to be reckoned from the date the installment became already sold his share to Rodolfo. Rodolfo admitted having
due; second, the seller must give the buyer a notice of remitted only P250,000.00 to Nicolas. He asserted that he would
cancellation/demand for rescission by notarial act if the pay the balance of the purchase price to Nicolas only after the
buyer fails to pay the installments due at the expiration of the latter shall have executed a deed of absolute sale.
said grace period; and third, the seller may actually cancel the
contract only after thirty (30) days from the buyer’s receipt of the RTC:
said notice of cancellation/demand for rescission by notarial act. After trial on the merits, or on April 19, 2005, the trial court
rendered its Decision dismissing Civil Case No. 99-02971-D for
In the present case, the 60-day grace period automatically lack of merit and ordering Nicolas to execute a deed of absolute
operated in favor of the buyers, Sps. Jovellanos, and took effect sale in favor of Rodolfo upon payment by the latter of
from the time that the maturity dates of the installment payments the P250,000.00 balance of the agreed purchase price.
lapsed. With the said grace period having expired bereft of any
installment payment on the part of Sps. Jovellanos, Optimum CA:
then issued a notarized Notice of Delinquency and Cancellation Nicolas appealed to the CA which sustained the trial court’s
of Contract on April 10, 2006. Finally, in proceeding with the Decision in toto. The CA held that since there was a perfected
actual cancellation of the contract to sell, Optimum gave Sps. contract of sale between Nicolas and Rodolfo, the latter may
Jovellanos an additional thirty (30) days within which to settle compel the former to execute the proper sale document.
their arrears and reinstate the contract, or sell or assign their
rights to another. Besides, Nicolas’s insistence that he has since rescinded their
agreement in 1997 proved the existence of a perfected sale. It
It was only after the expiration of the thirty day (30) period did added that Nicolas could not validly rescind the contract
Optimum treat the contract to sell as effectively cancelled – because: "1) Rodolfo ha[d] already made a partial payment; 2)
making as it did a final demand upon Sps. Jovellanos to vacate Nicolas ha[d] already partially performed his part regarding the
the subject property only on May 25, 2006. Thus, based on the contract; and 3) Rodolfo opposes the rescission.
foregoing, the Court finds that there was a valid and effective
cancellation of the Contract to Sell in accordance with Section 4 The CA then proceeded to rule that since no period was
of RA 6552 and since Sps. Jovellanos had already lost their right stipulated within which Rodolfo shall deliver the balance of the
to retain possession of the subject property as a consequence of purchase price, it was incumbent upon Nicolas to have filed a
such cancellation, their refusal to vacate and turn over civil case to fix the same. But because he failed to do so, Rodolfo
possession to Optimum makes out a valid case for unlawful cannot be considered to be in delay or default.
detainer as properly adjudged by the MeTC.
Finally, the CA made another interesting pronouncement, that by
virtue of the agreement Nicolas entered into with Rodolfo, he had
DIEGO VS. DIEGO already transferred his ownership over the subject property and
as a consequence, Rodolfo is legally entitled to collect the fruits
FACTS: In 1993, petitioner Nicolas P. Diego (Nicolas) and his thereof in the form of rentals. Nicolas’ remaining right is to
brother Rodolfo, respondent herein, entered into an oral contract demand payment of the balance of the purchase price, provided
to sell covering Nicolas’s share, fixed at P500,000.00, as co- that he first executes a deed of absolute sale in favor of Rodolfo.
owner of the family’s Diego Building situated in Dagupan City.
Rodolfo made a downpayment of P250,000.00. It was agreed ISSUE: WON there is a perfected contract of sale between a
that the deed of sale shall be executed upon payment of the petitioner and respondent over Nicolas’ share of the building.
remaining balance of P250,000.00. However, Rodolfo failed to
pay the remaining balance. HELD: The contract entered into by Nicolas and Rodolfo
Meanwhile, the building was leased out to third parties, but was a contract to sell.
Nicolas’s share in the rents were not remitted to him by herein
respondent Eduardo, another brother of Nicolas and designated a) The stipulation to execute a deed of sale upon full
administrator of the Diego Building. Instead, Eduardo gave payment of the purchase price is a unique and
Nicolas’s monthly share in the rents to Rodolfo. Despite distinguishing characteristic of a contract to sell. It also
demands and protestations by Nicolas, Rodolfo and Eduardo shows that the vendor reserved title to the property until full
failed to render an accounting and remit his share in the rents payment.
and fruits of the building, and Eduardo continued to hand them
over to Rodolfo. There is no dispute that in 1993, Rodolfo agreed to buy Nicolas’s
share in the Diego Building for the price ofP500,000.00. There is
Thus, on May 17, 1999, Nicolas filed a Complaint against also no dispute that of the total purchase price, Rodolfo paid, and
Rodolfo and Eduardo before the RTC of Dagupan CitY. Nicolas Nicolas received,P250,000.00. Significantly, it is also not
prayed that Eduardo be ordered to render an accounting of all disputed that the parties agreed that the remaining amount
the transactions over the Diego Building; that Eduardo and of P250,000.00 would be paid after Nicolas shall have executed
Rodolfo be ordered to deliver to Nicolas his share in the rents; a deed of sale.
and that Eduardo and Rodolfo be held solidarily liable for
attorney’s fees and litigation expenses. This stipulation, i.e., to execute a deed of absolute sale upon full
payment of the purchase price, is a unique and distinguishing
Rodolfo and Eduardo filed their Answer with Counterclaim for characteristic of a contract to sell. In Reyes v. Tuparan, this
damages and attorney’s fees. They argued that Nicolas had no Court ruled that a stipulation in the contract, "[w]here the
more claim in the rents in the Diego Building since he had vendor promises to execute a deed of absolute sale upon
COMPILED BY: WIGMORE #WIGMOREFOREVER 119
SALES Case Digest (Atty. Sarona)
Compiled by: Wigmore #wigmoreforever

the completion by the vendee of the payment of the out of the total consideration ofP500,000.00, the amount
price," indicates that the parties entered into a contract to sell. of P250,000.00 had already been paid while the
According to this Court, this particular provision is tantamount to remaining P250,000.00 would be paid after the execution of the
a reservation of ownership on the part of the vendor. Explicitly Deed of Sale, he never testified that there was a stipulation as
stated, the Court ruled that the agreement to execute a deed of regards delivery of title or possession.
sale upon full payment of the purchase price"shows that the
vendors reserved title to the subject property until full It is also quite understandable why Nicolas belatedly demanded
payment of the purchase price." the payment of the rentals. Records show that the structural
integrity of the Diego Building was severely compromised when
In the instant case, records show that Nicolas signed a mere an earthquake struck Dagupan City in 1990. In order to
receipt acknowledging partial payment ofP250,000.00 from rehabilitate the building, the co-owners obtained a loan from a
Rodolfo. It states: bank. Starting May 1994, the property was leased to third parties
and the rentals received were used to pay off the loan. It was
July 8, 1993 only in 1996, or after payment of the loan that the co-owners
Received the amount of [P250,000.00] for 1 share of Diego started receiving their share in the rentals. During this time,
Building as partial payment for Nicolas Diego. Nicolas was in the USA but immediately upon his return, he
(signed) demanded for the payment of his share in the rentals which
Nicolas Diego Eduardo remitted to Rodolfo. Failing which, he filed the instant
Complaint. To us, this bolsters our findings that Nicolas did not
The parties’ agreement was likewise embodied only in a receipt. intend to immediately transfer title over the property.
Also, Nicolas did not want to sign the deed of sale unless he is
fully paid. On the other hand, Rodolfo did not want to pay unless It must be stressed that it is anathema in a contract to sell that
a deed of sale is duly executed in his favor. We thus say, the prospective seller should deliver title to the property to the
pursuant to our ruling in Chua v. Court of Appeals that the prospective buyer pending the latter’s payment of the price in full.
agreement between Nicolas and Rodolfo is a contract to sell. It certainly is absurd to assume that in the absence of stipulation,
a buyer under a contract to sell is granted ownership of the
This Court cannot subscribe to the appellate court’s view that property even when he has not paid the seller in full. If this were
Nicolas should first execute a deed of absolute sale in favor of the case, then prospective sellers in a contract to sell would in all
Rodolfo, before the latter can be compelled to pay the balance of likelihood not be paid the balance of the price.
the price. This is patently ridiculous, and goes against every rule
in the book. This pronouncement virtually places the prospective This ponente has had occasion to rule that "[a] contract to sell is
seller in a contract to sell at the mercy of the prospective buyer, one where the prospective seller reserves the transfer of title to
and sustaining this point of view would place all contracts to sell the prospective buyer until the happening of an event, such as
in jeopardy of being rendered ineffective by the act of the full payment of the purchase price. What the seller obliges
prospective buyers, who naturally would demand that the deeds himself to do is to sell the subject property only when the entire
of absolute sale be first executed before they pay the balance of amount of the purchase price has already been delivered to him.
the price. Surely, no prospective seller would accommodate. ‘In other words, the full payment of the purchase price partakes
of a suspensive condition, the nonfulfillment of which prevents
In fine, "the need to execute a deed of absolute sale upon the obligation to sell from arising and thus, ownership is retained
completion of payment of the price generally indicates that by the prospective seller without further remedies by the
it is a contract to sell, as it implies the reservation of title in prospective buyer.’ It does not, by itself, transfer ownership to the
the vendor until the vendee has completed the payment of buyer.”
the price." In addition, "[a] stipulation reserving ownership in the
vendor until full payment of the price is x x x typical in a contract The contract to sell is terminated or cancelled.
to sell." Thus, contrary to the pronouncements of the trial and
appellate courts, the parties to this case only entered into a Having established that the transaction was a contract to sell,
contract to sell; as such title cannot legally pass to Rodolfo until what happens now to the parties’ agreement?
he makes full payment of the agreed purchase price.
The remedy of rescission is not available in contracts to sell. As
Nicolas did not surrender or deliver title or possession to explained in Spouses Santos v. Court of Appeals:
Rodolfo.
In view of our finding in the present case that the agreement
Moreover, there could not even be a surrender or delivery of title between the parties is a contract to sell, it follows that the
or possession to the prospective buyer Rodolfo. This was made appellate court erred when it decreed that a judicial rescission of
clear by the nature of the agreement, by Nicolas’s repeated said agreement was necessary. This is because there was no
demands for the return of all rents unlawfully and unjustly rescission to speak of in the first place. As we earlier pointed out,
remitted to Rodolfo by Eduardo, and by Rodolfo and Eduardo’s in a contract to sell, title remains with the vendor and does not
repeated demands for Nicolas to execute a deed of sale which, pass on to the vendee until the purchase price is paid in full.
as we said before, is a recognition on their part that ownership
over the subject property still remains with Nicolas. Thus, in a contract to sell, the payment of the purchase price is a
positive suspensive condition. Failure to pay the price agreed
Significantly, when Eduardo testified, he claimed to be upon is not a mere breach, casual or serious, but a situation that
knowledgeable about the terms and conditions of the transaction prevents the obligation of the vendor to convey title from
between Nicolas and Rodolfo. However, aside from stating that acquiring an obligatory force. This is entirely different from the
COMPILED BY: WIGMORE #WIGMOREFOREVER 120
SALES Case Digest (Atty. Sarona)
Compiled by: Wigmore #wigmoreforever

situation in a contract of sale, where non-payment of the price is for they certainly had free rein over Nicolas’s interest in the
a negative resolutory condition. The effects in law are not Diego Building. Rodolfo put off payment of the balance of the
identical. In a contract of sale, the vendor has lost ownership of price, yet, with the aid of Eduardo, collected and appropriated for
the thing sold and cannot recover it, unless the contract of sale is himself the rents which belonged to Nicolas.
rescinded and set aside. In a contract to sell, however, the
vendor remains the owner for as long as the vendee has not Eduardo is solidarily liable with Rodolfo as regards the
complied fully with the condition of paying the purchase price. If share of Nicolas in the rents.
the vendor should eject the vendee for failure to meet the
condition precedent, he is enforcing the contract and not For his complicity, bad faith and abuse of authority as the Diego
rescinding it. When the petitioners in the instant case Building administrator, Eduardo must be held solidarily liable with
repossessed the disputed house and lot for failure of private Rodolfo for all that Nicolas should be entitled to from 1993 up to
respondents to pay the purchase price in full, they were merely the present, or in respect of actual damages suffered in relation
enforcing the contract and not rescinding it. As petitioners to his interest in the Diego Building. Eduardo was the primary
correctly point out, the Court of Appeals erred when it ruled that cause of Nicolas’s loss, being directly responsible for making and
petitioners should have judicially rescinded the contract pursuant causing the wrongful payments to Rodolfo, who received them
to Articles 1592 and 1191 of the Civil Code. Article 1592 speaks under obligation to return them to Nicolas, the true recipient.
of non-payment of the purchase price as a resolutory condition. It
does not apply to a contract to sell. As to Article 1191, it is As such, Eduardo should be principally responsible to Nicolas as
subordinated to the provisions of Article 1592 when applied to well. Suffice it to state that every person must, in the exercise of
sales of immovable property. Neither provision is applicable in his rights and in the performance of his duties, act with justice,
the present case. give everyone his due, and observe honesty and good faith; and
every person who, contrary to law, wilfully or negligently causes
Similarly, we held in Chua v. Court of Appeals that "Article 1592 damage to another, shall indemnify the latter for the same.
of the Civil Code permits the buyer to pay, even after the
expiration of the period, as long as no demand for rescission of WHEREFORE, premises considered, the Petition is GRANTED.
the contract has been made upon him either judicially or by
notarial act. However, Article 1592 does not apply to a contract to
sell where the seller reserves the ownership until full payment of PART VIII: CONDITIONS AND WARRANTIES
the price," as in this case.
A. Conditions
Applying the above jurisprudence, we hold that when Rodolfo
failed to fully pay the purchase price, the contract to sell was CATUNGAL VS. RODRIGUEZ
deemed terminated or cancelled. As we have held in Chua v.
Court of Appeals,"[s]ince the agreement x x x is a mere contract FACTS: Agapita Catungal owned a parcel of land in Barrio
to sell, the full payment of the purchase price partakes of a Talamban, Cebu City. On April 232, 1990, Agapita, with the
suspensive condition. The non-fulfillment of the condition consent of her husband (Atty. Jose Catungal), entered a Contract
prevents the obligation to sell from arising and ownership is to Sell with respondent Angel Rodriguez. This Contract to Sell
retained by the seller without further remedies by the buyer." was further upgraded into a Conditional Deed of Sale where it
Similarly, we held in Reyes v. Tuparan that "petitioner’s obligation was stipulated that the sum of P25 million will be payable as
to sell the subject properties becomes demandable only upon the follows:
happening of the positive suspensive condition, which is the
respondent’s full payment of the purchase price. Without a) P500, 000 down payment upon signing of the
respondent’s full payment, there can be no breach of agreement;
contract to speak of because petitioner has no obligation b) The balance of P24, 500, 000 will be payable in five
yet to turn over the title. separate checks:

Respondent’s failure to pay in full the purchase price in full is not First check shall be for P4, 500, 000 while the remaining balance
the breach of contract contemplated under Article 1191 of the to be paid in four checks in the amount of P5 million each will be
New Civil Code but rather just an event that prevents the payable only after Rodriguez (Vendee) has successfully
petitioner from being bound to convey title to respondent." negotiated, secured, and provided a Road Right of Way. If
Otherwise stated, Rodolfo has no right to compel Nicolas to however the Road Right of Way could not be negotiated,
transfer ownership to him because he failed to pay in full the Rodriguez shall notify the Catungals for them to reassess and
purchase price. Correlatively, Nicolas has no obligation to solve the problem by taking other options and should the
transfer his ownership over his share in the Diego Building to situation ultimately prove futile, he shall take steps to rescind or
Rodolfo. cancel the herein Conditional Deed of Sale.

On the other hand, the respondents’ additional submission – that It was also stipulated that the access road or Road Right of Way
Nicolas cheated them by "vanishing and hibernating" in the USA leading to the lot shall be the responsibility of the VENDEE to
after receiving Rodolfo’s P250,000.00 downpayment, only to secure and any or all cost relative to the acquisition thereof shall
come back later and claim that the amount he received was a be borne solely by the VENDEE. He shall, however, be accorded
mere loan – cannot be believed. How the respondents could with enough time necessary for the success of his endeavor,
have been cheated or disadvantaged by Nicolas’s leaving is granting him a free hand in negotiating for the passage.
beyond comprehension. If there was anybody who benefited
from Nicolas’s perceived "hibernation", it was the respondents, Spouses Catungal requested an advance of P5 million on the
COMPILED BY: WIGMORE #WIGMOREFOREVER 121
SALES Case Digest (Atty. Sarona)
Compiled by: Wigmore #wigmoreforever

purchase price for personal reasons. However, Rodriguez If still warranted, respondent Angel S. Rodriguez is given a
refused on the ground that the amount was not due under the period of thirty (30) days from the finality of this Decision to
terms of their agreement. Further, he learned that the Catungals negotiate a road right of way. In the event no road right of way is
were offering the property for sale to third parties who are willing secured by respondent at the end of said period, the parties shall
to pay a higher amount of money for a Road Right of Way than reassess and discuss other options as stipulated in paragraph
what Rodriguez has initially negotiated. In other words, instead 1(b) of the Conditional Deed of Sale and, for this purpose, they
of assisting Rodriguez in successfully negotiating, the Catungals are given a period of thirty (30) days to agree on a course of
allegedly maliciously defeated his efforts so to justify the action. Should the discussions of the parties prove futile after the
rescission. Rodriguez then received letters signed by Atty. Jose said thirty (30)-day period, immediately upon the expiration of
Catungal demanding him to make up his mind about buying the said period for discussion, Rodriguez may (a) exercise his option
land or exercising his option to buy because they needed money to rescind the contract, subject to the return of his down
to pay personal obligations or else the Catungals warned that payment, in accordance with the provisions of paragraphs 1(b)
they would consider the contract cancelled. and 5 of the Conditional Deed of Sale or (b) waive the road right
of way and pay the balance of the deducted purchase price as
RTC ruled in favor of Rodriguez finding that his obligation to pay determined in the RTC Decision dated May 30, 1992.
the balance arises only after successfully negotiating a Road
Right of Way. CA affirmed the RTC’s decision but the defendants
filed a motion for reconsideration and raised for the first time the B. Warranties
contention that the court erred in not finding their stipulations null
for violating the principle of mutuality of contracts.

ISSUE: Whether or not the stipulations of their Conditional Deed 1. Express Warranties
of Sale constitute a potestative condition (one that is subject to
the will of one of the parties – either the debtor or creditor).
HARRISON MOTORS CORPORATION vs. RACHEL A.
HELD: NO. the condition in their Conditional Deed of Sale NAVARRO
stating that respondent shall pay the balance of the purchase
price when he has successfully negotiated and secured a road FACTS: Sometime in June of 1987 Harrison Motors Corporation
right of way, is not a condition on the perfection of the contract through its president, Renato Claros, sold 2 Isuzu Elf trucks to
nor on the validity of the entire contract or its compliance as Rachel Navarro, owner of RN Freight Lines. Prior to the sale,
contemplated in Article 1308. It is a condition imposed only on Renato Claros represented to Navarro that all the BIR taxes and
respondent's obligation to pay the remainder of the purchase customs duties for the parts used on the two 2 trucks had been
price. In our view and applying Article 1182, such a condition is paid for.
not purely potestative as petitioners contend. It is not dependent
on the sole will of the debtor but also on the will of third persons On 10 September 1987 the Bureau of Internal Revenue (BIR)
who own the adjacent land and from whom the road right of way and the Land Transportation Office (LTO) entered into a
shall be negotiated. Ina manner of speaking, such a condition is Memorandum of Agreement (MOA) which provided that prior to
likewise dependent on chance as there is no guarantee that registration in the LTO of any assembled or re-assembled motor
respondent and the third party-landowners would come to vehicle which used imported parts, a Certificate of Payment
an agreement regarding the road right of way. This type of mixed should first be obtained from the BIR to prove payment of all
condition is expressly allowed under Article 1182 of the Civil taxes required under existing laws.
Code.
On 16 June 1988 the BIR, BOC and LTO entered into a tripartite
IN RELATION TO ARTICLE 1197 – The Catungals also argued MOA which provided that prior to the registration in the LTO of
that Rodriguez has been given enough time to negotiate for the any locally assembled motor vehicle using imported component
Road Right of Way. However, no stipulation regarding specific parts, a Certificate of Payment should first be obtained from the
time can be found in their agreement. SC said that Even BIR and the BOC to prove that all existing taxes and customs
assuming arguendo that the Catungals were correct that the duties have been paid.
respondent's obligation to negotiate a road right of way was one
with an uncertain period, their rescission of the Conditional Deed In December of 1988 government agents seized and detained
of Sale would still be unwarranted. What the Catungals should the two (2) Elf trucks of Navarro after discovering that there were
have done was to first file an action in court to fix the period still unpaid BIR taxes and customs duties thereon. The BIR and
within which Rodriguez should accomplish the successful the BOC ordered NAvarro to pay the proper assessments or her
negotiation of the road right of way pursuant to the above quoted trucks would be impounded. Navarro went to Claros to ask for
provision. Thus, the Catungals' demand for Rodriguez to make the receipts evidencing payment of BIR taxes and customs
an additional payment of P5 million was premature and duties; however, Claros refused to comply. Navarro then
Rodriguez's failure to accede to such demand did not justify the demanded from Claros but her demands were again ignored.
rescission of the contract.
But wanting to secure the immediate release of the trucks to
WHEREFORE, the Decision dated August 8, 2000 and the comply with her business commitments, Navarro paid the
Resolution dated January 30, 2001 of the Court of Appeals are assessed BIR taxes and customs duties amounting to
AFFIRMED with the following P32,943.00. Consequently, she returned to Harrison‘s office to
ask for reimbursement, but it again refused, prompting her to
MODIFICATION: send a demand letter through her lawyer. When Harrison still
COMPILED BY: WIGMORE #WIGMOREFOREVER 122
SALES Case Digest (Atty. Sarona)
Compiled by: Wigmore #wigmoreforever

ignored her letter, she filed a complaint for a sum of money on 24 would have been avoided had Harrison simply furnished private
September 1990 with the Regional Trial Court of Makati. respondent with the receipts evidencing payment of BIR taxes
and customs duties. If only Navarro had the receipts to prove
On 5 March 1992 the trial court rendered a decision ordering payment of such assessments then she would have easily
Harrison to reimburse private respondent in the amount of secured the release of her two (2) Elf trucks. But Harrison
P32,943.00 for the customs duties and internal revenue taxes arbitrarily and unjustly denied Navarro‘s demands. Instead,
the latter had to pay to discharge her 2 Elf trucks from Harrison obstinately insisted that it was no longer concerned with
government custody. the problem involving the two (2) trucks since it no longer owned
the vehicles after the consummation of the sale.
Harrison argues that it was no longer obliged to pay for the
additional taxes and customs duties imposed on the imported It is true that the ownership of the trucks shifted to private
component parts by the Memorandum Orders and the two (2) respondent after the sale. But petitioner must remember that
Memoranda of Agreement based on non-impairment clause of prior to its consummation it expressly intimated to her that it had
the Constitution but also the principle of non-retroactivity of laws already paid the taxes and customs duties. Such representation
provided in Art. 4 of the Civil Code. shall be considered as a seller‘s express warranty under Art.
1546 of the Civil Code which covers any affirmation of fact or any
The records however reveal that the Memorandum Orders and promise by the seller which induces the buyer to purchase the
Memoranda of Agreement do not impose any additional BIR thing and actually purchases it relying on such affirmation or
taxes or customs duties. The MOA mandated that prior to promise. It includes all warranties which are derived from
registration in the LTO of any assembled automobile using express language, whether the language is in the form of a
imported parts, a Certificate of Payment should first be obtained promise or representation. Presumably, therefore, private
from the BIR which would then transmit the Certificate to the LTO respondent would not have purchased the two (2) Elf trucks were
to prove that all the BIR taxes required under existing laws have it not for petitioner‘s assertion and assurance that all taxes on its
been paid. imported parts were already settled.

The MOA provided that prior to registration with the LTO of any This express warranty was breached the moment petitioner
assembled motor vehicle using imported component parts, a refused to furnish private respondent with the corresponding
Certificate of Payment should first be secured from the BIR or receipts since such documents were the best evidence she could
the BOC which should then be duly forwarded to LTO. The present to the government to prove that all BIR taxes and
Certificate would serve as proof that all taxes and customs duties customs duties on the imported component parts were fully paid.
required under existing laws, rules and regulations had already Without evidence of payment, she was powerless to prevent the
been settled. trucks from being impounded.

ISSUE: Who should pay the BIR taxes and customs duties which Under Art. 1599 of the Civil Code, once an express warranty is
the administrative regulations sought to enforce? breached the buyer can accept or keep the goods and maintain
an action against the seller for damages. This was what private
HELD: Harrison contends that Navarro should be the one to pay respondent did. She opted to keep the two (2) trucks which she
the internal revenue taxes and customs duties. It claims that at apparently needed for her business and filed a complaint for
the time the Memorandum Orders and the two (2) Memoranda of damages, particularly seeking the reimbursement of the amount
Agreement took effect the two (2) Elf trucks were already sold to she paid to secure the release of her vehicles.
Navarro, thus, it no longer owned the vehicles.

Thus, although the Whereas clause in the MOA provides that


private respondent is the one required by the administrative
regulations to secure the Certificate of Payment for the purpose MOLES vs IAC
of registration, petitioner as the importer and the
assembler/manufacturer of the two (2) Elf trucks is still the one FACTS: In 1977, petitioner needed a linotype printing machine
liable for payment of revenue taxes and customs duties. for his printing business, The LM Press at Bacolod City, and
applied for an industrial loan with the Development Bank of the
Harrison‘s obligation to pay does not arise from the Philippines. (DBP) for the purchase thereof. An agent of Smith,
administrative regulations but from the tax laws existing at the Bell and Co. who is a friend of petitioner introduced the latter to
time of importation. Hence, even if Navarro already owned the private respondent, owner of the Diolosa Publishing House in
two (2) trucks when the Memorandum Orders and Memoranda of Iloilo City, who had two available machines. Thereafter, petitioner
Agreement took effect, the fact remains that Harrison was still went to Iloilo City to inspect the two machines offered for sale
the one duty-bound to pay for the BIR taxes and customs duties. and was informed that the same were secondhand but
functional.
It is also quite obvious that as between Harrison, who is the
importer- assembler/manufacturer, and Navarro, who is merely Sometime between April and May, 1977, the machine was
the buyer, it is petitioner which has the obligation to pay taxes to delivered to petitioner's publishing house where it was installed
the BIR and the BOC. Harrison would be unjustly enriched if by an employee of Diolosa. Prior to the release of the loan, a
private respondent should be denied reimbursement. representative from the DBP, Bacolod, supposedly inspected the
machine but he merely looked at it to see that it was there . The
Besides, Harrison‘s allegation that it already paid the BIR taxes inspector's recommendation was favorable and, thereafter,
and customs duties is highly doubtful. This entire controversy petitioner's loan of P50,000.00 was granted and released.
COMPILED BY: WIGMORE #WIGMOREFOREVER 123
SALES Case Digest (Atty. Sarona)
Compiled by: Wigmore #wigmoreforever

To repeat, in the case before Us, a certification to the effect that


But on November 29, 1977, petitioner wrote private respondent the linotype machine bought by petitioner was in A-1 condition
that the machine was not functioning properly as it needed a new was issued by private respondent in favor of the former. This
distributor bar. Private respondent made no reply to said letter, cannot but be considered as an express warranty. However, it is
so petitioner engaged the services of other technicians. Later, private respondent's submission, that the same is not binding on
after several telephone calls regarding the defects in the him, not being a part of the contract of sale between them.
machine, private respondent sent two technicians to make the
necessary repairs but they failed to put the machine in running It must be remembered that the certification was a condition sine
condition. In fact, since then petitioner was never able to use the qua non for the release of petitioner's loan which was to be used
machine. Petitioner again wrote private respondent, this time as payment for the purchase price of the machine. Private
with the warning that he would be forced to seek legal remedies respondent failed to refute this material fact. Neither does he
to protect his interest. explain why he made that express warranty on the condition of
the machine if he had not intended to be bound by it. In fact, the
Obviously in response to the foregoing letter, private respondent respondent court, in declaring that petitioner should have availed
decided to purchase a new distributor bar and private respondent of the remedy of requiring repairs as provided for in said
delivered this spare part to petitioner. However, when thereafter certification, thereby considered the same as part and parcel of
petitioner asked private respondent to pay for the price of the the verbal contract between the parties.
distributor bar, the latter asked petitioner to share the cost with
him. Petitioner thus finally decided to indorse the matter to his Private respondents express warranty as to the A-1
lawyer. condition of the machine was not merely dealer's talk.

An expert witness for the petitioner declared that he inspected Private respondent was not a dealer of printing or linotype
the linotype machine involved in this case at the instance of machines to whom could be ascribed the supposed resort to the
petitioner. In his inspection thereof, he found several defects. usual exaggerations of trade in said items. His certification as to
the condition of the machine was not made to induce petitioner to
ISSUE/S: purchase it but to confirm in writing for purposes of the financing
W/N private respondent is bound by an express warranty. aspect of the transaction his representations thereon. Ordinarily,
W/N private respondent’s express warrantly was a mere dealer’s what does not appear on the face of the written instrument
talk. should be regarded as dealer's or trader's talk conversely, what
W/N hidden defects in the machine is sufficient to warrant a is specifically represented as true in said document, as in the
rescission of the contract between the parties. instant case, cannot be considered as mere dealer's talk.

HELD: Private respondent is bound by the express warranty On the question as to whether the hidden defects in the machine
which he executed in favor of the petitioners. is sufficient to warrant a rescission of the contract between the
parties, we have to consider the rule on redhibitory defects
When an article is sold as a secondhand item, a question arises contemplated in Article 1561 of the Civil Code. A redhibitory
as to whether there is an implied warranty of its quality or fitness. defect must be an imperfection or defect of such nature as to
It is generally held that in the sale of a designated and specific engender a certain degree of importance. An imperfection or
article sold as secondhand, there is no implied warranty as to its defect of little consequence does not come within the category of
quality or fitness for the purpose intended, at least where it is being redhibitory.
subject to inspection at the time of the sale. On the other hand,
there is also authority to the effect that in a sale of a secondhand As already narrated, an expert witness for the petitioner
articles there may be, under some circumstances, an implied categorically established that the machine required major repairs
warranty of fitness for the ordinary purpose of the article sold or before it could be used. This, plus the fact that petitioner never
for the particular purpose of the buyer. made appropriate use of the machine from the time of purchase
until an action was filed, attest to the major defects in said
In a line of decisions rendered by the United States Supreme machine, by reason of which the rescission of the contract of
Court, it had theretofore been held that there is no implied sale is sought. The factual finding, therefore, of the trial court that
warranty as to the condition, adaptation, fitness, or suitability for the machine is not reasonably fit for the particular purpose for
the purpose for which made, or the quality, of an article sold as which it was intended must be upheld, there being ample
and for a secondhand article. Said general rule, however, is not evidence to sustain the same.
without exceptions. Article 1562 of our Civil Code, which was
taken from the Uniform Sales Act, provides: At a belated stage of this appeal, private respondent came up for
the first time with the contention that the action for rescission is
Art. 1562. In a sale of goods, there is an implied warranty or barred by prescription. While it is true that Article 1571 of the
condition as to the quality or fitness of the goods, as follows: Civil Code provides for a prescriptive period of six months for a
redhibitory action a cursory reading of the ten preceding articles
Where the buyer, expressly or by implication, makes known to to which it refers will reveal that said rule may be applied only in
the seller the particular purpose for which the goods are case of implied warranties. The present case involves one with
acquired, and it appears that the buyer relies on the seller's skill and express warranty. Consequently, the general rule on
27
or judgment (whether he be the grower or manufacturer or not), rescission of contract, which is four years shall apply.
there is an implied warranty that the goods shall be reasonably fit Considering that the original case for rescission was filed only
for such purpose; one year after the delivery of the subject machine, the same is
well within the prescriptive period.
COMPILED BY: WIGMORE #WIGMOREFOREVER 124
SALES Case Digest (Atty. Sarona)
Compiled by: Wigmore #wigmoreforever

2. Implied Warranties company’s president was a close friend of Evangelista. The


various animal feeds were paid and covered by checks with due
dates from July 1993-September 1993.
ESCALER V. CA AND SPS. REYNOSO
Initially, the spouses were good paying customers. However,
FACTS: On March 7, 1958, the spouses Reynoso sold to there were instances when they failed to issue checks despite the
petitioners a parcel of land situated in Antipolo, Rizal with an delivery of goods. Consequently, the respondents incurred an
area of 239,479 sqm and covered by TCT No. 57400. However, aggregate unsettled account with Nutrimix amounting to
on April 21, 1961, the Register of Deeds of Rizal and A. Doronilla P766,151
Resources Development, Inc. filed a case (Case no. 4252) for
the cancellation of OCT No. 1526 issued in the name of Angelina When the checks were deposited by the petitioner, the same
C. Reynoso (the predecessors-in-interest of the spouses were dishonored (closed account). Despite several demands from
Reynoso) on the ground that the subject property is already the petitioner, the spouses refused to pay the remaining balance
covered by TCT No. 42999 issued under A. Doronilla. The court
favored A. Doronilla in this case. Thereafter, Nutrimix filed a complaint against Evangelista for
collection of money with damages.
Thereafter, herein petitioners filed the present case against the
respondents, for the recovery of the value of the property sold to The respondents admitted their unpaid obligation but impugned
them plus damages on the ground that the spouses violated the their liability. The nine checks issued were made to guarantee the
vendors’ “warranty against eviction.” The lower court ruled in payment of the purchases, which was previously determined to
favor of the petitioners, which was reversed in the CA. Hence, be procured from the expected proceeds in the sale of their
the present petition. broilers and hogs. They contended that inasmuch as the sudden
and massive death of their animals was caused by the
ISSUE: W/n Articles 1558 and 1559 of the Civil Code are to be contaminated products of the petitioner, the nonpayment of their
strictly applied in this case. obligation was based on a just and legal ground.

HELD: YES. (See Articles 1548, 1558 and 1559). In order that a The respondents also lodged a complaint for damages against
vendor’s liability for eviction may be enforced, the following the petitioner, for the untimely and unforeseen death of their
requisites must concur: animals supposedly effected by the adulterated animal feeds the
petitioner sold to them.
a. There must be a final judgment;
b. The purchaser has been deprived of the whole or part Nutrimix alleged that the death of the respondents’ animals was
of the thing sold; due to the widespread pestilence in their farm. The petitioner,
c. Said deprivation was by virtue of a right prior to the sale likewise, maintained that it received information that the
made by the vendor; and respondents were in an unstable financial condition and even
d. The vendor has been summoned and made co- sold their animals to settle their obligations from other enraged
defendant in the suit for eviction at the instance of the and insistent creditors. It, moreover, theorized that it was the
vendee. respondents who mixed poison to its feeds to make it appear that
the feeds were contaminated.
In the case at bar, the fourth requisite—that of being summoned
in the suit for eviction (Case no. 4252) at the instance of the The trial court held in favor of petitioner on the ground that it
vendee—is not present. All that petitioners did, per their very cannot be held liable under Articles 1561 and 1566 of the Civil
admission, was to furnish respondents, by registered mail, with a Code governing “hidden defects” of commodities sold. The trial
copy of the opposition they (petitioners) filed in the suit. court is predisposed to believe that the subject feeds were
Decidedly, this is not the kind of notice prescribed by Articles contaminated sometime between their storage at the bodega of
1558 and 1559 of the Civil Code. the Evangelistas and their consumption by the poultry and hogs
fed therewith, and that the contamination was perpetrated by
The term “unless he is summoned in the suit for eviction at the unidentified or unidentifiable ill-meaning mischief-maker(s) over
instance of the vendee” means that the respondents as vendor/s whom Nutrimix had no control in whichever way.
should be made parties to the suit at the instance of petitioners-
vendees, either by way of asking that the former be made a co- CA modified the decision of the trial court, citing that respondents
defendant or by the filing of a third-party complaint against said were not obligated to pay their outstanding obligation to the
vendors. Nothing of that sort appeared to have been done by the petitioner in view of its breach of warranty against hidden defects.
petitioners in the instant case. The CA gave much credence to the testimony of Dr. Rodrigo
Diaz, who attested that the sample feeds distributed to the
various governmental agencies for laboratory examination were
NUTRIMIX FEEDS CORP. V. CA taken from a sealed sack bearing the brand name Nutrimix

FACTS: In 1993, private respondent spouses Evangelista ISSUE: WON Nutrimix is guilty of breach of warranty due to
procured various animal feeds from petitioner Nutrimix Feeds hidden defects
Corp. the petitioner gave the respondents a credit period of 30-
45 days to postdate checks to be issued as payment for the HELD: NO. The provisions on warranty against hidden defects
feeds. The accommodation was made apparently because the are found in Articles 1561 and 1566 of the New Civil Code of the
Philippines. A hidden defect is one which is unknown or could not
COMPILED BY: WIGMORE #WIGMOREFOREVER 125
SALES Case Digest (Atty. Sarona)
Compiled by: Wigmore #wigmoreforever

have been known to the vendee. Under the law, the requisites to and peaceful possession in favor of the petitioners.
recover on account of hidden defects are as follows:
The property is mortgage to PNP and as such, petitioners filed a
a) the defect must be hidden; request to assume responsibility of the mortgage. Because of
b) the defect must exist at the time the sale was made; petitioners failure to produce the required papers, their petition
c) the defect must ordinarily have been excluded from the was denied.
contract; Petitioners allege that the contract should be rescinded because
d) the defect, must be important (renders thing UNFIT or of failure of delivery.
considerably decreases FITNESS);
e) the action must be instituted within the statute of ISSUE: WON the contract is recissible due to breach of contract.
limitations
HELD: There is no breach of contact in this case since there is
In the sale of animal feeds, there is an implied warranty that it is no provision in the contract that imposes the obligation to the
reasonably fit and suitable to be used for the purpose which both respondents to eject the people occupying the property.
parties contemplated. To be able to prove liability on the basis of
breach of implied warranty, three things must be established by There was also a constructive delivery because the deed of sale
the respondents. The first is that they sustained injury because was made in a public document. The contention of the petitioners
of the product; the second is that the injury occurred because the that there could be no constructive delivery because the
product was defective or unreasonably unsafe; and finally, the respondents are not in possession of the property is of no merit.
defect existed when the product left the hands of the petitioner. A
manufacturer or seller of a product cannot be held liable for any What matters in a constructive delivery is control and not
damage allegedly caused by the product in the absence of any possession. Control was placed in the hands of the petitioners
proof that the product in question was defective. The defect must that is why they were able to file an ejectment case. Prior
be present upon the delivery or manufacture of the product; or physical delivery or possession is not legally required and the
when the product left the seller’s or manufacturer’s control; or execution of the deed of sale is deemed equivalent to delivery.
when the product was sold to the purchaser; or the product must
have reached the user or consumer without substantial change
in the condition it was sold. Tracing the defect to the petitioner SUPERCARS MANAGEMENT AND DEVELOPMENT
requires some evidence that there was no tampering with, or CORPORATION VS. FLORES
changing of the animal feeds. The nature of the animal feeds
makes it necessarily difficult for the respondents to prove that the FACTS: Respondent Flores bought an Isuzu Carter Crew Cab
defect was existing when the product left the premises of the from petitioner. The RCBC financed the balance of the purchase
petitioner. price. Its payment was secured by a chattel mortgage of the
same vehicle. However, defects of the car emerged when
A review of the facts of the case would reveal that the petitioner respondent was using it. These defects persuaded respondent
delivered the animal feeds, allegedly containing rat poison, on Flores to rescind the contract with petitioner and stop the
July 26, 1993; but it is astonishing that the respondents had the payment of the balance for the aforesaid car.
animal feeds examined only on October 20, 1993, or barely three
months after their broilers and hogs had died. A difference of As a result, RCBC bank opted to file a petition for Extrajudicial
approximately three months enfeebles the respondents’ theory Foreclosure of Chattel Mortgage. The car was then sold at a
that the petitioner is guilty of breach of warranty by virtue of public auction and RCBC acquired the same. It was later sold to
hidden defects. In a span of three months, the feeds could have a third person. Petitioner contends that respondent has "no right
already been contaminated by outside factors and subjected to to rescind the contract of sale" because the motor vehicle in
many conditions unquestionably beyond the control of the question is already in the hands of a third party.
petitioner.
Hence, Article 1191 can no longer be availed of by the
Even more surprising is the fact that during the meeting with respondent.
Nutrimix President Mr. Bartolome, the respondents claimed that
their animals were plagued by disease, and that they needed ISSUE: Whether or not Article 1191 can no longer be availed of
more time to settle their obligations with the petitioner. It was by respondent Flores.
only after a few months that the respondents changed their
justification for not paying their unsettled accounts, claiming RULING: Article 1191 is applicable. Rescission is proper if one of
anew that their animals were poisoned with the animal feeds the parties to a contract commits a substantial breach of its
supplied by the petitioner. provision. It creates an obligation to return the object of the
contract. It can be carried out only when the one who demands
rescission can return whatever he may obliged to restore.
POWER COMMERCIAL V. CA (June 20, 1997)
Rescission abrogates the contract from its inception and requires
FACTS: Petitioner asbestos manufacturer Power Commercial a mutual restitution of the benefits received. Respondent is not
and industrial corporation bought the property of spouses obliged to return the car; while petitioner is obliged to return what
Reynaldo and Angelita Quiambao located in Makati City. has been paid.

Since there are lessees occupying the subject land, part of the
deed of sale is a warranty of respondents that will defend its title
COMPILED BY: WIGMORE #WIGMOREFOREVER 126
SALES Case Digest (Atty. Sarona)
Compiled by: Wigmore #wigmoreforever

PART IX: EXTINGUISHMENT OF SALE also went to the Registry of Deeds to verify the records on file
and ascertained that OCT No. 535 was clean and had no lien
A. Grounds and encumbrances. After the necessary verification, petitioner
decided to buy the Antipolo property.
B. Conventional Redemption
On March 20, 1995, the Eniceo heirs executed a deed of
absolute sale in favor of petitioner covering lots 3 and 4 of the
KINGS PROPERTIES CORP., VS. CANUTO A. GALIDO Antipolo property for P500,000.00.

FACTS: This case involves an action for cancellation of On August 17, 1995, the Secretary of Department of
certificates of title, registration of deed of sale and issuance of Environment and Natural Resources (DENR Secretary)
certificates of title filed by Canuto A. Galido before the RTC of approved the deed of sale between the Eniceo heirs and
Antipolo City. On April 18, 1966, the heirs of Eniceo, namely respondent. On January 1996, respondent filed a civil complaint
Rufina and Maria Eniceo, were awarded with Homestead Patent with the trial court against the Eniceo heirs and petitioner praying
consisting of four parcels of land located in San Isidro, Antipolo, for the cancellation of the certificates
Rizal. The Antipolo property with a total area of 14.8882 hectares of title issued in favor of petitioner, and the registration of the
was registered under OCT No. 535. deed of sale and issuance of a new transfer certificate of title in
favor of respondent.
The issuance of the homestead patent was subject to the
following conditions: The trial court rendered it's decision dismissing the case for lack
of legal and factual basis. However, the CA reversed the trial
“To have and to hold the said tract of land, with the court's decision.
appurtenances thereunto of right belonging unto the said Heirs of
Domingo Eniceo and to his heir or heirs and assigns forever, ISSUE: Whether the deed of sale delivered to respondent should
subject to the provisions of sections 118, 121, 122 and 124 of be presumed an equitable mortgage pursuant to Article 1602(2)
Commonwealth Act No. 141, as amended, which provide that and 1604 of the Civil Code.
except in favor of the Government or any of its branches, units or
institutions, the land hereby acquired shall be inalienable and HELD: Validity of the deed of sale to respondent
shall not be subject to incumbrance for a period of five (5) years The contract between the Eniceo heirs and respondent executed
on 10 September 1973 was a perfected contract of sale. A
next following the date of this patent, and shall not be liable for
the satisfaction of any debt contracted prior to the expiration of contract is perfected once there is consent of the contracting
that period; that it shall not be alienated, transferred or conveyed parties on the object certain and on the cause of the obligation.
after five (5) years and before twenty-five (25) years next In the present case, the object of the sale is the Antipolo property
following the issuance of title, without the approval of the and the price certain is P250,000.
Secretary of Agriculture and Natural Resources; that it shall not
be incumbered, alienated, or transferred to any person, The contract of sale has also been consummated because the
corporation, association, or partnership not qualified to acquire vendors and vendee have performed their respective obligations
public lands under the said Act and its amendments; x x x” under the contract. In a contract of sale, the seller obligates
himself to transfer the ownership of the determinate thing sold,
On September 1973, a deed of sale covering the Antipolo and to deliver the same to the buyer, who obligates himself to
property was executed between Rufina Eniceo and Maria Eniceo pay a price certain to the seller. The execution of the notarized
as vendors and respondent as vendee. The property was sold to deed of sale and the delivery of the owners duplicate copy of
respondent for P250,000. A certain Carmen Aldana delivered the OCT No. 535 to respondent is tantamount to a constructive
owner's duplicate copy of OCT No. 535 to respondent. delivery of the object of the sale.

On 1988, the Eniceo heirs registered with the Registry of Deeds Petitioner invokes the belated approval by the DENR Secretary,
a Notice of Loss of the owner's copy of OCT No. 535. made within 25 years from the issuance of the homestead, to
nullify the sale of the Antipolo property. The sale of the Antipolo
RTC rendered a decision finding that the certified true copy of property cannot be annulled on the ground that the DENR
OCT No. 535 contained no annotation in favor of any person, Secretary gave his approval after 21 years from the date the
corporation or entity. The RTC ordered the Registry of Deeds to deed of sale in favor of respondent was executed.
issue a second owner's copy of OCT No. 535 in favor of the
Eniceo heirs and declared the original owner's copy of OCT No. Equitable Mortgage
535 canceled and considered no further value. Petitioner contends that the deed of sale in favor of respondent is
an equitable mortgage because the Eniceo heirs remained in
Petitioner states that as early as 1991, respondent knew of the possession of the Antipolo property despite the execution of the
RTC decision because respondent filed a criminal case against deed of sale.
Rufina Eniceo and Leonila Bolinas for giving false testimony
upon a material fact during the trial. They alleged that sometime An equitable mortgage is one which although lacking in some
in 1995, Bolinas came to the office of Alberto Tronio Jr., formality, or form or words, or other requisites demanded by a
petitioner's general manager, and offered to sell the Antipolo statute, nevertheless reveals the intention of the parties to
property. During an on-site inspection, Tronio saw a house and charge real property as security for a debt, and contains nothing
ascertained that the occupants were Bolina's relatives. Tronio impossible or contrary to law. The essential requisites of an
equitable mortgage are:
COMPILED BY: WIGMORE #WIGMOREFOREVER 127
SALES Case Digest (Atty. Sarona)
Compiled by: Wigmore #wigmoreforever

1. The parties entered into a contract denominated as a loan from the Rizal commercial Banking Corporation in the
contract of sale; and amount of P2mil and executed a Real Estate Mortgage over the
2. Their intention was to secure existing debt by way of a property as security thereof. On motion of the petitioners, they
mortgage. were granted leave to file an amended complaint impleading the
bank as additional party defendant. The court issued an order
Petitioner claims that an equitable mortgage can be presumed rejecting the amended complaint of the petitioners. Likewise, the
because the Eniceo heirs remained in possession of the Antipolo court dismissed the complaint and held that pursuant to Article
property. Apart from the fact that the Eniceo heirs remained in 493 of the Civil Code, a co-owner is not invalidated by the
possession of the Antipolo property, petitioner has failed to absence of the consent of the co-owners. Hence, the sale by
substantiate its claim that the contract of sale was intended Esperanza of the property was valid; the excess from her
to secure an existing debt by way of mortgage. In fact, mere undivided share should be taken from the undivided shares of
tolerated possession is not enough to prove that the Criseta and Antonio, who expressly agreed to and benefited from
transaction was an equitable mortgage. the sale.

Furthermore, petitioner has not shown any proof that the Eniceo The CA likewise held that the sale was valid and binding insofar
heirs were indebted to respondent. On the contrary, the deed of as Esperanza's undivided share of the property was concerned.
sale executed in favor of respondent was drafted clearly to It affirmed the RTC ruling that the lack of consent of the co-
convey that the Eniceo heirs sold and transferred the Antipolo owners did not nullify the sale.
property to respondent. The deed of sale even inserted a
provision about defrayment of registration expenses to effect the ISSUE: Whether or not the Deed of Absolute Sale was valid. -
transfer of title to respondent. Deed of Absolute Sale was merely relatively simulated, it
remains valid and enforceable.
The Court notes that the Eniceo heirs have not appealed the
CAs decision, hence, as to the Eniceo heirs, the CAs decision HELD:
that the contract was a sale and not an equitable mortgage is
now final. Since petitioner merely assumed the rights of the Validity of the Sale
Eniceo heirs, petitioner is now estopped from questioning the We have before us an example of a simulated contract. Article
deed of sale dated 10 September 1973. 1345 of the Civil Code provides that the simulation of a contract
may either be absolute or relative. In absolute simulation, there is
a colorable contract but without any substance, because the
HEIRS OF THE LATE SPOUSES BALITE VS. LIM parties have no intention to be bound by it. An absolutely
simulated contract is void, and the parties may recover from
FACTS: Spouses Aurelio and Esperanza Balite were the owners each other what they may have given under the contract. On the
of a parcel of land. When Aurelio died intestate, his wife other hand, if the parties state a false cause in the contract to
Esperanza and their children inherited the subject property and conceal their real agreement, such a contract is relatively
became co-owners thereof. Esperanza became ill and was in simulated. Here, the parties’ real agreement binds them.
dire need of money for her hospital expenses. She, through her
daughter, Criseta, offered to sell to Rodrigo Lim, her undivided In the present case, the parties intended to be bound by the
share for the price of P1 mil. Esperanza and Rodrigo agreed that Contract, even if it did not reflect the actual purchase price of the
under the Deed of Absolute Sale, it will be made to appear that property. That the parties intended the agreement to produce
the purchase price of the property was P150,000 although the legal effect is revealed by the letter of Esperanza Balite to
actual price agreed upon by them for the property was P1mil. respondent dated October 23, 1996 and petitioners admission
that there was a partial payment of P320,000 made on the basis
On April 16, 1996, Esperanza executed a Deed of Absolute Sale of the Deed of Absolute Sale. There was an intention to transfer
in favor of Rodrigo. They also executed on the same day a Joint the ownership of over 10,000 square meters of the property .
Affidavit under which they declared that the real price of the Clear from the letter is the fact that the objections of her children
property was P1mil. payable to Esperanza by installments. Only prompted Esperanza to unilaterally withdraw from the
Esperanza and two of her children Antonio and Criseta knew transaction.
about the said transaction. When the rest of the children knew of
the sale, they wrote to the Register of Deeds saying that their Since the Deed of Absolute Sale was merely relatively simulated,
mother did not inform them of the sale of a portion of the said it remains valid and enforceable. All the essential requisites
property nor did they give consent thereto. Nonetheless, Rodrigo prescribed by law for the validity and perfection of contracts are
made partial payments to Antonio who is authorized by his present. However, the parties shall be bound by their real
mother through a Special Power of Attorney. agreement for a consideration of P1,000,000 as reflected in their
Joint Affidavit.
Esperanza signed a letter addressed to Rodrigo informing the
latter that her children did not agree to the sale of the property to Deed of Sale not an Equitable Mortgage
him and that she was withdrawing all her commitments until the For Articles 1602 and 1604 to apply, two requisites must concur:
validity of the sale is finally resolved. Then Esperanza died one, the parties entered into a contract denominated as a
intestate and was survived by her children. Meanwhile, Rodrigo contract of sale; and, two, their intention was to secure an
caused to be published the Deed of Absolute Sale. existing debt by way of mortgage.

Petitioners filed a complaint against Rodrigo for the annulment of In the present case, however, the Contract does not merely
sale, quieting of title, injunction and damages. Rodrigo secured a purport to be an absolute sale. The records and the documentary
COMPILED BY: WIGMORE #WIGMOREFOREVER 128
SALES Case Digest (Atty. Sarona)
Compiled by: Wigmore #wigmoreforever

evidence introduced by the parties indubitably show that the On July 9, 1955, Leoncia and her three sons executed a deed
Contract is, indeed, one of absolute sale. There is no clear and denominated Kasulatan ng Biling Mabibiling Muli, whereby they
convincing evidence that the parties agreed upon a sold the land and its existing improvements to the Spouses
mortgage of the subject property. Benedicto Francia and Monica Ajoco (Spouses Francia) for
P500.00, subject to the vendors right to repurchase for the same
Furthermore, the voluntary, written and unconditional acceptance amount sa oras na sila'y makinabang. Potencianas heirs did not
of contractual commitments negates the theory of equitable assent to that deed. Nonetheless, Teofilo and Jose, Jr. and their
mortgage. There is nothing doubtful about the terms of, or the respective families remained in possession of the property and
circumstances surrounding, the Deed of Sale that would call for paid the realty taxes thereon.
the application of Article 1602. The Joint Affidavit indisputably
confirmed that the transaction between the parties was a sale. Leoncia and her children did not repay the amount of P500.00.

We find no basis to conclude that the purchase price of the The Spouses Francia both died intestate. Alejandro, the son of
property was grossly inadequate. Petitioners did not present any Jose, Sr., first partially paid to the Spouses Francia the amount
witness to testify as to the market values of real estate in the of P265.00 for the obligation of Leoncia, his uncles and his
subjects locale. They made their claim on the basis alone of the father. Alejandro later paid the balance of P235.00. Thus, on
P2,000,000 loan that respondent had been able to obtain from August 11, 1970, the heirs of Spouses Francia executed a deed
the Rizal Commercial Banking Corporation. This move did not entitled Pagsasa-ayos ng Pag-aari at Pagsasalin, whereby they
sufficiently show the alleged inadequacy of the purchase price. A transferred and conveyed to Alejandro all their rights and
mortgage is a mere security for a loan. There was no showing interests in the property for P500.00.
that the property was the only security relied upon by the bank;
or that the borrowers had no credit worthiness, other than the On August 21, 1970, Alejandro executed a Kasulatan ng
property offered as collateral. Pagmeme-ari, wherein he declared that he had acquired all the
rights and interests of the heirs of the Spouses Francia, including
Co-Ownership the ownership of the property, after the vendors had failed to
The appellate court was correct in affirming the validity of the repurchase within the given period. On the basis of the
sale of the property insofar as the pro indiviso share of Kasulatan ng Pagmeme-ari, Tax Declaration was issued to
Esperanza Balite was concerned. Alejandro. From then on, he had paid the realty taxes for the
property.
Article 493 of the Civil Code gives the owner of an undivided
interest in the property the right to freely sell and dispose of such Nevertheless, on October 17, 1970, Alejandro, his grandmother
interest. The co-owner, however, has no right to sell or alienate a (Leoncia), and his father (Jose, Sr.) executed a Magkakalakip na
specific or determinate part of the thing owned in common, Salaysay, by which Alejandro acknowledged the right of Leoncia,
because such right over the thing is represented by an aliquot or Jose, Jr., and Jose, Sr. to repurchase the property at any time for
ideal portion without any physical division. Nonetheless, the the same amount of P500.00.
mere fact that the deed purports to transfer a concrete portion
does not per se render the sale void. The sale is valid, but only On October 22, 1970, Leoncia died intestate. She was survived
with respect to the aliquot share of the selling co-owner. by Jose, Sr., Teofilo, Jose, Jr. and the heirs of Potenciana. Even
Furthermore, the sale is subject to the results of the partition after Leonicas death, Teofilo and Jose, Jr., with their respective
upon the termination of the co-ownership. families, continued to reside in the property.

Hence, the transaction between Esperanza Balite and On September 2, 1993, Alejandro also died intestate. Surviving
respondent could be legally recognized only in respect to him were his wife, Amanda, and their children. In 1994,
the formers pro indiviso share in the co-ownership. As a respondent Amanda Reyes asked the heirs of Teofilo and Jose,
matter of fact, the Deed of Absolute Sale executed between the Jr., to vacate the property because she and her children already
parties expressly referred to the 10,000-square-meter portion of needed it. After the petitioners refused to comply, she filed a
the land sold to respondent as the share of Esperanza in the complaint against the petitioners in the barangay, seeking their
conjugal property. Her clear intention was to sell merely her ideal eviction from the property. When no amicable settlement was
or undivided share in it. No valid objection can be made against reached, the Barangay Lupon issued a certification to file action
that intent. Clearly then, the sale can be given effect to the extent to the respondents on September 26, 1994.
of 9,751 square meters, her ideal share in the property as found
by both the trial and the appellate courts. In the interim, petitioner Nenita R. de la Cruz and her brother
Romeo Reyes also constructed their respective houses on the
property.
HEIRS OF REYES VS. REYES
ISSUE: Whether or not the Court of Appeals erred in finding that
FACTS: Antonio Reyes and his wife, Leoncia Mag-isa Reyes respondents (were) already barred from claiming that the
(Leoncia), were owners of a parcel of residential land located in transaction entered into by their predecessors-in-interest was an
Pulilan, Bulacan. On that land they constructed their dwelling. equitable mortgage and not a pacto de retro sale.
The couple had four children, namely: Jose, Sr., Teofilo, Jose, Jr.
and Potenciana. Antonio Reyes died intestate, and was survived RULING: The true agreement of the parties vis--vis the
by Leoncia and their three sons. Potenciana also died intestate, Kasulatan ng Biling Mabibiling Muli was an equitable
survived by her children. mortgage, not a pacto de retro sale. There was no dispute that
the purported vendors had continued in the possession of the
COMPILED BY: WIGMORE #WIGMOREFOREVER 129
SALES Case Digest (Atty. Sarona)
Compiled by: Wigmore #wigmoreforever

property even after the execution of the agreement; and that the conceal the true nature of a contract, that is, a loan secured by a
property had remained declared for taxation purposes under mortgage. It is a reality that grave financial distress renders
Leoncias name, with the realty taxes due being paid by Leoncia, persons hard-pressed to meet even their basic needs or to
despite the execution of the agreement. Such established respond to an emergency, leaving no choice to them but to sign
circumstances are among the badges of an equitable mortgage deeds of absolute sale of property or deeds of sale with pacto de
enumerated in Article 1602, paragraphs 2 and 5 of the Civil retro if only to obtain the much-needed loan from unscrupulous
Code, to wit: money lenders.[30] This reality precisely explains why the
pertinent provision of the Civil Code includes a peculiar rule
Art. 1602. The contract shall be presumed to be an equitable concerning the period of redemption, to wit:
mortgage, in any of the following cases:
xxx Art. 1602. The contract shall be presumed to be an equitable
(2) When the vendor remains in possession as lessee or mortgage, in any of the following cases:
otherwise; xxx
xxx (3)When upon or after the expiration of the right to repurchase
(5) When the vendor binds himself to pay the taxes on the thing another instrument extending the period of redemption or
sold; granting a new period is executed;
xxx xxx

The existence of any one of the conditions enumerated under Ostensibly, the law allows a new period of redemption to be
Article 1602 of the Civil Code, not a concurrence of all or of a agreed upon or granted even after the expiration of the equitable
majority thereof, suffices to give rise to the presumption that the mortgagors right to repurchase, and treats such extension as
contract is an equitable mortgage. Consequently, the contract one of the indicators that the true agreement between the parties
between the vendors and vendees (Spouses Francia) was an is an equitable mortgage, not a sale with right to repurchase. It
equitable mortgage. was indubitable, therefore, that the Magkasanib na Salaysay
effectively afforded to Leoncia, Teofilo, Jose, Sr. and Jose, Jr. a
Are the petitioners now barred from claiming that the transaction fresh period within which to pay to Alejandro the redemption
under the Kasulatan ng Biling Mabibiling Muli was an equitable price of P500.00.
mortgage by their failure to redeem the property for a long period
of time?
ALUDOS VS SUERTE
Considering that sa oras na silay makinabang, the period of
redemption stated in the Kasulatan ng Biling Mabibiling Muli, FACTS: Lomises acquired from Baguio City Government the
signified that no definite period had been stated, the period to right to occupy two stalls in the Hangar Market in Baguio City.
redeem should be ten years from the execution of the contract, Lomises entered into an agreement with respondent johnny
pursuant to Articles 1142 and 1144 of the Civil Code. Thus, the Suerte for the transfer of all improvements and rights over the
full redemption price should have been paid by July 9, 1955; and two market stalls. Suerte gave down payment and there is a
upon the expiration of said 10-year period, mortgagees Spouses receipt. Suerte gave another payment but before the completion
Francia or their heirs should have foreclosed the mortgage, but of payment, Lomises backed out of the agreement and returned
they did not do so. Instead, they accepted Alejandros payments, what was paid and there was a receipt for this. Johnny did not
until the debt was fully satisfied by August 11, 1970. want the return of his money and wanted the continuation and
enforcement of his agreement. He filed for specific performance
The acceptance of the payments even beyond the 10-year with damages.
period of redemption estopped the mortgagees heirs from
insisting that the period to redeem the property had already RTC: nullified the agreement between johnny and Lomises for
expired. Their actions impliedly recognized the continued failure to secure the consent of the baguio city government and
existence of the equitable mortgage. The conduct of the original held that Lomises are merely lessees and the government was
parties as well as of their successors-in-interest manifested that the lessor owner of stalls. Lomises appealed to CA: the real
the parties to the Kasulatan ng Biling Mabibiling Muli really agreement between the parties was merely one of loan and not
intended their transaction to be an equitable mortgage, not a for sale. The load had been extinguished upon the return of
pacto de retro sale. amount to Johnny’s mother Domes.

Both the trial court and the CA declared that the Magkasanib na CA: there are two agreements entered into between Johnny and
Salaysay, which extended the redemption period of the Lomises: one for assignment of leasehold rights and the other
mortgaged property, was inefficacious, because the period to was for the sale of the improvements on the market stalls. The
redeem could no longer be extended after the original assignment of leasehold rights was void for lack of consent of the
redemption period had already expired. lessor, Baguio government. The sale of improvements was valid
because there were Lomises private properties.
The provisions of the Civil Code governing equitable mortgages
disguised as sale contracts, like the one herein, are primarily Petitioner: agreement was a loan.
designed to curtail the evils brought about by contracts of sale
with right to repurchase, particularly the circumvention of the ISSUE: W/N the CA was correct in characterizing the agreement
usury law and pactum commissorium.[29] Courts have taken between Johnny and Lomises as a sale of improvements and
judicial notice of the well-known fact that contracts of sale with assignment of leasehold rights
right to repurchase have been frequently resorted to in order to
COMPILED BY: WIGMORE #WIGMOREFOREVER 130
SALES Case Digest (Atty. Sarona)
Compiled by: Wigmore #wigmoreforever

HELD: What existed was an equitable mortgage, as


contemplated in Article 1602, in relation with Article 1604, of the Lomises contends that of the P68,000.00 given by Johnny, he
Civil Code. “An equitable mortgage has been defined ‘as one only received P48,000.00, with the remaining P20,000.00
which although lacking in some formality, or form or words, or retained by Johnny as interest on the loan. However, the
other requisites demanded by a statute, nevertheless reveals the testimonies of the witnesses presented during trial, including
intention of the parties to charge real property as security Lomises himself, negate this claim. Judge Rodolfo Rodrigo
for a debt, there being no impossibility nor anything contrary to (RTC of Baguio City, Branch VII) asked Lomises’ lawyer, Atty.
law in this intent. Article 1602 of the Civil Code lists down the Lockey, if they deny receipt of the P68,000.00; Atty. Lockey said
circumstances that may indicate that a contract is an equitable that they were not denying receipt, and added that they had in
mortgage: fact returned the same amount. Judge Rodrigo accurately
summarized their point by stating that “there is no need to
Art. 1602. The contract shall be presumed to be an dispute whether the P68,000.00 was given, because if [Lomises]
equitable mortgage, in any of the following cases: tried to return that x x x he had received that. Witness Atty. Albert
Umaming said he counted the money before he drafted the
When the price of a sale with right to repurchase is October 9, 1985 receipt evidencing the return; he said that
unusually inadequate; Lomises returned P68,000.00 in total. Thus, if the transaction
When the vendor remains in possession as lessee or was indeed a loan and the P20,000.00 interest was already
otherwise; prepaid by Lomises, the return of the full amount of P68,000.00
When upon or after the expiration of the right to repurchase by Lomises to Johnny (through his mother, Domes) would not
another instrument extending the period of redemption or make sense.
granting a new period is executed;
When the purchaser retains for himself a part of the That Lomises retained possession of the market stalls even after
purchase price; the execution of his agreement with Johnny is also not an
When the vendor binds himself to pay the taxes on the indication that the true transaction between them was one of
thing sold; loan. Johnny had yet to complete his payment and, until
Lomises decided to forego with their agreement, had four more
In any other case where it may be fairly inferred that the months to pay; until then, Lomises retained ownership and
real intention of the parties is that the transaction shall possession of the market stalls.
secure the payment of a debt or the performance of any
other obligation. Lomises cannot feign ignorance of the import of the terms of the
receipt of September 8, 1984 by claiming that he was an illiterate
In any of the foregoing cases, any money, fruits, or other old man. A witness (Ana Comnad) testified not only of the fact of
benefit to be received by the vendee as rent or otherwise the sale, but also that Lomises’ daughter, Dolores, translated the
shall be considered as interest. terms of the agreement from English to Ilocano for Lomises’
benefit. Lomises himself admitted this fact. If Lomises believed
Based on Lomises’ allegations in his pleadings, we consider that the receipt of September 8, 1984 did not express the parties’
three circumstances to determine whether his claim is well true intent, he could have refused to sign it or subsequently
supported. First, Johnny was a mere college student dependent requested for a reformation of its terms. Lomises rejected the
on his parents for support when the agreement was executed, agreement only after Johnny sought to enforce it.
and it was Johnny’s mother, Domes, who was the party actually
interested in acquiring the market stalls. Second, Lomises Hence, the CA was correct in characterizing the agreement
received only P48,000.00 of the P68,000.00 that Johnny claimed between Johnny and Lomises as a sale of improvements and
he gave as down payment; Lomises said that the P20,000.00 assignment of leasehold rights.
represented interests on the loan. Third, Lomises retained
possession of the market stalls even after the execution of the
agreement. Whether separately or taken together, these
circumstances do not support a conclusion that the parties C. Legal Redemption
only intended to enter into a contract of loan.

That Johnny was a mere student when the agreement was PART XI: ASSIGNMENT
executed does not indicate that he had no financial capacity to
pay the purchase price. At that time, Johnny was a 26-year old
third year engineering student who operated as a businessman PART XII: LEASE
as a sideline activity and who helped his family sell goods in the
Hangar Market. During trial, Johnny was asked where he was to
get the funds to pay the P260,000.00 purchase price, and he REGINA DIZON ET AL V. CA AND OVERLAND EXPRESS
said he would get a loan from his grandfather. That he did not LINES, INC.
have the full amount at the time the agreement was executed G.R. No. 122544 January 28, 1999
does not necessarily negate his capacity to pay the purchase
price, since he had 16 months to complete the payment. Apart
FACTS: Overland Express Lines, Inc. entered into a Contract of
from Lomises’ bare claim that it was Johnny’s mother, Domes,
Lease with Option to Buy with petitioners involving a 1,755.80
who was interested in acquiring his market stalls, we find no
square meter parcel of land situated at Diliman, Quezon City.
other evidence supporting the claim that Johnny was merely
The term of the lease was for 1 year commencing from May
acting as a dummy for his mother.
COMPILED BY: WIGMORE #WIGMOREFOREVER 131
SALES Case Digest (Atty. Sarona)
Compiled by: Wigmore #wigmoreforever

16,1974 up to May 15, 1975. During this period, Overland


Express Lines was granted an option to purchase for the amount
of P3,000.00 per square meter. Thereafter, the lease shall be on VIEGELY SAMELO vs MANOTOK SERVICES, INC.
a per month basis with a monthly rental of P3,000.00. G.R. NO. 170509 – June 27, 2012

For failure of Overland Express Lines to pay the increased rental FACTS: On January 31, 1997, the respondent entered into a
of P8,000.00 per month effective June 1976, petitioners filed an contract with the petitioner for the lease of a portion of the lot for
action for ejectment against it. Overland Express Lines were a period of 1 year. Upon the expiration of the lease contract on
ordered to vacate the leased premises and to pay the sum of December 31, 1997, the petitioner continued occupying the
P624,000.00 representing rentals in arrears and/or as damages subject premises without paying the rent. On August 5, 1998, the
in the form of reasonable compensation for the use and respondent sent a letter to the petitioner demanding that she
occupation of the premises during the period of illegal detainer vacate the subject premises and pay compensation for its use
from June 1976 to November 1982. and occupancy however, petitioner refused to heed these
demands.
Overland Express Lines Inc. then endorsed P300,000.00 as
partial payment for the leased property and as an attempt to On November 18, 1998, the respondent filed a complaint for
resurrect the lapsed option of purchasing the property, which unlawful detainer against the petitioner befor the MeTC praying
petitioners accepted (through Alice A. Dizon,) with the issuance that the petitioner be ordered to vacate the subject premises and
of an official receipt. to pay compensation for its use and occupancy.

ISSUES: In her answer, the petitioner alleged that the respondent had no
1. Whether Alice Dizon was authorized to receive the sum of right to collect rentals because the subject premises are located
P300,000.00 on behalf of petitioners and validly consider it as inside the property of the Philippine national Railways (PNR).
the partial payment of the property to be purchased by the She also added that the petitioner had no certificate of title over
respondent(Overland Express Lines, Inc.). the subject premises and further claimed that her signature in the
2. Whether there was a perfected contract of sale between the contract of lease was obtained through respondent’s
parties. misrepresentation and likewise maintained that she is now the
owner of the subject premises as she had been in possession
HELD: since 1944.
1. Alice Dizon is not authorized to receive the sum of
P300,000.00 on behalf of petitioners, therefore will not validly The MeTC decided in favor of the respondent and held that the
bind the petitioners with the private respondents to a contract of only issue to be resolved in an unlawful detainer case is physical
sale. possession or possession de facto, and that the respondent had
established its right of possession over the subject premises. It
Article 1874 of the Civil Code is explicit that: "When a sale of a added that the petitioner’s right under the lease contract already
piece of land or any interest therein is through an agent, the ceased upon the expiration of the said contract. It further ruled
authority of the latter shall be in writing; otherwise, the sale shall that the petitioner is already stopped from questioning the right of
be void." There was no written proof and valid consent by the the respondent over the subject premises when she entered into
petitioners (as co-owners of the leased premises) on the a contract of lease with the respondent.
supposed sale entered into by Alice A. Dizon, as petitioners’
alleged agent, and Overland Express Lines. On appeal, the RTC set aside the decision of the MeTC and
dismissed the complaint for unlawful detainer. The RTC held that
The sum of P300,000.00 received by Alice Dizon cannot be the respondent had no right to collect rentals as it failed to show
considered as partial payment for the purchase of the property that it had authority to administer subject premises and to enter
for the reason that the option given to the private respondent in into a contract of lease with the petitioner.
purchasing the property as an added condition in the contract of
lease already expired on May 1975. Aggrieved by the reversal, the respondent filed a petition for
review with the CA which reversed and set aside the RTC
As provided in Art 1670 of the Civil Code, the provision entitling decision and reinstated the MeTC judgment. The CA held that
the lessee the option to purchase the leased premises is not the petitioner is now estopped from questioning the right of the
deemed incorporated in the impliedly renewed contract because respondent over the subject property. It explained that in an
it is alien to the possession of the lessee. Private respondent's action involving the possession of the subject premises, a tenant
right to exercise the option to purchase expired with the cannot controvert the title of his landlord or assert any right
termination of the original contract of lease for one year. adverse to that title, without first delivering to the landlord the
premises acquired by virtue of the agreement between
2. There was no perfected contract of sale in the first place themselves. It further held that the only issue in an ejectment suit
because Alice Dizon was not an authorized agent of petitioner, is physical or material possession. The issue of ownership is not
therefore she cannot do any legal transactions with the required to determine the issue of possession since the petitioner
respondent. tacitly admitted that she is a lessee of the subject premises.

As enshrined in Art 1868, “By the contract of agency a person Petitioner moved for reconsideration but the CA denied her in its
binds himself to render some service or to do something in motion, hence this petition.
representation or on behalf of another, with the consent or
authority of the latter. ISSUE: The ejectment suit is physical or material possession.
COMPILED BY: WIGMORE #WIGMOREFOREVER 132
SALES Case Digest (Atty. Sarona)
Compiled by: Wigmore #wigmoreforever

(The issue of ownership is not required to determine the issue of act on the part of the lessor that it no longer consents to the
possession since the petitioner tacitly admitted that she is a continued occupation by the lessee of its property. After such
lessee of the subject premises.) notice, lessee’s right to continue in possession ceases and her
possession becomes one of detainer.
RULING: The SC held that an action for unlawful detainer exists
when a person unlawfully withholds possession of any land or Wherefore, petition was denied and the decisions of CA were
building against or from a lessor, vendor, vendee or other affirmed with modification on the unpaid rentals due.
persons, after the expiration or termination of the right to hold
possession, by virtue of any contract, express or implied. The
only issue to be resolved in an unlawful detainer case is physical SPS. MAMARIL VS. BOY SCOUT OF THE PHILIPPINES
or material possession of the property involved, independent of G.R. NO. 179382 | JANUARY 14, 2013
any claim of ownership by any of the parties involved. Thus, any
attempt of the parties to inject the question of ownership into the FACTS: PUJ operators Sps. Mamaril would park their 6
case is futile, except insofar as it might throw light on the right of passenger jeepneys every night at BSP’s compound in Malate,
possession. Manila for a fee of P300.00 per month for each unit. One day,
one of the vehicles was missing and was never recovered.
In the instant case, the lease contract was for the period of one According to the security guards Peña and Gaddi of AIB Security
year with a monthly rental of P3,960 commencing on January 31, Agency with whom BSP had contracted for its security and
1997 and expiring on December 31, 1997. It bears emphasis that protection, a male person who looked familiar to them took the
it was only on August 5, 1998 that a notice to vacate was sent subject vehicle out of the compound. Sps. Mamaril prayed that
and the petitioner continued enjoying the subject premises for Peña and Gaddi, together with AIB and BSP, be held liable for:
more than 15 days, without objection from the respondent. By (a) the value of the subject vehicle; (b) amount representing daily
the inaction of the respondent as lessor, there can be no loss of income/boundary reckoned from the day the vehicle was
inference that it intended to discontinue the lease contract, lost; (c) exemplary damages; (d) moral damages; (e) attorney's
therefore, an implied new lease was therefore created pursuant fees; and (f) cost of suit.
to Article 1670 of the Civil Code which provides:
BSP denied any liability contending that not only did Sps.
Article 1670. If at the end of the contract of lease the lessee Mamaril directly deal with AIB with respect to the manner by
should continue enjoying the thing leased for 15 days with the which the parked vehicles would be handled, but the parking
acquiescence of the lessor, and unless a contrary by either party ticket itself expressly stated that the "Management shall not be
has previously been given, it is understood that there is implied responsible for loss of vehicle or any of its accessories or article
new lease, not for the period of the original contract, but for the left therein." It also claimed that Sps. Mamaril erroneously relied
time established in Articles 1682 and 1687. The other terms of on the Guard Service Contract. Apart from not being parties
the original contract shall be revived. thereto, its provisions cover only the protection of BSP's
properties, its officers, and employees.
An implied new lease or tacita reconduccion will set in when the
following requisites are found to exist: ISSUE: Whether or not BSP may be held liable for the loss of the
(a) The term of the original contract of lease has expired; vehicle caused by the negligence of its security guards. NO
(b) The lessor has not given the lessee a demand to vacate; and
(c) The lessee continued enjoying the thing leased for 15 days HELD: The proximate cause of the loss of Sps. Mamaril's vehicle
with the acquiescence of the lessor. was the negligent act of security guards Peña and Gaddi in
allowing an unidentified person to drive out the subject vehicle.
Article 1687 of the CC on implied new lease provides:
The records are bereft of any finding of negligence on the part of
Article 1687. If the period for the lease has not been fixed, it is BSP. Neither will the vicarious liability of an employer under
understood to be from year to year, if the rent is to be paid Article 2180 of the Civil Code apply in this case. Peña and Gaddi
annual; from month to month if it is monthly; from week to week, were assigned as security guards by AIB to BSP pursuant to the
if the rent is weekly; and from day to day, if the rent id to be paid Guard Service Contract. No employer-employee relationship
daily. However, even though a monthly rent is paid, and no existed between BSP and the security guards assigned in its
period for the lease has been set, the court may fix a longer term premises. Sps. Mamaril are not parties to the Guard Service
for the lease after the lessee has occupied the premises for over Contract. Guard Service Contract between defendant-appellant
one year. If the rent id weekly, the court may likewise determine BSP and defendant AIB Security Agency is purely between the
a longer period after the lessee has been in possession for over parties therein.
6 months. In case of daily rent, the courts may fix a longer period
after the lessee has stayed in the place for over one month. Contracts take effect only between the parties, their assigns and
heirs, except in case where the rights and obligations arising
Since the rent was paid on a monthly basis, the period of lease is from the contract are not transmissible by their nature, or by
considered to be from month to month. A lease from month to stipulation or by provision of law. The heir is not liable beyond the
month is considered to be one with a definite period, which value of the property he received from the decedent. If a contract
expires at the end of each month upon a demand to vacate by should contain some stipulation in favor of a third person, he may
the lessor. When the respondent sent a letter to vacate to the demand its fulfillment provided he communicated his acceptance
petitioner on August 5, 1998, the tacita reconduccion was to the obligor before its revocation. A mere incidental benefit or
aborted, and the contract of lease is deemed to have expired at interest of a person is not sufficient. The contracting parties must
the end of that month. A notice to vacate constitutes an express have clearly and deliberately conferred a favor upon a third
COMPILED BY: WIGMORE #WIGMOREFOREVER 133
SALES Case Digest (Atty. Sarona)
Compiled by: Wigmore #wigmoreforever

person.

Thus, in order that a third person benefited by the second


paragraph of Article 1311, referred to as a stipulation pour autrui,
may demand its fulfillment, the following requisites must concur:
(1) There is a stipulation in favor of a third person; (2) The
stipulation is a part, not the whole, of the contract; (3) The
contracting parties clearly and deliberately conferred a favor to
the third person - the favor is not merely incidental; (4) The favor
is unconditional and uncompensated; (5) The third person
communicated his or her acceptance of the favor before its
revocation; and (6) The contracting parties do not represent, or
are not authorized, by the third party. However, none of the
foregoing elements obtains in this case. There is absolutely
nothing in the said contract that would indicate any obligation
and/or liability on the part of the parties therein in favor of third
persons such as herein plaintiffs-appellees.

Moreover, the Court concurs with the finding of the CA that


the contract between the parties herein was one of lease as
defined under Article 1643 of the Civil Code. It has been held
that the act of parking a vehicle in a garage, upon payment
of a fixed amount, is a lease.

A lessor-lessee relationship existed between Spouses


Mamaril and BSP. Article 1664 of the same Code states that
the lessor is not obliged to answer for a mere act of
trespass which a third person may cause on the use of the
thing leased; but the lessee shall have a direct action
against the intruder. Here, BSP was not remiss in its
obligation to provide Spouses Mamaril a suitable parking
space for their jeepneys as it even hired security guards to
secure the premises; hence, it should not be held liable for
the loss suffered by Spouses Mamaril.

The agreement with respect to the ingress and egress of


Sps. Mamaril's vehicles were coordinated only with AIB and
its security guards, without the knowledge and consent of
BSP. Accordingly, the mishandling of the parked vehicles
that resulted in herein complained loss should be recovered
only from the tort feasors (Peña and Gaddi) and their
employer, AIB; and not against the lessor, BSP.

END

COMPILED BY: WIGMORE #WIGMOREFOREVER 134

S-ar putea să vă placă și